Acquisition by Creation

3. Acquisition by Creation

 

3.1. Unfair Competition

 

International News Service v. Associated Press,

248 U.S. 215 (1918)

 

 

 

Messrs. Samuel Untermyer, of New York City, Hiram W. Johnson, of San Francisco, Cal., and Henry A. Wise and William A De Ford, both of New York City, for petitioner.

 

Mr. Frederic W. Lehmann, of St. Louis, Mo., for respondent.

 

 

 

Mr. Justice Pitney delivered the opinion of the Court.

 

The parties are competitors in the gathering and distribution of news and its publication for profit in newspapers throughout the United States. The Associated Press, which was complainant in the District Court, is a co-operative organization, incorporated under the Membership Corporations Law of the state of New York, its members being individuals who are either proprietors or representatives of about 950 daily newspapers published in all parts of the United States. That a corporation may be organized under that act for the purpose of gathering news for the use and benefit of its members and for publication in newspapers owned or represented by them, is recognized by an amendment enacted in 1901 (Laws N. Y. 1901, c. 436). Complainant gathers in all parts of the world, by means of various instrumentalities of its own, by exchange with its members, and by other appropriate means, news and intelligence of current and recent events of interest to newspaper readers and distributes it daily to its members for publication in their newspapers. The cost of the service, amounting approximately to $3,500,000 per annum, is assessed upon the members and becomes a part of their costs of operation, to be recouped, presumably with profit, through the publication of their several newspapers. Under complainant’s by-laws each member agrees upon assuming membership that news received through complainant’s service is received exclusively for publication in a particular newspaper, language, and place specified in the certificate of membership, that no other use of it shall be permitted, and that no member shall furnish or permit any one in his employ or connected with his newspaper to furnish any of complainant’s news in advance of publication to any person not a member. And each member is required to gather the local news of his district and supply it to the Associated Press and to no one else.

 

Defendant is a corporation organized under the laws of the state of New Jersey, whose business is the gathering and selling of news to its customers and clients, consisting of newspapers published throughout the United States, under contracts by which they pay certain amounts at stated times for defendant’s service. It has widespread news-gathering agencies; the cost of its operations amounts, it is said, to more than $2,000,000 per annum; and it serves about 400 newspapers located in the various cities of the United States and abroad, a few of which are represented, also, in the membership of the Associated Press.

 

The parties are in the keenest competition between themselves in the distribution of news throughout the United States; and so, as a rule, are the newspapers that they serve, in their several districts.

 

Complainant in its bill, defendant in its answer, have set forth in almost identical terms the rather obvious circumstances and conditions under which their business is conducted. The value of the service, and of the news furnished, depends upon the promptness of transmission, as well as upon the accuracy and impartiality of the news; it being essential that the news be transmitted to members or subscribers as early or earlier than similar information can be furnished to competing newspapers by other news services, and that the news furnished by each agency shall not be furnished to newspapers which do not contribute to the expense of gathering it. And further, to quote from the answer:

 

 

Prompt knowledge and publication of worldwide news is essential to the conduct of a modern newspaper, and by reason of the enormous expense incident to the gathering and distribution of such news, the only practical way in which a proprietor of a newspaper can obtain the same is, either through co-operation with a considerable number of other newspaper proprietors in the work of collecting and distributing such news, and the equitable division with them of the expenses thereof, or by the purchase of such news from some existing agency engaged in that business.

 

 

 

 

The bill was filed to restrain the pirating of complainant’s news by defendant in three ways: First, by bribing employes of newspapers published by complainant’s members to furnish Associated Press news to defendant before publication, for transmission by telegraph and telephone to defendant’s clients for publication by them; second, by inducing Associated Press members to violate its by-laws and permit defendant to obtain news before publication; and, third, by copying news from bulletin boards and from early editions of complainant’s newspapers and selling this, either bodily or after rewriting it, to defendant’s customers.

 

The District Court, upon consideration of the bill and answer, with voluminous affidavits on both sides, granted a preliminary injunction under the first and second heads, but refused at that stage to restrain the systematic practice admittedly pursued by defendant, of taking news bodily from the bulletin boards and early editions of complainant’s newspapers and selling it as its own. The court expressed itself as satisfied that this practice amounted to unfair trade, but as the legal question was one of first impression it considered that the allowance of an injunction should await the outcome of an appeal. 240 Fed. 983, 996. Both parties having appealed, the Circuit Court of Appeals sustained the injunction order so far as it went, and upon complainant’s appeal modified it and remanded the cause, with directions to issue an injunction also against any bodily taking of the words or substance of complainant’s news until its commercial value as news had passed away. 245 Fed. 244, 253. The present writ of certiorari was then allowed. 245 U. S. 644.

 

The only matter that has been argued before us is whether defendant may lawfully be restrained from appropriating news taken from bulletins issued by complainant or any of its members, or from newspapers published by them, for the purpose of selling it to defendant’s clients. Complainant asserts that defendant’s admitted course of conduct in this regard both violates complainant’s property right in the news and constitutes unfair competition in business. And notwithstanding the case has proceeded only to the stage of a preliminary injunction, we have deemed it proper to consider the underlying questions, since they go to the very merits of the action and are presented upon facts that are not in dispute. As presented in argument, these questions are: (1) Whether there is any property in news; (2) Whether, if there be property in news collected for the purpose of being published, it survives the instant of its publication in the first newspaper to which it is communicated by the news-gatherer; and (3) whether defendant’s admitted course of conduct in appropriating for commercial use matter taken from bulletins or early editions of Associated Press publications constitutes unfair competition in trade.

 

The federal jurisdiction was invoked because of diversity of citizenship, not upon the ground that the suit arose under the copyright or other laws of the United States. Complainant’s news matter is not copyrighted. It is said that it could not, in practice, be copyrighted, because of the large number of dispatches that are sent daily; and, according to complainant’s contention, news is not within the operation of the copyright act. Defendant, while apparently conceding this, nevertheless invokes the analogies of the law of literary property and copyright, insisting as its principal contention that, assuming complainant has a right of property in its news, it can be maintained (unless the copyright act by complied with) only by being kept secret and confidential, and that upon the publication with complainant’s consent of uncopyrighted news of any of complainant’s members in a newspaper or upon a bulletin board, the right of property is lost, and the subsequent use of the news by the public or by defendant for any purpose whatever becomes lawful.

 

A preliminary objection to the form in which the suit is brought may be disposed of at the outset. It is said that the Circuit Court of Appeals granted relief upon considerations applicable to particular members of the Associated Press, and that this was erroneous because the suit was brought by complainant as a corporate entity, and not by its members; the argument being that their interests cannot be protected in this proceeding any more than the individual rights of a stockholder can be enforced in an action brought by the corporation. From the averments of the bill, however, it is plain that the suit in substance was brought for the benefit of complainant’s members, and that they would be proper parties, and, except for their numbers, perhaps necessary parties. Complainant is a proper party to conduct the suit as representing their interest; and since no specific objection, based upon the want of parties, appears to have been made below, we will treat the objection as waived. See Equity Rules 38, 43, 44 (33 Sup. Ct. xxix, xxx).

 

In considering the general question of property in news matter, it is necessary to recognize its dual character, distinguishing between the substance of the information and the particular form or collocation of words in which the writer has communicated it.

 

No doubt news articles often possess a literary quality, and are the subject of literary property at the common law; nor do we question that such an article, as a literary production, is the subject of copyright by the terms of the act as it now stands. In an early case at the circuit Mr. Justice Thompson held in effect that a newspaper was not within the protection of the copyright acts of 1790 (1 Stat. 124) and 1802 (2 Stat. 171). Clayton v. Stone, 2 Paine, 382. But the present act is broader; it provides that the works for which copyright may be secured shall include ‘all the writings of an author,’ and specifically mentions ‘periodicals, including newspapers.’ Act of March 4, 1909, c. 320, §§ 4 and 5, 35 Stat. 1075, 1076. Evidently this admits to copyright a contribution to a newspaper, notwithstanding it also may convey news; and such is the practice of the copyright office, as the newspapers of the day bear witness. See Copyright Office Bulletin No. 15 (1917) pp. 7, 14, 16, 17.

 

But the news element – the information respecting current events contained in the literary production – is not the creation of the writer, but is a report of matters that ordinarily are publici juris; it is the history of the day. It is not to be supposed that the framers of the Constitution, when they empowered Congress ‘to promote the progress of science and useful arts, by securing for limited times to authors and inventors the exclusive right to their respective writings and discoveries’ (Const. art. 1, § 8, par. 8), intended to confer upon one who might happen to be the first to report a historic event the exclusive right for any period to spread the knowledge of it.

 

We need spend no time, however, upon the general question of property in news matter at common law, or the application of the copyright act, since it seems to us the case must turn upon the question of unfair competition in business. And, in our opinion, this does not depend upon any general right of property analogous to the common-law right of the proprietor of an unpublished work to prevent its publication without his consent; nor is it foreclosed by showing that the benefits of the copyright act have been waived. We are dealing here not with restrictions upon publication but with the very facilities and processes of publication. The peculiar value of news is in the spreading of it while it is fresh; and it is evident that a valuable property interest in the news, as news, cannot be maintained by keeping it secret. Besides, except for matters improperly disclosed, or published in breach of trust or confidence, or in violation of law, none of which is involved in this brance of the case, the news of current events may be regarded as common property. What we are concerned with is the business of making it known to the world, in which both parties to the present suit are engaged. That business consists in maintaining a prompt, sure, steady, and reliable service designed to place the daily events of the world at the breakfast table of the millions at a price that, while of trifling moment to each reader, is sufficient in the aggregate to afford compensation for the cost of gathering and distributing it, with the added profit so necessary as an incentive to effective action in the commercial world. The service thus performed for newspaper readers is not only innocent but extremely useful in itself, and indubitably constitutes a legitimate business. The parties are competitors in this field; and, on fundamental principles, applicable here as elsewhere, when the rights or privileges of the one are liable to conflict with those of the other, each party is under a duty so to conduct its own business as not unnecessarily or unfairly to injure that of the other. Hitchman Coal & Coke Co. v. Mitchell, 245 U. S. 229, 254.

 

Obviously, the question of what is unfair competition in business must be determined with particular reference to the character and circumstances of the business. The question here is not so much the rights of either party as against the public but their rights as between themselves. See Morison v. Moat, 9 Hare, 241, 258. And, although we may and do assume that neither party has any remaining property interest as against the public in uncopyrighted news matter after the moment of its first publication, it by no means follows that there is no remaining property interest in it as between themselves. For, to both of them alike, news matter, however little susceptible of ownership or dominion in the absolute sense, is stock in trade, to be gathered at the cost of enterprise, organization, skill, labor, and money, and to be distributed and sold to those who will pay money for it, as for any other merchandise. Regarding the news, therefore, as but the material out of which both parties are seeking to make profits at the same time and in the same field, we hardly can fail to recognize that for this purpose, and as between them, it must be regarded as quasi property, irrespective of the rights of either as against the public.

 

… .

 

The question, whether one who has gathered general information or news at pains and expense for the purpose of subsequent publication through the press has such an interest in its publication as may be protected from interference, has been raised many times, although never, perhaps, in the precise form in which it is now presented.

 

Board of Trade v. Christie Grain & Stock Co., 198 U. S. 236, 250, related to the distribution of quotations of prices on dealings upon a board of trade, which were collected by plaintiff and communicated on confidential terms to numerous persons under a contract not to make them public. This court held that, apart from certain special objections that were overruled, plaintiff’s collection of quotations was entitled to the protection of the law; that, like a trade secret, plaintiff might keep to itself the work done at its expense, and did not lose its right by communicating the result to persons, even if many, in confidential relations to itself, under a contract not to make it public; and that strangers should be restrained from getting at the knowledge by inducing a breach of trust.

 

In National Tel. News Co. v. Western Union Tel. Co., 119 Fed. 294, the Circuit Court of Appeals for the Seventh Circuit dealt with news matter gathered and transmitted by a telegraph company, and consisting merely of a notation of current events having but a transient value due to quick transmission and distribution; and, while declaring that this was not copyrightable although printed on a tape by tickers in the offices of the recipients, and that it was a commercial not a literary product, nevertheless held that the business of gathering and communicating the news – the service of purveying it was a legitimate business, meeting a distinctive commercial want and adding to the facilities of the business world, and partaking of the nature of property in a sense that entitled it to the protection of a court of equity against piracy.

 

Other cases are cited, but none that we deem it necessary to mention.

 

Not only do the acquisition and transmission of news require elaborate organization and a large expenditure of money, skill, and effort; not only has it an exchange value to the gatherer, dependent chiefly upon its novelty and freshness, the regularity of the service, its reputed reliability and thoroughness, and its adaptability to the public needs; but also, as is evident, the news has an exchange value to one who can misappropriate it.

 

The peculiar features of the case arise from the fact that, while novelty and freshness form so important an element in the success of the business, the very processes of distribution and publication necessarily occupy a good deal of time. Complainant’s service, as well as defendant’s, is a daily service to daily newspapers; most of the foreign news reaches this country at the Atlantic seaboard, principally at the city of New York, and because of this, and of time differentials due to the earth’s rotation, the distribution of news matter throughout the country is principally from east to west; and, since in speed the telegraph and telephone easily outstrip the rotation of the earth, it is a simple matter for defendant to take complainant’s news from bulletins or early editions of complainant’s members in the eastern cities and at the mere cost of telegraphic transmission cause it to be published in western papers issued at least as warly as those served by complainant. Besides this, and irrespective of time differentials, irregularities in telegraphic transmission on different lines, and the normal consumption of time in printing and distributing the newspaper, result in permitting pirated news to be placed in the hands of defendant’s readers sometimes simultaneously with the service of competing Associated Press papers, occasionally even earlier.

 

Defendant insists that when, with the sanction and approval of complainant, and as the result of the use of its news for the very purpose for which it is distributed, a portion of complainant’s members communicate it to the general public by posting it upon bulletin boards so that all may read, or by issuing it to newspapers and distributing it indiscriminately, complainant no longer has the right to control the use to be made of it; that when it thus reaches the light of day it becomes the common possession of all to whom it is accessible; and that any purchaser of a newspaper has the right to communicate the intelligence which it contains to anybody and for any purpose, even for the purpose of selling it for profit to newspapers published for profit in competition with complainant’s members.

 

The fault in the reasoning lies in applying as a test the right of the complainant as against the public, instead of considering the rights of complainant and defendant, competitors in business, as between themselves. The right of the purchaser of a single newspaper to spread knowledge of its contents gratuitously, for any legitimate purpose not unreasonably interfering with complainant’s right to make merchandise of it, may be admitted; but to transmit that news for commercial use, in competition with complainant – which is what defendant has done and seeks to justify – is a very different matter. In doing this defendant, by its very act, admits that it is taking material that has been acquired by complainant as the result of organization and the expenditure of labor, skill, and money, and which is salable by complainant for money, and that defendant in appropriating it and selling it as its own is endeavoring to reap where it has not sown, and by disposing of it to newspapers that are competitors of complainant’s members is appropriating to itself the harvest of those who have sown. Stripped of all disguises, the process amounts to an unauthorized interference with the normal operation of complainant’s legitimate business precisely at the point where the profit is to be reaped, in order to divert a material portion of the profit from those who have earned it to those who have not; with special advantage to defendant in the competition because of the fact that it is not burdened with any part of the expense of gathering the news. The transaction speaks for itself and a court of equity ought not to hesitate long in characterizing it as unfair competition in business.

 

… .

 

The contention that the news is abandoned to the public for all purposes when published in the first newspaper is untenable. Abandonment is a question of intent, and the entire organization of the Associated Press negatives such a purpose. The cost of the service would be prohibited if the reward were to be so limited. No single newspaper, no small group of newspapers, could sustain the expenditure. Indeed, it is one of the most obvious results of defendant’s theory that, by permitting indiscriminate publication by anybody and everybody for purposes of profit in competition with the news-gatherer, it would render publication profitless, or so little profitable as in effect to cut off the service by rendering the cost prohibitive in comparison with the return. The practical needs and requirements of the business are reflected in complainant’s by-laws which have been referred to. Their effect is that publication by each member must be deemed not by any means an abandonment of the news to the world for any and all purposes, but a publication for limited purposes; for the benefit of the readers of the bulletin or the newspaper as such; not for the purpose of making merchandise of it as news, with the result of depriving complainant’s other members of their reasonable opportunity to obtain just returns for their expenditures.

 

It is to be observed that the view we adopt does not result in giving to complainant the right to monopolize either the gathering or the distribution of the news, or, without complying with the copyright act, to prevent the reproduction of its news articles, but only postpones participation by complainant’s competitor in the processes of distribution and reproduction of news that it has not gathered, and only to the extent necessary to prevent that competitor from reaping the fruits of complainant’s efforts and expenditure, to the partial exclusion of complainant. and in violation of the principle that underlies the maxim ‘sic utere tuo,’ etc.

 

It is said that the elements of unfair competition are lacking because there is no attempt by defendant to palm off its goods as those of the complainant, characteristic of the most familiar, if not the most typical, cases of unfair competition. Howe Scale Co. v. Wyckoff, Seamans, etc., 198 U. S. 118, 140. But we cannot concede that the right to equitable relief is confined to that class of cases. In the present case the fraud upon complainant’s rights is more direct and obvious. Regarding news matter as the mere material from which these two competing parties are endeavoring to make money, and treating it, therefore, as quasi property for the purposes of their business because they are both selling it as such, defendant’s conduct differs from the ordinary case of unfair competition in trade principally in this that, instead of selling its own goods as those of complainant, it substitutes misappropriation in the place of misrepresentation, and sells complainant’s goods as its own.

 

Besides the misappropriation, there are elements of imitation, of false pretense, in defendant’s practices. The device of rewriting complainant’s news articles, frequently resorted to, carries its own comment. The habitual failure to give credit to complainant for that which is taken is significant. Indeed, the entire system of appropriating complainant’s news and transmitting it as a commercial product to defendant’s clients and patrons amounts to a false representation to them and to their newspaper readers that the news transmitted is the result of defendant’s own investigation in the field. But these elements, although accentuating the wrong, are not the essence of it. It is something more than the advantage of celebrity of which complainant is being deprived.

 

The doctrine of unclean hands is invoked as a bar to relief; it being insisted that defendant’s practices against which complainant seeks an injunction are not different from the practice attributed to complainant, of utilizing defendant’s news published by its subscribers. At this point it becomes necessary to consider a distinction that is drawn by complainant, and, as we understand it, was recognized by defendant also in the submission of proofs in the District Court, between two kinds of use that may be made by one news agency of news taken from the bulletins and newspapers of the other. The first is the bodily appropriation of a statement of fact or a news article, with or without rewriting, but without independent investigation or other expense. This form of pirating was found by both courts to have been pursued by defendant systematically with respect to complainant’s news, and against it the Circuit Court of Appeals granted an injunction. This practice complainant denies having pursued and the denial was sustained by the finding of the District Court. It is not contended by defendant that the finding can be set aside, upon the proofs as they now stand. The other use is to take the news of a rival agency as a ‘tip’ to be investigated, and if verified by independent investigation the news thus gathered is sold. This practice complainant admits that it has pursued and still is willing that defendant shall employ.

 

Both courts held that complainant could not be debarred on the ground of unclean hands upon the score of pirating defendant’s news, because not shown to be guilty of sanctioning this practice.

 

As to securing ‘tips’ from a competing news agency the District Court (240 Fed. 991, 995), while not sanctioning the practice, found that both parties had adopted it in accordance with common business usage, in the belief that their conduct was technically lawful, and hence did not find in it any sufficient ground for attributing unclean hands to complainant. The Circuit Court of Appeals (245 Fed. 247, 157) found that the tip habit, though discouraged by complainant, was ‘incurably journalistic,’ and that there was ‘no difficulty in discriminating between the utilization of tips and the bodily appropriation of another’s labor in accumulating and stating information.’

 

We are inclined to think a distinction may be drawn between the utilization of tips and the bodily appropriation of news matter, either in its original form or after rewriting and without independent investigation and verification; whatever may appear at the final hearing, the proofs as they now stand recognize such a distinction; both parties avowedly recognize the practice of taking tips, and neither party alleges it to be unlawful or to amount to unfair competition in business. In a line of English cases a somewhat analogous practice has been held not to amount to an infringement of the copyright of a directory or other book containing compiled information. In Kelly v. Morris, L. R. 1 Eq. 697, 701, 702, Vice Chancellor Sir William Page Wood (afterwards Lord Hatherly), dealing with such a case, said that defendant was

 

 

not entitled to take one word of the information previously published without independently working out the matter for himself, so as to arrive at the same result from the same common sources of information, and the only use that he can legitimately make of a previous publication is to verify his own calculations and results when obtained.

 

 

This was followed by Vice Chancellor Giffard in Morris v. Ashbee, L. R. 7 Eq. 34, where he said:

 

 

In a case such as this no one has a right to take the results of the labour and expense incurred by another for the purposes of a rival publication, and thereby save himself the expense and labour of working out and arriving at these results by some independent road.

 

 

A similar view was adopted by Lord Chancellor Hatherly and the former Vice Chancellor, then Giffard, L. J., in Pike v. Nicholas, L. R. 5 Ch. App. Cas. 251, and shortly afterwards by the latter judge in Morris v. Wright, L. R. 5 Ch. App. Cas. 279, 287, where he said, commenting upon Pike v. Nicholas:

 

 

It was a perfectly legitimate course for the defendant to refer to the plaintiff’s book, and if, taking that book as his guide, he went to the original authorities and compiled his book from them, he made no unfair or improper use of the plaintiff’s book; and so here, if the fact be that Mr. Wright used the plaintiff’s book in order to guide himself to the persons on whom it would be worth his while to call, and for no other purpose, he made a perfectly legitimate use of the plaintiff’s book.

 

 

A like distinction was recognized by the Circuit Court of Appeals for the Second Circuit in Edward Thompson Co. v. American Law Book Co., 122 Fed. 922 and in West Pub. Co. v. Edward Thompson Co., 176 Fed. 833, 838.

 

In the case before us, in the present state of the pleadings and proofs, we need go no further than to hold, as we do, that the admitted pursuit by complainant of the practice of taking news items published by defendant’s subscribers as tips to be investigated, and, if verified, the result of the investigation to be sold – the practice having been followed by defendant also, and by news agencies generally – is not shown to be such as to constitute an unconscientious or inequitable attitude towards its adversary so as to fix upon complainant the taint of unclean hands, and debar it on this ground from the relief to which it is otherwise entitled.

 

There is some criticism of the injunction that was directed by the District Court upon the going down of the mandate from the Circuit Court of Appeals. In brief, it restrains any taking or gainfully using of the complainant’s news, either bodily or in substance from bulletins issued by the complainant or any of its members, or from editions of their newspapers, ’until its commercial value as news to the complainant and all of its members has passed away.’ The part complained of is the clause we have italicized; but if this be indefinite, it is no more so than the criticism. Perhaps it would be better that the terms of the injunction be made specific, and so framed as to confine the restraint to an extent consistent with the reasonable protection of complainant’s newspapers, each in its own area and for a specified time after its publication, against the competitive use of pirated news by defendant’s customers. But the case presents practical difficulities; and we have not the materials, either in the way of a definite suggestion of amendment, or in the way of proofs, upon which to frame a specific injunction; hence, while not expressing approval of the form adopted by the District Court, we decline to modify it at this preliminary stage of the case, and will leave that court to deal with the matter upon appropriate application made to it for the purpose.

 

The decree of the Circuit court of Appeals will be

 

Affirmed.

 

Mr. Justice Clarke took no part in the consideration or decision of this case.

 

 

 

Mr. Justice Holmes, dissenting.

 

When an uncopyrighted combination of words is published there is no general right to forbid other people repeating them – in other words there is no property in the combination or in the thoughts or facts that the words express. Property, a creation of law, does not arise from value, although exchangeable – a matter of fact. Many exchangeable values may be destroyed intentionally without compensation. Property depends upon exclusion by law from interference, and a person is not excluded from using any combination of words merely because some one has used it before, even if it took labor and genius to make it. If a given person is to be prohibited from making the use of words that his neighbors are free to make some other ground must be found. One such ground is vaguely expressed in the phrase unfair trade. This means that the words are repeated by a competitor in business in such a way as to convey a misrepresentation that materially injures the person who first used them, by appropriating credit of some kind which the first user has earned. The ordinary case is a representation by device, appearance, or other indirection that the defendant’s goods come from the plaintiff. But the only reason why it is actionable to make such a representation is that it tends to give the defendant an advantage in his competition with the plaintiff and that it is thought undesirable that an advantage should be gained in that way. Apart from that the defendant may use such unpatented devices and uncopyrighted combinations of words as he likes. The ordinary case, I say, is palming off the defendant’s product as the plaintiff’s but the same evil may follow from the opposite falsehood – from saying whether in words or by implication that the plaintiff’s product is the defendant’s, and that, it seems to me, is what has happened here.

 

Fresh news is got only by enterprise and expense. To produce such news as it is produced by the defendant represents by implication that it has been acquired by the defendant’s enterprise and at its expense. When it comes from one of the great news collecting agencies like the Associated Press, the source generally is indicated, plainly importing that credit; and that such a representation is implied may be inferred with some confidence from the unwillingness of the defendant to give the credit and tell the truth. If the plaintiff produces the news at the same time that the defendant does, the defendant’s presentation impliedly denies to the plaintiff the credit of collecting the facts and assumes that credit to the defendant. If the plaintiff is later in Western cities it naturally will be supposed to have obtained its information from the defendant. The falsehood is a little more subtle, the injury, a little more indirect, than in ordinary cases of unfair trade, but I think that the principle that condemns the one condemns the other. It is a question of how strong an infusion of fraud is necessary to turn a flavor into a poison. The does seems to me strong enough here to need a remedy from the law. But as, in my view, the only ground of complaint that can be recognized without legislation is the implied misstatement, it can be corrected by stating the truth; and a suitable acknowledgment of the source is all that the plaintiff can require. I think that within the limits recognized by the decision of the Court the defendant should be enjoined from publishing news obtained from the Associated Press for hours after publication by the plaintiff unless it gives express credit to the Associated Press; the number of hours and the form of acknowledgment to be settled by the District Court.

 

Mr. Justice McKenna concurs in this opinion.

 

 

 

Mr. Justice Brandeis, dissenting.

 

There are published in the United States about 2,500 daily papers. More than 800 of them are supplied with domestic and foreign news of general interest by the Associated Press – a corporation without capital stock which does not sell news or earn or seek to earn profits, but serves merely as an instrumentality by means of which these papers supply themselves at joint expense with such news. Papers not members of the Associated Press depend for their news of general interest largely upon agencies organized for profit. Among these agen cies is the International News Service which supplies news to about 400 subscribing papers. It has, like the Associated Press, bureaus and correspondents in this and foreign countries; and its annual expenditures in gathering and distributing news is about $2,000,000. Ever since its organization in 1909, it has included among the sources from which it gathers news, copies (purchased in the open market of early editions of some papers published by members of the Associated Press and the bulletins publicly posted by them. These items, which constitute but a small part of the news transmitted to its subscribers, are generally verified by the International News Service before transmission; but frequently items are transmitted without verification; and occasionally even without being re-written. In no case is the fact disclosed that such item was suggested by or taken from a paper or bulletin published by an Associated Press member.

 

No question of statutory copyright is involved. The sole question for our consideration is this: Was the International News Service properly enjoined from using, or causing to be used gainfully, news of which it acquired knowledge by lawful means (namely, by reading publicly posted bulletins or papers purchased by it in the open market) merely because the news had been originally gathered by the Associated Press and continued to be of value to some of its members, or because it did not reveal the source from which it was acquired?

 

The ‘ticker’ cases, the cases concerning literary and artistic compositions, and cases of unfair competition were relied upon in support of the injunction. But it is admitted that none of those cases affords a complete analogy with that before us. The question presented for decision is new, and it is important.

 

News is a report of recent occurrences. The business of the news agency is to gather systematically knowledge of such occurrences of interest and to distribute reports thereof. The Associated Press contended that knowledge so acquired is property, because it costs money and labor to produce and because it has value for which those who have it not are ready to pay; that it remains property and is entitled to protection as long as it has commercial value as news; and that to protect it effectively, the defendant must be enjoined from making, or causing to be made, any gainful use of it while it retains such value. An essential element of individual property is the legal right to exclude others from enjoying it. If the property is private, the right of exclusion may be absolute; if the property is affected with a public interest, the right of exclusion is qualified. But the fact that a product of the mind has cost its producer money and labor, and has a value for which others are willing to pay, is not sufficient to ensure to it this legal attribute of property. The general rule of law is, that the noblest of human productions – knowledge, truths ascertained, conceptions, and ideas – became, after voluntary communication to others, free as the air to common use. Upon these incorporeal productions the attribute of property is continued after such communication only in certain classes of cases where public policy has seemed to demand it. These exceptions are confined to productions which, in some degree, involve creation, invention, or discovery. But by no means all such are endowed with this attribute of property. The creations which are recognized as property by the common law are literary, dramatic, musical, and other artistic creations; and these have also protection under the copyright statutes. The inventions and discoveries upon which this attribute of property is conferred only by statute, are the few comprised within the patent law. There are also many other cases in which courts interfere to prevent curtailment of plaintiff’s enjoyment of incorporal productions; and in which the right to relief is often called a property right, but is such only in a special sense. In those cases, the plaintiff has no absolute right to the protection of his production; he has merely the qualified right to be protected as against the defendant’s acts, because of the special relation in which the latter stands or the wrongful method or means employed in acquiring the knowledge or the manner in which it is used. Protection of this character is afforded where the suit is based upon breach of contract or of trust or upon unfair competition.

 

The knowledge for which protection is sought in the case at bar is not of a kind upon which the law has heretofore conferred the attributes of property; nor is the manner of its acquisition or use nor the purpose to which it is applied, such as has heretofore been recognized as entitling a plaintiff to relief.

 

First. Plaintiff’s principal reliance was upon the ‘ticker’ cases; but they do not support its contention. The leading cases on this subject rest the grant of relief, not upon the existence of a general property right in news, but upon the breach of a contract or trust concerning the use of news communicated; and that element is lacking here… . .

 

… .

 

If the news involved in the case at bar had been posted in violation of any agreement between the Associated Press and its members, questions similar to those in the ‘ticker’ cases might have arisen. But the plaintiff does not contend that the posting was wrongful or that any papers were wrongfully issued by its subscribers. On the contrary it is conceded that both the bulletins and the papers were issued in accordance with the regulations of the plaintiff. Under such circumstances, for a reader of the papers purchased in the open market, or a reader of the bulletins publicly posted, to procure and use gainfully, information therein contained, does not involve inducing any one to commit a breach either of contract or of trust, or committing or in any way abetting a breach of confidence.

 

Second. Plaintiff also relied upon the cases which hold that the common law right of the producer to prohibit copying is not lost by the private circulation of a literary composition, the delivery of a lecture, the exhibition of a painting, or the performance of a dramatic or musical composition. These cases rest upon the ground that the common law recognizes such productions as property which, despite restricted communication, continues until there is a dedication to the public under the copyright statutes or otherwise. But they are inapplicable for two reasons: (1) At common law, as under the copyright acts, intellectual productions are entitled to such protection only if there is underneath something evincing the mind of a creator or originator, however modest the requirement. The mere record of isolated happenings, whether in words or by photographs not involving artistic skill, are denied such protection. (2) At common law, as under the copyright acts, the element in intellectual productions which secures such protection, is not the knowledge, truths, ideas, or emotions which the composition expresses, but the form or sequence in which they are expressed; that is, ‘some new collocation of visible or audible points – of lines, colors, sounds, or words.’ See White-Smith Music Co. v. Apollo Co., 209 U. S. 1, 19; Kalem Co. v. Harper Bros., 222 U. S. 55, 63. An author’s theories, suggestions, and speculations, or the systems, plans, methods, and arrangements of an originator, derive no such protection from the statutory copyright of the book in which they are set forth; and they are likewise denied such protection at common law.

 

That news is not property in the strict sense is illustrated by the case of Sports and General Press Agency, Ltd., v. ‘Our Dogs’ Publishing Co., Ltd., [1916] 2 K. B. 880, where the plaintiff, the assignee of the right to photograph the exhibits at a dog show, was refused an injunction against defendant who had also taken pictures of the show and was publishing them. The court said that, except in so far as the possession of the land occupied by the show enabled the proprietors to exclude people or permit them on condition that they agree not to take photographs (which condition was not imposed in that case), the proprietors had no exclusive right to photograph the show and could therefore grant no such right. And it was further stated that, at any rate, no matter what conditions might be imposed upon those entering the grounds, if the defendant had been on top of a house or in some position where he could photograph the show without interfering with the physical property of the plaintiff, the plaintiff would have no right to stop him. If, when the plaintiff creates the event recorded, he is not entitled to the exclusive first publication of the news (in that case a photograph) of the event, no reason can be shown why he should be accorded such protection as to events which he simply records and transmits to other parts of the world, though with great expenditure of time and money.

 

Third. If news be treated as possessing the characteristics not of a trade secret, but of literary property, then the earliest issue of a paper of general circulation or the earliest public posting of a bulletin which embodies such news would, under the established rules governing literary property, operate as a publication, and all property in the news would then cease. Resisting this conclusion, plaintiff relied upon the cases which hold that uncopyrighted intellectual and artistic property survives private circulation or a restricted publication; and it contended that in each issue of each paper, a restriction is to be implied, that the news shall not be used gainfully in competition with the Associated Press or any of its members. There is no basis for such an implication. But it is, also, well settled that where the publication is in fact a general one – even express words of restriction upon use are inoperative. In other words, a general publication is effective to dedicate literary property to the public, regardless of the actual intent of its owner. In the cases dealing with lectures, dramatic and musical performances, and art exhibitions, upon which plaintiff relied, there was no general publication in print comparable to the issue of daily newspapers or the unrestricted public posting of bulletins. The principles governing those cases differ more or less in application, if not in theory, from the principles governing the issue of printed copies; and in so far as they do differ, they have no application to the case at bar.

 

Fourth. Plaintiff further contended that defendant’s practice constitutes unfair competition, because there is ‘appropriation without cost to itself of values created by’ the plaintiff; and it is upon this ground that the decision of this court appears to be based. To appropriate and use for profit, knowledge and ideas produced by other men, without making compensation or even acknowledgment, may be inconsistent with a finer sense of propriety; but, with the exceptions indicated above, the law has heretofore sanctioned the practice. Thus it was held that one may ordinarily make and sell anything in any form, may copy with exactness that which another has produced, or may otherwise use his ideas without his consent and without the payment of compensation, and yet not inflict a legal injury; and that ordinarily one is at perfect liberty to find out, if he can by lawful means, trade secrets of another, however valuable, and then use the knowledge so acquired gainfully, although it cost the original owner much in effort and in money to collect or produce.

 

Such taking and gainful use of a product of another which, for reasons of public policy, the law has refused to endow with the attributes of property, does not become unlawful because the product happens to have been taken from a rival and is used in competition with him. The unfairness in competition which hitherto has been recognized by the law as a basis for relief, lay in the manner or means of conducting the business; and the manner or means held legally unfair, involves either fraud or force or the doing of acts otherwise prohibited by law. In the ‘passing off’ cases (the typical and most common case of unfair competition), the wrong consists in fraudulently representing by word or act that defendant’s goods are those of plaintiff. See Hanover Milling Co. v. Metcalf, 240 U. S. 403, 412, 413. In the other cases, the diversion of trade was effected through physical or moral coercion, or by inducing breaches of contract or of trust or by enticing away employes. In some others, called cases of simulated competition, relief was granted because defendant’s purpose was unlawful; namely, not competition but deliberate and wanton destruction of plaintiff’s business.

 

That competition is not unfair in a legal sence, merely because the profits gained are unearned, even if made at the expense of a rival, is shown by many cases besides those referred to above. He who follows the pioneer into a new market, or who engages in the manufacture of an article newly introduced by another, seeks profits due largely to the labor and expense of the first adventurer; but the law sanctions, indeed encourages, the pursuit. He who makes a city known through his product, must submit to sharing the resultant trade with others who, perhaps for that reason, locate there later. Canal Co. v. Clark, 13 Wall. 311; Elgin National Watch Co. v. Illinois Watch Co., 179 U. S. 665, 673. He who has made his name a guaranty of quality, protests in vain when another with the same name engages, perhaps for that reason, in the same lines of business; provided, precaution is taken to prevent the public from being deceived into the belief that what he is selling, was made by his competitor. One bearing a name made famous by another is permitted to enjoy the unearned benefit which necessarily flows from such use, even though the use proves harmful to him who gave the name value. Brown Chemical Co. v. Meyer, 139 U. S. 540, 544 [other citations omitted].

 

The means by which the International News Service obtains news gathered by the Associated Press is also clearly unobjectionable. It is taken from papers bought in the open market or from bulletins publicly posted. No breach of contract such as the court considered to exist in Hitchman Coal & Coke Co. v. Mitchell, 245 U. S. 229, 254, or of trust such as was present in Morison v. Moat, 9 Hare, 241, and neither fraud nor force is involved. The manner of use is likewise unobjectionable. No reference is made by word or by act to the Associated Press, either in transmitting the news to subscribers or by them in publishing it in their papers. Neither the International News Service nor its subscribers is gaining or seeking to gain in its business a benefit from the reputation of the Associated Press. They are merely using its product without making compensation. See Bamforth v. Douglass Post Card & Machine Co. (C. C.) 158 Fed. 355; Tribune Co. of Chicago v. Associated Press (C. C.) 116 Fed. 126. That they have a legal right to do, because the product is not property, and they do not stand in any relation to the Associated Press, either of contract or of trust, which otherwise precludes such use. The argument is not advanced by characterizing such taking and use a misappropriation.

 

It is also suggested that the fact that defendant does not refer to the Associated Press as the source of the news may furnish a basis for the relief. But the defendant and its subscribers, unlike members of the Associated Press, were under no contractual obligation to disclose the source of the news; and there is no rule of law requiring acknowledgment to be made where uncopyrighted matter is reproduced. The International News Service is said to mislead its subscribers into believing that the news transmitted was originally gathered by it and that they in turn mislead their readers. There is, in fact, no representation by either of any kind. Sources of information are sometimes given because required by contract; sometimes because naming the source gives authority to an otherwise incredible statement; and sometimes the source is named because the agency does not wish to take the responsibility itself of giving currency to the news. But no representation can properly be implied from omission to mention the source of information except that the International News Service is transmitting news which it believes to be credible.

 

Nor is the use made by the International News Service of the information taken from papers or bulletins of Associated Press members legally objectionable by reason of the purpose for which it was employed. The acts here complained of were not done for the purpose of injuring the business of the Associated Press. Their purpose was not even to divert its trade, or to put it at a disadvantage by lessening defendant’s necessary expenses. The purpose was merely to supply subscribers of the International News Service promptly with all available news. The suit is, as this court declares, in substance one brought for the benefit of the members of the Associated Press, who would be proper, and except for their number perhaps necessary, parties; and the plaintiff conducts the suit as representing their interests. It thus appears that the protection given by the injunction is not actually to the business of the complainant news agency; for this agency does not sell news nor seek to earn profits, but is a mere instrumentality by which 800 or more newspapers collect and distribute news. It is these papers severally which are protected; and the protection afforded is not from competition of the defendant, but from possible competition of one or more of the 400 other papers which receive the defendant’s service. Furthermore, the protection to these Associated Press members consists merely in denying to other papers the right to use as news, information which by authority of all concerned, had theretofore been given to the public by some of those who joined in gathering it; and to which the law denies the attributes of property. There is in defendant’s purpose nothing on which to base a claim for relief.

 

It is further said that, while that for which the Associated Press spends its money is too fugitive to be recognized as property in the common-law courts, the defendant cannot be heard to say so in a court of equity, where the question is one of unfair competition. The case presents no elements of equitable title or of breach of trust. The only possible reason for resort to a court of equity in a case like this is that the remedy which the law gives is inadequate. If the plaintiff has no legal cause of action, the suit necessarily fails. Levy v. Walker, L. R. 10 Ch. D. 436, 449. There is nothing in the situation of the parties which can estop the defendant from saying so.

 

Fifth. The great development of agencies now furnishing country-wide distribution of news, the vastness of our territory, and improvements in the means of transmitting intelligence, have made it possible for a news agency or newspapers to obtain, without paying compensation, the fruit of another’s efforts and to use news so obtained gainfully in competition with the original collector. The injustice of such action is obvious. But to give relief against it would involve more than the application of existing rules of law to new facts. It would require the making of a new rule in analogy to existing ones. The unwritten law possesses capacity for growth; and has often satisfied new demands for justice by invoking analogies or by expanding a rule or principle. This process has been in the main wisely applied and should not be discontinued. Where the problem is relatively simple, as it is apt to be when private interests only are involved, it generally proves adequate. But with the increasing complexity of society, the public interest tends to become omnipresent; and the problems presented by new demands for justice cease to be simple. Then the creation or recognition by courts of a new private right may work serious injury to the general public, unless the boundaries of the right are definitely established and wisely guarded. In order to reconcile the new private right with the public interest, it may be necessary to prescribe limitations and rules for its enjoyment; and also to provide administrative machinery for enforcing the rules. It is largely for this reason that, in the effort to meet the many new demands for justice incident to a rapidly changing civilization, resort to legislation has latterly been had with increasing frequency.

 

The rule for which the plaintiff contends would effect an important extension of property rights and a corresponding curtailment of the free use of knowledge and of ideas; and the facts of this case admonish us of the danger involved in recognizing such a property right in news, without imposing upon news-gatherers corresponding obligations. A large majority of the newspapers and perhaps half the newspaper readers of the United States are dependent for their news of general interest upon agencies other than the Associated Press. The channel through which about 400 of these papers received, as the plaintiff alleges, ‘a large amount of news relating to the European war of the greatest importance and of intense interest to the newspaper reading public’ was suddenly closed. The closing to the International News Service of these channels for foreign news (if they were closed) was due not to unwillingness on its part to pay the cost of collecting the news, but to the prohibitions imposed by foreign governments upon its securing news from their respective countries and from using cable or telegraph lines running therefrom. For aught that appears, this prohibition may have been wholly undeserved; and at all events the 400 papers and their readers may be assumed to have been innocent. For aught that appears, the International News Service may have sought then to secure temporarily by arrangement with the Associated Press the latter’s foreign news service. For aught that appears, all of the 400 subscribers of the International News Service would gladly have then become members of the Associated Press, if they could have secured election thereto. It is possible, also, that a large part of the readers of these papers were so situated that they could not secure prompt access to papers served by the Associated Press. The prohibition of the foreign governments might as well have been extended to the channels through which news was supplied to the more than a thousand other daily papers in the United States not served by the Associated Press; and a large part of their readers may also be so located that they cannot procure prompt access to papers served by the Associated Press.

 

A Legislature, urged to enact a law by which one news agency or newspaper may prevent appropriation of the fruits of its labors by another, would consider such facts and possibilities and others which appropriate inquiry might disclose. Legislators might conclude that it was impossible to put an end to the obvious injustice involved in such appropriation of news, without opening the door to other evils, greater than that sought to be remedied. Such appears to have been the opinion of our Senate which reported unfavorably a bill to give news a few hours’ protection; and which ratified, on February 15, 1911, the convention adopted at the Fourth International American Conference; and such was evidently the view also of the signatories to the International Copyright Union of November 13, 1908, as both these conventions expressly exclude news from copyright protection.

 

Or legislators dealing with the subject might conclude, that the right to news values should be protected to the extent of permitting recovery of damages for any unauthorized use, but that protection by injunction should be denied, just as courts of equity ordinarily refuse (perhaps in the interest of free speech) to restrain actionable libels, and for other reasons decline to protect by injunction mere political rights;and as Congress has prohibited courts from enjoining the illegal assessment or collection of federal taxes. If a Legislature concluded to recognize property in published news to the extent of permitting recovery at law, it might, with a view to making the remedy more certain and adequate, provide a fixed measure of damages, as in the case of copyright infringement.

 

Or again, a Legislature might conclude that it was unwise to recognize even so limited a property right in published news as that above indicated; but that a news agency should, on some conditions, be given full protection of its business; and to that end a remedy by injunction as well as one for damages should be granted, where news collected by it is gainfully used without permission. If a Legislature concluded (as at least one court has held, New York and Chicago Grain and Stock Exchange v. Board of Trade, 127 Ill. 153) that under certain circumstances news-gathering is a business affected with a public interest; it might declare that, in such cases, news should be protected against appropriation, only if the gatherer assumed the obligation of supplying it at resonable rates and without discrimination, to all papers which applied therefor. If legislators reached that conclusion, they would probably go further, and prescribe the conditions under which and the extent to which the protection should be afforded; and they might also provide the administrative machinery necessary for insuring to the public, the press, and the news agencies, full enjoyment of the rights so conferred.

 

Courts are ill-equipped to make the investigations which should precede a determination of the limitations which should be set upon any property right in news or of the circumstances under which news gathered by a private agency should be deemed affected with a public interest. Courts would be powerless to prescribe the detailed regulations essential to full enjoyment of the rights conferred or to introduce the machinery required for enforcement of such regulations. Considerations such as these should lead us to decline to establish a new rule of law in the effort to redress a newly disclosed wrong, although the propriety of some remedy appears to be clear.

 

 

Outline of INS v. AP

 

Three opinions: Pitney (majority); Holmes; Brandeis

 

In a nutshell:

 

 

Pitney: AP has relative title to news it gathers such that it can prevent competitors from free riding for some period after it is first reported. This is in order to keep alive the incentive to gather news.

 

 

 

Holmes: The harm here is only that INS is publishing news under its name that in fact was gathered by the AP – potentially misleading consumers about the source of the news. INS should be required to credit AP but no more.

 

 

 

Brandeis: Just because there is value in news does not mean that the initial gatherer should receive the legal right to exclude others from using and reporting that news. There is value in free riding, at least where, as here, the public isn’t confused about the source of news. The law does not protect creators of value in every case, only in certain areas (patent and copyright, for example). And in any event, establishing exclusion rights in news is a change that will affect the interests of the general public and getting the balance right will require careful review of the industry and, perhaps, administrative machinery that courts are not in the best position to provide. This issue should be left to the legislature.

 

 

 

 

Majority opinion

 

 

I. Background and posture of the case

 

  • Background on industry and organization of INS and AP.

 

  • Allegations: Injury exists in taking the advantage of freshness

 

  • Procedural history – we’re here on an an interlocutory appeal from the grant of a preliminary injunction.

 

  • Only issue is whether INS can use news published by AP, on bulletin boards or in papers, as the sole basis for its own stories.

 

  • This is a case raising only state law issues of unfair competition (the parties are diverse, coming from different states, thus giving fed courts jurisdiction) – not copyright. Copyright would only protect the expression, the literary quality or wording of news stories, not the facts that form the basis of the stories. And the facts are all that was taken here.

 

II. Positive Legal Analysis

 

  • News is valuable because it is, well, new. Those in the news business compete to report facts first, or at least timely.

 

  • As business competitors they are under a legal duty, when conflict between them is likely, to “conduct their business as not unnecessarily or unfairly to injure the of the other.” If there is a right to exclude here, it is only relative (RELATIVE TITLE). Each may have a right to exclude the other, but not the general public, who may do with the news what they wish.

 

  • So is there such a right?

 

  • Canvass precedent – two cases suggesting that finding relative title to news would fit with existing law.

 

  • Justification – news is valuable to those who gather it and to those who would appropriate it. HERE, The time difference between the East and West coast allows a competitor to appropriate that value, publishing nearly simultaneously with the gatherer on the West Coast.

 

  • Since news is expensive to gather, an interloper should not be allowed to reap what he has not sown. (Argument that property rights, at least as between competitors, should be awarded to protect labor. Contra Pierson v. Post.) That’s happened here, so liability.

 

III. Rebuttals to arguments against

 

  • News is not “abandoned” when published. There’s no intent to abandon, as is evidenced by the organizational structure of the AP. They couldn’t stay in business if they couldn’t profit from publication of the news. Their bylaws attempt to prevent transmission to competitors before publication.

 

  • The holding does not create a monopoly of gathering and distributing news, but only partially keeps out competitors who would “reproduce” AP’s news and only for long enough to give AP enough value to keep alive its incentive to gather news.

 

  • The harm here is not (only) in the fraud upon consumers, the reverse palming off of AP’s news as coming from INS. And so INS’s giving credit would not fix the problem.

 

  • AP uses other organizations’ news stories as leads or tips that are independently verified before publication. But this does not mean that AP is guilty of that of which they accuse INS (which guilt might establish the equitable defense of unclean hands).

 

 

 

Holmes

 

In general, the law does not prevent one from repeating what others have said. The news is not copyrighted and so is free to be used by others, including competitors. But here, using AP’s news and publishing it at the same time AP papers publish it, in the west, amounts to a kind of fraud. INS should be required to name the source.

 

 

 

Brandeis

 

  • Facts

 

  • Framing the question: Can AP enjoin a competitor from using AP-published news that the competitor lawfully acquires? No cases answer this question.

 

  • The background rule is that “a product of the mind,” even if it requires laborious effort to produce, is free to be ripped off by others. There are exceptions to this only where public policy requires it – as in patent, copyright, and certain cases where the relationship between the parties demands it.

 

  • This is not such a case (rebuts arguments one by one):

 

  1. Distinguish precedent – not like cases where the appropriation involved a breach of contract or trust. No such relationship here.

 

  1. Distinguish adverse precedent based on a kind of common law copyright by using a Feist rationale – to have such a cause of action, the expression must evince at least some originality, not a bare recitation of facts.

 

  1. There was a general publication here, and so once published, the news is not protectable literary property (don’t worry about this).

 

  1. Rebuts the bare argument that it’s unfair competition to free-ride. But unfair competition doesn’t protect against free-riding, only against fraud or force or other legal violations used to gain a competitive edge. No such unfair means were used here (papers were bought in the open market). And the law generally sanctions and even encourages free riders, those who follow pioneers. Also, Brandeis disagrees that a failure to attribute is misleading. Doesn’t think people assume the publisher is the source of all the news it publishes.

 

  1. (LEGAL PROCESS) Technological change and the “vastness of our territory” do threaten to erode the incentive to engage in the worthwhile endeavor of news gathering and dissemination. But we’d need to develop new law to deal with this, and doing so would affect the public at large, perhaps adversely. Courts won’t do a good job, and so should leave this for the legislature (which can take broader evidence about the problem and produce more finely tuned rules enforceable by administrative machinery).

 

Cheney Bros. V. Doris Silk Corporation,

35 F.2d 279 (SDNY, 1929)

 

Harry D. Nims, of New York City (Minturn DeS. Verdi and Wallace H. Martin, both of New York City, on the brief), for appellant.

 

Epstein & Bros., of New York City (Arthur J. Brothers, of New York City, of counsel), for appellee.

 

Before Manton, L. Hand, and Swan, Circuit Judges.

 

 

 

L. Hand, Circuit Judge

 

The plaintiff, a corporation, is a manufacturer of silks, which puts out each season many new patterns, designed to attract purchasers by their novelty and beauty. Most of these fail in that purpose, so that no much more than a fifth catch the public fancy. Moreover, they have only a short life, for the most part no more than a single season of eight or nine months. It is in practice impossible, and it would be very onerous if it were not, to secure design patents upon all of these; it would also be impossible to know in advance which would sell well, and patent only those. Besides, it is probable that for the most part they have no such originality as would support a design patent. Again, it is impossible to copyright them under the Copyright Act (17 USCA s 1 et seq.), or at least so the authorities of the Copyright Office hold. So it is easy for any one to copy such as prove successful, and the plaintiff, which is put to much ingenuity and expense in fabricating them, finds itself without protection of any sort for its pains.

 

Taking advantage of this situation, the defendant copied one of the popular designs in the season beginning in October, 1928, and undercut the plaintiff’s price. This is the injury of which it complains. The defendant, though it duplicated the design in question, denies that it knew it to be the plaintiff’s, and there thus arises an issue which might be an answer to the motion. However, the parties wish a decision upon the equity of the bill, and, since it is within our power to dismiss it, we shall accept its allegation, and charge the defendant with knowledge.

 

The plaintiff asks for protection only during the season, and needs no more, for the designs are all ephemeral. It seeks in this way to disguise the extent of the proposed innovation, and to persuade us that, if we interfere only a little, the solecism, if there be one, may be pardonable. But the reasoning which would justify any interposition at all demands that it cover the whole extent of the injury. A man whose designs come to harvest in two years, or in five, has prima facie as good right to protection as one who deals only in annuals. Nor could we consistently stop at designs; processes, machines, and secrets have an equal claim. The upshot must be that, whenever any one has contrived any of these, others may be forbidden to copy it. That is not the law. In the absence of some recognized right at common law, or under the statutes- and the plaintiff claims neither- a man’s property is limited to the chattels which embody his invention. Others may imitate these at their pleasure. Flagg Mfg. Co. v. Holway, 178 Mass. 83, 59 N.E. 667; Keystone Co. v. Portland Publishing Co., 186 F. 690 (C.C.A. 1); Heide v. Wallace, 135 F. 346 (C.C.A. 3); Upjohn Co. v. Merrell Co., 269 F. 209 (C.C.A. 6); Hudson Co. v. Apco Co. (D.C.) 288 F. 871; Crescent Tool Co. v. Kilborn & Bishop Co., 247 F. 299 (C.C.A. 2); Hamilton Co. v. Tubbs Co. (D.C.) 216 F. 401; Montegut v. Hickson, 178 App.Div. 94, 164 N.Y.S. 858.

 

This is confirmed by the doctrine of ‘non-functional’ features, under which it is held that to imitate these is to impute to the copy the same authorship as the original. Enterprise Co. v. Landers, 131 F. 240 (C.C.A. 2); Yale & Towne Co. v. Adler, 154 F. 37 (C.C.A. 2); Rushmore v. Manhattan Co., 163 F. 939, 19 L.R.A.(N.S.) 269 (C.C.A. 2); Rushmore v. Badger Co., 198 F. 379 (C.C.A. 2); Fox v. Glynn, 191 Mass. 344, 78 N.E. 89, 9 L.R.A.(N.S.) 1096, 114 Am.St.Rep. 619. These decisions imply that, except as to these elements, any one may copy the original at will. Unless, therefore, there has been some controlling authority to the contrary, the bill at bar stands upon no legal right and must fail.

 

Of the cases on which the plaintiff relies, the chief is International News Service v. Associated Press, 248 U.S. 215, 39 S.Ct. 68, 63 L.Ed. 211, 2 A.L.R. 293. Although that concerned another subject-matter- printed news dispatches- we agree that, if it meant to lay down a general doctrine, it would cover this case; at least, the language of the majority opinion goes so far. We do not believe that it did. While it is of course true that law ordinarily speaks in general terms, there are cases where the occasion is at once the justification for, and the limit of, what is decided. This appears to us such an instance; we think that no more was covered than situations substantially similar to those then at bar. The difficulties of understanding it otherwise are insuperable. We are to suppose that the court meant to create a sort of common-law patent or copyright for reasons of justice. Either would flagrantly conflict with the scheme which Congress has for more than a century devised to cover the subject-matter.

 

Qua patent, we should at least have to decide, as tabula rasa, whether the design or machine was new and required invention; further, we must ignore the Patent Office whose action has always been a condition upon the creation of this kind of property. Qua copyright, although it would be simpler to decide upon the merits, we should equally be obliged to dispense with the conditions imposed upon the creation of the right. Nor, if we went so far, should we know whether the property so recognized should be limited to the periods prescribed in the statutes, or should extend as long as the author’s grievance. It appears to us incredible that the Supreme Court should have had in mind any such consequences. To exclude others from the enjoyment of a chattel is one thing; to prevent any imitation of it, to set up a monopoly in the plan of its structure, gives the author a power over his fellows vastly greater, a power which the Constitution allows only Congress to create.

 

The other cases are easily distinguishable. Board of Trade v. Christie, 198 U.S. 236, 25 S.Ct. 637, 49 L.Ed. 1031, went upon the fact that the defendants had procured their information through a breach of contract between the plaintiff and its subscribers, or some surreptitious and dishonest conduct. Hunt v. N.Y. Cotton Exchange, 205 U.S. 322, 27 S.Ct. 529, 51 L.Ed. 821, was another instance of the same kind. There is, indeed, language in National Tel. News Co. v. West Un. Tel. Co., 119 F. 294, 60 L.R.A. 805 (C.C.A. 7), which goes further, but we take it that the authoritative statement of the doctrine must be found in Board of Trade v. Christie 221 F. 305 (C.C.A. 2). Though the limitations there imposed have indeed been extended in International News Service v. Associated Press, they still comprise no more than cases involving news and perhaps market quotations. Prest-O-Lite v. Bogen (C.C.) 209 F. 915, and Prest-O-Lite v. Davis (D.C.) 209 F. 917, were cases of passing off. In Kiernan v. Manhattan Co., 50 How.Prac.(N.Y.) 194, Dodge v. Construction Co., 183 Mass. 63, 66 N.E. 204, 60 L.R.A. 810, 97 Am.St.Rep. 412; Exchange Co. v. Gregory, L.R. (1896) 1 Q.B. 147 (C.A.) and Exchange Co. v. Central News, L.R. (1897) 2 Ch. 48, again, either there was a breach of contract between the plaintiff and its subscriber, or the defendant had dishonestly procured the information. They are like Board of Trade v. Christie.

 

True, it would seem as though the plaintiff had suffered a grievance for which there should be a remedy, perhaps by an amendment of the Copyright Law, assuming that this does not already cover the case, which is not urged here. It seems a lame answer in such a case to turn the injured party out of court, but there are larger issues at stake than his redress. Judges have only a limited power to amend the law; when the subject has been confided to a Legislature, they must stand aside, even though there be an hiatus in completed justice. An omission in such cases must be taken to have been as deliberate as though it were express, certainly after long-standing action on the subject-matter. Indeed, we are not in any position to pass upon the questions involved, as Brandeis, J., observed in International News Service v. Associated Press. We must judge upon records prepared by litigations, which do not contain all that may be relevant to the issues, for they cannot disclose the conditions of this industry, or of the others which may be involved. Congress might see its way to create some sort of temporary right, or it might not. Its decision would certainly be preceded by some examination of the result upon the other interests affected. Whether these would prove paramount we have no means of saying; it is not for us to decide. Our vision is inevitably contracted, and the whole horizon may contain much which will compose a very different picture.

 

The order is affirmed, and, as the bill cannot in any event succeed, it may be dismissed, if the defendant so desires.

 

 

National Basketball Association v. Motorola, Inc.,

105 F.3d 841

 

Before: Van Graafeiland, Winter, and Altimari, Circuit Judges.

 

 

 

Winter, Circuit Judge:

 

… .

 

 

 

I. BACKGROUND

 

 

 

The facts are largely undisputed. Motorola manufactures and markets the SportsTrax paging device while STATS supplies the game information that is transmitted to the pagers. The product became available to the public in January 1996, at a retail price of about $200. SportsTrax’s pager has an inch-and-a-half by inch-and-a-half screen and operates in four basic modes: “current,” “statistics,” “final scores” and “demonstration.” It is the “current” mode that gives rise to the present dispute. In that mode, SportsTrax displays the following information on NBA games in progress: (i) the teams playing; (ii) score changes; (iii) the team in possession of the ball; (iv) whether the team is in the free-throw bonus; (v) the quarter of the game; and (vi) time remaining in the quarter. The information is updated every two to three minutes, with more frequent updates near the end of the first half and the end of the game. There is a lag of approximately two or three minutes between events in the game itself and when the information appears on the pager screen.

 

SportsTrax’s operation relies on a “data feed” supplied by STATS reporters who watch the games on television or listen to them on the radio. The reporters key into a personal computer changes in the score and other information such as successful and missed shots, fouls, and clock updates. The information is relayed by modem to STATS’s host computer, which compiles, analyzes, and formats the data for retransmission. The information is then sent to a common carrier, which then sends it via satellite to various local FM radio networks that in turn emit the signal received by the individual SportsTrax pagers.

 

Although the NBA’s complaint concerned only the SportsTrax device, the NBA offered evidence at trial concerning STATS’s America On-Line (“AOL”) site. Starting in January, 1996, users who accessed STATS’s AOL site, typically via a modem attached to a home computer, were provided with slightly more comprehensive and detailed real-time game information than is displayed on a SportsTrax pager. On the AOL site, game scores are updated every 15 seconds to a minute, and the player and team statistics are updated each minute. The district court’s original decision and judgment, National Basketball Ass’n v. Sports Team Analysis and Tracking Sys. Inc., 931 F. Supp. 1124 (S.D.N.Y. 1996), did not address the AOL site, because “NBA’s complaint and the evidence proffered at trial were devoted largely to SportsTrax.” National Basketball Ass’n v. Sports Team Analysis and Tracking Sys. Inc., 939 F.Supp. 1071, 1074 n. 1 (S.D.N.Y. 1996). Upon motion by the NBA, however, the district court amended its decision and judgment and enjoined use of the real-time game information on STATS’s AOL site. Id. at 1075 n. 1. Because the record on appeal, the briefs of the parties, and oral argument primarily addressed the SportsTrax device, we similarly focus on that product. However, we regard the legal issues as identical with respect to both products, and our holding applies equally to SportsTrax and STATS’s AOL site.

 

The NBA’s complaint asserted six claims for relief: (i) state law unfair competition by misappropriation; (ii) false advertising under Section 43(a) of the Lanham Act, 15 U.S.C. § 1125(a); (iii) false representation of origin under Section 43(a) of the Lanham Act; (iv) state and common law unfair competition by false advertising and false designation of origin; (v) federal copyright infringement; and (vi) unlawful interception of communications under the Communications Act of 1934, 47 U.S.C. § 605. Motorola counterclaimed, alleging that the NBA unlawfully interfered with Motorola’s contractual relations with four individual NBA teams that had agreed to sponsor and advertise SportsTrax.

 

The district court dismissed all of the NBA’s claims except the first-misappropriation under New York law. The court also dismissed Motorola’s counterclaim. Finding Motorola and STATS liable for misappropriation, Judge Preska entered the permanent injunction, reserved the calculation of damages for subsequent proceedings, and stayed execution of the injunction pending appeal. Motorola and STATS appeal from the injunction … .

 

… .

 

 

 

 

C. The State-Law Misappropriation Claim

 

 

 

 

… .

 

In our view, the elements central to an INS claim are: (i) the plaintiff generates or collects information at some cost or expense; (ii) the value of the information is highly time-sensitive; (iii) the defendant’s use of the information constitutes free-riding on the plaintiff’s costly efforts to generate or collect it; (iv) the defendant’s use of the information is in direct competition with a product or service offered by the plaintiff; (v) the ability of other parties to free-ride on the efforts of the plaintiff would so reduce the incentive to produce the product or service that its existence or quality would be substantially threatened.1

 

INS is not about ethics; it is about the protection of property rights in time-sensitive information so that the information will be made available to the public by profit seeking entrepreneurs. If services like AP were not assured of property rights in the news they pay to collect, they would cease to collect it. The ability of their competitors to appropriate their product at only nominal cost and thereby to disseminate a competing product at a lower price would destroy the incentive to collect news in the first place. The newspaper-reading public would suffer because no one would have an incentive to collect “hot news.”

 

We therefore find the extra elements-those in addition to the elements of copyright infringement-that allow a “hotnews” claim to survive preemption are: (i) the time-sensitive value of factual information, (ii) the free-riding by a defendant, and (iii) the threat to the very existence of the product or service provided by the plaintiff.

 

 

 

 

 

2 . The Legality of SportsTrax

 

 

 

 

 

We conclude that Motorola and STATS have not engaged in unlawful misappropriation under the “hot-news” test set out above. To be sure, some of the elements of a “hot-news” INS claim are met. The information transmitted to SportsTrax is not precisely contemporaneous, but it is nevertheless time-sensitive. Also, the NBA does provide, or will shortly do so, information like that available through SportsTrax. It now offers a service called “Gamestats” that provides official play-by-play game sheets and half-time and final box scores within each arena. It also provides such information to the media in each arena. In the future, the NBA plans to enhance Gamestats so that it will be networked between the various arenas and will support a pager product analogous to SportsTrax. SportsTrax will of course directly compete with an enhanced Gamestats.

 

However, there are critical elements missing in the NBA’s attempt to assert a “hot-news” INS-type claim. As framed by the NBA, their claim compresses and confuses three different informational products. The first product is generating the information by playing the games; the second product is transmitting live, full descriptions of those games; and the third product is collecting and retransmitting strictly factual information about the games. The first and second products are the NBA’s primary business: producing basketball games for live attendance and licensing copyrighted broadcasts of those games. The collection and retransmission of strictly factual material about the games is a different product: e.g., box-scores in newspapers, summaries of statistics on television sports news, and real-time facts to be transmitted to pagers. In our view, the NBA has failed to show any competitive effect whatsoever from SportsTrax on the first and second products and a lack of any free-riding by SportsTrax on the third.

 

With regard to the NBA’s primary products-producing basketball games with live attendance and licensing copyrighted broadcasts of those games-there is no evidence that anyone regards SportsTrax or the AOL site as a substitute for attending NBA games or watching them on television. In fact, Motorola markets SportsTrax as being designed “for those times when you cannot be at the arena, watch the game on TV, or listen to the radio …”

 

The NBA argues that the pager market is also relevant to a “hot-news” INS-type claim and that SportsTrax’s future competition with Gamestats satisfies any missing element. We agree that there is a separate market for the real-time transmission of factual information to pagers or similar devices, such as STATS’s AOL site. However, we disagree that SportsTrax is in any sense free-riding off Gamestats.

 

An indispensable element of an INS “hot-news” claim is free riding by a defendant on a plaintiff’s product, enabling the defendant to produce a directly competitive product for less money because it has lower costs. SportsTrax is not such a product. The use of pagers to transmit real-time information about NBA games requires: (i) the collecting of facts about the games; (ii) the transmission of these facts on a network; (iii) the assembling of them by the particular service; and (iv) the transmission of them to pagers or an on-line computer site. Appellants are in no way free-riding on Gamestats. Motorola and STATS expend their own resources to collect purely factual information generated in NBA games to transmit to SportsTrax pagers. They have their own network and assemble and transmit data themselves.

 

To be sure, if appellants in the future were to collect facts from an enhanced Gamestats pager to retransmit them to SportsTrax pagers, that would constitute free-riding and might well cause Gamestats to be unprofitable because it had to bear costs to collect facts that SportsTrax did not. If the appropriation of facts from one pager to another pager service were allowed, transmission of current information on NBA games to pagers or similar devices would be substantially deterred because any potential transmitter would know that the first entrant would quickly encounter a lower cost competitor free-riding on the originator’s transmissions.2

 

However, that is not the case in the instant matter. SportsTrax and Gamestats are each bearing their own costs of collecting factual information on NBA games, and, if one produces a product that is cheaper or otherwise superior to the other, that producer will prevail in the marketplace. This is obviously not the situation against which INS was intended to prevent: the potential lack of any such product or service because of the anticipation of free-riding.

 

For the foregoing reasons, the NBA has not shown any damage to any of its products based on free-riding by Motorola and STATS, and the NBA’s misappropriation claim based on New York law is preempted.

 

… .

 


  1. Some authorities have labeled this element as requiring direct competition between the defendant and the plaintiff in a primary market. ”In most of the small number of cases in which the misappropriation doctrine has been determinative, the defendant’s appropriation, like that in INS, resulted in direct competition in the plaintiffs’ primary market … Appeals to the misappropriation doctrine are almost always rejected when the appropriation does not intrude upon the plaintiff’s primary market.”, Restatement (Third) of Unfair Competition, § 38 cmt. c, at 412-13; see also National Football League v. Governor of State of Delaware, 435 F. Supp. 1372 (D. Del. 1977). In that case, the NFL sued Delaware over the state’s lottery game which was based on NFL games. In dismissing the wrongful misappropriation claims, the court stated: “While courts have recognized that one has a right to one’s own harvest, this proposition has not been construed to preclude others from profiting from demands for collateral services generated by the success of one’s business venture.” Id. at 1378. The court also noted, “It is true that Delaware is thus making profits it would not make but for the existence of the NFL, but I find this difficult to distinguish from the multitude of charter bus companies who generate profit from servicing those of plaintiffs’ fans who want to go to the stadium or, indeed, the sidewalk popcorn salesman who services the crowd as it surges towards the gate.” Id.

 

  1. It may well be that the NBA’s product, when enhanced, will actually have a competitive edge because its Gamestats system will apparently be used for a number of in-stadium services as well as the pager market, resulting in a certain amount of cost sharing. Gamestats might also have a temporal advantage in collecting and transmitting official statistics. Whether this is so does not affect our disposition of this matter, although it does demonstrate the gulf between this case and INS, where the free-riding created the danger of no wire service being viable.

 

 

 

3.2. Trademarks and Domain Names

 

Qualitex Co. v. Jacobson Products Co.,

514 U.S. 159 (1995)

 

 

 

Justice Breyer delivered the opinion of the Court.

 

The question in this case is whether the Lanham Trademark Act of 1946 (Lanham Act), 15 U.S.C. §§ 1051-1127 (1988 ed. and Supp. V), permits the registration of a trademark that consists, purely and simply, of a color. We conclude that, sometimes, a color will meet ordinary legal trademark requirements. And, when it does so, no special legal rule prevents color alone from serving as a trademark.

 

The case before us grows out of petitioner Qualitex Company’s use (since the 1950’s) of a special shade of green-gold color on the pads that it makes and sells to dry cleaning firms for use on dry cleaning presses. In 1989 respondent Jacobson Products (a Qualitex rival) began to sell its own press pads to dry cleaning firms; and it colored those pads a similar green-gold. In 1991 Qualitex registered the special green-gold color on press pads with the Patent and Trademark Office as a trademark. Registration No. 1,633,711 (Feb. 5, 1991). Qualitex subsequently added a trademark infringement count, 15 U.S.C. § 1114(1), to an unfair competition claim, § 1125(a), in a lawsuit it had already filed challenging Jacobson’s use of the green-gold color.

 

Qualitex won the lawsuit in the District Court. 21 U.S.P.Q.2d 1457, 1991 WL 318798 (CD Cal.1991). But, the Court of Appeals for the Ninth Circuit set aside the judgment in Qualitex’s favor on the trademark infringement claim because, in that Circuit’s view, the Lanham Act does not permit Qualitex, or anyone else, to register “color alone” as a trademark. 13 F.3d 1297, 1300, 1302 (1994).

 

The courts of appeals have differed as to whether or not the law recognizes the use of color alone as a trademark. Compare NutraSweet Co. v. Stadt Corp., 917 F.2d 1024, 1028 (CA7 1990) (absolute prohibition against protection of color alone), with In re Owens-Corning Fiberglas Corp., 774 F.2d 1116, 1128 (CA Fed.1985) (allowing registration of color pink for fiberglass insulation), and Master Distributors, Inc. v. Pako Corp., 986 F.2d 219, 224 (CA8 1993) (declining to establish per se prohibition against protecting color alone as a trademark). Therefore, this Court granted certiorari. 512 U.S. —-, 115 S.Ct. 40, 129 L.Ed.2d 935 (1994). We now hold that there is no rule absolutely barring the use of color alone, and we reverse the judgment of the Ninth Circuit.

 

 

 

 

II

 

 

 

 

The Lanham Act gives a seller or producer the exclusive right to “register” a trademark, 15 U.S.C. § 1052 (1988 ed. and Supp. V), and to prevent his or her competitors from using that trademark, § 1114(1). Both the language of the Act and the basic underlying principles of trademark law would seem to include color within the universe of things that can qualify as a trademark. The language of the Lanham Act describes that universe in the broadest of terms. It says that trademarks “include any word, name, symbol, or device, or any combination thereof.” § 1127. Since human beings might use as a “symbol” or “device” almost anything at all that is capable of carrying meaning, this language, read literally, is not restrictive. The courts and the Patent and Trademark Office have authorized for use as a mark a particular shape (of a Coca-Cola bottle), a particular sound (of NBC’s three chimes), and even a particular scent (of plumeria blossoms on sewing thread). See, e.g., Registration No. 696,147 (Apr. 12, 1960); Registration Nos. 523,616 (Apr. 4, 1950) and 916,522 (July 13, 1971); In re Clarke, 17 U.S.P.Q.2d 1238, 1240 (TTAB 1990). If a shape, a sound, and a fragrance can act as symbols why, one might ask, can a color not do the same?

 

A color is also capable of satisfying the more important part of the statutory definition of a trademark, which requires that a person “use” or “intend to use” the mark

 

 

“to identify and distinguish his or her goods, including a unique product, from those manufactured or sold by others and to indicate the source of the goods, even if that source is unknown.” 15 U.S.C. § 1127.

 

 

True, a product’s color is unlike “fanciful,” “arbitrary,” or “suggestive” words or designs, which almost automatically tell a customer that they refer to a brand. Abercrombie & Fitch Co. v. Hunting World, Inc., 537 F.2d 4, 9-10 (CA2 1976) (Friendly, J.); see Two Pesos, Inc. v. Taco Cabana, Inc., 505 U.S. —-, —-, 112 S.Ct. 2753, 2757, 120 L.Ed.2d 615 (1992). The imaginary word “Suntost,” or the words “Suntost Marmalade,” on a jar of orange jam immediately would signal a brand or a product “source”; the jam’s orange color does not do so. But, over time, customers may come to treat a particular color on a product or its packaging (say, a color that in context seems unusual, such as pink on a firm’s insulating material or red on the head of a large industrial bolt) as signifying a brand. And, if so, that color would have come to identify and distinguish the goods – i.e. “to “indicate” their “source” – much in the way that descriptive words on a product (say, “Trim” on nail clippers or “Car-Freshner” on deodorizer) can come to indicate a product’s origin. See, e.g., J. Wiss & Sons Co. v. W.E. Bassett Co., 59 C.C.P.A. 1269, 1271 (Pat.), 462 F.2d 567, 569 (1972); Car-Freshner Corp. v. Turtle Wax, Inc., 268 F.Supp. 162, 164 (SDNY 1967). In this circumstance, trademark law says that the word (e.g., “Trim”), although not inherently distinctive, has developed “secondary meaning.” See Inwood Laboratories, Inc. v. Ives Laboratories, Inc., 456 U.S. 844, 851, n. 11, 102 S.Ct. 2182, 2187, n. 11, 72 L.Ed.2d 606 (1982) (“secondary meaning” is acquired when “in the minds of the public, the primary significance of a product feature … is to identify the source of the product rather than the product itself”). Again, one might ask, if trademark law permits a descriptive word with secondary meaning to act as a mark, why would it not permit a color, under similar circumstances, to do the same?

 

We cannot find in the basic objectives of trademark law any obvious theoretical objection to the use of color alone as a trademark, where that color has attained “secondary meaning” and therefore identifies and distinguishes a particular brand (and thus indicates its “source”). In principle, trademark law, by preventing others from copying a source-identifying mark, “reduces the customer’s costs of shopping and making purchasing decisions,” 1 J. McCarthy, McCarthy on Trademarks and Unfair Competition § 2.012, p. 2-3 (3d ed. 1994) (hereinafter McCarthy), for it quickly and easily assures a potential customer that this item – the item with this mark – is made by the same producer as other similarly marked items that he or she liked (or disliked) in the past. At the same time, the law helps assure a producer that it (and not an imitating competitor) will reap the financial, reputation-related rewards associated with a desirable product. The law thereby “encourages the production of quality products,” ibid., and simultaneously discourages those who hope to sell inferior products by capitalizing on a consumer’s inability quickly to evaluate the quality of an item offered for sale. See, e.g., 3 L. Altman, Callmann on Unfair Competition, Trademarks and Monopolies § 17.03 (4th ed. 1983); Landes & Posner, The Economics of Trademark Law, 78 T.M. Rep. 267, 271-272 (1988); Park ‘N Fly, Inc. v. Dollar Park and Fly, Inc., 469 U.S. 189, 198, 105 S.Ct. 658, 663, 83 L.Ed.2d 582 (1985); S.Rep. No. 100-515, p. 4 (1988) U.S.Code Cong. & Admin.News, 1988, pp. 5577, 5580. It is the source-distinguishing ability of a mark – not its ontological status as color, shape, fragrance, word, or sign – that permits it to serve these basic purposes. See Landes & Posner, Trademark Law: An Economic Perspective, 30 J.Law & Econ. 265, 290 (1987). And, for that reason, it is difficult to find, in basic trademark objectives, a reason to disqualify absolutely the use of a color as a mark.

 

Neither can we find a principled objection to the use of color as a mark in the important “functionality” doctrine of trademark law. The functionality doctrine prevents trademark law, which seeks to promote competition by protecting a firm’s reputation, from instead inhibiting legitimate competition by allowing a producer to control a useful product feature. It is the province of patent law, not trademark law, to encourage invention by granting inventors a monopoly over new product designs or functions for a limited time, 35 U.S.C. §§ 154, 173, after which competitors are free to use the innovation. If a product’s functional features could be used as trademarks, however, a monopoly over such features could be obtained without regard to whether they qualify as patents and could be extended forever (because trademarks may be renewed in perpetuity). See Kellogg Co. v. National Biscuit Co., 305 U.S. 111, 119-120, 59 S.Ct. 109, 113-114, 83 L.Ed. 73 (1938) (Brandeis, J.); Inwood Laboratories, Inc., supra, 456 U.S., at 863, 102 S.Ct., at 2193 (White, J., concurring in result) (“A functional characteristic is ‘an important ingredient in the commercial success of the product,’ and, after expiration of a patent, it is no more the property of the originator than the product itself”) (citation omitted). Functionality doctrine therefore would require, to take an imaginary example, that even if customers have come to identify the special illumination-enhancing shape of a new patented light bulb with a particular manufacturer, the manufacturer may not use that shape as a trademark, for doing so, after the patent had expired, would impede competition – not by protecting the reputation of the original bulb maker, but by frustrating competitors’ legitimate efforts to produce an equivalent illumination-enhancing bulb. See, e.g., Kellogg Co., supra, 305 U.S., at 119-120, 59 S.Ct., at 113-114 (trademark law cannot be used to extend monopoly over “pillow” shape of shredded wheat biscuit after the patent for that shape had expired). This Court consequently has explained that, ”in general terms, a product feature is functional,” and cannot serve as a trademark, “if it is essential to the use or purpose of the article or if it affects the cost or quality of the article,” that is, if exclusive use of the feature would put competitors at a significant non-reputation-related disadvantage. Inwood Laboratories, Inc., 456 U.S., at 850, n. 10, 102 S.Ct., at 2186, n. 10. Although sometimes color plays an important role (unrelated to source identification) in making a product more desirable, sometimes it does not. And, this latter fact – the fact that sometimes color is not essential to a product’s use or purpose and does not affect cost or quality – indicates that the doctrine of “functionality” does not create an absolute bar to the use of color alone as a mark. See Owens-Corning, 774 F.2d, at 1123 (pink color of insulation in wall “performs no nontrademark function”).

 

It would seem, then, that color alone, at least sometimes, can meet the basic legal requirements for use as a trademark. It can act as a symbol that distinguishes a firm’s goods and identifies their source, without serving any other significant function. See U.S. Dept. of Commerce, Patent and Trademark Office, Trademark Manual of Examining Procedure § 1202.04(e), p. 1202-13 (2d ed. May, 1993) (hereinafter PTO Manual) (approving trademark registration of color alone where it “has become distinctive of the applicant’s goods in commerce,” provided that “there is no competitive need for colors to remain available in the industry” and the color is not “functional”); see also 1 McCarthy §§ 3.011, 7.26 (“requirements for qualification of a word or symbol as a trademark” are that it be (1) a “symbol,” (2) “used … as a mark,” (3) “to identify and distinguish the seller’s goods from goods made or sold by others,” but that it not be “functional”). Indeed, the District Court, in this case, entered findings (accepted by the Ninth Circuit) that show Qualitex’s green-gold press pad color has met these requirements. The green-gold color acts as a symbol. Having developed secondary meaning (for customers identified the green-gold color as Qualitex’s), it identifies the press pads’ source. And, the green-gold color serves no other function. (Although it is important to use some color on press pads to avoid noticeable stains, the court found “no competitive need in the press pad industry for the green-gold color, since other colors are equally usable.” 21 U.S.P.Q.2d, at 1460, 1991 WL 318798.) Accordingly, unless there is some special reason that convincingly militates against the use of color alone as a trademark, trademark law would protect Qualitex’s use of the green-gold color on its press pads.

 

 

 

 

III

 

 

 

 

Respondent Jacobson Products says that there are four special reasons why the law should forbid the use of color alone as a trademark. We shall explain, in turn, why we, ultimately, find them unpersuasive.

 

First, Jacobson says that, if the law permits the use of color as a trademark, it will produce uncertainty and unresolvable court disputes about what shades of a color a competitor may lawfully use. Because lighting (morning sun, twilight mist) will affect perceptions of protected color, competitors and courts will suffer from “shade confusion” as they try to decide whether use of a similar color on a similar product does, or does not, confuse customers and thereby infringe a trademark. Jacobson adds that the “shade confusion” problem is “more difficult” and “far different from” the “determination of the similarity of words or symbols.” Brief for Respondent 22.

 

We do not believe, however, that color, in this respect, is special. Courts traditionally decide quite difficult questions about whether two words or phrases or symbols are sufficiently similar, in context, to confuse buyers. They have had to compare, for example, such words as “Bonamine” and “Dramamine” (motion-sickness remedies); “Huggies” and “Dougies” (diapers); “Cheracol” and “Syrocol” (cough syrup); “Cyclone” and “Tornado” (wire fences); and “Mattres” and “1-800-Mattres” (mattress franchisor telephone numbers). See, e.g., G.D. Searle & Co. v. Chas. Pfizer & Co., 265 F.2d 385, 389 (CA7 1959); Kimberly-Clark Corp. v. H. Douglas Enterprises, Ltd., 774 F.2d 1144, 1146-1147 (CA Fed.1985); Upjohn Co. v. Schwartz, 246 F.2d 254, 262 (CA2 1957); Hancock v. American Steel & Wire Co., 40 C.C.P.A. of New Jersey, 931, 935 (Pat.), 203 F.2d 737, 740-741 (1953); Dial-A-Mattress Franchise Corp. v. Page, 880 F.2d 675, 678 (CA2 1989). Legal standards exist to guide courts in making such comparisons. See, e.g., 2 McCarthy § 15.08; 1 McCarthy §§ 11.24-11.25 (”Strong” marks, with greater secondary meaning, receive broader protection than “weak” marks). We do not see why courts could not apply those standards to a color, replicating, if necessary, lighting conditions under which a colored product is normally sold. See Ebert, Trademark Protection in Color: Do It By the Numbers!, 84 T.M.Rep. 379, 405 (1994). Indeed, courts already have done so in cases where a trademark consists of a color plus a design, i.e., a colored symbol such as a gold stripe (around a sewer pipe), a yellow strand of wire rope, or a “brilliant yellow” band (on ampules). See, e.g., Youngstown Sheet & Tube Co. v. Tallman Conduit Co., 149 U.S.P.Q. 656, 657 (TTAB 1966); Amsted Industries, Inc. v. West Coast Wire Rope & Rigging Inc., 2 U.S.P.Q.2d 1755, 1760 (TTAB 1987); In re Hodes-Lange Corp., 167 U.S.P.Q. 255, 256 (TTAB 1970).

 

Second, Jacobson argues, as have others, that colors are in limited supply. See, e.g., NutraSweet Co., 917 F.2d, at 1028; Campbell Soup Co. v. Armour & Co., 175 F.2d 795, 798 (CA3 1949). Jacobson claims that, if one of many competitors can appropriate a particular color for use as a trademark, and each competitor then tries to do the same, the supply of colors will soon be depleted. Put in its strongest form, this argument would concede that ”hundreds of color pigments are manufactured and thousands of colors can be obtained by mixing.” L. Cheskin, Colors: What They Can Do For You 47 (1947). But, it would add that, in the context of a particular product, only some colors are usable. By the time one discards colors that, say, for reasons of customer appeal, are not usable, and adds the shades that competitors cannot use lest they risk infringing a similar, registered shade, then one is left with only a handful of possible colors. And, under these circumstances, to permit one, or a few, producers to use colors as trademarks will “deplete” the supply of usable colors to the point where a competitor’s inability to find a suitable color will put that competitor at a significant disadvantage.

 

This argument is unpersuasive, however, largely because it relies on an occasional problem to justify a blanket prohibition. When a color serves as a mark, normally alternative colors will likely be available for similar use by others. See, e.g., Owens-Corning, 774 F.2d, at 1121 (pink insulation). Moreover, if that is not so – if a “color depletion” or “color scarcity” problem does arise – the trademark doctrine of “functionality” normally would seem available to prevent the anticompetitive consequences that Jacobson’s argument posits, thereby minimizing that argument’s practical force.

 

The functionality doctrine, as we have said, forbids the use of a product’s feature as a trademark where doing so will put a competitor at a significant disadvantage because the feature is “essential to the use or purpose of the article” or “affects its cost or quality.” Inwood Laboratories, Inc., 456 U.S., at 850, n. 10, 102 S.Ct., at 2186, n. 10. The functionality doctrine thus protects competitors against a disadvantage (unrelated to recognition or reputation) that trademark protection might otherwise impose, namely their inability reasonably to replicate important non-reputation-related product features. For example, this Court has written that competitors might be free to copy the color of a medical pill where that color serves to identify the kind of medication (e.g., a type of blood medicine) in addition to its source. See id., at 853, 858, n. 20, 102 S.Ct., at 2188, 2190, n. 20 (”Some patients commingle medications in a container and rely on color to differentiate one from another”); see also J. Ginsburg, D. Goldberg, & A. Greenbaum, Trademark and Unfair Competition Law 194-195 (1991) (noting that drug color cases “have more to do with public health policy” regarding generic drug substitution “than with trademark law”). And, the federal courts have demonstrated that they can apply this doctrine in a careful and reasoned manner, with sensitivity to the effect on competition. Although we need not comment on the merits of specific cases, we note that lower courts have permitted competitors to copy the green color of farm machinery (because customers wanted their farm equipment to match) and have barred the use of black as a trademark on outboard boat motors (because black has the special functional attributes of decreasing the apparent size of the motor and ensuring compatibility with many different boat colors). See Deere & Co. v. Farmhand, Inc., 560 F.Supp. 85, 98 (SD Iowa 1982), aff’d, 721 F.2d 253 (CA8 1983); Brunswick Corp. v. British Seagull Ltd., 35 F.3d 1527, 1532 (CA Fed.1994), cert. pending, No. 94-1075; see also Nor-Am Chemical v. O.M. Scott & Sons Co., 4 U.S.P.Q.2d 1316, 1320, 1987 WL 13742 (ED Pa.1987) (blue color of fertilizer held functional because it indicated the presence of nitrogen). The Restatement (Third) of Unfair Competition adds that, if a design’s “aesthetic value” lies in its ability to “confer a significant benefit that cannot practically be duplicated by the use of alternative designs,” then the design is “functional.” Restatement (Third) of Unfair Competition § 17, Comment c, pp. 175-176 (1995). The “ultimate test of aesthetic functionality,” it explains, “is whether the recognition of trademark rights would significantly hinder competition.” Id., at 176.

 

The upshot is that, where a color serves a significant nontrademark function – whether to distinguish a heart pill from a digestive medicine or to satisfy the “noble instinct for giving the right touch of beauty to common and necessary things,” G.K. Chesterton, Simplicity and Tolstoy 61 (1912) – courts will examine whether its use as a mark would permit one competitor (or a group) to interfere with legitimate (nontrademark-related) competition through actual or potential exclusive use of an important product ingredient. That examination should not discourage firms from creating aesthetically pleasing mark designs, for it is open to their competitors to do the same. See, e.g., W.T. Rogers Co. v. Keene, 778 F.2d 334, 343 (CA7 1985) (Posner, J.). But, ordinarily, it should prevent the anticompetitive consequences of Jacobson’s hypothetical “color depletion” argument, when, and if, the circumstances of a particular case threaten “color depletion.”

 

Third, Jacobson points to many older cases – including Supreme Court cases – in support of its position. In 1878, this Court described the common-law definition of trademark rather broadly to “consist of a name, symbol, figure, letter, form, or device, if adopted and used by a manufacturer or merchant in order to designate the goods he manufactures or sells to distinguish the same from those manufactured or sold by another.” McLean v. Fleming, 96 U.S. 245, 254, 24 L.Ed. 828. Yet, in interpreting the Trademark Acts of 1881 and 1905, 21 Stat. 502, 33 Stat. 724, which retained that common-law definition, the Court questioned ”whether mere color can constitute a valid trade-mark,” A. Leschen & Sons Rope Co. v. Broderick & Bascom Rope Co., 201 U.S. 166, 171, 26 S.Ct. 425, 426, 50 L.Ed. 710 (1906), and suggested that the “product including the coloring matter is free to all who make it.” Coca-Cola Co. v. Koke Co. of America, 254 U.S. 143, 147, 41 S.Ct. 113, 114, 65 L.Ed. 189 (1920). Even though these statements amounted to dicta, lower courts interpreted them as forbidding protection for color alone. See, e.g., Campbell Soup Co., 175 F.2d, at 798, and n. 9; Life Savers Corp. v. Curtiss Candy Co., 182 F.2d 4, 9 (CA7 1950) (quoting Campbell Soup ).

 

These Supreme Court cases, however, interpreted trademark law as it existed b efore 1946, when Congress enacted the Lanham Act. The Lanham Act significantly changed and liberalized the common law to “dispense with mere technical prohibitions,” S.Rep. No. 1333, 79th Cong., 2d Sess., 3 (1946), most notably, by permitting trademark registration of descriptive words (say, “U-Build-It” model airplanes) where they had acquired “secondary meaning.” See Abercrombie & Fitch Co., 537 F.2d, at 9 (Friendly, J.). The Lanham Act extended protection to descriptive marks by making clear that (with certain explicit exceptions not relevant here),

 

 

“nothing … shall prevent the registration of a mark used by the applicant which has become distinctive of the applicant’s goods in commerce.” 15 U.S.C. § 1052(f) (1988 ed., Supp. V).

 

 

This language permits an ordinary word, normally used for a nontrademark purpose (e.g., description), to act as a trademark where it has gained “secondary meaning.” Its logic would appear to apply to color as well. Indeed, in 1985, the Federal Circuit considered the significance of the Lanham Act’s changes as they related to color and held that trademark protection for color was consistent with the

 

 

“jurisprudence under the Lanham Act developed in accordance with the statutory principle that if a mark is capable of being or becoming distinctive of the applicant’s goods in commerce, then it is capable of serving as a trademark.” Owens-Corning, 774 F.2d, at 1120.

 

 

In 1988 Congress amended the Lanham Act, revising portions of the definitional language, but left unchanged the language here relevant. § 134, 102 Stat. 3946, 15 U.S.C. § 1127. It enacted these amendments against the following background: (1) the Federal Circuit had decided Owens-Corning; (2) the Patent and Trademark Office had adopted a clear policy (which it still maintains) permitting registration of color as a trademark, see PTO Manual § 1202.04(e) (at p. 1200-12 of the January 1986 edition and p. 1202-13 of the May 1993 edition); and (3) the Trademark Commission had written a report, which recommended that “the terms ‘symbol, or device’ … not be deleted or narrowed to preclude registration of such things as a color, shape, smell, sound, or configuration which functions as a mark,” The United States Trademark Association Trademark Review Commission Report and Recommendations to USTA President and Board of Directors, 77 T.M.Rep. 375, 421 (1987) (hereinafter Trademark Commission); see also 133 Cong.Rec. 32812 (1987) (statement of Sen. DeConcini) (“The bill I am introducing today is based on the Commission’s report and recommendations”). This background strongly suggests that the language “any word, name, symbol, or device,” 15 U.S.C. § 1127, had come to include color. And, when it amended the statute, Congress retained these terms. Indeed, the Senate Report accompanying the Lanham Act revision explicitly referred to this background understanding, in saying that the “revised definition intentionally retains … the words ‘symbol or device’ so as not to preclude the registration of colors, shapes, sounds or configurations where they function as trademarks.” S.Rep. No. 100-515, at 44 U.S.Code Cong. & Admin.News, 1988, p. 5607. (In addition, the statute retained language providing that ”no trademark by which the goods of the applicant may be distinguished from the goods of others shall be refused registration … on account of its nature” (except for certain specified reasons not relevant here). 15 U.S.C. § 1052 (1988 ed., Supp. V)).

 

This history undercuts the authority of the precedent on which Jacobson relies. Much of the pre-1985 case law rested on statements in Supreme Court opinions that interpreted pre-Lanham Act trademark law and were not directly related to the holdings in those cases. Moreover, we believe the Federal Circuit was right in 1985 when it found that the 1946 Lanham Act embodied crucial legal changes that liberalized the law to permit the use of color alone as a trademark (under appropriate circumstances). At a minimum, the Lanham Act’s changes left the courts free to reevaluate the preexisting legal precedent which had absolutely forbidden the use of color alone as a trademark. Finally, when Congress re-enacted the terms “word, name, symbol, or device” in 1988, it did so against a legal background in which those terms had come to include color, and its statutory revision embraced that understanding.

 

Fourth, Jacobson argues that there is no need to permit color alone to function as a trademark because a firm already may use color as part of a trademark, say, as a colored circle or colored letter or colored word, and may rely upon “trade dress” protection, under § 43(a) of the Lanham Act, if a competitor copies its color and thereby causes consumer confusion regarding the overall appearance of the competing products or their packaging, see 15 U.S.C. § 1125(a) (1988 ed., Supp. V). The first part of this argument begs the question. One can understand why a firm might find it difficult to place a usable symbol or word on a product (say, a large industrial bolt that customers normally see from a distance); and, in such instances, a firm might want to use color, pure and simple, instead of color as part of a design. Neither is the second portion of the argument convincing. Trademark law helps the holder of a mark in many ways that “trade dress” protection does not. See 15 U.S.C. § 1124 (ability to prevent importation of confusingly similar goods); § 1072 (constructive notice of ownership); § 1065 (incontestible status); § 1057(b) (prima facie evidence of validity and ownership). Thus, one can easily find reasons why the law might provide trademark protection in addition to trade dress protection.

 

 

 

 

IV

 

 

 

 

Having determined that a color may sometimes meet the basic legal requirements for use as a trademark and that respondent Jacobson’s arguments do not justify a special legal rule preventing color alone from serving as a trademark (and, in light of the District Court’s here undisputed findings that Qualitex’s use of the green-gold color on its press pads meets the basic trademark requirements), we conclude that the Ninth Circuit erred in barring Qualitex’s use of color as a trademark. For these reasons, the judgment of the Ninth Circuit is

 

Reversed.

 

 

Virtual Works, Inc. v. Volkswagen of America, Inc.,

238 F.3d 264 (4th Cir. 2001)

 

 

 

William Herbert Bode, Bode & Beckman, L.L.P., Washington, D.C., for Appellant.

 

Thomas Rex Lee, Howard, Phillips & Andersen, P.C., Salt Lake City, Utah, for Appellees.

 

Before Wilkinson, Chief Judge, and Michael and Traxler, Circuit Judges.

 

 

 

Wilkinson, Chief Judge:

 

Volkswagen challenges Virtual Works, Inc.’s use of the domain name vw.net under the 1999 Anticybersquatting Consumer Protection Act (ACPA). Volkswagen claims that Virtual Works registered vw.net with the purpose of one day selling it to Volkswagen. The district court agreed, holding that Virtual Works had a bad faith intent to profit from the vw.net domain name and that its use of vw.net diluted and infringed upon the VW mark. Virtual Works, Inc. v. Network Solutions, Inc., 106 F. Supp.2d 845 (E.D. Va. 2000). The district court therefore ordered Virtual Works to relinquish to Volkswagen the rights to vw.net. Because the district court did not err in holding that Virtual Works violated the ACPA, we affirm the judgment.

 

 

 

 

I.

 

 

 

 

On October 23, 1996, Virtual Works registered the domain name vw.net with Network Solutions, Inc. (NSI). At that time, NSI was the only company authorized by the government to serve as a registrar for Internet domain names. A domain name tells users where they can find a particular web page, much like a street address tells people where they can find a particular home or business. Domain names consist of two parts: the top level domain name (TLD) and secondary level domain name (SLD). The TLD is the suffix, identifying the nature of the site. The SLD is the prefix, identifying the site’s owner. Thus in the domain name Duke.edu, “.edu” is the TLD, identifying the site as affiliated with an educational institution. “Duke” is the SLD, identifying the owner as Duke University. There are various other TLDs. The most common are .com, .net, and .org for commercial users and .gov for governmental entities. At one point there was a distinction between the .com, .org, and .net TLDs. The .net TLD was reserved for Internet service providers (ISPs). The .org TLD was reserved for non-commercial or non-profit users. In September 1995, however, NSI stopped enforcing these distinctions. Thus, after 1995, commercial businesses could register domain names with the .net, .org, or .com TLD.

 

At the time Virtual Works registered vw.net, two of its principals, Christopher Grimes and James Anderson, were aware that some Internet users might think that vw.net was affiliated with Volkswagen. According to Grimes, he and Anderson “talked about Volkswagen and decided that [they] would use the domain name for [the] company, but if Volkswagen offered to work out a deal for services or products, that [they] would sell it to [Volkswagen] for a lot of money.” When Virtual Works registered vw.net , many other domain names were available for its use. For instance, vwi.net, vwi.org, virtual works. net, and virtual works. org, were still available.

 

Virtual Works used the vw.net domain name for approximately two years as a part of its ISP business. In December 1998, various Volkswagen dealerships contacted Virtual Works and expressed an interest in purchasing the rights to the vw.net domain name. Virtual Works, in turn, called Volkswagen, offering to sell vw.net. The terms of Virtual Works’ offer, however, were somewhat unusual. Anderson left a voice mail message for Linda Scipione in Volkswagen’s trademark department. In the message, Anderson stated that he owned the rights to vw.net. He also said that unless Volkswagen bought the rights to vw.net, Virtual Works would sell the domain name to the highest bidder. Anderson gave Volkswagen twenty-four hours to respond.

 

In response to what it perceived as a threat to the VW mark, Volkswagen invoked NSI’s dispute resolution procedure. NSI in turn told Virtual Works that Virtual Works would lose the vw.net domain name unless it filed a declaratory judgment action against Volkswagen. Virtual Works complied. Volkswagen subsequently counterclaimed, alleging trademark dilution, infringement, and cybersquatting under the ACPA. 15 U.S.C. § 1125(d). The district court granted Volkswagen’s motion for summary judgment on its cybersquatting, dilution, and infringement counterclaims and dismissed Virtual Works’ cross-motions on the same. Accordingly, the district court ordered Virtual Works to relinquish to Volkswagen the rights to the vw.net domain name. Virtual Works appeals.

 

 

 

 

II.

 

 

 

 

 

 

A.

 

 

 

 

 

The ACPA was enacted in 1999 in response to concerns over the proliferation of cybersquatting -the Internet version of a land grab. According to the Senate Report accompanying the Act: “Trademark owners are facing a new form of piracy on the Internet caused by acts of ‘cybersquatting,’ which refers to the deliberate, bad-faith, and abusive registration of Internet domain names in violation of the rights of trademark owners.” S. Rep. No. 106-140, at 4 (1999). Cybersquatting is the practice of registering “well-known brand names as Internet domain names” in order to force the rightful owners of the marks “to pay for the right to engage in electronic commerce under their own brand name.” Id. at 5. See also H.R. Rep. No. 106-412, at 5-7 (1999). Cybersquatting is profitable because while it is inexpensive for a cybersquatter to register the mark of an established company as a domain name, such companies are often vulnerable to being forced into paying substantial sums to get their names back. Sporty’s Farm, L.L.C. v. Sportsman’s Market, Inc., 202 F.3d 489, 493 (2d Cir. 2000).

 

Congress viewed the practice of cybersquatting as harmful because it threatened “the continued growth and vitality of the Internet as a platform” for “communication, electronic commerce, education, entertainment, and countless other yet-to-be-determined uses.” S. Rep. No. 106-140, at 8. New legislation was required to address this situation because then-current law did not expressly prohibit the act of cybersquatting and cybersquatters had started to take the necessary precautions to insulate themselves from liability under the Federal Trademark Dilution Act. Id. at 7. Accordingly, Congress passed, and the President signed, the ACPA in 1999. Pub. L. No. 106-113, 113 Stat. 1536 (codified at 15 U.S.C. § 1125(d)).

 

 

 

 

 

B.

 

 

 

 

 

Under the ACPA, a person alleged to be a cybersquatter is liable to the owner of a protected mark if that person:

 

 

(i) has a bad faith intent to profit from that mark …; and

 

 

 

(ii) registers, traffics in, or uses a domain name that-(I) in the case of a mark that is distinctive …, is identical or confusingly similar to that mark;(II) in the case of a famous mark …, is identical or confusingly similar to or dilutive of that mark;

 

 

15 U.S.C. § 1125(d)(1)(A). With respect to the bad faith determination, the statute provides that:

 

 

(B)(i) In determining whether a person has a bad faith intent … a court may consider factors such as, but not limited to

 

 

(I) the trademark or other intellectual property rights of the person, if any, in the domain name;

 

 

 

 

 

(II) the extent to which the domain name consists of the legal name of the person or a name that is otherwise commonly used to identify that person;

 

 

 

 

 

(III) the person’s prior use, if any, of the domain name in connection with the bona fide offering of any goods or services;

 

 

 

 

 

(IV) the person’s bona fide noncommercial or fair use of the mark in a site accessible under the domain name;

 

 

 

 

 

(V) the person’s intent to divert consumers from the mark owner’s online location to a site … that could harm the goodwill represented by the mark, either for commercial gain or with the intent to tarnish or disparage the mark …;

 

 

 

 

 

(VI) the person’s offer to transfer, sell, or otherwise assign the domain name to the mark owner or any third party for financial gain without having used … the domain name in the bona fide offering of any goods or services …;

 

 

 

 

 

(VII) the person’s provision of material and misleading false contact information when applying for the registration of the domain name …;

 

 

 

 

 

(VIII) the person’s registration or acquisition of multiple domain names which the person knows are identical or confusingly similar to marks of others …; and

 

 

 

 

 

(IX) the extent to which the mark incorporated in the person’s domain name registration is or is not distinctive and famous … .

 

 

 

15 U.S.C. § 1125(d)(1)(B)(i). In addition to listing these nine factors, the Act contains a safe harbor. The safe harbor provision states that bad faith intent “shall not be found in any case in which the court determines that the person believed and had reasonable grounds to believe that the use of the domain name was fair use or otherwise lawful.” 15 U.S.C. § 1225(d)(1)(B)(ii).

 

A court is not limited to considering these nine factors when determining the presence or absence of bad faith. 15 U.S.C. § 1125(d)(1)(B)(i). The Second Circuit, in the first court of appeals case addressing the ACPA, noted that the most important grounds for finding bad faith “are the unique circumstances of th[e] case, which do not fit neatly into the specific factors enumerated by Congress but may nevertheless be considered under the statute.” Sporty’s Farm, 202 F.3d at 499.

 

The remedies available under the ACPA depend on when the unlawful activity took place. A person who unlawfully registers, traffics in, or uses a domain name after the ACPA’s date of enactment, November 29, 1999, can be liable for monetary damages under 15 U.S.C. § 1117(d) and can have the domain name transferred to the owner of the mark or canceled under 15 U.S.C. § 1125(d)(2)(D)(i). The only remedy available for ACPA violations that occurred before November 29, 1999, however, is to have the domain name transferred to the owner of the mark or canceled. Anticybersquatting Consumer Protection Act, Pub. L. No. 106-113, § 3010, 113 Stat. 1536. Since Virtual Works’ alleged cybersquatting occurred before the ACPA’s date of enactment, Volkswagen sought only the right to use vw.net for itself.

 

 

 

 

III.

 

 

 

 

Having discussed the statutory purpose and framework of the ACPA, we must now determine whether Virtual Works violated the Act. The district court found that a number of the ACPA’s nine bad faith factors supported Volkswagen’s claim that Virtual Works’ registration of vw.net constituted cybersquatting under the Act. Virtual Works, 106 F. Supp.2d at 848. With respect to the first and second factors, for example, the district court held that Virtual Works had no right to or interest in the VW mark and that Virtual Works had never been referred to or done business under the name VW. Id. at 847. With respect to the fifth factor, the district court held that the disparaging comments posted by Virtual Works harmed the goodwill of the VW mark. Id. Finally, the district court found that, under the ninth factor, the famousness of the VW mark also favored Volkswagen. Id. The district court thus granted summary judgment to Volkswagen, which we review de novo.

 

 

 

 

 

A.

 

 

 

 

 

The first inquiry under the ACPA is whether Virtual Works acted with a bad faith intent to profit from a protected mark. 15 U.S.C. § 1125(d)(1)(A)(i). Virtual Works claims that the district court erred in holding that it did. We need not, however, march through the nine factors seriatim because the ACPA itself notes that use of the listed criteria is permissive. As the Second Circuit noted in Sporty’s Farms, the factors are “expressly described as indicia that ‘may’ be considered along with other facts.” 202 F.3d at 498 (citing 15 U.S.C. § 1125(d)(1)(B)(i)).

 

We are mindful that the instant case comes to us on summary judgment and involves a contested determination of Virtual Works’ intent. Unfortunately for Virtual Works, however, there is both circumstantial and direct evidence establishing bad faith. The following uncontested facts all provide circumstantial evidence of Virtual Works’ bad faith with respect to the VW mark: 1) the famousness of the VW mark; 2) the similarity of vw.net to the VW mark; 3) the admission that Virtual Works never once did business as VW nor identified itself as such; and 4) the availability of vwi.org and vwi.net at the time Virtual Works registered vw.net. Notably, either of these domain names would have satisfied Virtual Works’ own stated criterion of registering a domain name that used only two or three letters and would have eliminated any risk of confusion with respect to the VW mark.

 

We consider such circumstantial factors cautiously, however. We do not suggest that these four facts would alone resolve the question of Virtual Works’ intent on summary judgment. The fact that a domain resembles a famous trademark, for example, hardly in and of itself establishes bad faith. Moreover, domain names that are abbreviations of a company’s formal name are quite common. To view the use of such names as tantamount to bad faith would chill Internet entrepreneurship with the prospect of endless litigation.

 

Volkswagen, however, points to direct evidence regarding Virtual Works’ intent -the statements made at registration. Grimes’ deposition reveals that when registering vw.net, he and Anderson specifically acknowledged that vw.net might be confused with Volkswagen by some Internet users. They nevertheless decided to register the address for their own use, but left open the possibility of one day selling the site to Volkswagen “for a lot of money.” Volkswagen claims that this is sufficient to establish bad faith registration in violation of the ACPA.

 

Viewing the facts in the light most favorable to Virtual Works, as we must on summary judgment, the statement at registration establishes that Virtual Works had a dual purpose in selecting vw.net. Contrary to Virtual Works’ claim, the fact that it used vw.net for two years as a part of an ISP business is not dispositive of the question of intent. Virtual Works chose vw.net over other domain names not just because “vw” reflected the company’s own initials, but also because it foresaw the ability to profit from the natural association of vw.net with the VW mark. Indeed, it is obvious even to a casual observer that the similarity between vw.net and the VW mark is overwhelming.

 

Moreover, the facts in the summary judgment record affirmatively support the claim that Virtual Works had a bad faith intent to profit when it attempted to sell vw.net to Volkswagen. It is true that a mere offer to sell a domain name is not itself evidence of unlawful trafficking. H.R. Conf. Rep. No. 106-464, at 111 (1999). The ACPA was not enacted to put an end to the sale of all domain names. This case, however, involves much more than a plain vanilla offer to sell a domain name.

 

Indeed, the second piece of direct evidence regarding Virtual Works’ intent is the terms of its offer to Volkswagen. Virtual Works told Volkswagen that vw.net would be sold to the highest bidder if Volkswagen did not make an offer within twenty-four hours. Virtual Works also stated that others would jump at the chance to own a valuable domain name like vw.net because Internet users would instinctively associate the site with Volkswagen. Virtual Works knew, both when it registered vw.net and when it offered to sell the site, that consumers would associate vw.net with Volkswagen. It sought to maximize the advantage of this association by threatening to auction off the site. And it hoped that in an effort to protect its mark, Volkswagen would respond with a hefty offer of its own.

 

Likewise, Virtual Works cannot take refuge in the ACPA’s safe harbor provision. The safe harbor is only available when the defendant both “believed and had reasonable grounds to believe that the use of the domain name was fair use or otherwise lawful.” 15 U.S.C. § 1125(d)(1)(B)(ii). The openly admitted hope of profiting from consumer confusion of vw.net with the VW mark disqualifies Virtual Works from the ACPA’s safe harbor. A defendant who acts even partially in bad faith in registering a domain name is not, as a matter of law, entitled to benefit from the Act’s safe harbor provision. Virtual Works knew it was registering a domain name bearing strong resemblance to a federally protected trademark. And it did so, at least in part, with the idea of selling the site “for a lot of money” to the mark’s owner.

 

Just as we are reluctant to interpret the ACPA’s liability provisions in an overly aggressive manner, we decline to construe the safe harbor so broadly as to undermine the rest of the statute. All but the most blatant cybersquatters will be able to put forth at least some lawful motives for their behavior. To hold that all such individuals may qualify for the safe harbor would frustrate Congress’ purpose by artificially limiting the statute’s reach. We do not think Congress intended the safe harbor to protect defendants operating, at least in part, with unlawful intent.

 

The ACPA allows a court to view the totality of the circumstances in making the bad faith determination. 15 U.S.C.S 1125(d)(1)(B)(i). Here, that means looking at the purely circumstantial indicia of bad faith, as well as the direct evidence of the statements made at the time of registration and the direct evidence regarding terms of the sale. Whether our decision would be the same in the absence of any particular piece of evidence is a question we need not address. Viewed in its totality, the evidence establishes that at the time Virtual Works proposed to sell vw.net to Volkswagen, it was motivated by a bad faith intent to profit from the famousness of the VW mark. This is the sort of misconduct that Congress sought to discourage.

 

 

 

 

 

B.

 

 

 

 

 

The second inquiry under the ACPA is whether Virtual Works 1) registered, trafficked in, or used a domain name; 2) that is identical or confusingly similar to a distinctive mark; or 3) is identical, confusingly similar to, or dilutive of a famous mark. 15 U.S.C. § 1125(d)(1)(A)(ii). There is no dispute that Virtual Works registered, trafficked in, and used vw.net. There is also no dispute that the VW mark is famous. The sole point of contention is whether vw.net is identical, confusingly similar to, or dilutive of Volkswagen’s famous VW mark.

 

Virtual Works claims it is not similar because there is a distinction between the .net and .com TLD. According to Virtual Works, Volkswagen could not have registered vw.net in October of 1996 because it is an automaker and not an Internet service provider. This claim, however, is unavailing in light of the fact that NSI stopped enforcing the .com/.net distinction over a year before Virtual Works registered vw.net. The claim is also undermined by Virtual Works’ admission that at the time of registration it was aware of the potential confusion with the VW mark, and by its statement to Volkswagen that users would instinctively use the vw.net address to link to Volkswagen’s web site. Cf. Shade’s Landing, Inc. v. Williams , 76 F. Supp.2d 983, 990 (D. Minn. 1999) (“Because all domain names include one of these extensions, the distinction between a domain name ending with ‘.com’ and the same name ending with ‘.net’ is not highly significant.”). The district court was correct, therefore, in holding that vw.net is confusingly similar to the famous VW mark.

 

 

 

 

IV.

 

 

 

 

The remedy that Volkswagen sought in district court was the right to use vw.net for itself. The ACPA allows a court to order “the transfer of the domain name to the owner of the mark” if the Act is violated. 15 U.S.C. § 1125(d)(2)(D)(i). Because Virtual Works’ violation of the ACPA supports the remedy Volkswagen seeks, we need not address Volkswagen’s claims of trademark infringement or dilution.

 

The ACPA was not enacted to give companies the right to fence off every possible combination of letters that bears any similarity to a protected mark. Rather, it was enacted to prevent the expropriation of protected marks in cyberspace and to abate the consumer confusion resulting therefrom. The resolution of this case turns on the unique facts and circumstances which it presents. Ultimately, we believe the evidence is sufficient to establish that, as a matter of law, Virtual Works attempted to profit in bad faith from Volkswagen’s famous mark. 15 U.S.C. § 1125(d)(1)(A). The district court thus did not err in ordering Virtual Works to turn over vw.net to Volkswagen. For the foregoing reasons, we affirm the judgment.

 

 

PROBLEMS

 

 

 

If Qualitex developed a new material for their dryer pads that featured distinctive, small bumps that aid in the evenness of drying, Qualitex … (For each answer, state whether the resulting statement is true or false, with a very, very brief explanation.)

 

(a) could trademark the bumpy pad.

 

(b) would be barred from trademark because a bumpy texture is not a word, name, symbol or device.

 

(c) would be barred from trademark under the functionality doctrine.

 

(d) would be barred from obtaining a patent for the texture itself, because the bumps aren’t a “process,” under the patent statutes.

 

(e) might be able to patent the process for creating the material even if materials featuring a similar texture are already on the market.

 

 

 

Answers

 

 

(a) could trademark the bumpy pad.

 

 

False. This is just a pure application of the functionality doctrine – see below.

 

 

(b) would be barred from trademark because a bumpy texture is not a word, name, symbol or device.

 

 

False. As Breyer stated in Qualitex, “[s]ince human beings might use as a ‘symbol’ or ‘device’ almost anything at all that is capable of carrying meaning, this language, read literally, is not restrictive.” Indeed, a bumpy texture is capable of carrying meaning. Even the more demanding part of the statute might be met here, since the bumpy texture quite possibly could be used to distinguish Qualitex as the source of the pads.

 

 

(c) would be barred from trademark under the functionality doctrine.

 

 

True. The texture “aid[s] in the evenness of drying.” Because the texture results in a better performing pad, and thus “affects the … quality” of the pad, the functionality doctrine would bar trademark registration of the texture.

 

 

(d) would be barred from obtaining a patent for the texture itself, because the bumps aren’t a “process,” under the patent statutes.

 

 

False. Even if the bumps aren’t a “process,” they might be a “manufacture” or “composition of matter,” and thus patentable subject matter under §101 of the Patent Act. The Act allows one to patent inventions other than processes.

 

 

(e) might be able to patent the process for creating the material even if materials featuring a similar texture are already on the market.

 

 

True. If similar materials are on the market already, then one would not be able to patent the material itself. This is because the material would not be novel, which is required to obtain a patent. However, if Qualitex has invented a new process for creating a known material and that process otherwise meets the novel and non-obvious requirements, then the process may be patentable.

 

 

3.3. Patents

 

Diamond v. Chakrabarty,

447 U.S. 303 (1980)

 

 

 

Lawrence G. Wallace, Washington, D. C., for petitioner.

 

Edward F. McKie, Jr., Washington, D. C., for respondent.

 

 

 

Mr. Chief Justice Burger delivered the opinion of the Court.

 

We granted certiorari to determine whether a live, human-made micro-organism is patentable subject matter under 35 U.S.C. § 101.

 

 

 

 

I

 

 

 

 

In 1972, respondent Chakrabarty, a microbiologist, filed a patent application, assigned to the General Electric Co. The application asserted 36 claims related to Chakrabarty’s invention of “a bacterium from the genus Pseudomonas containing therein at least two stable energy-generating plasmids, each of said plasmids providing a separate hydrocarbon degradative pathway.”1 This human-made, genetically engineered bacterium is capable of breaking down multiple components of crude oil. Because of this property, which is possessed by no naturally occurring bacteria, Chakrabarty’s invention is believed to have significant value for the treatment of oil spills.2

 

Chakrabarty’s patent claims were of three types: first, process claims for the method of producing the bacteria; second, claims for an inoculum comprised of a carrier material floating on water, such as straw, and the new bacteria; and third, claims to the bacteria themselves. The patent examiner allowed the claims falling into the first two categories, but rejected claims for the bacteria. His decision rested on two grounds: (1) that micro-organisms are “products of nature,” and (2) that as living things they are not patentable subject matter under 35 U.S.C. § 101.

 

Chakrabarty appealed the rejection of these claims to the Patent Office Board of Appeals, and the Board affirmed the Examiner on the second ground.3 Relying on the legislative history of the 1930 Plant Patent Act, in which Congress extended patent protection to certain asexually reproduced plants, the Board concluded that § 101 was not intended to cover living things such as these laboratory created micro-organisms.

 

The Court of Customs and Patent Appeals, by a divided vote, reversed on the authority of its prior decision in In re Bergy, 563 F.2d 1031, 1038 (1977), which held that “the fact that microorganisms … are alive … [is] without legal significance” for purposes of the patent law.4 Subsequently, we granted the Acting Commissioner of Patents and Trademarks’ petition for certiorari in Bergy, vacated the judgment, and remanded the case “for further consideration in light of Parker v. Flook, 437 U.S. 584, [98 S.Ct. 2522, 57 L.Ed.2d 451] (1978).” 438 U.S. 902, 98 S.Ct. 3119, 57 L.Ed.2d 1145 (1978). The Court of Customs and Patent Appeals then vacated its judgment in Chakrabarty and consolidated the case with Bergy for reconsideration. After re-examining both cases in the light of our holding in Flook, that court, with one dissent, reaffirmed its earlier judgments. 596 F.2d 952 (1979).

 

The Commissioner of Patents and Trademarks again sought certiorari, and we granted the writ as to both Bergy and Chakrabarty. 444 U.S. 924, 100 S.Ct. 261, 62 L.Ed.2d 180 (1979). Since then, Bergy has been dismissed as moot, 444 U.S. 1028, 100 S.Ct. 696, 62 L.Ed.2d 664 (1980), leaving only Chakrabarty for decision.

 

 

 

 

II

 

 

 

 

The Constitution grants Congress broad power to legislate to “promote the Progress of Science and useful Arts, by securing for limited Times to Authors and Inventors the exclusive Right to their respective Writings and Discoveries.” Art. I, § 8, cl. 8. The patent laws promote this progress by offering inventors exclusive rights for a limited period as an incentive for their inventiveness and research efforts. Kewanee Oil Co. v. Bicron Corp., 416 U.S. 470, 480-481, 94 S.Ct. 1879, 1885-1886, 40 L.Ed.2d 315 (1974); Universal Oil Co. v. Globe Co., 322 U.S. 471, 484, 64 S.Ct. 1110, 1116, 88 L.Ed. 1399 (1944). The authority of Congress is exercised in the hope that “[t]he productive effort thereby fostered will have a positive effect on society through the introduction of new products and processes of manufacture into the economy, and the emanations by way of increased employment and better lives for our citizens.” Kewanee, supra, 416 U.S., at 480, 94 S.Ct., at 1885-86.

 

The question before us in this case is a narrow one of statutory interpretation requiring us to construe 35 U.S.C. § 101, which provides:

 

 

Whoever invents or discovers any new and useful process, machine, manufacture, or composition of matter, or any new and useful improvement thereof, may obtain a patent therefor, subject to the conditions and requirements of this title.

 

 

Specifically, we must determine whether respondent’s micro-organism constitutes a “manufacture” or “composition of matter” within the meaning of the statute.5

 

 

 

 

III

 

 

 

 

In cases of statutory construction we begin, of course, with the language of the statute. Southeastern Community College v. Davis, 442 U.S. 397, 405, 99 S.Ct. 2361, 2366, 60 L.Ed.2d 980 (1979). And “unless otherwise defined, words will be interpreted as taking their ordinary, contemporary common meaning.” Perrin v. United States, 444 U.S. 37, 42, 100 S.Ct. 311, 314, 62 L.Ed.2d 199 (1979). We have also cautioned that courts “should not read into the patent laws limitations and conditions which the legislature has not expressed.” United States v. Dubilier Condenser Corp., 289 U.S. 178, 199, 53 S.Ct. 554, 561, 77 L.Ed. 1114 (1933).

 

Guided by these canons of construction, this Court has read the term “manufacture” in § 101 in accordance with its dictionary definition to mean “the production of articles for use from raw or prepared materials by giving to these materials new forms, qualities, properties, or combinations, whether by hand-labor or by machinery.” American Fruit Growers, Inc. v. Brogdex Co., 283 U.S. 1, 11, 51 S.Ct. 328, 330, 75 L.Ed. 801 (1931). Similarly, “composition of matter” has been construed consistent with its common usage to include “all compositions of two or more substances and … all composite articles, whether they be the results of chemical union, or of mechanical mixture, or whether they be gases, fluids, powders or solids.” Shell Development Co. v. Watson, 149 F.Supp. 279, 280 (D.C.1957) (citing 1 A. Deller, Walker on Patents § 14, p. 55 (1st ed. 1937)). In choosing such expansive terms as “manufacture” and “composition of matter,” modified by the comprehensive “any,” Congress plainly contemplated that the patent laws would be given wide scope.

 

The relevant legislative history also supports a broad construction. The Patent Act of 1793, authored by Thomas Jefferson, defined statutory subject matter as “any new and useful art, machine, manufacture, or composition of matter, or any new or useful improvement [thereof].” Act of Feb. 21, 1793, § 1, 1 Stat. 319. The Act embodied Jefferson’s philosophy that “ingenuity should receive a liberal encouragement.” 5 Writings of Thomas Jefferson 75-76 (Washington ed. 1871). See Graham v. John Deere Co., 383 U.S. 1, 7-10, 86 S.Ct. 684, 688-690, 15 L.Ed.2d 545 (1966). Subsequent patent statutes in 1836, 1870, and 1874 employed this same broad language. In 1952, when the patent laws were recodified, Congress replaced the word “art” with “process,” but otherwise left Jefferson’s language intact. The Committee Reports accompanying the 1952 Act inform us that Congress intended statutory subject matter to “include anything under the sun that is made by man.” S.Rep.No.1979, 82d Cong., 2d Sess., 5 (1952); H.R.Rep.No.1923, 82d Cong., 2d Sess., 6 (1952).6

 

This is not to suggest that § 101 has no limits or that it embraces every discovery. The laws of nature, physical phenomena, and abstract ideas have been held not patentable. See Parker v. Flook, 437 U.S. 584, 98 S.Ct. 2522, 57 L.Ed.2d 451 (1978); Gottschalk v. Benson, 409 U.S. 63, 67, 93 S.Ct. 253, 255, 34 L.Ed.2d 273 (1972); Funk Brothers Seed Co. v. Kalo Inoculant Co., 333 U.S. 127, 130, 68 S.Ct. 440, 441, 92 L.Ed. 588 (1948); O’Reilly v. Morse, 15 How. 62, 112-121, 14 L.Ed. 601 (1854); Le Roy v. Tatham, 14 How. 156, 175, 14 L.Ed. 367 (1853). Thus, a new mineral discovered in the earth or a new plant found in the wild is not patentable subject matter. Likewise, Einstein could not patent his celebrated law that E=mc7; nor could Newton have patented the law of gravity. Such discoveries are “manifestations of … nature, free to all men and reserved exclusively to none.” Funk, supra, 333 U.S., at 130, 68 S.Ct., at 441.

 

Judged in this light, respondent’s micro-organism plainly qualifies as patentable subject matter. His claim is not to a hitherto unknown natural phenomenon, but to a nonnaturally occurring manufacture or composition of matter – a product of human ingenuity “having a distinctive name, character [and] use.” Hartranft v. Wiegmann, 121 U.S. 609, 615, 7 S.Ct. 1240, 1243, 30 L.Ed. 1012 (1887). The point is underscored dramatically by comparison of the invention here with that in Funk. There, the patentee had discovered that there existed in nature certain species of root-nodule bacteria which did not exert a mutually inhibitive effect on each other. He used that discovery to produce a mixed culture capable of inoculating the seeds of leguminous plants. Concluding that the patentee had discovered “only some of the handiwork of nature,” the Court ruled the product non-patentable:

 

 

Each of the species of root-nodule bacteria contained in the package infects the same group of leguminous plants which it always infected. No species acquires a different use. The combination of species produces no new bacteria, no change in the six species of bacteria, and no enlargement of the range of their utility. Each species has the same effect it always had. The bacteria perform in their natural way. Their use in combination does not improve in any way their natural functioning. They serve the ends nature originally provided and act quite independently of any effort of the patentee.

 

 

333 U.S., at 131, 68 S.Ct., at 442.

 

Here, by contrast, the patentee has produced a new bacterium with markedly different characteristics from any found in nature and one having the potential for significant utility. His discovery is not nature’s handiwork, but his own; accordingly it is patentable subject matter under § 101.

 

 

 

 

IV

 

 

 

 

Two contrary arguments are advanced, neither of which we find persuasive.

 

 

 

 

 

(A)

 

 

 

 

 

The petitioner’s first argument rests on the enactment of the 1930 Plant Patent Act, which afforded patent protection to certain asexually reproduced plants, and the 1970 Plant Variety Protection Act, which authorized protection for certain sexually reproduced plants but excluded bacteria from its protection.8 In the petitioner’s view, the passage of these Acts evidences congressional understanding that the terms “manufacture” or “composition of matter” do not include living things; if they did, the petitioner argues, neither Act would have been necessary.

 

We reject this argument. Prior to 1930, two factors were thought to remove plants from patent protection. The first was the belief that plants, even those artificially bred, were products of nature for purposes of the patent law. This position appears to have derived from the decision of the patent office in Ex parte Latimer, 1889 Dec.Com.Pat. 123, in which a patent claim for fiber found in the needle of the Pinus australis was rejected. The Commissioner reasoned that a contrary result would permit “patents [to] be obtained upon the trees of the forest and the plants of the earth, which of course would be unreasonable and impossible.” Id., at 126. The Latimer case, it seems, came to “se[t] forth the general stand taken in these matters” that plants were natural products not subject to patent protection. Thorne, Relation of Patent Law to Natural Products, 6 J. Pat.Off.Soc. 23, 24 (1923).9 The second obstacle to patent protection for plants was the fact that plants were thought not amenable to the “written description” requirement of the patent law. See 35 U.S.C. § 112. Because new plants may differ from old only in color or perfume, differentiation by written description was often impossible. See Hearings on H.R.11372 before the House Committee on Patents, 71st Cong., 2d Sess. 7 (1930) (memorandum of Patent Commissioner Robertson).

 

In enacting the Plant Patent Act, Congress addressed both of these concerns. It explained at length its belief that the work of the plant breeder “in aid of nature” was patentable invention. S.Rep.No.315, 71st Cong., 2d Sess., 6-8 (1930); H.R.Rep.No.1129, 71st Cong., 2d Sess., 7-9 (1930). And it relaxed the written description requirement in favor of “a description … as complete as is reasonably possible.” 35 U.S.C. § 162. No Committee or Member of Congress, however, expressed the broader view, now urged by the petitioner, that the terms “manufacture” or “composition of matter” exclude living things. The sole support for that position in the legislative history of the 1930 Act is found in the conclusory statement of Secretary of Agriculture Hyde, in a letter to the Chairmen of the House and Senate Committees considering the 1930 Act, that “the patent laws … at the present time are understood to cover only inventions or discoveries in the field of inanimate nature.” See S.Rep.No.315, supra, at Appendix A; H.R.Rep.No.1129, supra, at Appendix A. Secretary Hyde’s opinion, however, is not entitled to controlling weight. His views were solicited on the administration of the new law and not on the scope of patentable subject matter – an area beyond his competence. Moreover, there is language in the House and Senate Committee Reports suggesting that to the extent Congress considered the matter it found the Secretary’s dichotomy unpersuasive. The Reports observe:

 

 

There is a clear and logical distinction between the discovery of a new variety of plant and of certain inanimate things, such, for example, as a new and useful natural mineral. The mineral is created wholly by nature unassisted by man… . On the other hand, a plant discovery resulting from cultivation is unique, isolated, and is not repeated by nature, nor can it be reproduced by nature unaided by man… .

 

 

S.Rep.No.315, supra, at 6; H.R.Rep.No.1129,supra, at 7 (emphasis added).

 

Congress thus recognized that the relevant distinction was not between living and inanimate things, but between products of nature, whether living or not, and human-made inventions. Here, respondent’s micro-organism is the result of human ingenuity and research. Hence, the passage of the Plant Patent Act affords the Government no support.

 

Nor does the passage of the 1970 Plant Variety Protection Act support the Government’s position. As the Government acknowledges, sexually reproduced plants were not included under the 1930 Act because new varieties could not be reproduced true-to-type through seedlings. Brief for Petitioner 27, n. 31. By 1970, however, it was generally recognized that true-to-type reproduction was possible and that plant patent protection was therefore appropriate. The 1970 Act extended that protection. There is nothing in its language or history to suggest that it was enacted because § 101 did not include living things.

 

In particular, we find nothing in the exclusion of bacteria from plant variety protection to support the petitioner’s position. See n. 7, supra. The legislative history gives no reason for this exclusion. As the Court of Customs and Patent Appeals suggested, it may simply reflect congressional agreement with the result reached by that court in deciding In re Arzberger, 27 C.C.P.A. (Pat.) 1315, 112 F.2d 834 (1940), which held that bacteria were not plants for the purposes of the 1930 Act. Or it may reflect the fact that prior to 1970 the Patent Office had issued patents for bacteria under § 101.10 In any event, absent some clear indication that Congress “focused on [the] issues … directly related to the one presently before the Court,” SEC v. Sloan, 436 U.S. 103, 120-121, 98 S.Ct. 1702, 1713, 56 L.Ed.2d 148 (1978), there is no basis for reading into its actions an intent to modify the plain meaning of the words found in § 101. See TVA v. Hill, 437 U.S. 153, 189-193, 98 S.Ct. 2279, 2299-2301, 57 L.Ed.2d 117 (1978); United States v. Price, 361 U.S. 304, 313, 80 S.Ct. 326, 331, 4 L.Ed.2d 334 (1960).

 

 

 

 

 

(B)

 

 

 

 

 

The petitioner’s second argument is that micro-organisms cannot qualify as patentable subject matter until Congress expressly authorizes such protection. His position rests on the fact that genetic technology was unforeseen when Congress enacted § 101. From this it is argued that resolution of the patentability of inventions such as respondent’s should be left to Congress. The legislative process, the petitioner argues, is best equipped to weigh the competing economic, social, and scientific considerations involved, and to determine whether living organisms produced by genetic engineering should receive patent protection. In support of this position, the petitioner relies on our recent holding in Parker v. Flook, 437 U.S. 584, 98 S.Ct. 2522, 57 L.Ed.2d 451 (1978), and the statement that the judiciary “must proceed cautiously when … asked to extend patent rights into areas wholly unforeseen by Congress.” Id., at 596, 98 S.Ct. at 2529.

 

It is, of course, correct that Congress, not the courts, must define the limits of patentability; but it is equally true that once Congress has spoken it is “the province and duty of the judicial department to say what the law is.” Marbury v. Madison, 1 Cranch 137, 177, 2 L.Ed. 60 (1803). Congress has performed its constitutional role in defining patentable subject matter in § 101; we perform ours in construing the language Congress has employed. In so doing, our obligation is to take statutes as we find them, guided, if ambiguity appears, by the legislative history and statutory purpose. Here, we perceive no ambiguity. The subject-matter provisions of the patent law have been cast in broad terms to fulfill the constitutional and statutory goal of promoting “the Progress of Science and the useful Arts” with all that means for the social and economic benefits envisioned by Jefferson. Broad general language is not necessarily ambiguous when congressional objectives require broad terms.

 

Nothing in Flook is to the contrary. That case applied our prior precedents to determine that a “claim for an improved method of calculation, even when tied to a specific end use, is unpatentable subject matter under § 101.” 437 U.S., at 595, n. 18, 98 S.Ct., at 2528, n. 18. The Court carefully scrutinized the claim at issue to determine whether it was precluded from patent protection under “the principles underlying the prohibition against patents for ‘ideas’ or phenomena of nature.” Id., at 593, 98 S.Ct. at 2527. We have done that here. Flook did not announce a new principle that inventions in areas not contemplated by Congress when the patent laws were enacted are unpatentable per se.

 

To read that concept into Flook would frustrate the purposes of the patent law. This Court frequently has observed that a statute is not to be confined to the “particular application[s] … contemplated by the legislators.” Barr v. United States, 324 U.S. 83, 90, 65 S.Ct. 522, 525, 89 L.Ed. 765 (1945). Accord, Browder v. United States, 312 U.S. 335, 339, 61 S.Ct. 599, 601, 85 L.Ed. 862 (1941); Puerto Rico v. Shell Co., 302 U.S. 253, 257, 58 S.Ct. 167, 169, 82 L.Ed. 235 (1937). This is especially true in the field of patent law. A rule that unanticipated inventions are without protection would conflict with the core concept of the patent law that anticipation undermines patentability. See Graham v. John Deere Co., 383 U.S., at 12-17, 86 S.Ct., at 691-693. Mr. Justice Douglas reminded that the inventions most benefiting mankind are those that “push back the frontiers of chemistry, physics, and the like.” Great A. & P. Tea Co. v. Supermarket Corp., 340 U.S. 147, 154, 71 S.Ct. 127, 131, 95 L.Ed. 162 (1950) (concurring opinion). Congress employed broad general language in drafting § 101 precisely because such inventions are often unforeseeable.11

 

To buttress his argument, the petitioner, with the support of amicus, points to grave risks that may be generated by research endeavors such as respondent’s. The briefs present a gruesome parade of horribles. Scientists, among them Nobel laureates, are quoted suggesting that genetic research may pose a serious threat to the human race, or, at the very least, that the dangers are far too substantial to permit such research to proceed apace at this time. We are told that genetic research and related technological developments may spread pollution and disease, that it may result in a loss of genetic diversity, and that its practice may tend to depreciate the value of human life. These arguments are forcefully, even passionately, presented; they remind us that, at times, human ingenuity seems unable to control fully the forces it creates – that with Hamlet, it is sometimes better “to bear those ills we have than fly to others that we know not of.”

 

It is argued that this Court should weigh these potential hazards in considering whether respondent’s invention is patentable subject matter under § 101. We disagree. The grant or denial of patents on micro-organisms is not likely to put an end to genetic research or to its attendant risks. The large amount of research that has already occurred when no researcher had sure knowledge that patent protection would be available suggests that legislative or judicial fiat as to patentability will not deter the scientific mind from probing into the unknown any more than Canute could command the tides. Whether respondent’s claims are patentable may determine whether research efforts are accelerated by the hope of reward or slowed by want of incentives, but that is all.

 

What is more important is that we are without competence to entertain these arguments – either to brush them aside as fantasies generated by fear of the unknown, or to act on them. The choice we are urged to make is a matter of high policy for resolution within the legislative process after the kind of investigation, examination, and study that legislative bodies can provide and courts cannot. That process involves the balancing of competing values and interests, which in our democratic system is the business of elected representatives. Whatever their validity, the contentions now pressed on us should be addressed to the political branches of the Government, the Congress and the Executive, and not to the courts.12

 

We have emphasized in the recent past that “[o]ur individual appraisal of the wisdom or unwisdom of a particular [legislative] course … is to be put aside in the process of interpreting a statute.” TVA v. Hill, 437 U.S., at 194, 98 S.Ct., at 2302. Our task, rather, is the narrow one of determining what Congress meant by the words it used in the statute; once that is done our powers are exhausted. Congress is free to amend § 101 so as to exclude from patent protection organisms produced by genetic engineering. Cf. 42 U.S.C. § 2181(a), exempting from patent protection inventions “useful solely in the utilization of special nuclear material or atomic energy in an atomic weapon.” Or it may chose to craft a statute specifically designed for such living things. But, until Congress takes such action, this Court must construe the language of § 101 as it is. The language of that section fairly embraces respondent’s invention.

 

Accordingly, the judgment of the Court of Customs and Patent Appeals is

 

Affirmed.

 

 

 

Mr. Justice Brennan, with whom Mr. Justice White, Mr. Justice Marshall, and Mr. Justice POWELL join, dissenting.

 

I agree with the Court that the question before us is a narrow one. Neither the future of scientific research, nor even, the ability of respondent Chakrabarty to reap some monopoly profits from his pioneering work, is at stake. Patents on the processes by which he has produced and employed the new living organism are not contested. The only question we need decide is whether Congress, exercising its authority under Art. I, § 8, of the Constitution, intended that he be able to secure a monopoly on the living organism itself, no matter how produced or how used. Because I believe the Court has misread the applicable legislation, I dissent.

 

The patent laws attempt to reconcile this Nation’s deep seated antipathy to monopolies with the need to encourage progress. Deepsouth Packing Co. v. Laitram Corp., 406 U.S. 518, 530-531, 92 S.Ct. 1700, 1707-1708, 32 L.Ed.2d 273 (1972); Graham v. John Deere Co., 383 U.S. 1, 7-10, 86 S.Ct. 684, 668-690, 15 L.Ed.2d 545 (1966). Given the complexity and legislative nature of this delicate task, we must be careful to extend patent protection no further than Congress has provided. In particular, were there an absence of legislative direction, the courts should leave to Congress the decisions whether and how far to extend the patent privilege into areas where the common understanding has been that patents are not available.13 Cf. Deepsouth Packing Co. v. Laitram Corp., supra.

 

In this case, however, we do not confront a complete legislative vacuum. The sweeping language of the Patent Act of 1793, as re-enacted in 1952, is not the last pronouncement Congress has made in this area. In 1930 Congress enacted the Plant Patent Act affording patent protection to developers of certain asexually reproduced plants. In 1970 Congress enacted the Plant Variety Protection Act to extend protection to certain new plant varieties capable of sexual reproduction. Thus, we are not dealing – as the Court would have it – with the routine problem of “unanticipated inventions.” Ante, at 316. In these two Acts Congress has addressed the general problem of patenting animate inventions and has chosen carefully limited language granting protection to some kinds of discoveries, but specifically excluding others. These Acts strongly evidence a congressional limitation that excludes bacteria from patentability.2

 

First, the Acts evidence Congress’ understanding, at least since 1930, that § 101 does not include living organisms. If newly developed living organisms not naturally occurring had been patentable under § 101, the plants included in the scope of the 1930 and 1970 Acts could have been patented without new legislation. Those plants, like the bacteria involved in this case, were new varieties not naturally occurring.14 Although the Court, ante, at 311, rejects this line of argument, it does not explain why the Acts were necessary unless to correct a pre-existing situation.15 I cannot share the Court’s implicit assumption that Congress was engaged in either idle exercises or mere correction of the public record when it enacted the 1930 and 1970 Acts. And Congress certainly thought it was doing something significant. The Committee Reports contain expansive prose about the previously unavailable benefits to be derived from extending patent protection to plants.16 H.R. Rep. No. 91-1605, pp. 1-3 (1970), U.S.Code Cong. & Admin.News 1970, p. 5082; S.Rep.No.315, 71st Cong., 2d Sess., 1-3 (1930). Because Congress thought it had to legislate in order to make agricultural “human-made inventions” patentable and because the legislation Congress enacted is limited, it follows that Congress never meant to make items outside the scope of the legislation patentable.

 

Second, the 1970 Act clearly indicates that Congress has included bacteria within the focus of its legislative concern, but not within the scope of patent protection. Congress specifically excluded bacteria from the coverage of the 1970 Act. 7 U.S.C. § 2402(a). The Court’s attempts to supply explanations for this explicit exclusion ring hollow. It is true that there is no mention in the legislative history of the exclusion, but that does not give us license to invent reasons. The fact is that Congress, assuming that animate objects as to which it had not specifically legislated could not be patented, excluded bacteria from the set of patentable organisms.

 

The Court protests that its holding today is dictated by the broad language of § 101, which cannot “be confined to the ‘particular application[s] … contemplated by the legislators.’ ” Ante, at 315, quoting Barr v. United States, 324 U.S. 83, 90, 65 S.Ct. 522, 525, 89 L.Ed. 765 (1945). But as I have shown, the Court’s decision does not follow the unavoidable implications of the statute. Rather, it extends the patent system to cover living material even though Congress plainly has legislated in the belief that § 101 does not encompass living organisms. It is the role of Congress, not this Court, to broaden or narrow the reach of the patent laws. This is especially true where, as here, the composition sought to be patented uniquely implicates matters of public concern.

 


  1. Plasmids are hereditary units physically separate from the chromosomes of the cell. In prior research, Chakrabarty and an associate discovered that plasmids control the oil degradation abilities of certain bacteria. In particular, the two researchers discovered plasmids capable of degrading camphor and octane, two components of crude oil. In the work represented by the patent application at issue here, Chakrabarty discovered a process by which four different plasmids, capable of degrading four different oil components, could be transferred to and maintained stably in a single Pseudomonas bacterium, which itself has no capacity for degrading oil.

 

  1. At present, biological control of oil spills requires the use of a mixture of naturally occurring bacteria, each capable of degrading one component of the oil complex. In this way, oil is decomposed into simpler substances which can serve as food for aquatic life. However, for various reasons, only a portion of any such mixed culture survives to attack the oil spill. By breaking down multiple components of oil, Chakrabarty’s micro-organism promises more efficient and rapid oil-spill control.

 

  1. The Board concluded that the new bacteria were not “products of nature,” because Pseudomonas bacteria containing two or more different energy-generating plasmids are not naturally occurring.

 

  1. Bergy involved a patent application for a pure culture of the micro-organism Streptomyces vellosus found to be useful in the production of lincomycin, an antibiotic.

 

  1. This case does not involve the other “conditions and requirements” of the patent laws, such as novelty and nonobviousness. 35 U.S.C. §§ 102, 103.

 

  1. This same language was employed by P. J. Federico, a principal draftsman of the 1952 recodification, in his testimony regarding that legislation: “[U]nder section 101 a person may have invented a machine or a manufacture, which may include anything under the sun that is made by man… . ” Hearings on H.R. 3760 before Subcommittee No. 3 of the House Committee on the Judiciary, 82d Cong., 1st Sess., 37 (1951).

 

  1. The Plant Patent Act of 1930, 35 U.S.C. § 161, provides in relevant part:

     

     

    Whoever invents or discovers and asexually reproduces any distinct and new variety of plant, including cultivated sports, mutants, hybrids, and newly found seedlings, other than a tuber propogated plant or a plant found in an uncultivated state, may obtain a patent therefor … .

     

     

    The Plant Variety Protection Act of 1970, provides in relevant part:

     

     

    The breeder of any novel variety of sexually reproduced plant (other than fungi, bacteria, or first generation hybrids) who has so reproduced the variety, or his successor in interest, shall be entitled to plant variety protection therefor … .

     

     

    84 Stat. 1547, 7 U.S.C. § 2402(a). See generally, 3 A. Deller, Walker on Patents, ch. IX (2d ed. 1964); R. Allyn, The First Plant Patents (1934).

 

  1. Writing three years after the passage of the 1930 Act, R. Cook, Editor of the Journal of Heredity, commented: “It is a little hard for plant men to understand why [Art. I, § 8] of the Constitution should not have been earlier construed to include the promotion of the art of plant breeding. The reason for this is probably to be found in the principle that natural products are not patentable.” Florists Exchange and Horticultural Trade World, July 15, 1933, p. 9.

 

  1. In 1873, the Patent Office granted Louis Pasteur a patent on “yeast, free from organic germs of disease, as an article of manufacture.” And in 1967 and 1968, immediately prior to the passage of the Plant Variety Protection Act, that Office granted two patents which, as the petitioner concedes, state claims for living micro-organisms. See Reply Brief for Petitioner 3, and n. 2.

 

  1. Even an abbreviated list of patented inventions underscores the point: telegraph (Morse, No. 1,647); telephone (Bell, No. 174,465); electric lamp (Edison, No. 223,898); airplane (the Wrights, No. 821,393); transistor (Bardeen & Brattain, No. 2,524,035); neutronic reactor (Fermi & Szilard, No. 2,708,656); laser (Schawlow & Townes, No. 2,929,922). See generally Revolutionary Ideas, Patents & Progress in America, United States Patent and Trademark Office (1976).

 

  1. We are not to be understood as suggesting that the political branches have been laggard in the consideration of the problems related to genetic research and technology. They have already taken action. In 1976, for example, the National Institutes of Health released guidelines for NIH-sponsored genetic research which established conditions under which such research could be performed. 41 Fed.Reg. 27902. In 1978 those guidelines were revised and relaxed. 43 Fed.Reg. 60080, 60108, 60134. And Committees of the Congress have held extensive hearings on these matters. See, e. g., Hearings on Genetic Engineering before the Subcommittee on Health of the Senate Committee on Labor and Public Welfare, 94th Cong., 1st Sess. (1975); Hearings before the Subcommittee on Science, Technology, and Space of the Senate Committee on Commerce, Science, and Transportation, 95th Cong., 1st Sess. (1977); Hearings on H.R. 4759 et al. before the Subcommittee on Health and the Environment of the House Committee on Interstate and Foreign Commerce, 95th Cong., 1st Sess. (1977).

 

  1. I read the Court to admit that the popular conception, even among advocates of agricultural patents, was that living organisms were unpatentable. See ante, at 311-312, and n. 8.

 

  1. But even if I agreed with the Court that the 1930 and 1970 Acts were not dispositive, I would dissent. This case presents even more cogent reasons than Deepsouth Packing Co. not to extend the patent monopoly in the face of uncertainty. At the very least, these Acts are signs of legislative attention to the problems of patenting living organisms, but they give no affirmative indication of congressional intent that bacteria be patentable. The caveat of Parker v. Flook, 437 U.S. 584, 596, 90 S.Ct. 2522, 2529, 57 L.Ed.2d 451 (1978), an admonition to “proceed cautiously when we are asked to extend patent rights into areas wholly unforeseen by Congress,” therefore becomes pertinent. I should think the necessity for caution is that much greater when we are asked to extend patent rights into areas Congress has foreseen and considered but has not resolved.

 

  1. The Court refers to the logic employed by Congress in choosing not to perpetuate the “dichotomy” suggested by Secretary Hyde. Ante, at 313. But by this logic the bacteria at issue here are distinguishable from a “mineral … created wholly by nature” in exactly the same way as were the new varieties of plants. If a new Act was needed to provide patent protection for the plants, it was equally necessary for bacteria. Yet Congress provided for patents on plants but not on these bacteria. In short, Congress decided to make only a subset of animate “human-made inventions,” ibid., patentable.

 

  1. If the 1930 Act’s only purpose were to solve the technical problem of description referred to by the Court, ante, at 312, most of the Act, and in particular its limitation to asexually reproduced plants, would have been totally unnecessary.

 

  1. Secretary Hyde’s letter was not the only explicit indication in the legislative history of these Acts that Congress was acting on the assumption that legislation was necessary to make living organisms patentable. The Senate Judiciary Committee Report on the 1970 Act states the Committee’s understanding that patent protection extended no further than the explicit provisions of these Acts:

     

     

    Under the patent law, patent protection is limited to those varieties of plants which reproduce asexually, that is, by such methods as grafting or budding. No protection is available to those varieties of plants which reproduce sexually, that is, generally by seeds.

     

     

    S.Rep.No.91-1246, p. 3 (1970).

     

    Similarly, Representative Poage, speaking for the 1970 Act, after noting the protection accorded asexually developed plants, stated that “for plants produced from seed, there has been no such protection.” 116 Cong.Rec. 40295 (1970).

 

 

 

3.4 Copyright

 

3.4.1. Eligibility

 

Feist Publications, Inc. v. Rural Telephone Service Co.,

499 U.S. 340 (1991)

 

 

 

O’Connor J., delivered the opinion of the Court, in which Rehnquist, C.J., and White, Marshall, Stevens, Scalia, Kennedy, and Souter, JJ., joined. Blackmun, J., concurred in the judgment.

 

Kyler Knobbe, Cimarron, Kan., for petitioner.

 

James M. Caplinger, Jr., Topeka, Kan., for respondent.

 

 

 

Justice O’Connor delivered the opinion of the Court.

 

This case requires us to clarify the extent of copyright protection available to telephone directory white pages.

 

 

 

 

I

 

 

 

 

Rural Telephone Service Company, Inc., is a certified public utility that provides telephone service to several communities in northwest Kansas. It is subject to a state regulation that requires all telephone companies operating in Kansas to issue annually an updated telephone directory. Accordingly, as a condition of its monopoly franchise, Rural publishes a typical telephone directory, consisting of white pages and yellow pages. The white pages list in alphabetical order the names of Rural’s subscribers, together with their towns and telephone numbers. The yellow pages list Rural’s business subscribers alphabetically by category and feature classified advertisements of various sizes. Rural distributes its directory free of charge to its subscribers, but earns revenue by selling yellow pages advertisements.

 

Feist Publications, Inc., is a publishing company that specializes in area-wide telephone directories. Unlike a typical directory, which covers only a particular calling area, Feist’s area-wide directories cover a much larger geographical range, reducing the need to call directory assistance or consult multiple directories. The Feist directory that is the subject of this litigation covers 11 different telephone service areas in 15 counties and contains 46,878 white pages listings – compared to Rural’s approximately 7,700 listings. Like Rural’s directory, Feist’s is distributed free of charge and includes both white pages and yellow pages. Feist and Rural compete vigorously for yellow pages advertising.

 

As the sole provider of telephone service in its service area, Rural obtains subscriber information quite easily. Persons desiring telephone service must apply to Rural and provide their names and addresses; Rural then assigns them a telephone number. Feist is not a telephone company, let alone one with monopoly status, and therefore lacks independent access to any subscriber information. To obtain white pages listings for its area-wide directory, Feist approached each of the 11 telephone companies operating in northwest Kansas and offered to pay for the right to use its white pages listings.

 

Of the 11 telephone companies, only Rural refused to license its listings to Feist. Rural’s refusal created a problem for Feist, as omitting these listings would have left a gaping hole in its area-wide directory, rendering it less attractive to potential yellow pages advertisers. In a decision subsequent to that which we review here, the District Court determined that this was precisely the reason Rural refused to license its listings. The refusal was motivated by an unlawful purpose “to extend its monopoly in telephone service to a monopoly in yellow pages advertising.” Rural Telephone Service Co. v. Feist Publications, Inc., 737 F.Supp. 610, 622 (Kan.1990).

 

Unable to license Rural’s white pages listings, Feist used them without Rural’s consent. Feist began by removing several thousand listings that fell outside the geographic range of its area-wide directory, then hired personnel to investigate the 4,935 that remained. These employeesveri fied the data reported by Rural and sought to obtain additional information. As a result, a typical Feist listing includes the individual’s street address; most of Rural’s listings do not. Notwithstanding these additions, however,1,309 of the 46,878 listings in Feist’s 1983 directory were identical to listings in Rural’s 1982-1983 white pages. App. 54 (¶ 15-16), 57. Four of these were fictitious listings that Rural had inserted into its directory to detect copying.

 

Rural sued for copyright infringement in the District Court for the District of Kansas taking the position that Feist, in compiling its own directory, could not use the information contained in Rural’s white pages. Rural asserted that Feist’s employees were obliged to travel door-to-door or conduct a telephone survey to discover the same information for themselves. Feist responded that such efforts were economically impractical and, in any event, unnecessary because the information copied was beyond the scope of copyright protection. The District Court granted summary judgment to Rural, explaining that “[c]ourts have consistently held that telephone directories are copyrightable” and citing a string of lower court decisions. 663 F.Supp. 214, 218 (1987). In an unpublished opinion, the Court of Appeals for the Tenth Circuit affirmed “for substantially the reasons given by the district court.” App. to Pet. for Cert. 4a, judgt. order reported at 916 F.2d 718 (1990). We granted certiorari, 498 U.S. 808, 111 S.Ct. 40, 112 L.Ed.2d 17 (1990), to determine whether the copyright in Rural’s directory protects the names, towns, and telephone numbers copied by Feist.

 

 

 

 

II

 

 

 

 

 

 

A.

 

 

 

 

 

This case concerns the interaction of two well-established propositions. The first is that facts are not copyrightable; the other, that compilations of facts generally are. Each of these propositions possesses an impeccable pedigree. That there can be no valid copyright in facts is universally understood. The most fundamental axiom of copyright law is that “[n]o author may copyright his ideas or the facts he narrates.” Harper & Row, Publishers, Inc. v. Nation Enterprises, 471 U.S. 539, 556, 105 S.Ct. 2218, 2228, 85 L.Ed.2d 588 (1985). Rural wisely concedes this point, noting in its brief that “[f]acts and discoveries, of course, are not themselves subject to copyright protection.” Brief for Respondent 24. At the same time, however, it is beyond dispute that compilations of facts are within the subject matter of copyright. Compilations were expressly mentioned in the Copyright Act of 1909, and again in the Copyright Act of 1976.

 

There is an undeniable tension between these two propositions. Many compilations consist of nothing but raw data i.e., wholly factual information not accompanied by any original written expression. On what basis may one claim a copyright in such a work? Common sense tells us that 100 uncopyrightable facts do not magically change their status when gathered together in one place. Yet copyright law seems to contemplate that compilations that consist exclusively of facts are potentially within its scope.

 

The key to resolving the tension lies in understanding why facts are not copyrightable. The sine qua non of copyright is originality. To qualify for copyright protection, a work must be original to the author. See Harper & Row, supra, at 547-549, 105 S.Ct., at 2223-2224. Original, as the term is used in copyright, means only that the work was independently created by the author (as opposed to copied from other works), and that it possesses at least some minimal degree of creativity. 1 M. Nimmer & D. Nimmer, Copyright §§ 2.01[A], [B] (1990) (hereinafter Nimmer). To be sure, the requisite level of creativity is extremely low; even a slight amount will suffice. The vast majority of works make the grade quite easily, as they possess some creative spark, “no matter how crude, humble or obvious” it might be. Id., § 1.08[C][1]. Originality does not signify novelty; a work may be original even though it closely resembles other works so long as the similarity is fortuitous, not the result of copying. To illustrate, assume that two poets, each ignorant of the other, compose identical poems. Neither work is novel, yet both are original and, hence, copyrightable. See Sheldon v. Metro-Goldwyn Pictures Corp., 81 F.2d 49, 54 (CA2 1936).

 

Originality is a constitutional requirement. The source of Congress’ power to enact copyright laws is Article I, § 8, cl. 8, of the Constitution, which authorizes Congress to “secur[e] for limited Times to Authors … the exclusive Right to their respective Writings.” In two decisions from the late 19th century The Trade-Mark Cases, 100 U.S. 82, 25 L.Ed. 550 (1879); and Burrow-Giles Lithographic Co. v. Sarony, 111 U.S. 53, 4 S.Ct. 279, 28 L.Ed. 349 (1884) – this Court defined the crucial terms “authors” and “writings.” In so doing, the Court made it unmistakably clear that these terms presuppose a degree of originality.

 

In The Trade-Mark Cases, the Court addressed the constitutional scope of “writings.” For a particular work to be classified “under the head of writings of authors,” the Court determined, “originality is required.” 100 U.S., at 94. The Court explained that originality requires independent creation plus a modicum of creativity: “[W]hile the word writings may be liberally construed, as it has been, to include original designs for engraving, prints, &c., it is only such as are original, and are founded in the creative powers of the mind. The writings which are to be protected are the fruits of intellectual labor, embodied in the form of books, prints, engravings, and the like.” Ibid. (emphasis in original).

 

In Burrow-Giles, the Court distilled the same requirement from the Constitution’s use of the word “authors.” The Court defined “author,” in a constitutional sense, to mean “he to whom anything owes its origin; originator; maker.” 111 U.S., at 58, 4 S.Ct., at 281 (internal quotation marks omitted). As in The Trade-Mark Cases, the Court emphasized the creative component of originality. It described copyright as being limited to “original intellectual conceptions of the author,” 111 U.S., at 58, 4 S.Ct., at 281, and stressed the importance of requiring an author who accuses another of infringement to prove “the existence of those facts of originality, of intellectual production, of thought, and conception.” Id., at 59-60, 4 S.Ct., at 281-282.

 

The originality requirement articulated in The Trade-Mark Cases and Burrow-Giles remains the touchstone of copyright protection today. See Goldstein v. California, 412 U.S. 546, 561-562, 93 S.Ct. 2303, 2312, 37 L.Ed.2d 163 (1973). It is the very “premise of copyright law.” Miller v. Universal City Studios, Inc., 650 F.2d 1365, 1368 (CA5 1981). Leading scholars agree on this point. As one pair of commentators succinctly puts it: “The originality requirement is constitutionally mandated for all works.” Patterson & Joyce, Monopolizing the Law: The Scope of Copyright Protection for Law Reports and Statutory Compilations, 36 UCLA L.Rev. 719, 763, n. 155 (1989) (emphasis in original) (hereinafter Patterson & Joyce). Accord, id., at 759-760, and n. 140; Nimmer § 1.06[A] (“[O]riginality is a statutory as well as a constitutional requirement”); id., § 1.08[C][1] (“[A] modicum of intellectual labor … clearly constitutes an essential constitutional element”).

 

It is this bedrock principle of copyright that mandates the law’s seemingly disparate treatment of facts and factual compilations. “No one may claim originality as to facts.” Id., § 2.11[A], p. 2-157. This is because facts do not owe their origin to an act of authorship. The distinction is one between creation and discovery: The first person to find and report a particular fact has not created the fact; he or she has merely discovered its existence. To borrow from Burrow-Giles, one who discovers a fact is not its “maker” or “originator.” 111 U.S., at 58, 4 S.Ct., at 281. “The discoverer merely finds and records.” Nimmer § 2.03[E]. Census takers, for example, do not “create” the population figures that emerge from their efforts; in a sense, they copy these figures from the world around them. Denicola, Copyright in Collections of Facts: A Theory for the Protection of Nonfiction Literary Works, 81 Colum.L.Rev. 516, 525 (1981) (hereinafter Denicola). Census data therefore do not trigger copyright because these data are not “original” in the constitutional sense. Nimmer § 2.03[E]. The same is true of all facts – scientific, historical, biographical, and news of the day. “[T]hey may not be copyrighted and are part of the public domain available to every person.” Miller, supra, at 1369.

 

Factual compilations, on the other hand, may possess the requisite originality. The compilation author typically chooses which facts to include, in what order to place them, and how to arrange the collected data so that they may be used effectively by readers. These choices as to selection and arrangement, so long as they are made independently by the compiler and entail a minimal degree of creativity, are sufficiently original that Congress may protect such compilations through the copyright laws. Nimmer §§ 2.11[D], 3.03; Denicola 523, n. 38. Thus, even a directory that contains absolutely no protectible written expression, only facts, meets the constitutional minimum for copyright protection if it features an original selection or arrangement. See Harper & Row, 471 U.S., at 547, 105 S.Ct., at 2223. Accord, Nimmer § 3.03.

 

This protection is subject to an important limitation. The mere fact that a work is copyrighted does not mean that every element of the work may be protected. Originality remains the sine qua non of copyright; accordingly, copyright protection may extend only to those components of a work that are original to the author. Patterson & Joyce 800-802; Ginsburg, Creation and Commercial Value: Copyright Protection of Works of Information, 90 Colum.L.Rev. 1865, 1868, and n. 12 (1990) (hereinafter Ginsburg). Thus, if the compilation author clothes facts with an original collocation of words, he or she may be able to claim a copyright in this written expression. Others may copy the underlying facts from the publication, but not the precise words used to present them. In Harper & Row, for example, we explained that President Ford could not prevent others from copying bare historical facts from his autobiography, see 471 U.S., at 556-557, 105 S.Ct., at 2228-2229, but that he could prevent others from copying his “subjective descriptions and portraits of public figures.” Id., at 563, 105 S.Ct., at 2232. Where the compilation author adds no written expression but rather lets the facts speak for themselves, the expressive element is more elusive. The only conceivable expression is the manner in which the compiler has selected and arranged the facts. Thus, if the selection and arrangement are original, these elements of the work are eligible for copyright protection. See Patry, Copyright in Compilations of Facts (or Why the “White Pages” Are Not Copyrightable), 12 Com. & Law 37, 64 (Dec. 1990) (hereinafter Patry). No matter how original the format, however, the facts themselves do not become original through association. See Patterson & Joyce 776.

 

This inevitably means that the copyright in a factual compilation is thin. Notwithstanding a valid copyright, a subsequent compiler remains free to use the facts contained in another’s publication to aid in preparing a competing work, so long as the competing work does not feature the same selection and arrangement. As one commentator explains it: “[N]o matter how much original authorship the work displays, the facts and ideas it exposes are free for the taking… . [T]he very same facts and ideas may be divorced from the context imposed by the author, and restated or reshuffled by second comers, even if the author was the first to discover the facts or to propose the ideas.” Ginsburg 1868.

 

It may seem unfair that much of the fruit of the compiler’s labor may be used by others without compensation. As Justice Brennan has correctly observed, however, this is not “some unforeseen byproduct of a statutory scheme.” Harper & Row, 471 U.S., at 589, 105 S.Ct., at 2245 (dissenting opinion). It is, rather, “the essence of copyright,” ibid., and a constitutional requirement. The primary objective of copyright is not to reward the labor of authors, but “[t]o promote the Progress of Science and useful Arts.” Art. I, § 8, cl. 8. Accord, Twentieth Century Music Corp. v. Aiken, 422 U.S. 151, 156, 95 S.Ct. 2040, 2044, 45 L.Ed.2d 84 (1975). To this end, copyright assures authors the right to their original expression, but encourages others to build freely upon the ideas and information conveyed by a work. Harper & Row, supra, 471 U.S., at 556-557, 105 S.Ct., at 2228-2229. This principle, known as the idea/expression or fact/expression dichotomy, applies to all works of authorship. As applied to a factual compilation, assuming the absence of original written expression, only the compiler’s selection and arrangement may be protected; the raw facts may be copied at will. This result is neither unfair nor unfortunate. It is the means by which copyright advances the progress of science and art.

 

This Court has long recognized that the fact/expression dichotomy limits severely the scope of protection in fact-based works. More than a century ago, the Court observed: “The very object of publishing a book on science or the useful arts is to communicate to the world the useful knowledge which it contains. But this object would be frustrated if the knowledge could not be used without incurring the guilt of piracy of the book.” Baker v. Selden, 101 U.S. 99, 103, 25 L.Ed. 841 (1880). We reiterated this point in Harper & Row:

 

 

[N]o author may copyright facts or ideas. The copyright is limited to those aspects of the work – termed ‘expression’ – that display the stamp of the author’s originality. [C]opyright does not prevent subsequent users from copying from a prior author’s work those constituent elements that are not original – for example … facts, or materials in the public domain – as long as such use does not unfairly appropriate the author’s original contributions.

 

 

471 U.S., at 547-548, 105 S.Ct., at 2223-2224 (citation omitted).

 

This, then, resolves the doctrinal tension: Copyright treats facts and factual compilations in a wholly consistent manner. Facts, whether alone or as part of a compilation, are not original and therefore may not be copyrighted. A factual compilation is eligible for copyright if it features an original selection or arrangement of facts, but the copyright is limited to the particular selection or arrangement. In no event may copyright extend to the facts themselves.

 

 

 

 

 

B.

 

 

 

 

 

As we have explained, originality is a constitutionally mandated prerequisite for copyright protection. The Court’s decisions announcing this rule predate the Copyright Act of 1909, but ambiguous language in the 1909 Act caused some lower courts temporarily to lose sight of this requirement.

 

The 1909 Act embodied the originality requirement, but not as clearly as it might have. See Nimmer § 2.01. The subject matter of copyright was set out in §§ 3 and 4 of the Act. Section 4 stated that copyright was available to “all the writings of an author.” 35 Stat. 1076. By using the words “writings” and “author” – the same words used in Article I, § 8, of the Constitution and defined by the Court in The Trade-Mark Cases and Burrow-Giles – the statute necessarily incorporated the originality requirement articulated in the Court’s decisions. It did so implicitly, however, thereby leaving room for error.

 

Section 3 was similarly ambiguous. It stated that the copyright in a work protected only “the copyrightable component parts of the work.” It thus stated an important copyright principle, but failed to identify the specific characteristic originality – that determined which component parts of a work were copyrightable and which were not.

 

Most courts construed the 1909 Act correctly, notwithstanding the less-than-perfect statutory language. They understood from this Court’s decisions that there could be no copyright without originality. See Patterson & Joyce 760-761. As explained in the Nimmer treatise: “The 1909 Act neither defined originality, nor even expressly required that a work be ‘original’ in order to command protection. However, the courts uniformly inferred the requirement from the fact that copyright protection may only be claimed by ‘authors’… . It was reasoned that since an author is ‘the … creator, originator’ it follows that a work is not the product of an author unless the work is original.” Nimmer § 2.01 (footnotes omitted) (citing cases).

 

But some courts misunderstood the statute. See, e.g., Leon v. Pacific Telephone & Telegraph Co., 91 F.2d 484 (CA9 1937); Jeweler’s Circular Publishing Co. v. Keystone Publishing Co., 281 F. 83 (CA2 1922). These courts ignored §§ 3 and 4, focusing their attention instead on § 5 of the Act. Section 5, however, was purely technical in nature: It provided that a person seeking to register a work should indicate on the application the type of work, and it listed 14 categories under which the work might fall. One of these categories was “[b]ooks, including composite and cyclopaedic works, directories, gazetteers, and other compilations.” § 5(a). Section 5 did not purport to say that all compilations were automatically copyrightable. Indeed, it expressly disclaimed any such function, pointing out that “the subject-matter of copyright [i]s defined in section four.” Nevertheless, the fact that factual compilations were mentioned specifically in § 5 led some courts to infer erroneously that directories and the like were copyrightable per se, “without any further or precise showing of original – personal – authorship.” Ginsburg 1895.

 

Making matters worse, these courts developed a new theory to justify the protection of factual compilations. Known alternatively as “sweat of the brow” or “industrious collection,” the underlying notion was that copyright was a reward for the hard work that went into compiling facts. The classic formulation of the doctrine appeared in Jeweler’s Circular Publishing Co., 281 F., at 88:

 

 

The right to copyright a book upon which one has expended labor in its preparation does not depend upon whether the materials which he has collected consist or not of matters which are publici juris, or whether such materials show literary skill or originality, either in thought or in language, or anything more than industrious collection. The man who goes through the streets of a town and puts down the names of each of the inhabitants, with their occupations and their street number, acquires material of which he is the author.

 

 

(emphasis added).

 

The “sweat of the brow” doctrine had numerous flaws, the most glaring being that it extended copyright protection in a compilation beyond selection and arrangement – the compiler’s original contributions – to the facts themselves. Under the doctrine, the only defense to infringement was independent creation. A subsequent compiler was “not entitled to take one word of information previously published,” but rather had to “independently wor[k] out the matter for himself, so as to arrive at the same result from the same common sources of information.” Id., at 88-89 (internal quotations omitted). “Sweat of the brow” courts thereby eschewed the most fundamental axiom of copyright law – that no one may copyright facts or ideas. See Miller v. Universal City Studios, Inc., 650 F.2d, at 1372 (criticizing “sweat of the brow” courts because “ensur[ing] that later writers obtain the facts independently … is precisely the scope of protection given … copyrighted matter, and the law is clearthat facts are not entitled to such protection”).

 

Decisions of this Court applying the 1909 Act make clear that the statute did not permit the “sweat of the brow” approach. The best example is International News Service v. Associated Press, 248 U.S. 215, 39 S.Ct. 68, 63 L.Ed. 211 (1918). In that decision, the Court stated unambiguously that the 1909 Act conferred copyright protection only on those elements of a work that were original to the author. International News Service had conceded taking news reported by Associated Press and publishing it in its own newspapers. Recognizing that § 5 of the Act specifically mentioned ” ‘periodicals, including newspapers,’ ” § 5(b), the Court acknowledged that news articles were copyrightable. Id., at 234, 39 S.Ct., at 70. It flatly rejected, however, the notion that the copyright in an article extended to the factual information it contained: “[T]he news element – the information respecting current events contained in the literary production – is not the creation of the writer, but is a report of matters that ordinarily are publici juris; it is the history of the day.” Ibid.1

 

Without a doubt, the “sweat of the brow” doctrine flouted basic copyright principles. Throughout history, copyright law has “recognize[d] a greater need to disseminate factual works than works of fiction or fantasy.” Harper & Row, 471 U.S., at 563, 105 S.Ct., at 2232. Accord, Gorman, Fact or Fancy: The Implications for Copyright, 29 J. Copyright Soc. 560, 563 (1982). But “sweat of the brow” courts took a contrary view; they handed out proprietary interests in facts and declared that authors are absolutely precluded from saving time and effort by relying upon the facts contained in prior works. In truth, “[i]t is just such wasted effort that the proscription against the copyright of ideas and facts … [is] designed to prevent.” Rosemont Enterprises, Inc. v. Random House, Inc., 366 F.2d 303, 310 (CA2 1966), cert. denied 385 U.S. 1009, 87 S.Ct. 714, 17 L.Ed.2d 546 (1967). “Protection for the fruits of such research … may in certain circumstances be available under a theory of unfair competition. But to accord copyright protection on this basis alone distorts basic copyright principles in that it creates a monopoly in public domain materials without the necessary justification of protecting and encouraging the creation of ‘writings’ by ‘authors.’ ” Nimmer § 3.04, p. 3-23 (footnote omitted).

 

 

 

 

 

C.

 

 

 

 

 

“Sweat of the brow” decisions did not escape the attention of the Copyright Office. When Congress decided to overhaul the copyright statute and asked the Copyright Office to study existing problems, see Mills Music, Inc. v. Snyder, 469 U.S. 153, 159, 105 S.Ct. 638, 642, 83 L.Ed.2d 556 (1985), the Copyright Office promptly recommended that Congress clear up the confusion in the lower courts as to the basic standards of copyrightability. The Register of Copyrights explained in his first report to Congress that “originality” was a “basic requisit[e]” of copyright under the 1909 Act, but that “the absence of any reference to [originality] in the statute seems to have led to misconceptions as to what is copyrightable matter.” Report of the Register of Copyrights on the General Revision of the U.S. Copyright Law, 87th Cong., 1st Sess., p. 9 (H. Judiciary Comm. Print 1961). The Register suggested making the originality requirement explicit. Ibid.

 

Congress took the Register’s advice. In enacting the Copyright Act of 1976, Congress dropped the reference to “all the writings of an author” and replaced it with the phrase “original works of authorship.” 17 U.S.C. § 102(a). In making explicit the originality requirement, Congress announced that it was merely clarifying existing law: “The two fundamental criteria of copyright protection [are] originality and fixation in tangible form… . The phrase ‘original works of authorship,’ which is purposely left undefined, is intended to incorporate without change the standard of originality established by the courts under the present [1909] copyright statute.” H.R.Rep. No. 94-1476, p. 51 (1976) (emphasis added) (hereinafter H.R.Rep.); S.Rep. No. 94-473, p. 50 (1975), U.S.Code Cong. & Admin.News 1976, pp. 5659, 5664 (emphasis added) (hereinafter S.Rep.). This sentiment was echoed by the Copyright Office: “Our intention here is to maintain the established standards of originality… .” Supplementary Report of the Register of Copyrights on the General Revision of U.S. Copyright Law, 89th Cong., 1st Sess., pt. 6, p. 3 (H. Judiciary Comm. Print 1965) (emphasis added).

 

To ensure that the mistakes of the “sweat of the brow” courts would not be repeated, Congress took additional measures. For example, § 3 of the 1909 Act had stated that copyright protected only the “copyrightable component parts” of a work, but had not identified originality as the basis for distinguishing those component parts that were copyrightable from those that were not. The 1976 Act deleted this section and replaced it with § 102(b), which identifies specifically those elements of a work for which copyright is not available: “In no case does copyright protection for an original work of authorship extend to any idea, procedure, process, system, method of operation, concept, principle, or discovery, regardless of the form in which it is described, explained, illustrated, or embodied in such work.” Section 102(b) is universally understood to prohibit any copyright in facts. Harper & Row, supra, at 547, 556, 105 S.Ct., at 2223, 2228. Accord, Nimmer § 2.03[E] (equating facts with “discoveries”). As with § 102(a), Congress emphasized that § 102(b) did not change the law, but merely clarified it: “Section 102(b) in no way enlarges or contracts the scope of copyright protection under the present law. Its purpose is to restate … that the basic dichotomy between expression and idea remains unchanged.” H.R.Rep., at 57; S.Rep., at 54, U.S.Code Cong. & Admin.News 1976, p. 5670.

 

Congress took another step to minimize confusion by deleting the specific mention of “directories … and other compilations” in § 5 of the 1909 Act. As mentioned, this section had led some courts to conclude that directories were copyrightable per se and that every element of a directory was protected. In its place, Congress enacted two new provisions. First, to make clear that compilations were not copyrightable per se, Congress provided a definition of the term “compilation.” Second, to make clear that the copyright in a compilation did not extend to the facts themselves, Congress enacted § 103.

 

The definition of “compilation” is found in § 101 of the 1976 Act. It defines a “compilation” in the copyright sense as “a work formed by the collection and assembling of preexisting materials or of data that are selected, coordinated, or arranged in such a way that the resulting work as a whole constitutes an original work of authorship” (emphasis added).

 

The purpose of the statutory definition is to emphasize that collections of facts are not copyrightable per se. It conveys this message through its tripartite structure, as emphasized above by the italics. The statute identifies three distinct elements and requires each to be met for a work to qualify as a copyrightable compilation: (1) the collection and assembly of pre-existing material, facts, or data; (2) the selection, coordination, or arrangement of those materials; and (3) the creation, by virtue of the particular selection, coordination, or arrangement, of an “original” work of authorship. “[T]his tripartite conjunctive structure is self-evident, and should be assumed to ‘accurately express the legislative purpose.’ ” Patry 51, quoting Mills Music, 469 U.S., at 164, 105 S.Ct., at 645.

 

At first glance, the first requirement does not seem to tell us much. It merely describes what one normally thinks of as a compilation – a collection of pre-existing material, facts, or data. What makes it significant is that it is not the sole requirement. It is not enough for copyright purposes that an author collects and assembles facts. To satisfy the statutory definition, the work must get over two additional hurdles. In this way, the plain language indicates that not every collection of facts receives copyright protection. Otherwise, there would be a period after “data.”

 

The third requirement is also illuminating. It emphasizes that a compilation, like any other work, is copyrightable only if it satisfies the originality requirement (“an original work of authorship”). Although § 102 states plainly that the originality requirement applies to all works, the point was emphasized with regard to compilations to ensure that courts would not repeat the mistake of the “sweat of the brow” courts by concluding that fact-based works are treated differently and measured by some other standard. As Congress explained it, the goal was to “make plain that the criteria of copyrightable subject matter stated in section 102 apply with full force to works … containing preexisting material.” H.R.Rep., at 57; S.Rep., at 55, U.S.Code Cong. & Admin.News 1976, p. 5670.

 

The key to the statutory definition is the second requirement. It instructs courts that, in determining whether a fact-based work is an original work of authorship, they should focus on the manner in which the collected facts have been selected, coordinated, and arranged. This is a straightforward application of the originality requirement. Facts are never original, so the compilation author can claim originality, if at all, only in the way the facts are presented. To that end, the statute dictates that the principal focus should be on whether the selection, coordination, and arrangement are sufficiently original to merit protection.

 

Not every selection, coordination, or arrangement will pass muster. This is plain from the statute. It states that, to merit protection, the facts must be selected, coordinated, or arranged “in such a way” as to render the work as a whole original. This implies that some “ways” will trigger copyright, but that others will not. See Patry 57, and n. 76. Otherwise, the phrase “in such a way” is meaningless and Congress should have defined “compilation” simply as “a work formed by the collection and assembly of preexisting materials or data that are selected, coordinated, or arranged.” That Congress did not do so is dispositive. In accordance with “the established principle that a court should give effect, if possible, to every clause and word of a statute,” Moskal v. United States, 498 U.S. 103, 109-110, 111 S.Ct. 461, 466, 112 L.Ed.2d 449 (1990) (internal quotation marks omitted), we conclude that the statute envisions that there will be some fact-based works in which the selection, coordination, and arrangement are not sufficiently original to trigger copyright protection.

 

As discussed earlier, however, the originality requirement is not particularly stringent. A compiler may settle upon a selection or arrangement that others have used; novelty is not required. Originality requires only that the author make the selection or arrangement independently (i.e., without copying that selection or arrangement from another work), and that it display some minimal level of creativity. Presumably, the vast majority of compilations will pass this test, but not all will. There remains a narrow category of works in which the creative spark is utterly lacking or so trivial as to be virtually nonexistent. See generally Bleistein v. Donaldson Lithographing Co., 188 U.S. 239, 251, 23 S.Ct. 298, 300, 47 L.Ed. 460 (1903) (referring to “the narrowest and most obvious limits”). Such works are incapable of sustaining a valid copyright. Nimmer § 2.01[B].

 

Even if a work qualifies as a copyrightable compilation, it receives only limited protection. This is the point of § 103 of the Act. Section 103 explains that “[t]he subject matter of copyright … includes compilations,” § 103(a), but that copyright protects only the author’s original contributions – not the facts or information conveyed:

 

 

The copyright in a compilation … extends only to the material contributed by the author of such work, as distinguished from the preexisting material employed in the work, and does not imply any exclusive right in the preexisting material.

 

 

§ 103(b).

 

As § 103 makes clear, copyright is not a tool by which a compilation author may keep others from using the facts or data he or she has collected. “The most important point here is one that is commonly misunderstood today: copyright … has no effect one way or the other on the copyright or public domain status of the preexisting material.” H.R.Rep., at 57; S.Rep., at 55, U.S.Code Cong. & Admin. News 1976, p. 5670. The 1909 Act did not require, as “sweat of the brow” courts mistakenly assumed, that each subsequent compiler must start from scratch and is precluded from relying on research undertaken by another. See, e.g., Jeweler’s Circular Publishing Co., 281 F., at 88-89. Rather, the facts contained in existing works may be freely copied because copyright protects only the elements that owe their origin to the compiler – the selection, coordination, and arrangement of facts.

 

In summary, the 1976 revisions to the Copyright Act leave no doubt that originality, not “sweat of the brow,” is the touchstone of copyright protection in directories and other fact-based works. Nor is there any doubt that the same was true under the 1909 Act. The 1976 revisions were a direct response to the Copyright Office’s concern that many lower courts had misconstrued this basic principle, and Congress emphasized repeatedly that the purpose of the revisions was to clarify, not change, existing law. The revisions explain with painstaking clarity that copyright requires originality, § 102(a); that facts are never original, § 102(b); that the copyright in a compilation does not extend to the facts it contains, § 103(b); and that a compilation is copyrightable only to the extent that it features an original selection, coordination, or arrangement, § 101.

 

The 1976 revisions have proven largely successful in steering courts in the right direction. A good example is Miller v. Universal City Studios, Inc., 650 F.2d, at 1369-1370: “A copyright in a directory … is properly viewed as resting on the originality of the selection and arrangement of the factual material, rather than on the industriousness of the efforts to develop the information. Copyright protection does not extend to the facts themselves, and the mere use of information contained in a directory without a substantial copying of the format does not constitute infringement” (citation omitted). Additionally, the Second Circuit, which almost 70 years ago issued the classic formulation of the “sweat of the brow” doctrine in Jeweler’s Circular Publishing Co., has now fully repudiated the reasoning of that decision. See, e.g., Financial Information, Inc. v. Moody’s Investors Service, Inc., 808 F.2d 204, 207 (CA2 1986), cert. denied, 484 U.S. 820, 108 S.Ct. 79, 98 L.Ed.2d 42 (1987); Financial Information, Inc. v. Moody’s Investors Service, Inc., 751 F.2d 501, 510 (CA2 1984) (Newman, J., concurring); Hoehling v. Universal City Studios, Inc., 618 F.2d 972, 979 (CA2 1980). Even those scholars who believe that “industrious collection” should be rewarded seem to recognize that this is beyond the scope of existing copyright law. See Denicola 516 (“[T]he very vocabulary of copyright is ill suited to analyzing property rights in works of nonfiction”); id., at 520-521, 525; Ginsburg 1867, 1870.

 

 

 

 

III

 

 

 

 

There is no doubt that Feist took from the white pages of Rural’s directory a substantial amount of factual information. At a minimum, Feist copied the names, towns, and telephone numbers of 1,309 of Rural’s subscribers. Not all copying, however, is copyright infringement. To establish infringement, two elements must be proven: (1) ownership of a valid copyright, and (2) copying of constituent elements of the work that are original. See Harper & Row, 471 U.S., at 548, 105 S.Ct., at 2224. The first element is not at issue here; Feist appears to concede that Rural’s directory, considered as a whole, is subject to a valid copyright because it contains some foreword text, as well as original material in its yellow pages advertisements. See Brief for Petitioner 18; Pet. for Cert. 9.

 

The question is whether Rural has proved the second element. In other words, did Feist, by taking 1,309 names, towns, and telephone numbers from Rural’s white pages, copy anything that was “original” to Rural? Certainly, the raw data does not satisfy the originality requirement. Rural may have been the first to discover and report the names, towns, and telephone numbers of its subscribers, but this data does not ” ‘ow[e] its origin’ ” to Rural. Burrow-Giles, 111 U.S., at 58, 4 S.Ct., at 281. Rather, these bits of information are uncopyrightable facts; they existed before Rural reported them and would have continued to exist if Rural had never published a telephone directory. The originality requirement “rule[s] out protecting … names, addresses, and telephone numbers of which the plaintiff by no stretch of the imagination could be called the author.” Patterson & Joyce 776.

 

Rural essentially concedes the point by referring to the names, towns, and telephone numbers as “preexisting material.” Brief for Respondent 17. Section 103(b) states explicitly that the copyright in a compilation does not extend to “the preexisting material employed in the work.”

 

The question that remains is whether Rural selected, coordinated, or arranged these uncopyrightable facts in an original way. As mentioned, originality is not a stringent standard; it does not require that facts be presented in an innovative or surprising way. It is equally true, however, that the selection and arrangement of facts cannot be so mechanical or routine as to require no creativity whatsoever. The standard of originality is low, but it does exist. See Patterson & Joyce 760, n. 144 (“While this requirement is sometimes characterized as modest, or a low threshold, it is not without effect”) (internal quotation marks omitted; citations omitted). As this Court has explained, the Constitution mandates some minimal degree of creativity, see The Trade-Mark Cases, 100 U.S., at 94; and an author who claims infringement must prove “the existence of … intellectual production, of thought, and conception.” Burrow-Giles, supra, 111 U.S., at 59-60, 4 S.Ct., at 281-282.

 

The selection, coordination, and arrangement of Rural’s white pages do not satisfy the minimum constitutional standards for copyright protection. As mentioned at the outset, Rural’s white pages are entirely typical. Persons desiring telephone service in Rural’s service area fill out an application and Rural issues them a telephone number. In preparing its white pages, Rural simply takes the data provided by its subscribers and lists it alphabetically by surname. The end product is a garden-variety white pages directory, devoid of even the slightest trace of creativity.

 

Rural’s selection of listings could not be more obvious: It publishes the most basic information – name, town, and telephone number – about each person who applies to it for telephone service. This is “selection” of a sort, but it lacks the modicum of creativity necessary to transform mere selection into copyrightable expression. Rural expended sufficientef fort to make the white pages directory useful, but insufficient creativity to make it original.

 

We note in passing that the selection featured in Rural’s white pages may also fail the originality requirement for another reason. Feist points out that Rural did not truly “select” to publish the names and telephone numbers of its subscribers; rather, it was required to do so by the Kansas Corporation Commission as part of its monopoly franchise. See 737 F.Supp., at 612. Accordingly, one could plausibly conclude that this selection was dictated by state law, not by Rural.

 

Nor can Rural claim originality in its coordination and arrangement of facts. The white pages do nothing more than list Rural’s subscribers in alphabetical order. This arrangement may, technically speaking, owe its origin to Rural; no one disputes that Rural undertook the task of alphabetizing the names itself. But there is nothing remotely creative about arranging names alphabetically in a white pages directory. It is an age-old practice, firmly rooted in tradition and so commonplace that it has come to be expected as a matter of course. See Brief for Information Industry Association et al. as Amici Curiae 10 (alphabetical arrangement “is universally observed in directories published by local exchange telephone companies”). It is not only unoriginal, it is practically inevitable. This time-honored tradition does not possess the minimal creative spark required by the Copyright Act and the Constitution.

 

We conclude that the names, towns, and telephone numbers copied by Feist were not original to Rural and therefore were not protected by the copyright in Rural’s combined white and yellow pages directory. As a constitutional matter, copyright protects only those constituent elements of a work that possess more than a de minimis quantum of creativity. Rural’s white pages, limited to basic subscriber information and arranged alphabetically, fall short of the mark. As a statutory matter, 17 U.S.C. § 101 does not afford protection from copying to a collection of facts that are selected, coordinated, and arranged in a way that utterly lacks originality. Given that some works must fail, we cannot imagine a more likely candidate. Indeed, were we to hold that Rural’s white pages pass muster, it is hard to believe that any collection of facts could fail.

 

Because Rural’s white pages lack the requisite originality, Feist’s use of the listings cannot constitute infringement. This decision should not be construed as demeaning Rural’s efforts in compiling its directory, but rather as making clear that copyright rewards originality, not effort. As this Court noted more than a century ago, ” ‘great praise may be due to the plaintiffs for their industry and enterprise in publishing this paper, yet the law does not contemplate their being rewarded in this way.’ ” Baker v. Selden, 101 U.S., at 105.

 

The judgment of the Court of Appeals is

 

Reversed.

 

Justice Blackmun concurs in the judgment.

 


  1. The Court ultimately rendered judgment for Associated Press on noncopyright grounds that are not relevant here. See 248 U.S., at 235, 241-242, 39 S.Ct., at 71, 73-74.

 

 

 

3.4.2. Calibration: Scope of Exclusion, a.k.a. Fair Use

 

Suntrust Bank v. Houghton Mifflin Co.,

268 F.3d 1257 (11th Cir. 2001)

 

 

 

Joseph M. Beck, Miles J. Alexander, Jerry Bailey Swann, Kilpatrick Stockton & Cody, Atlanta, Georgia, for Appellant.

 

Richard Kurnit, New York City, William B. Smith, Ralph R. Morrison, Anne Moody Johnson, Jones, Day, Reavis & Pogue, Atlanta, Georgia, for Appellee.

 

Leon Friedman, New York City, for Pen American Ctr., American Booksellers Foundation for Freedom of Exp., Freedom to Read Foundation, Washington Lawyers’ for the Arts, The First Amendment Project and National Coalition Against Censorship, Amicus Curiae,.

 

E. Edward Bruce, Covington & Burling, Washington, DC, for Microsoft Corp., Amicus Curiae,.

 

Hollie Manheimer, Decatur, Georgia, for Amicus Curiae.

 

 

 

Before Birch, Marcus and Wood1, Circuit Judges.

 

 

 

Birch, Circuit Judge:

 

 

 

In this opinion, we decide whether publication of The Wind Done Gone (“TWDG”), a fictional work admittedly based on Margaret Mitchell’s Gone With the Wind (“GWTW”), should be enjoined from publication based on alleged copyright violations. The district court granted a preliminary injunction against publication of TWDG because it found that Plaintiff-Appellee SunTrust Bank (“SunTrust”) met the four-part test governing preliminary injunctions. We VACATE the injunction and REMAND for consideration of the remaining claims.

 

 

 

I. BACKGROUND

 

 

 

 

A. Procedural History

 

 

 

 

SunTrust is the trustee of the Mitchell Trust, which holds the copyright in GWTW. Since its publication in 1936, GWTW has become one of the best-selling books in the world, second in sales only to the Bible. The Mitchell Trust has actively managed the copyright, authorizing derivative works and a variety of commercial items. It has entered into a contract authorizing, under specified conditions, a second sequel to GWTW to be published by St. Martin’s Press. The Mitchell Trust maintains the copyright in all of the derivative works as well. See 17 U.S.C. § 103.2

 

Alice Randall, the author of TWDG, persuasively claims that her novel is a critique of GWTW’s depiction of slavery and the Civil-War era American South. To this end, she appropriated the characters, plot and major scenes from GWTW into the first half of TWDG. According to SunTrust, TWDG “(1) explicitly refers to [GWTW] in its foreword; (2) copies core characters, character traits, and relationships from [GWTW]; (3) copies and summarizes famous scenes and other elements of the plot from [GWTW]; and (4) copies verbatim dialogues and descriptions from [GWTW].” SunTrust Bank v. Houghton Mifflin Co., 136 F. Supp. 2d 1357, 1364 (N.D.Ga. 2001), vacated, 252 F.3d 1165 (11th Cir. 2001). Defendant-Appellant Houghton Mifflin, the publisher of TWDG, does not contest the first three allegations,3 but nonetheless argues that there is no substantial similarity between the two works or, in the alternative, that the doctrine of fair use protects TWDG because it is primarily a parody of GWTW.

 

After discovering the similarities between the books, SunTrust asked Houghton Mifflin to refrain from publication or distribution of TWDG, but Houghton Mifflin refused the request. Subsequently, SunTrust filed an action alleging copyright infringement, violation of the Lanham Act, and deceptive trade practices, and immediately filed a motion for a temporary restraining order and a preliminary injunction.

 

After a hearing, the district court granted the motion, preliminarily enjoining Houghton Mifflin from “further production, display, distribution, advertising, sale, or offer for sale of” TWDG. SunTrust Bank, 136 F. Supp. 2d at 1386. In a thorough opinion, the court found that “the defendant’s publication and sale of [TWDG would] infringe the plaintiff’s copyright interests as protected under the copyright laws.” Id. Houghton Mifflin appealed. At oral argument, we issued an order vacating the injunction on the grounds that it was an unconstitutional prior restraint. SunTrust Bank v. Houghton Mifflin Co., 252 F. 3d 1165 (11th Cir. 2001). We now vacate that order and issue this more comprehensive opinion.

 

 

 

 

B. Standard of Review

 

 

 

 

“We review the district court’s grant of a preliminary injunction for abuse of discretion.” Warren Pub., Inc. v. Microdos Data Corp., 115 F.3d 1509, 1516 (11th Cir. 1997) (en banc). We review decisions of law de novo and findings of fact for clear error. Mitek Holdings, Inc. v. Arce Eng’g Co., Inc., 89 F.3d 1548, 1554 (11th Cir. 1996).

 

 

 

II. DISCUSSION

 

 

 

Our primary focus at this stage of the case is on the appropriateness of the injunctive relief granted by the district court. In our analysis, we must evaluate the merits of SunTrust’s copyright infringement claim, including Houghton Mifflin’s affirmative defense of fair use.4 As we assess the fair-use defense, we examine to what extent a critic may use a work to communicate her criticism of the work without infringing the copyright in that work. To approach these issues in the proper framework, we should initially review the history of the Constitution’s Copyright Clause and understand its relationship to the First Amendment.

 

 

 

 

A. History and Development of the Copyright Clause

 

 

 

 

The Copyright Clause finds its roots in England, where, in 1710, the Statute of Anne “was designed to destroy the booksellers’ monopoly of the booktrade and to prevent its recurrence.” L. Ray Patterson, Understanding the Copyright Clause, 47 J. Copyright Soc’y USA 365, 379 (2000). This Parliamentary statute assigned copyright in books to authors, added a requirement that only a new work could be copyrighted, and limited the duration, which had been perpetual, to two fourteen-year terms. It is clear that the goal of the Statute of Anne was to encourage creativity and ensure that the public would have free access to information by putting an end to “the continued use of copyright as a device of censorship.” Patterson at 379.5 The Framers of the U.S. Constitution relied on this statute when drafting the Copyright Clause of our Constitution, which reads,

 

 

The Congress shall have Power … to promote the Progress of Science … by securing for limited Times to Authors … the exclusive Right to their respective Writings … .

 

 

U.S. Const. art. 1, § 8, cl. 8. Congress directly transferred the principles from the Statute of Anne into the copyright law of the United States in 1783, first through a recommendation to the states to enact similar copyright laws, and then in 1790, with the passage of the first American federal copyright statute.

 

The Copyright Clause was intended “to be the engine of free expression.” Harper & Row Publishers, Inc. v. Nation Enters., 471 U.S. 539, 558 (1985). To that end, copyright laws have been enacted achieve the three main goals: the promotion of learning, the protection of the public domain, and the granting of an exclusive right to the author.

 

 

 

 

 

1. Promotion of Learning

 

 

 

 

 

In the United States, copyright has always been used to promote learning by guarding against censorship. Throughout the nineteenth century, the copyright in literature was limited to the right “to publish and vend books.” Patterson, at 383. The term “copy” was interpreted literally; an author had the right only to prevent others from copying and selling her particular literary work. See Stowe v. Thomas, 23 F. Cas. 201 (C.C.E.D. Pa. 1853) (holding that a translation of Uncle Tom’s Cabin into German was not a copyright infringement because it was not a copy of the work as it was published).6 This limited right ensured that a maximum number of new works would be created and published. It was not until the 1909 Act, which codified the concept of a derivative work, that an author’s right to protect his original work against imitation was established. This change more closely represents current statutory copyright law and is consistent with copyright’s constitutional mandate.

 

As a further protection of the public interest, until 1976, statutory copyright law required that a work be published before an author was entitled to a copyright in that work. Therefore, in order to have the sole right of publication for the statutory period, the author was first required to make the work available to the public. In 1976, copyright was extended to include any work “fixed in any tangible medium of expression” in order to adapt the law to technological advances. § 102(a). Thus, the publication requirement was removed, but the fair use right was codified to maintain the constitutionally mandated balance to ensure that the public has access to knowledge.

 

The Copyright Act promotes public access to knowledge because it provides an economic incentive for authors to publish books and disseminate ideas to the public. Harper & Row, 471 U.S. at 558 (“By establishing a marketable right to the use of one’s expression, copyright supplies the economic incentive to create and disseminate ideas.”). The Supreme Court has recognized that “[t]he monopoly created by copyright thus rewards the individual author in order to benefit the public.” Id. at 546. Without the limited monopoly, authors would have little economic incentive to create and publish their work. Therefore, by providing this incentive, the copyright law promotes the public access to new ideas and concepts.

 

 

 

 

 

2. Protection of the Public Domain

 

 

 

 

 

The second goal of the Copyright Clause is to ensure that works enter the public domain after an author’s rights, exclusive, but limited, have expired. Parallel to the patent regime, the limited time period of the copyright serves the dual purpose of ensuring that the work will enter the public domain and ensuring that the author has received “a fair return for [her] labors.” Harper & Row, 471 U.S. at 546. This limited grant “is intended to motivate the creative activity of authors … by the provision of a special reward, and to allow the public access to the products of their genius after the limited period of exclusive control has expired.” Sony, 464 U.S. at 429. The public is protected in two ways: the grant of a copyright encourages authors to create new works, as discussed in section II.A.1., and the limitation ensures that the works will eventually enter the public domain, which protects the public’s right of access and use.

 

 

 

 

 

3. Exclusive Rights of the Author

 

 

 

 

 

Finally, the Copyright Clause grants the author limited exclusive rights in order to encourage the creation of original works. Before our copyright jurisprudence developed, there were two separate theories of copyright in England – the natural law copyright, which was the right of first publication, and the statutory copyright, which was the right of continued publication. The natural law copyright, which is not a part of our system, implied an ownership in the work itself, and thus was preferred by the booksellers and publishers striving to maintain their monopoly over literature as well as by the Crown to silence “seditious” writings. Even after passage of the Statute of Anne, the publishers and booksellers resisted the loss of their monopoly in the courts for more than sixty years. Finally, in 1774, the House of Lords ruled that the natural law copyright, that is, the ownership of the work itself, expires upon publication of the book, when the statutory copyright attaches. Patterson at 382.

 

This bifurcated system was carried over into our copyright law. As of the 1909 Act, an author had “state common law protection [that] persisted until the moment of general publication.” Estate of Martin Luther King, Jr. v. CBS, Inc., 194 F.3d 1211, 1214 (11th Cir. 1999). After the work was published, the author was entitled to federal statutory copyright protection if she had complied with certain federal requirements (i.e. publication with notice). If not, the work was released into the public domain. Id. The system illustrates that the author’s ownership is in the copyright, and not in the work itself, for if the author had an ownership interest in the work itself, she would not lose that right if she published the book without complying with federal statutory copyright requirements. Compliance with the copyright law results in the guarantee of copyright to the author for a limited time, but the author never owns the work itself. § 202 (“Ownership of a copyright, or of any of the exclusive rights under a copyright, is distinct from ownership of any material object in which the work is embodied.”).

 

This has an important impact on modern interpretation of copyright, as it emphasizes the distinction between ownership of the work, which an author does not possess, and ownership of the copyright, which an author enjoys for a limited time. In a society oriented toward property ownership, it is not surprising to find many that erroneously equate the work with the copyright in the work and conclude that if one owns the copyright, they must also own the work. However, the fallacy of that understanding is exposed by the simple fact that the work continues to exist after the term of copyright associated with the work has expired. “The copyright is not a natural right inherent in authorship. If it were, the impact on market values would be irrelevant; any unauthorized taking would be obnoxious.” Pierre Leval, Towards a Fair Use Standard, 105 Harv. L. Rev. 1105, 1124 (1990).

 

 

 

B. The Union of Copyright and the First Amendment

 

 

 

The Copyright Clause and the First Amendment, while intuitively in conflict,7 were drafted to work together to prevent censorship; copyright laws were enacted in part to prevent private censorship and the First Amendment was enacted to prevent public censorship. There are “[c]onflicting interests that must be accommodated in drawing a definitional balance” between the Copyright Clause and the First Amendment. 1 Nimmer § 1.10[B][1]. In establishing this balance “[o]n the copyright side, economic encouragement for creators must be preserved and the privacy of unpublished works recognized. Freedom of speech[, on the other hand,] requires the preservation of a meaningful public or democratic dialogue, as well as the uses of speech as a safety valve against violent acts, and as an end in itself.” Id.

 

In copyright law, the balance between the First Amendment and copyright is preserved, in part, by the idea/expression dichotomy and the doctrine of fair use.

 

 

 

 

 

1. The Idea/Expression Dichotomy

 

 

 

 

 

Copyright cannot protect an idea, only the expression of that idea. The result is that “copyright assures authors the right to their original expression, but encourages others to build freely upon the ideas and information conveyed by the work.” Feist, 499 U.S. at 349-50. It is partly through this idea/expression dichotomy that copyright law embodies the First Amendment’s underlying goal of encouraging open debate and the free exchange of ideas. Holding an infringer liable in copyright for copying the expression of another author’s ideas does not impede First Amendment goals because the public purpose has been served – the public already has access to the idea or the concepts. A new author may use or discuss the idea, but must do so using her own original expression.

 

 

 

 

 

2. Fair Use

 

 

 

 

 

First Amendment privileges are also preserved through the doctrine of fair use. Until codification of the fair-use doctrine in the 1976 Act, fair use was a judge-made right developed to preserve the constitutionality of copyright legislation by protecting First Amendment values. Had fair use not been recognized as a right under the 1976 Act, the statutory abandonment of publication as a condition of copyright that had existed for over 200 years would have jeopardized the constitutionality of the new Act because there would be no statutory guarantee that new ideas, or new expressions of old ideas, would be accessible to the public. Included in the definition of fair use are “purposes such as criticism, comment, news reporting, teaching …, scholarship, or research.” § 107. The exceptions carved out for these purposes are at the heart of fair use’s protection of the First Amendment, as they allow later authors to use a previous author’s copyright to introduce new ideas or concepts to the public. Therefore, within the limits of the fair-use test, any use of a copyright is permitted to fulfill one of the important purposes listed in the statute.

 

Because of the First Amendment principles built into copyright law through the idea/expression dichotomy and the doctrine of fair use, courts often need not entertain related First Amendment arguments in a copyright case. [citations omitted]

 

The case before us calls for an analysis of whether a preliminary injunction was properly granted against an alleged infringer who, relying largely on the doctrine of fair use, made use of another’s copyright for comment and criticism. As discussed herein, copyright does not immunize a work from comment and criticism. Therefore, the narrower question in this case is to what extent a critic may use the protected elements of an original work of authorship to communicate her criticism without infringing the copyright in that work. As will be discussed below, this becomes essentially an analysis of the fair use factors. As we turn to the analysis required in this case, we must remain cognizant of the First Amendment protections interwoven into copyright law.

 

 

 

 

C. Appropriateness of Injunctive Relief

 

 

 

 

“The chief function of a preliminary injunction is to preserve the status quo until the merits of the controversy can be fully and fairly adjudicated.” The Copyright Act specifically vests the federal courts with power to grant injunctions “to prevent or restrain infringement of a copyright.” § 502(a). While injunctive relief may be particularly appropriate in cases involving simple copying or “piracy” of a copyrighted work, the Supreme Court has cautioned that such relief may not be consistent with the goals of copyright law in cases in which the alleged infringer of the copyright has a colorable fair-use defense.

 

The basic framework for our analysis remains, however, the standard test governing the issuance of preliminary injunctions. SunTrust is not entitled to relief in the form of a preliminary injunction unless it has proved each of the following four elements: “(1) a substantial likelihood of success on the merits, (2) a substantial threat of irreparable injury if the injunction were not granted, (3) that the threatened injury to the plaintiff outweighs the harm an injunction may cause the defendant, and (4) that granting the injunction would not disserve the public interest.”

 

 

 

 

 

1. Substantial Likelihood of Success on the Merits

 

 

 

 

 

 

 

 

a. Prima Facie Copyright Infringement

 

 

 

 

 

 

The first step in evaluating the likelihood that SunTrust will succeed on the merits is to determine whether it has established the prima facie elements of a copyright infringement claim: (1) that SunTrust owns a valid copyright in GWTW and (2) that Randall copied original elements of GWTW in TWDG. The district court found that SunTrust had carried its burden on both of these elements.

 

The first element, SunTrust’s ownership of a valid copyright in GWTW, is not disputed. Houghton Mifflin does assert, however, that SunTrust did not establish the second element of infringement, that TWDG appropriates copyright-protected expression from GWTW. In order to prove copying, SunTrust was required to show a “substantial similarity” between the two works such that “an average lay observer would recognize the alleged copy as having been appropriated from the copyrighted work.” Not all copying of a work is actionable, however, for, as discussed in section II.B.1., “no author may copyright facts or ideas. The copyright is limited to those aspects of the work-termed ‘expression’-that display the stamp of the author’s originality.” Thus, we are concerned with substantial similarities between TWDG and GWTW only to the extent that they involve the copying of original, protected expression.8

 

There is no bright line that separates the protectable expression from the nonprotectable idea in a work of fiction. While often referred to as a test for distinguishing the idea from the expression, Judge Learned Hand’s famous statement in Nichols v. Universal Pictures Corp., 45 F.2d 119 (2d Cir. 1930), is actually nothing more than a concise restatement of the problem facing the courts:

 

 

Upon any work, and especially upon a play, a great number of patterns of increasing generality will fit equally well, as more and more of the incident is left out. The last may perhaps be no more than the most general statement of what the play is about, and at time might consist only of its title; but there is a point in this series of abstractions where they are no longer protected, since otherwise the playwright could prevent the use of his ‘ideas,’ to which, apart from their expression, his property is never extended.

 

 

Id. at 121. At one end of the spectrum, scenes a faire – the stock scenes and hackneyed character types that “naturally flow from a common theme” – are considered “ideas,” and therefore are not copyrightable. Beal v. Paramount Pictures Corp., 20 F.3d 454, 459-60 (11th Cir. 1994). But as plots become more intricately detailed and characters become more idiosyncratic, they at some point cross the line into “expression” and are protected by copyright.

 

After conducting a thorough comparison of the two works, the district court found that TWDG copied far more than unprotected scenes a faire from GWTW: “[TWDG] uses fifteen fictional characters from [GWTW], incorporating their physical attributes, mannerisms, and the distinct features that Ms. Mitchell used to describe them, as well as their complex relationships with each other. Moreover, the various [fictional] locales, … settings, characters, themes, and plot of [TWDG] closely mirror those contained in [GWTW].” SunTrust, 136 F.Supp.2d at 1367.

 

Our own review of the two works reveals substantial use of GWTW. TWDG appropriates numerous characters, settings, and plot twists from GWTW. For example, Scarlett O’Hara, Rhett Butler, Bonnie Butler, Melanie Wilkes, Ashley Wilkes, Gerald O’Hara, Ellen O’Hara, Mammy, Pork, Dilcey, Prissy, Belle Watling, Carreen O’Hara, Stuart and Brenton Tarleton, Jeems, Philippe, and Aunt Pittypat, all characters in GWTW, appear in TWDG. Many of these characters are renamed in TWDG: Scarlett becomes “Other,” Rhett Butler becomes “R.B.,” Pork becomes “Garlic,” Prissy becomes “Miss Priss,” Philippe becomes “Feleepe,” Aunt Pittypat becomes “Aunt Pattypit,” etc. In several instances, Randall renamed characters using Mitchell’s descriptions of those characters in GWTW: Ashley becomes “Dreamy Gentleman,” Melanie becomes “Mealy Mouth,” Gerald becomes “Planter.” The fictional settings from GWTW receive a similarly transparent renaming in TWDG: Tara becomes “Tata,” Twelve Oaks Plantation becomes “Twelve Slaves Strong as Trees.” TWDG copies, often in wholesale fashion, the descriptions and histories of these fictional characters and places from GWTW, as well as their relationships and interactions with one another. TWDG appropriates or otherwise explicitly references many aspects of GWTW’s plot as well, such as the scenes in which Scarlett kills a Union soldier and the scene in which Rhett stays in the room with his dead daughter Bonnie, burning candles. After carefully comparing the two works, we agree with the district court that, particularly in its first half, TWDG is largely “an encapsulation of [GWTW] [that] exploit[s] its copyrighted characters, story lines, and settings as the palette for the new story.” SunTrust, 136 F.Supp.2d at 1367.

 

Houghton Mifflin argues that there is no substantial similarity between TWDG and GWTW because the retelling of the story is an inversion of GWTW: the characters, places, and events lifted from GWTW are often cast in a different light, strong characters from the original are depicted as weak (and vice-versa) in the new work, the institutions and values romanticized in GWTW are exposed as corrupt in TWDG. While we agree with Houghton Mifflin that the characters, settings, and plot taken from GWTW are vested with a new significance when viewed through the character of Cynara9 in TWDG, it does not change the fact that they are the very same copyrighted characters, settings, and plot.

 

 

 

 

 

 

b. Fair Use

 

 

 

 

 

 

Randall’s appropriation of elements of GWTW in TWDG may nevertheless not constitute infringement of SunTrust’s copyright if the taking is protected as a “fair use.” The codification of the fair-use doctrine in the Copyright Act provides:

 

 

Notwithstanding the provisions of sections 106 and 106A, the fair use of a copyrighted work … for purposes such as criticism, comment, news reporting, teaching (including multiple copies for classroom use), scholarship, or research, is not an infringement of copyright. In determining whether the use made of a work in any particular case is a fair use the factors to be considered shall include

 

 

(1) the purpose and character of the use, including whether such use is of a commercial nature or is for nonprofit educational purposes;

 

 

 

(2) the nature of the copyrighted work;

 

 

 

(3) the amount and substantiality of the portion used in relation to the copyrighted work as a whole; and

 

 

 

(4) the effect of the use upon the potential market for or value of the copyrighted work.

 

 

 

§ 107. In assessing whether a use of a copyright is a fair use under the statute, we bear in mind that the examples of possible fair uses given are illustrative rather than exclusive, and that “[a]ll [of the four factors] are to be explored, and the results weighed together in light of the purposes of copyright.” Campbell, 510 U.S. at 577-78. In light of the discussion in §§ IIA and B, one of the most important purposes to consider is the free flow of ideas – particularly criticism and commentary.

 

Houghton Mifflin argues that TWDG is entitled to fair-use protection as a parody of GWTW. In Campbell, the Supreme Court held that parody, although not specifically listed in § 107, is a form of comment and criticism that may constitute a fair use of the copyrighted work being parodied. Id. at 579. Parody, which is directed toward a particular literary or artistic work, is distinguishable from satire, which more broadly addresses the institutions and mores of a slice of society. Id. at 580-81, 581 n.15. Thus, “[p]arody needs to mimic an original to make its point, and so has some claim to use the creation of its victim’s … imagination, whereas satire can stand on its own two feet and so requires justification for the very act of borrowing.” Id. at 580-81.

 

The fact that parody by definition must borrow elements from an existing work, however, does not mean that every parody is shielded from a claim of copyright infringement as a fair use. “The [Copyright] Act has no hint of an evidentiary preference for parodists over their victims, and no workable presumption for parody could take account of the fact that parody often shades into satire when society is lampooned through its creative artifacts, or that a work may contain both parodic and nonparodic elements.” Id. at 581. Therefore, Houghton Mifflin’s fair-use defense of parody, like any other claim of fair use, must be evaluated in light of the factors set out in § 107 and the constitutional purposes of copyright law. Id..

 

Before considering a claimed fair-use defense based on parody, however, the Supreme Court has required that we ensure that “a parodic character may reasonably be perceived” in the allegedly infringing work. Id. at 582. The Supreme Court’s definition of parody in Campbell, however, is somewhat vague. On the one hand, the Court suggests that the aim of parody is “comic effect or ridicule,” but it then proceeds to discuss parody more expansively in terms of its “commentary” on the original. Id. at 580. In light of the admonition in Campbell that courts should not judge the quality of the work or the success of the attempted humor in discerning its parodic character, we choose to take the broader view. For purposes of our fair-use analysis, we will treat a work as a parody if its aim is to comment upon or criticize a prior work by appropriating elements of the original in creating a new artistic, as opposed to scholarly or journalistic, work. Under this definition, the parodic character of TWDG is clear. TWDG is not a general commentary upon the Civil-War-era American South, but a specific criticism of and rejoinder to the depiction of slavery and the relationships between blacks and whites in GWTW. The fact that Randall chose to convey her criticisms of GWTW through a work of fiction, which she contends is a more powerful vehicle for her message than a scholarly article, does not, in and of itself, deprive TWDG of fair-use protection. We therefore proceed to an analysis of the four fair-use factors.

 

 

 

 

 

 

 

i. Purpose and Character of the Work

 

 

 

 

 

 

 

The first factor in the fair-use analysis, the purpose and character of the allegedly infringing work, has several facets. The first is whether TWDG serves a commercial purpose or nonprofit educational purpose. § 107(1). Despite whatever educational function TWDG may be able to lay claim to, it is undoubtedly a commercial product. As the Supreme Court has stated, “[t]he crux of the profit/nonprofit distinction is not whether the sole motive of the use is monetary gain but whether the user stands to profit from exploitation of the copyrighted material without paying the customary price.” Harper & Row, 471 U.S. at 562. The fact that TWDG was published for profit is the first factor weighing against a finding of fair use. However, TWDG’s for-profit status is strongly overshadowed and outweighed in view of its highly transformative use of GWTC’s copyrighted elements. “[T]he more transformative the new work, the less will be the significance of other factors, like commercialism, that may weigh against a finding of fair use.” Campbell, 510 U.S. at 579. “[T]he goal of copyright, to promote science and the arts, is generally furthered by the creation of transformative works.” Id. A work’s transformative value is of special import in the realm of parody, since a parody’s aim is, by nature, to transform an earlier work.

 

The second factor in the “purpose and character” analysis relevant to this case is to what extent TWDG’s use of copyrighted elements of GWTW can be said to be “transformative.” The inquiry is “whether the new work merely supersedes the objects of the original creation, or instead adds something new, with a further purpose or different character, altering the first with new expression, meaning, or message.” Campbell, 510 U.S. at 579. The issue of transformation is a double-edged sword in this case. On the one hand, the story of Cynara and her perception of the events in TWDG certainly adds new “expression, meaning, [and] message” to GWTW. From another perspective, however, TWDG’s success as a pure work of fiction depends heavily on copyrighted elements appropriated from GWTW to carry its own plot forward.

 

However, as noted above, TWDG is more than an abstract, pure fictional work. It is principally and purposefully a critical statement that seeks to rebut and destroy the perspective, judgments, and mythology of GWTW. Randall’s literary goal is to explode the romantic, idealized portrait of the antebellum South during and after the Civil War. In the world of GWTW, the white characters comprise a noble aristocracy whose idyllic existence is upset only by the intrusion of Yankee soldiers, and, eventually, by the liberation of the black slaves. Through her characters as well as through direct narration, Mitchell describes how both blacks and whites were purportedly better off in the days of slavery: “The more I see of emancipation the more criminal I think it is. It’s just ruined the darkies,” says Scarlett O’Hara. GWTW at 639. Free blacks are described as “creatures of small intelligence … [l]ike monkeys or small children turned loose among treasured objects whose value is beyond their comprehension, they ran wild – either from perverse pleasure in destruction or simply because of their ignorance.” Id. at 654. Blacks elected to the legislature are described as spending “most of their time eating goobers and easing their unaccustomed feet into and out of new shoes.” Id. at 904.

 

As the district court noted: “The earlier work is a third-person epic, whereas the new work is told in the first-person as an intimate diary of the life of Cynara. Thematically, the new work provides a different viewpoint of the antebellum world.” 136 F. Supp. 2d at 1367. While told from a different perspective, more critically, the story is transformed into a very different tale, albeit much more abbreviated. Cynara’s very language is a departure from Mitchell’s original prose; she acts as the voice of Randall’s inversion of GWTW. She is the vehicle of parody; she is its means – not its end. It is clear within the first fifty pages of Cynara’s fictional diary that Randall’s work flips GWTW’s traditional race roles, portrays powerful whites as stupid or feckless, and generally sets out to demystify GWTW and strip the romanticism from Mitchell’s specific account of this period of our history. Approximately the last half of TWDG tells a completely new story that, although involving characters based on GWTW characters, features plot elements found nowhere within the covers of GWTW.

 

Where Randall refers directly to Mitchell’s plot and characters, she does so in service of her general attack on GWTW. In GWTW, Scarlett O’Hara often expresses disgust with and condescension towards blacks; in TWDG, Other, Scarlett’s counterpart, is herself of mixed descent. In GWTW, Ashley Wilkes is the initial object of Scarlett’s affection; in TWDG, he is homosexual. In GWTW, Rhett Butler does not consort with black female characters and is portrayed as the captain of his own destiny. In TWDG, Cynara ends her affair with Rhett’s counterpart, R., to begin a relationship with a black Congressman; R. ends up a washed out former cad. In TWDG, nearly every black character is given some redeeming quality – whether depth, wit, cunning, beauty, strength, or courage – that their GWTW analogues lacked.

 

In light of this, we find it difficult to conclude that Randall simply tried to “avoid the drudgery in working up something fresh.” Campbell, 510 U.S. at 580. It is hard to imagine how Randall could have specifically criticized GWTW without depending heavily upon copyrighted elements of that book. A parody is a work that seeks to comment upon or criticize another work by appropriating elements of the original. “Parody needs to mimic an original to make its point, and so has some claim to use the creation of its victim’s (or collective victims’) imagination.” Campbell, 510 U.S. at 580-81. Thus, Randall has fully employed those conscripted elements from GWTW to make war against it. Her work, TWDG, reflects transformative value because it “can provide social benefit, by shedding light on an earlier work, and, in the process, creating a new one.” Campbell, 510 U.S. at 579.

 

While “transformative use is not absolutely necessary for a finding of fair use, … the more transformative the new work, the less will be the significance of other factors.” Id. In the case of TWDG, consideration of this factor certainly militates in favor of a finding of fair use, and, informs our analysis of the other factors, particularly the fourth, as discussed below.

 

 

 

 

 

 

 

ii. Nature of the Copyrighted Work

 

 

 

 

 

 

 

The second factor, the nature of the copyrighted work, recognizes that there is a hierarchy of copyright protection in which original, creative works are afforded greater protection than derivative works or factual compilations. Id. at 586. GWTW is undoubtedly entitled to the greatest degree of protection as an original work of fiction. This factor is given little weight in parody cases, however, “since parodies almost invariably copy publicly known, expressive works.” Campbell, 510 U.S. at 586.

 

 

 

 

 

 

 

iii. Amount and Substantiality of the Portion Used

 

 

 

 

 

 

 

The third fair-use factor is “the amount and substantiality of the portion used in relation to the copyrighted work as a whole.” § 107(3). It is at this point that parody presents uniquely difficult problems for courts in the fair-use context, for “[p]arody’s humor, or in any event its comment, necessarily springs from recognizable allusion to its object through distorted imitation… . When parody takes aim at a particular original work, the parody must be able to ‘conjure up’ at least enough of that original to make the object of its critical wit recognizable.” Campbell, 510 U.S. at 588. Once enough has been taken to “conjure up” the original in the minds of the readership, any further taking must specifically serve the new work’s parodic aims.

 

GWTW is one of the most famous, popular, and enduring American novels ever written. Given the fame of the work and its primary characters, SunTrust argues that very little reference is required to conjure up GWTW. As we have already indicated in our discussion of substantial similarity, TWDG appropriates a substantial portion of the protected elements of GWTW. Houghton Mifflin argues that TWDG takes nothing from GWTW that does not serve a parodic purpose, the crux of the argument being that a large number of characters had to be taken from GWTW because each represents a different ideal or stereotype that requires commentary, and that the work as a whole could not be adequately commented upon without revisiting substantial portions of the plot, including its most famous scenes. Houghton Mifflin’s argument is similar to that made by the defendants in Harper & Row, who argued for “expanding the doctrine of fair use to create what amounts to a public figure exception to copyright.” 471 U.S. at 560. To the extent Houghton Mifflin argues for extra latitude in copying from GWTW because of its fame, the Supreme Court has squarely foreclosed any such privilege:

 

 

It is fundamentally at odds with the scheme of copyright to accord lesser rights in those works that are of greatest importance to the public… . To propose that fair use be imposed whenever the social value of dissemination outweighs any detriment to the artist, would be to propose depriving copyright owners of their right in the property precisely when they encounter those users who could afford to pay for it.

 

 

Id. at 559, 105 S. Ct. at 2229-30 (internal quotations and punctuation omitted). Notably, however, the Court did not go so far as to grant well-known works a special, higher copyright status either.

 

There are numerous instances in which TWDG appropriates elements of GWTW and then transforms them for the purpose of commentary. TWDG uses several of GWTW’s most famous lines, but vests them with a completely new significance… . .

 

On the other hand, however, we are told that not all of TWDG’s takings from GWTW are clearly justified as commentary. We have already determined that TWDG is a parody, but not every parody is a fair use. SunTrust contends that TWDG, at least at the margins, takes more of the protected elements of GWTW than was necessary to serve a parodic function.

 

For example, in a sworn declaration to the district court, Randall stated that she needed to reference the scene from GWTW in which Jeems is given to the Tarleton twins as a birthday present because she considers it “perhaps the single most repellent paragraph in Margaret Mitchell’s novel: a black child given to two white children as a birthday present … as if the buying and selling of children thus had no moral significance.” Clearly, such a scene is fair game for criticism. However, in this instance, SunTrust argues that TWDG goes beyond commentary on the occurrence itself, appropriating such nonrelevant details as the fact that the twins had red hair and were killed at Gettysburg. There are several other scenes from GWTW … that are retold or alluded to without serving any apparent parodic purpose. Similar taking of the descriptions of characters and the minor details of their histories and interactions that arguably are not essential to the parodic purpose of the work recur throughout… . .

 

The Supreme Court in Campbell did not require that parodists take the bare minimum amount of copyright material necessary to conjure up the original work. Parody “must be able to conjure up at least enough of [the] original to make the object of its critical wit recognizable.” Campbell, 510 U.S. at 588 (emphasis added; quotations omitted). “Parody frequently needs to be more than a fleeting evocation of an original in order to make its humorous point… . [E]ven more extensive use [than necessary to conjure up the original] would still be fair use, provided the parody builds upon the original, using the original as a known element of modern culture and contributing something new for humorous effect or commentary.” Elsmere Music, Inc. v. National Broad’g Co., 623 F.2d 252, 253 n. 1 (2d Cir. 1980).

 

A use does not necessarily become infringing the moment it does more than simply conjure up another work. Rather, “[o]nce enough has been taken to assure identification, how much more is reasonable will depend, say, [1] on the extent to which the [work’s] overriding purpose and character is to parody the original or, in contrast, [2] the likelihood that the parody may serve as a market substitute for the original.” Campbell, 510 U.S. at 588. As to the first point, it is manifest that TWDG’s raison d’etre is to parody GWTW. The second point indicates that any material we suspect is “extraneous” to the parody is unlawful only if it negatively effects the potential market for or value of the original copyright. Based upon this record at this juncture, we cannot determine in any conclusive way whether “‘the quantity and value of the materials used’” are reasonable in relation to the purpose of the copying.’” Id., 510 U.S. at 586.

 

 

 

 

 

 

 

iv. Effect on the Market Value of the Original

 

 

 

 

 

 

 

The final fair-use factor requires us to consider the effect that the publication of TWDG will have on the market for or value of SunTrust’s copyright in GWTW, including the potential harm it may cause to the market for derivative works based on GWTW. Campbell, 510 U.S. at 590. In addressing this factor, we must “consider not only the extent of market harm caused by the particular actions of the alleged infringer, but also whether unrestricted and widespread conduct of the sort engaged in by the defendant [] would result in a substantially adverse impact on the potential market.” Id., 114 S. Ct. at 1177 (quotations omitted). More specifically, the Campbell Court continued: “[T]he only harm to derivatives that need concern us … is the harm of market substitution. The fact that a parody may impair the market for derivative uses by the very effectiveness of its critical commentary is no more relevant under copyright that the like threat to the original market.” Id., 510 U.S. at 593.

 

As for the potential market, SunTrust proffered evidence in the district court of the value of its copyright in GWTW. Several derivative works of GWTW have been authorized, including the famous movie of the same name and a book titled Scarlett: The Sequel. GWTW and the derivative works based upon it have generated millions of dollars for the copyright holders. SunTrust has negotiated an agreement with St. Martin’s Press permitting it to produce another derivative work based on GWTW, a privilege for which St. Martin’s paid “well into seven figures.” Part of this agreement was that SunTrust would not authorize any other derivative works prior to the publication of St. Martin’s book.

 

An examination of the record, with its limited development as to relevant market harm due to the preliminary injunction status of the case, discloses that SunTrust focuses on the value of GWTW and its derivatives, but fails to address and offers little evidence or argument to demonstrate that TWDG would supplant demand for SunTrust’s licensed derivatives. However, the Supreme Court and other appeals courts have made clear that, particularly in cases of parody, evidence of harm to the potential market for or value of the original copyright is crucial to a fair use determination. “[E]vidence about relevant markets” is also crucial to the fair use analysis. Campbell, 510 U.S. at 590. “Evidence of substantial harm to [a derivative market] would weigh against a finding of fair use.” Id. at 593. “What is necessary is a showing by a preponderance of the evidence that some meaningful likelihood of future harm exits.” Sony, 464 U.S. at 451… . .

 

In contrast, the evidence proffered in support of the fair use defense specifically and correctly focused on market substitution and demonstrates why Randall’s book is unlikely to displace sales of GWTW. Thus, we conclude, based on the current record, that SunTrust’s evidence falls far short of establishing that TWDG or others like it will act as market substitutes for GWTW or will significantly harm its derivatives. Accordingly, the fourth fair use factor weighs in favor of TWDG.

 

 

 

 

 

 

c. Summary of the Merits

 

 

 

 

 

 

We reject the district court’s conclusion that SunTrust has established its likelihood of success on the merits. To the contrary, based upon our analysis of the fair use factors we find, at this juncture, TWDG is entitled to a fair-use defense.

 

 

 

 

 

2. Irreparable Injury

 

 

 

 

 

… .

 

[A] lack of irreparable injury to SunTrust, together with the First Amendment concerns regarding comment and criticism and the likelihood that a fair use defense will prevail, make injunctive relief improper and we need not address the remaining factors, except to stress that the public interest is always served in promoting First Amendment values and in preserving the public domain from encroachment. Accordingly, we vacate the district court’s injunction.

 

 

 

III. CONCLUSION

 

 

 

In this case, we have found that to the extent SunTrust suffers injury from TWDG’s putative infringement of its copyright in GWTW, such harm can adequately be remedied through an award of monetary damages. Moreover, under the present state of the record, it appears that a viable fair use defense is available. Thus, the issuance of the injunction was at odds with the shared principles of the First Amendment and the copyright law, acting as a prior restraint on speech because the public had not had access to Randall’s ideas or viewpoint in the form of expression that she chose.

 

We REVERSE the judgment of the district court and REMAND the case for further proceedings consistent with this opinion.

 

 

 

Marcus, Circuit Judge, specially concurring:

 

 

 

I concur in Judge Birch’s thoughtful and thorough opinion but write separately to emphasize that, on this limited record, SunTrust has fallen well short of establishing a likelihood of success on its copyright infringement claim. I stress three points. First, the district court erred by finding that the critical or parodic element of The Wind Done Gone is anything but clear-cut. Far from amounting to “unabated piracy,” 136 F.Supp.2d 1357, 1369 (N.D. Ga. 2001), The Wind Done Gone is unequivocally parody, as both Judge Birch and the Supreme Court in Campbell v. Acuff-Rose Music, Inc., 510 U.S. 569 (1994), define that term… . . Moreover, the Mitchell estate seems to have made a specific practice of refusing to license just the sort of derivative use Randall has undertaken – a factor that further undermines SunTrust’s copyright claim.

 

… .

 

Finally, I wish to highlight a factor significant to the market harm inquiry: SunTrust’s apparent practice of placing certain editorial restrictions on the authors of its licensed derivatives. Pat Conroy, the author of The Prince of Tides and Beach Music, among other novels, attested to the sorts of constraints the Mitchell estate sought to place on him as a potential author of a sequel to Gone With the Wind:

 

 

I wrote an introduction to the sixtieth anniversary edition of [Gone With the Wind] … After the appearance of my introduction[,] which included my own deep appreciation for the artistry of GWTW, the estate of Margaret Mitchell contacted my agent, Julian Bach, in New York and asked if I would be interested in doing a sequel to GWTW… . When Julian Bach called me, he issued a strange decree from the estate that Julian said was non-negotiable… . He said, “You’re not going to like this, but the estate will require you to sign a pledge that says you will under no circumstances write anything about miscegenation or homosexuality.”10

 

 

In light of this, the The Wind Done Gone’s transformation of Ashley Wilkes into a homosexual, its depiction of interracial sex, and its multiple mulatto characters take on additional relevance. The Supreme Court in Campbell explained how a copyright holder’s reluctance to license certain kinds of derivatives affects the market harm analysis:

 

 

The market for potential derivative uses includes only those that creators of original works would in general develop or license others to develop. Yet the unlikelihood that creators of imaginative works will license critical reviews or lampoons of their own productions removes such uses from the very notion of a potential licensing market.

 

 

510 U.S. at 592.

 

Other courts have echoed the principle that “‘only traditional, reasonable, or likely to be developed markets’” ought to be considered when assessing the effect of a challenged use upon a potential market. [citations omitted] “In the cases where we have found the fourth factor to favor a defendant, the defendant’s work filled a market niche that the plaintiff simply had no interest in occupying.” Twin Peaks Prods., Inc. v. Publications Int’l, Ltd., 996 F.2d 1366, 1377 (2d Cir. 1993).

 

The preliminary record does not indicate why SunTrust sought to impose editorial restrictions on Conroy. To the extent that SunTrust may have done so to preserve Gone With the Wind’s reputation, or protect its story from “taint,” however, it may not now invoke copyright to further that goal. Of course, SunTrust can choose to license its derivatives however it wishes and insist that those derivatives remain free of content it deems disreputable. SunTrust may be vigilant of Gone With the Wind’s public image – but it may not use copyright to shield Gone With the Wind from unwelcome comment, a policy that would extend intellectual property protection “into the precincts of censorship,” in Pat Conroy’s words. “Because the social good is served by increasing the supply of criticism – and thus, potentially, of truth – creators of original works cannot be given the power to block the dissemination of critical derivative works.” Leibovitz, 137 F.3d at 115 n.3. “Copyright law is not designed to stifle critics. Destructive parodies play an important role in social and literary criticism and thus merit protection even though they may discourage or discredit an original author.” Fisher, 794 F.2d at 438 (citation and internal quotation marks omitted).

 

The law grants copyright holders a powerful monopoly in their expressive works. It should not also afford them windfall damages for the publication of the sorts of works that they themselves would never publish, or worse, grant them a power of indirect censorship.

 

Finally, Appellee warns that our decision in this case will prompt an endless parade of litigants to test the boundaries of the rule we establish here. This is at least possible, but such a phenomenon is not exactly alien to our common law tradition. And to the extent authors and publishers will be encouraged to experiment with new and different forms of storytelling, copyright’s fundamental purpose, “[t]o promote the Progress of Science and useful Arts,” will have been served. U.S. Const. Art. 1, § 8, cl. 8.

 


  1. Honorable Harlington Wood, Jr., U.S. Circuit Judge for the Seventh Circuit, sitting by designation.

 

  1. Hereafter, the Copyright Act of 1976 shall be referred to by only the section number of the Act.

 

  1. Houghton Mifflin denies that there are passages from GWTW copied verbatim in TWDG.

 

  1. I believe that fair use should be considered an affirmative right under the 1976 Act, rather than merely an affirmative defense, as it is defined in the Act as a use that is not a violation of copyright. See Bateman v. Mneumonics, Inc., 79 F.3d 1532, 1542 n.22 (11th Cir. 1996). However, fair use is commonly referred to an affirmative defense, see Campbell v. Acuff-Rose Music, Inc., 510 U.S. 569, 590, 114 S. Ct. 1164, 1177 (1994), and, as we are bound by Supreme Court precedent, we will apply it as such. See also David Nimmer, A Riff on Fair Use in the Digital Millennium Copyright Act, 148 U. Pa. L. Rev. 673, 714 n. 227 (2000) (citing Bateman). Nevertheless, the fact that the fair use right must be procedurally asserted as an affirmative defense does not detract from its constitutional significance as a guarantor to access and use for First Amendment purposes.

 

  1. The Statute of Anne providing for copyright is introduced as “[a]n act for the encouragement of learning,” and has a preamble that states one of the purposes as “the encouragement of learned men to compose and write useful books.” 8 Anne, C.19 (1710), reprinted in 8 Nimmer § 7-5.

 

  1. Under modern copyright, such a right to translate would enjoy protection as a “derivative work.” §§ 101 and 106. In Folsom v. Marsh, 9 F.Cas. 342 (C.C.Mass. 1841), Justice Story created the concept of “fair use,” which actually expanded the copyright monopoly, since until that time a translation or abridgement was not considered an infringement.

 

  1. While the First Amendment disallows laws that abridge the freedom of speech, the Copyright Clause calls specifically for such a law.

 

  1. Originally the word “copie” was a noun, indicating the manuscript. Ownership of the “copie” thus meant ownership of the manuscript for the purposes of publishing it. Today, “copy” has become a verb, meaning the act of reproduction of a work. But in the development of copyright law it was intended to be a term of art, indicating a reproduction of a work for publication. Failure to understand and apply this distinction has confused many courts (assisted by overzealous advocates) into too expansive a view of the scope of the copyright monopoly.

 

  1. “Cynara” is the name of the poem by Ernest Dowson, from which GWTW’s title is derived (“I have forgot much, Cynara! gone with the wind, …”).

 

  1. In a piece of documentary evidence submitted by SunTrust (Thomas Hal Clarke, attorney and member of the committee established by the trust instruments to direct the plaintiff SunTrust Bank), Conroy again indicates that the Mitchell Estate was loath to license a derivative work that contained such elements:

     

     

    All my resistance to your restrictions – all of them, and I include miscegenation, homosexuality, the rights of review and approval – I do because they begin inching toward the precincts of censorship.

     

     

    Fax to Owen Laster from Pat Conroy, Nov. 10, 1998.

 

 

 

PROBLEM

 

 

 

Suppose that Rural’s compilation of phone numbers had been determined to be copyrightable in Feist. In other words, suppose Feist had come out the other way. Go through the factors to argue whether Feist’s use was a fair use.

 

3.4.3. Calibration: Length of Term

 

Eldred v. Ashcroft,

537 U.S. 186 (2003)

 

 

 

Lawrence Lessig argued the cause for petitioners. With him on the briefs were Kathleen M. Sullivan, Alan B. Morrison, Edward Lee, Charles Fried, Geoffrey S. Stewart, Donald B. Ayer, Robert P. Ducatman, Daniel H. Bromberg, Charles R. Nesson, and Jonathan L. Zittrain.

 

Solicitor General Olson argued the cause for respondent. With him on the brief were Assistant Attorney General McCallum, Deputy Solicitor General Wallace, Jeffrey A. Lamken, William Kanter, and John S. Koppel.1

 

 

 

Justice Ginsburg delivered the opinion of the Court.

 

This case concerns the authority the Constitution assigns to Congress to prescribe the duration of copyrights. The Copyright and Patent Clause of the Constitution, Art. I, § 8, cl. 8, provides as to copyrights: “Congress shall have Power … [t]o promote the Progress of Science … by securing [to Authors] for limited Times … the exclusive Right to their … Writings.” In 1998, in the measure here under inspection, Congress enlarged the duration of copyrights by 20 years. Copyright Term Extension Act (CTEA), Pub. L. 105-298, §§ 102(b) and (d), 112 Stat. 2827-2828 (amending 17 U.S.C. §§ 302, 304). As in the case of prior extensions, principally in 1831, 1909, and 1976, Congress provided for application of the enlarged terms to existing and future copyrights alike.

 

Petitioners are individuals and businesses whose products or services build on copyrighted works that have gone into the public domain. They seek a determination that the CTEA fails constitutional review under both the Copyright Clause’s “limited Times” prescription and the First Amendment’s free speech guarantee. Under the 1976 Copyright Act, copyright protection generally lasted from the work’s creation until 50 years after the author’s death. Under the CTEA, most copyrights now run from creation until 70 years after the author’s death. Petitioners do not challenge the “life-plus-70-years” timespan itself. “Whether 50 years is enough, or 70 years too much,” they acknowledge, “is not a judgment meet for this Court.”2 Congress went awry, petitioners maintain, not with respect to newly created works, but in enlarging the term for published works with existing copyrights. The “limited Tim[e]” in effect when a copyright is secured, petitioners urge, becomes the constitutional boundary, a clear line beyond the power of Congress to extend. As to the First Amendment, petitioners contend that the CTEA is a content-neutral regulation of speech that fails inspection under the heightened judicial scrutiny appropriate for such regulations.

 

In accord with the District Court and the Court of Appeals, we reject petitioners’ challenges to the CTEA. In that 1998 legislation, as in all previous copyright term extensions, Congress placed existing and future copyrights in parity. In prescribing that alignment, we hold, Congress acted within its authority and did not transgress constitutional limitations.

 

 

 

 

I

 

 

 

 

 

 

A.

 

 

 

 

 

We evaluate petitioners’ challenge to the constitutionality of the CTEA against the backdrop of Congress’ previous exercises of its authority under the Copyright Clause. The Nation’s first copyright statute, enacted in 1790, provided a federal copyright term of 14 years from the date of publication, renewable for an additional 14 years if the author survived the first term. The 1790 Act’s renewable 14-year term applied to existing works (i. e., works already published and works created but not yet published) and future works alike. Congress expanded the federal copyright term to 42 years in 1831 (28 years from publication, renewable for an additional 14 years), and to 56 years in 1909 (28 years from publication, renewable for an additional 28 years). Both times, Congress applied the new copyright term to existing and future works; to qualify for the 1831 extension, an existing work had to be in its initial copyright term at the time the Act became effective.

 

In 1976, Congress altered the method for computing federal copyright terms. For works created by identified natural persons, the 1976 Act provided that federal copyright protection would run from the work’s creation, not – as in the 1790, 1831, and 1909 Acts – its publication; protection would last until 50 years after the author’s death. In these respects, the 1976 Act aligned United States copyright terms with the then-dominant international standard adopted under the Berne Convention for the Protection of Literary and Artistic Works. For anonymous works, pseudonymous works, and works made for hire, the 1976 Act provided a term of 75 years from publication or 100 years from creation, whichever expired first.

 

These new copyright terms, the 1976 Act instructed, governed all works not published by its effective date of January 1, 1978, regardless of when the works were created. For published works with existing copyrights as of that date, the 1976 Act granted a copyright term of 75 years from the date of publication, a 19-year increase over the 56-year term applicable under the 1909 Act.

 

The measure at issue here, the CTEA, installed the fourth major duration extension of federal copyrights.3 Retaining the general structure of the 1976 Act, the CTEA enlarges the terms of all existing and future copyrights by 20 years. For works created by identified natural persons, the term now lasts from creation until 70 years after the author’s death. This standard harmonizes the baseline United States copyright term with the term adopted by the European Union in 1993. For anonymous works, pseudonymous works, and works made for hire, the term is 95 years from publication or 120 years from creation, whichever expires first.

 

Paralleling the 1976 Act, the CTEA applies these new terms to all works not published by January 1, 1978. For works published before 1978 with existing copyrights as of the CTEA’s effective date, the CTEA extends the term to 95 years from publication. Thus, in common with the 1831, 1909, and 1976 Acts, the CTEA’s new terms apply to both future and existing copyrights.4

 

 

 

 

 

B.

 

 

 

 

 

The [District] court held that the CTEA does not violate the “limited Times” restriction of the Copyright Clause because the CTEA’s terms, though longer than the 1976 Act’s terms, are still limited, not perpetual, and therefore fit within Congress’ discretion. The court also held that “there are no First Amendment rights to use the copyrighted works of others.”

 

The Court of Appeals for the District of Columbia Circuit affirmed. … .

 

We granted certiorari to address two questions: whether the CTEA’s extension of existing copyrights exceeds Congress’ power under the Copyright Clause; and whether the CTEA’s extension of existing and future copyrights violates the First Amendment. 534 U.S. 1126 and 1160 (2002). We now answer those two questions in the negative and affirm.

 

 

 

 

II

 

 

 

 

 

 

A.

 

 

 

 

 

We address first the determination of the courts below that Congress has authority under the Copyright Clause to extend the terms of existing copyrights. Text, history, and precedent, we conclude, confirm that the Copyright Clause empowers Congress to prescribe “limited Times” for copyright protection and to secure the same level and duration of protection for all copyright holders, present and future.

 

The CTEA’s baseline term of life plus 70 years, petitioners concede, qualifies as a “limited Tim[e]” as applied to future copyrights.5 Petitioners contend, however, that existing copyrights extended to endure for that same term are not “limited.” … .

 

To comprehend the scope of Congress’ power under the Copyright Clause, “a page of history is worth a volume of logic.” New York Trust Co. v. Eisner, 256 U.S. 345, 349 (1921) (Holmes, J.). History reveals an unbroken congressional practice of granting to authors of works with existing copyrights the benefit of term extensions so that all under copyright protection will be governed evenhandedly under the same regime. As earlier recounted, the First Congress accorded the protections of the Nation’s first federal copyright statute to existing and future works alike. Since then, Congress has regularly applied duration extensions to both existing and future copyrights. 6

 

… .

 

The CTEA reflects judgments of a kind Congress typically makes, judgments we cannot dismiss as outside the Legislature’s domain. As respondent describes, a key factor in the CTEA’s passage was a 1993 European Union (EU) directive instructing EU members to establish a copyright term of life plus 70 years. Consistent with the Berne Convention, the EU directed its members to deny this longer term to the works of any non-EU country whose laws did not secure the same extended term. By extending the baseline United States copyright term to life plus 70 years, Congress sought to ensure that American authors would receive the same copyright protection in Europe as their European counterparts.7 The CTEA may also provide greater incentive for American and other authors to create and disseminate their work in the United States.8

 

In addition to international concerns,9 Congress passed the CTEA in light of demographic, economic, and technological changes,10 and rationally credited projections that longer terms would encourage copyright holders to invest in the restoration and public distribution of their works.11

 

In sum, we find that the CTEA is a rational enactment; we are not at liberty to second-guess congressional determinations and policy judgments of this order, however debatable or arguably unwise they may be. Accordingly, we cannot conclude that the CTEA – which continues the unbroken congressional practice of treating future and existing copyrights in parity for term extension purposes – is an impermissible exercise of Congress’ power under the Copyright Clause.

 

 

 

 

 

B.

 

 

 

 

 

Petitioners’ Copyright Clause arguments rely on several novel readings of the Clause. We next address these arguments and explain why we find them unpersuasive.

 

 

 

 

 

 

1

 

 

 

 

 

 

Petitioners contend that even if the CTEA’s 20-year term extension is literally a “limited Tim[e],” permitting Congress to extend existing copyrights allows it to evade the “limited Times” constraint by creating effectively perpetual copyrights through repeated extensions. We disagree.

 

As the Court of Appeals observed, a regime of perpetual copyrights “clearly is not the situation before us.” Nothing before this Court warrants construction of the CTEA’s 20-year term extension as a congressional attempt to evade or override the “limited Times” constraint.12 Critically, we again emphasize, petitioners fail to show how the CTEA crosses a constitutionally significant threshold with respect to “limited Times” that the 1831, 1909, and 1976 Acts did not. Those earlier Acts did not create perpetual copyrights, and neither does the CTEA.13

 

 

 

 

 

 

2

 

 

 

 

 

 

… . Petitioners contend that the CTEA’s extension of existing copyrights (1) overlooks the requirement of “originality,” (2) fails to “promote the Progress of Science,” and (3) ignores copyright’s quid pro quo. Petitioners’ “originality” argument draws on Feist Publications, Inc. v. Rural Telephone Service Co., 499 U.S. 340 (1991). [The Court turned aside the argument that existing works are not original and therefore not eligible for further protection, arguing that the originality requirement is not relevant to interpreting the “limited Times” language.]

 

… .

 

More forcibly, petitioners contend that [t]he CTEA’s extension of existing copyrights categorically fails to “promote the Progress of Science” …, because it does not stimulate the creation of new works but merely adds value to works already created.

 

… .

 

We have … stressed, however, that it is generally for Congress, not the courts, to decide how best to pursue the Copyright Clause’s objectives. The justifications we earlier set out for Congress’ enactment of the CTEA provide a rational basis for the conclusion that the CTEA “promote[s] the Progress of Science.”

 

On the issue of copyright duration, Congress, from the start, has routinely applied new definitions or adjustments of the copyright term to both future works and existing works not yet in the public domain.14 Such consistent congressional practice is entitled to “very great weight, and when it is remembered that the rights thus established have not been disputed during a period of [over two] centur[ies], it is almost conclusive.” Indeed, “[t]his Court has repeatedly laid down the principle that a contemporaneous legislative exposition of the Constitution when the founders of our Government and framers of our Constitution were actively participating in public affairs, acquiesced in for a long term of years, fixes the construction to be given [the Constitution’s] provisions.” Congress’ unbroken practice since the founding generation thus overwhelms petitioners’ argument that the CTEA’s extension of existing copyrights fails per se to “promote the Progress of Science.”

 

Closely related to petitioners’ preambular argument, or a variant of it, is their assertion that the Copyright Clause “imbeds a quid pro quo.” They contend, in this regard, that Congress may grant to an “Autho[r]” an “exclusive Right” for a “limited Tim[e],” but only in exchange for a “Writin[g].” Congress’ power to confer copyright protection, petitioners argue, is thus contingent upon an exchange: The author of an original work receives an “exclusive Right” for a “limited Tim[e]” in exchange for a dedication to the public thereafter. Extending an existing copyright without demanding additional consideration, petitioners maintain, bestows an unpaid-for benefit on copyright holders and their heirs, in violation of the quid pro quo requirement.

 

We can demur to petitioners’ description of the Copyright Clause as a grant of legislative authority empowering Congress “to secure a bargain – this for that.” But the legislative evolution earlier recalled demonstrates what the bargain entails. Given the consistent placement of existing copyright holders in parity with future holders, the author of a work created in the last 170 years would reasonably comprehend, as the “this” offered her, a copyright not only for the time in place when protection is gained, but also for any renewal or extension legislated during that time. Congress could rationally seek to “promote … Progress” by including in every copyright statute an express guarantee that authors would receive the benefit of any later legislative extension of the copyright term. Nothing in the Copyright Clause bars Congress from creating the same incentive by adopting the same position as a matter of unbroken practice.

 

We note, furthermore, that patents and copyrights do not entail the same exchange, and that our references to a quid pro quo typically appear in the patent context. This is understandable, given that immediate disclosure is not the objective of, but is exacted from, the patentee. It is the price paid for the exclusivity secured. For the author seeking copyright protection, in contrast, disclosure is the desired objective, not something exacted from the author in exchange for the copyright. Indeed, since the 1976 Act, copyright has run from creation, not publication.

 

Further distinguishing the two kinds of intellectual property, copyright gives the holder no monopoly on any knowledge. A reader of an author’s writing may make full use of any fact or idea she acquires from her reading. The grant of a patent, on the other hand, does prevent full use by others of the inventor’s knowledge. In light of these distinctions, one cannot extract from language in our patent decisions – language not trained on a grant’s duration – genuine support for petitioners’ bold view. Accordingly, we reject the proposition that a quid pro quo requirement stops Congress from expanding copyright’s term in a manner that puts existing and future copyrights in parity.15

 

… .

 

 

 

 

III

 

 

 

 

Petitioners separately argue that the CTEA is a content-neutral regulation of speech that fails heightened judicial review under the First Amendment. We reject petitioners’ plea for imposition of uncommonly strict scrutiny on a copyright scheme that incorporates its own speech-protective purposes and safeguards. … .

 

 

 

 

IV

 

 

 

 

If petitioners’ vision of the Copyright Clause held sway, it would do more than render the CTEA’s duration extensions unconstitutional as to existing works. Indeed, petitioners’ assertion that the provisions of the CTEA are not severable would make the CTEA’s enlarged terms invalid even as to tomorrow’s work. The 1976 Act’s time extensions, which set the pattern that the CTEA followed, would be vulnerable as well.

 

As we read the Framers’ instruction, the Copyright Clause empowers Congress to determine the intellectual property regimes that, overall, in that body’s judgment, will serve the ends of the Clause. Beneath the facade of their inventive constitutional interpretation, petitioners forcefully urge that Congress pursued very bad policy in prescribing the CTEA’s long terms. The wisdom of Congress’ action, however, is not within our province to second-guess. Satisfied that the legislation before us remains inside the domain the Constitution assigns to the First Branch, we affirm the judgment of the Court of Appeals.

 

It is so ordered.

 

 

 

Justice Stevens, dissenting.

 

… . [I]nsofar as the 1998 Sonny Bono Copyright Term Extension Act, 112 Stat. 2827, purported to extend the life of unexpired copyrights, it is invalid. Because the majority’s contrary conclusion rests on the mistaken premise that this Court has virtually no role in reviewing congressional grants of monopoly privileges to authors, inventors, and their successors, I respectfully dissent.

 

 

 

 

I

 

 

 

 

The authority to issue copyrights stems from the same Clause in the Constitution that created the patent power. It provides:

 

 

Congress shall have Power … To promote the Progress of Science and useful Arts, by securing for limited Times to Authors and Inventors the exclusive Right to their respective Writings and Discoveries.

 

 

Art. I, § 8, cl. 8.

 

It is well settled that the Clause is “both a grant of power and a limitation” and that Congress “may not overreach the restraints imposed by the stated constitutional purpose.” As we have made clear in the patent context, that purpose has two dimensions. Most obviously the grant of exclusive rights to their respective writings and discoveries is intended to encourage the creativity of “Authors and Inventors.” But the requirement that those exclusive grants be for “limited Times” serves the ultimate purpose of promoting the “Progress of Science and useful Arts” by guaranteeing that those innovations will enter the public domain as soon as the period of exclusivity expires:

 

 

Once the patent issues, it is strictly construed, it cannot be used to secure any monopoly beyond that contained in the patent, … and especially relevant here, when the patent expires the monopoly created by it expires, too, and the right to make the article – including the right to make it in precisely the shape it carried when patented – passes to the public.

 

 

Sears, Roebuck & Co. v. Stiffel Co., 376 U. S. 230 (1964).

 

It is that ultimate purpose that explains why a patent may not issue unless it discloses the invention in such detail that one skilled in the art may copy it. Complete disclosure as a precondition to the issuance of a patent is part of the quid pro quo that justifies the limited monopoly for the inventor as consideration for full and immediate access by the public when the limited time expires.

 

Almost two centuries ago the Court plainly stated that public access to inventions at the earliest possible date was the essential purpose of the Clause:

 

 

While one great object was, by holding out a reasonable reward to inventors and giving them an exclusive right to their inventions for a limited period, to stimulate the efforts of genius; the main object was ‘to promote the progress of science and useful arts;’ and this could be done best, by giving the public at large a right to make, construct, use, and vend the thing invented, at as early a period as possible, having a due regard to the rights of the inventor. If an inventor should be permitted to hold back from the knowledge of the public the secrets of his invention; if he should for a long period of years retain the monopoly, and make, and sell his invention publicly, and thus gather the whole profits of it, relying upon his superior skill and knowledge of the structure; and then, and then only, when the danger of competition should force him to secure the exclusive right, he should be allowed to take out a patent, and thus exclude the public from any farther use than what should be derived under it during his fourteen years; it would materially retard the progress of science and the useful arts, and give a premium to those, who should be least prompt to communicate their discoveries.

 

 

Pennock v. Dialogue, 2 Pet. 1, 18 (1829).

 

Pennock held that an inventor could not extend the period of patent protection by postponing his application for the patent while exploiting the invention commercially. As we recently explained, “implicit in the Patent Clause itself” is the understanding “that free exploitation of ideas will be the rule, to which the protection of a federal patent is the exception. Moreover, the ultimate goal of the patent system is to bring new designs and technologies into the public domain through disclosure.”

 

Neither the purpose of encouraging new inventions nor the overriding interest in advancing progress by adding knowledge to the public domain is served by retroactively increasing the inventor’s compensation for a completed invention and frustrating the legitimate expectations of members of the public who want to make use of it in a free market. Because those twin purposes provide the only avenue for congressional action under the Copyright/Patent Clause of the Constitution, any other action is manifestly unconstitutional.

 

 

 

 

II

 

 

 

 

We have recognized that these twin purposes of encouraging new works and adding to the public domain apply to copyrights as well as patents. Thus, with regard to copyrights on motion pictures, we have clearly identified the overriding interest in the “release to the public of the products of [the author’s] creative genius.” United States v. Paramount Pictures, Inc., 334 U. S. 131, 158 (1948). And, as with patents, we have emphasized that the overriding purpose of providing a reward for authors’ creative activity is to motivate that activity and “to allow the public access to the products of their genius after the limited period of exclusive control has expired.” Sony Corp. of America v. Universal City Studios, Inc., 464 U. S. 417, 429 (1984). Ex post facto extensions of copyrights result in a gratuitous transfer of wealth from the public to authors, publishers, and their successors in interest. Such retroactive extensions do not even arguably serve either of the purposes of the Copyright/Patent Clause. The reasons why such extensions of the patent monopoly are unconstitutional apply to copyrights as well.

 

Respondent, however, advances four arguments in support of the constitutionality of such retroactive extensions: (1) The first Copyright Act enacted shortly after the Constitution was ratified applied to works that had already been produced; (2) later Congresses have repeatedly authorized extensions of copyrights and patents; (3) such extensions promote the useful arts by giving copyright holders an incentive to preserve and restore certain valuable motion pictures; and (4) as a matter of equity, whenever Congress provides a longer term as an incentive to the creation of new works by authors, it should provide an equivalent reward to the owners of all unexpired copyrights. None of these arguments is persuasive. [Justice Stevens argues that both reason and the history of Copyright and Patent statutes contradict these arguments.]

 

 

 

 

VII

 

 

 

 

The express grant of a perpetual copyright would unquestionably violate the textual requirement that the authors’ exclusive rights be only “for limited Times.” Whether the extraordinary length of the grants authorized by the 1998 Act are invalid because they are the functional equivalent of perpetual copyrights is a question that need not be answered in this case because the question presented by the certiorari petition merely challenges Congress’ power to extend retroactively the terms of existing copyrights. Accordingly, there is no need to determine whether the deference that is normally given to congressional policy judgments may save from judicial review its decision respecting the appropriate length of the term. It is important to note, however, that a categorical rule prohibiting retroactive extensions would effectively preclude perpetual copyrights. More importantly, as the House of Lords recognized when it refused to amend the Statute of Anne in 1735, unless the Clause is construed to embody such a categorical rule, Congress may extend existing monopoly privileges ad infinitum under the majority’s analysis.

 

By failing to protect the public interest in free access to the products of inventive and artistic genius – indeed, by virtually ignoring the central purpose of the Copyright/Patent Clause – the Court has quitclaimed to Congress its principal responsibility in this area of the law. Fairly read, the Court has stated that Congress’ actions under the Copyright/Patent Clause are, for all intents and purposes, judicially unreviewable. That result cannot be squared with the basic tenets of our constitutional structure. It is not hyperbole to recall the trenchant words of Chief Justice John Marshall: “It is emphatically the province and duty of the judicial department to say what the law is.” Marbury v. Madison, 1 Cranch 137, 177 (1803). We should discharge that responsibility as we did in Chadha.

 

I respectfully dissent.

 

 

 

Justice Breyer, dissenting.

 

The Constitution’s Copyright Clause grants Congress the power to ”promote the Progress of Science … by securing for limited Times to Authors … the exclusive Right to their respective Writings.” Art. I, § 8, cl. 8 (emphasis added). The statute before us, the 1998 Sonny Bono Copyright Term Extension Act, extends the term of most existing copyrights to 95 years and that of many new copyrights to 70 years after the author’s death. The economic effect of this 20-year extension – the longest blanket extension since the Nation’s founding – is to make the copyright term not limited, but virtually perpetual. Its primary legal effect is to grant the extended term not to authors, but to their heirs, estates, or corporate successors. And most importantly, its practical effect is not to promote, but to inhibit, the progress of “Science” – by which word the Framers meant learning or knowledge.

 

The majority believes these conclusions rest upon practical judgments that at most suggest the statute is unwise, not that it is unconstitutional. Legal distinctions, however, are often matters of degree. And in this case the failings of degree are so serious that they amount to failings of constitutional kind. Although the Copyright Clause grants broad legislative power to Congress, that grant has limits. And in my view this statute falls outside them.

 

 

 

 

I

 

 

 

 

The “monopoly privileges” that the Copyright Clause confers “are neither unlimited nor primarily designed to provide a special private benefit.” Sony Corp. of America v. Universal City Studios, Inc., 464 U. S. 417, 429 (1984); cf. Graham v. John Deere Co. of Kansas City, 383 U. S. 1, 5 (1966). This Court has made clear that the Clause’s limitations are judicially enforceable. E. g., Trade-Mark Cases, 100 U. S. 82, 93-94 (1879). And, in assessing this statute for that purpose, I would take into account the fact that the Constitution is a single document, that it contains both a Copyright Clause and a First Amendment, and that the two are related.

 

The Copyright Clause and the First Amendment seek related objectives – the creation and dissemination of information. When working in tandem, these provisions mutually reinforce each other, the first serving as an “engine of free expression,” Harper & Row, Publishers, Inc. v. Nation Enterprises, 471 U. S. 539, 558 (1985), the second assuring that government throws up no obstacle to its dissemination. At the same time, a particular statute that exceeds proper Copyright Clause bounds may set Clause and Amendment at cross-purposes, thereby depriving the public of the speech-related benefits that the Founders, through both, have promised.

 

Consequently, I would review plausible claims that a copyright statute seriously, and unjustifiably, restricts the dissemination of speech somewhat more carefully than reference to this Court’s traditional Copyright Clause jurisprudence might suggest. There is no need in this case to characterize that review as a search for “‘congruence and proportionality,’” or as some other variation of what this Court has called “intermediate scrutiny.” Rather, it is necessary only to recognize that this statute involves not pure economic regulation, but regulation of expression, and what may count as rational where economic regulation is at issue is not necessarily rational where we focus on expression – in a Nation constitutionally dedicated to the free dissemination of speech, information, learning, and culture. In this sense only, and where line-drawing among constitutional interests is at issue, I would look harder than does the majority at the statute’s rationality – though less hard than precedent might justify.

 

Thus, I would find that the statute lacks the constitutionally necessary rational support (1) if the significant benefits that it bestows are private, not public; (2) if it threatens seriously to undermine the expressive values that the Copyright Clause embodies; and (3) if it cannot find justification in any significant Clause-related objective. Where, after examination of the statute, it becomes difficult, if not impossible, even to dispute these characterizations, Congress’ “choice is clearly wrong.”

 

 

 

 

II

 

 

 

 

 

 

A.

 

 

 

 

 

Because we must examine the relevant statutory effects in light of the Copyright Clause’s own purposes, we should begin by reviewing the basic objectives of that Clause. The Clause authorizes a “tax on readers for the purpose of giving a bounty to writers.” Why? What constitutional purposes does the “bounty” serve?

 

The Constitution itself describes the basic Clause objective as one of “promot[ing] the Progress of Science,” i. e., knowledge and learning. The Clause exists not to “provide a special private benefit,” Sony, supra, at 429, but “to stimulate artistic creativity for the general public good.” It does so by “motivat[ing] the creative activity of authors” through “the provision of a special reward.” Sony, supra, at 429. The “reward” is a means, not an end. And that is why the copyright term is limited. It is limited so that its beneficiaries – the public – “will not be permanently deprived of the fruits of an artist’s labors.”

 

That is how the Court previously has described the Clause’s objectives. And, in doing so, the Court simply has reiterated the views of the Founders.

 

Madison, like Jefferson and others in the founding generation, warned against the dangers of monopolies. Madison noted that the Constitution had “limited them to two cases, the authors of Books, and of useful inventions.” Madison on Monopolies 756. He thought that in those two cases monopoly is justified because it amounts to “compensation for” an actual community “benefit” and because the monopoly is “temporary” – the term originally being 14 years (once renewable). Madison concluded that “under that limitation a sufficient recompence and encouragement may be given.” But he warned in general that monopolies must be “guarded with strictness agst abuse.”

 

Many Members of the Legislative Branch have expressed themselves similarly. Those who wrote the House Report on the landmark Copyright Act of 1909, for example, said that copyright was not designed “primarily” to “benefit” the “author” or “any particular class of citizens, however worthy.” Rather, under the Constitution, copyright was designed “primarily for the benefit of the public,” for “the benefit of the great body of people, in that it will stimulate writing and invention.” And were a copyright statute not “believed, in fact, to accomplish” the basic constitutional objective of advancing learning, that statute “would be beyond the power of Congress” to enact. Similarly, those who wrote the House Report on legislation that implemented the Berne Convention for the Protection of Literary and Artistic Works said that “[t]he constitutional purpose of copyright is to facilitate the flow of ideas in the interest of learning.” H. R. Rep. No. 100-609, p. 22 (1988) (internal quotation marks omitted). They added:

 

 

Under the U. S. Constitution, the primary objective of copyright law is not to reward the author, but rather to secure for the public the benefits derived from the authors’ labors. By giving authors an incentive to create, the public benefits in two ways: when the original expression is created and … when the limited term … expires and the creation is added to the public domain.

 

 

Id., at 17.

 

For present purposes, then, we should take the following as well established: that copyright statutes must serve public, not private, ends; that they must seek “to promote the Progress” of knowledge and learning; and that they must do so both by creating incentives for authors to produce and by removing the related restrictions on dissemination after expiration of a copyright’s “limited Tim[e]” – a time that (like “a limited monarch”) is “restrain[ed]” and “circumscribe[d],” “not [left] at large,” 2 S. Johnson, A Dictionary of the English Language 1151 (4th rev. ed. 1773). I would examine the statute’s effects in light of these well-established constitutional purposes.

 

 

 

 

 

B.

 

 

 

 

 

This statute, like virtually every copyright statute, imposes upon the public certain expression-related costs in the form of (1) royalties that may be higher than necessary to evoke creation of the relevant work, and (2) a requirement that one seeking to reproduce a copyrighted work must obtain the copyright holder’s permission. The first of these costs translates into higher prices that will potentially restrict a work’s dissemination. The second means search costs that themselves may prevent reproduction even where the author has no objection. Although these costs are, in a sense, inevitable concomitants of copyright protection, there are special reasons for thinking them especially serious here.

 

First, the present statute primarily benefits the holders of existing copyrights, i. e., copyrights on works already created. And a Congressional Research Service (CRS) study prepared for Congress indicates that the added royalty-related sum that the law will transfer to existing copyright holders is large. E. Rappaport, CRS Report for Congress, Copyright Term Extension: Estimating the Economic Values (1998) (hereinafter CRS Report). In conjunction with official figures on copyright renewals, the CRS Report indicates that only about 2% of copyrights between 55 and 75 years old retain commercial value – i. e., still generate royalties after that time. But books, songs, and movies of that vintage still earn about $400 million per year in royalties. Hence, (despite declining consumer interest in any given work over time) one might conservatively estimate that 20 extra years of copyright protection will mean the transfer of several billion extra royalty dollars to holders of existing copyrights – copyrights that, together, already will have earned many billions of dollars in royalty “reward.”

 

The extra royalty payments will not come from thin air. Rather, they ultimately come from those who wish to read or see or hear those classic books or films or recordings that have survived. Even the $500,000 that United Airlines has had to pay for the right to play George Gershwin’s 1924 classic Rhapsody in Blue represents a cost of doing business, potentially reflected in the ticket prices of those who fly. See Ganzel, Copyright or Copywrong?, 39 Training 36, 42 (Dec. 2002). Further, the likely amounts of extra royalty payments are large enough to suggest that unnecessarily high prices will unnecessarily restrict distribution of classic works (or lead to disobedience of the law) – not just in theory but in practice. Cf. CRS Report 3 (“[N]ew, cheaper editions can be expected when works come out of copyright”); Brief for College Art Association et al. as Amici Curiae 24 (One year after expiration of copyright on Willa Cather’s My Antonia, seven new editions appeared at prices ranging from $2 to $24); Ganzel, supra, at 40-41, 44 (describing later abandoned plans to charge individual Girl Scout camps $257 to $1,439 annually for a license to sing songs such as God Bless America around a campfire).

 

A second, equally important, cause for concern arises out of the fact that copyright extension imposes a “permissions” requirement – not only upon potential users of “classic” works that still retain commercial value, but also upon potential users of any other work still in copyright. Again using CRS estimates, one can estimate that, by 2018, the number of such works 75 years of age or older will be about 350,000. See Brief for Petitioners 7. Because the Copyright Act of 1976 abolished the requirement that an owner must renew a copyright, such still-in-copyright works (of little or no commercial value) will eventually number in the millions.

 

The potential users of such works include not only movie buffs and aging jazz fans, but also historians, scholars, teachers, writers, artists, database operators, and researchers of all kinds – those who want to make the past accessible for their own use or for that of others. The permissions requirement can inhibit their ability to accomplish that task. Indeed, in an age where computer-accessible databases promise to facilitate research and learning, the permissions requirement can stand as a significant obstacle to realization of that technological hope.

 

The reason is that the permissions requirement can inhibit or prevent the use of old works (particularly those without commercial value): (1) because it may prove expensive to track down or to contract with the copyright holder, (2) because the holder may prove impossible to find, or (3) because the holder when found may deny permission either outright or through misinformed efforts to bargain. The CRS, for example, has found that the cost of seeking permission “can be prohibitive.” CRS Report 4. And amici, along with petitioners, provide examples of the kinds of significant harm at issue.

 

Thus, the American Association of Law Libraries points out that the clearance process associated with creating an electronic archive, Documenting the American South, “consumed approximately a dozen man-hours” per work. The College Art Association says that the costs of obtaining permission for use of single images, short excerpts, and other short works can become prohibitively high; it describes the abandonment of efforts to include, e. g., campaign songs, film excerpts, and documents exposing “horrors of the chain gang” in historical works or archives; and it points to examples in which copyright holders in effect have used their control of copyright to try to control the content of historical or cultural works. … . Petitioners point to music fees that may prevent youth or community orchestras, or church choirs, from performing early 20th-century music. Brief for Petitioners 3-5; see also App. 16-17 (Copyright extension caused abandonment of plans to sell sheet music of Maurice Ravel’s Alborada Del Gracioso). Amici for petitioners describe how electronic databases tend to avoid adding to their collections works whose copyright holders may prove difficult to contact, see, e. g., Arms, Getting the Picture: Observations from the Library of Congress on Providing Online Access to Pictorial Images, 48 Library Trends 379, 405 (1999) (describing how this tendency applies to the Library of Congress’ own digital archives).

 

As I have said, to some extent costs of this kind accompany any copyright law, regardless of the length of the copyright term. But to extend that term, preventing works from the 1920’s and 1930’s from falling into the public domain, will dramatically increase the size of the costs just as – perversely – the likely benefits from protection diminish. The older the work, the less likely it retains commercial value, and the harder it will likely prove to find the current copyright holder. The older the work, the more likely it will prove useful to the historian, artist, or teacher. The older the work, the less likely it is that a sense of authors’ rights can justify a copyright holder’s decision not to permit reproduction, for the more likely it is that the copyright holder making the decision is not the work’s creator, but, say, a corporation or a great-grandchild whom the work’s creator never knew. Similarly, the costs of obtaining permission, now perhaps ranging in the millions of dollars, will multiply as the number of holders of affected copyrights increases from several hundred thousand to several million. The costs to the users of nonprofit databases, now numbering in the low millions, will multiply as the use of those computer-assisted databases becomes more prevalent. See, e. g., Brief for Internet Archive et al. as Amici Curiae 2, 21, and n. 37 (describing nonprofit Project Gutenberg). And the qualitative costs to education, learning, and research will multiply as our children become ever more dependent for the content of their knowledge upon computer-accessible databases – thereby condemning that which is not so accessible, say, the cultural content of early 20th-century history, to a kind of intellectual purgatory from which it will not easily emerge.

 

The majority … invokes the “fair use” exception, and it notes that copyright law itself is restricted to protection of a work’s expression, not its substantive content. Neither the exception nor the restriction, however, would necessarily help those who wish to obtain from electronic databases material that is not there – say, teachers wishing their students to see albums of Depression Era photographs, to read the recorded words of those who actually lived under slavery, or to contrast, say, Gary Cooper’s heroic portrayal of Sergeant York with filmed reality from the battlefield of Verdun. Such harm, and more will occur despite the 1998 Act’s exemptions and despite the other “First Amendment safeguards” in which the majority places its trust.

 

I should add that the Motion Picture Association of America also finds my concerns overstated, at least with respect to films, because the extension will sometimes make it profitable to reissue old films, saving them from extinction. Other film preservationists note, however, that only a small minority of the many films, particularly silent films, from the 1920’s and 1930’s have been preserved. 1 Report of the Librarian of Congress, Film Preservation 1993, pp. 3-4 (Half of all pre-1950 feature films and more than 80% of all such pre-1929 films have already been lost); cf. Brief for Hal Roach Studios et al. as Amici Curiae 18 (Out of 1,200 Twenties Era silent films still under copyright, 63 are now available on digital video disc). They seek to preserve the remainder. See, e. g., Brief for Internet Archive et al. as Amici Curiae 22 (Nonprofit database digitized 1,001 public-domain films, releasing them online without charge); 1 Film Preservation 1993, supra, at 23 (reporting well over 200,000 titles held in public archives). And they tell us that copyright extension will impede preservation by forbidding the reproduction of films within their own or within other public collections.

 

Because this subsection concerns only costs, not countervailing benefits, I shall simply note here that, with respect to films as with respect to other works, extension does cause substantial harm to efforts to preserve and to disseminate works that were created long ago. And I shall turn to the second half of the equation: Could Congress reasonably have found that the extension’s toll-related and permissions-related harms are justified by extension’s countervailing preservationist incentives or in other ways?

 

 

 

 

 

C.

 

 

 

 

 

What copyright-related benefits might justify the statute’s extension of copyright protection? First, no one could reasonably conclude that copyright’s traditional economic rationale applies here. The extension will not act as an economic spur encouraging authors to create new works. See Mazer, 347 U. S., at 219 (The “economic philosophy” of the Copyright Clause is to “advance public welfare” by “encourag[ing] individual effort” through “personal gain”). No potential author can reasonably believe that he has more than a tiny chance of writing a classic that will survive commercially long enough for the copyright extension to matter. After all, if, after 55 to 75 years, only 2% of all copyrights retain commercial value, the percentage surviving after 75 years or more (a typical pre-extension copyright term) – must be far smaller. See CRS Report 7 (estimating that, even after copyright renewal, about 3.8% of copyrighted books go out of print each year). And any remaining monetary incentive is diminished dramatically by the fact that the relevant royalties will not arrive until 75 years or more into the future, when, not the author, but distant heirs, or shareholders in a successor corporation, will receive them. Using assumptions about the time value of money provided us by a group of economists (including five Nobel prize winners), it seems fair to say that, for example, a 1% likelihood of earning $100 annually for 20 years, starting 75 years into the future, is worth less than seven cents today.

 

What potential Shakespeare, Wharton, or Hemingway would be moved by such a sum? What monetarily motivated Melville would not realize that he could do better for his grandchildren by putting a few dollars into an interest-bearing bank account? The Court itself finds no evidence to the contrary. It refers to testimony before Congress (1) that the copyright system’s incentives encourage creation, and (2) (referring to Noah Webster) that income earned from one work can help support an artist who “‘continue[s] to create.’” But the first of these amounts to no more than a set of undeniably true propositions about the value of incentives in general. And the applicability of the second to this Act is mysterious. How will extension help today’s Noah Webster create new works 50 years after his death? Or is that hypothetical Webster supposed to support himself with the extension’s present discounted value, i. e., a few pennies? Or (to change the metaphor) is the argument that Dumas fils would have written more books had Dumas pere’s Three Musketeers earned more royalties?

 

Regardless, even if this cited testimony were meant more specifically to tell Congress that somehow, somewhere, some potential author might be moved by the thought of great-grandchildren receiving copyright royalties a century hence, so might some potential author also be moved by the thought of royalties being paid for two centuries, five centuries, 1,000 years, “‘til the End of Time.” And from a rational economic perspective the time difference among these periods makes no real difference. The present extension will produce a copyright period of protection that, even under conservative assumptions, is worth more than 99.8% of protection in perpetuity (more than 99.99% for a songwriter like Irving Berlin and a song like Alexander’s Ragtime Band). The lack of a practically meaningful distinction from an author’s ex ante perspective between (a) the statute’s extended terms and (b) an infinite term makes this latest extension difficult to square with the Constitution’s insistence on “limited Times.”

 

I am not certain why the Court considers it relevant in this respect that “[n]othing … warrants construction of the [1998 Act’s] 20-year term extension as a congressional attempt to evade or override the ‘limited Times’ constraint.” Of course Congress did not intend to act unconstitutionally. But it may have sought to test the Constitution’s limits. After all, the statute was named after a Member of Congress, who, the legislative history records, “wanted the term of copyright protection to last forever.” 144 Cong. Rec. H9952 (daily ed. Oct. 7, 1998) (statement of Rep. Mary Bono). See also Copyright Term, Film Labeling, and Film Preservation Legislation: Hearings on H. R. 989 et al. before the Subcommittee on Courts and Intellectual Property of the House Judiciary Committee, 104th Cong., 1st Sess., 94 (1995) (hereinafter House Hearings) (statement of Rep. Sonny Bono) (questioning why copyrights should ever expire); ibid. (statement of Rep. Berman) (“I guess we could … just make a permanent moratorium on the expiration of copyrights”); id., at 230 (statement of Rep. Hoke) (“Why 70 years? Why not forever? Why not 150 years?”); cf. ibid. (statement of the Register of Copyrights) (In Copyright Office proceedings, “[t]he Songwriters Guild suggested a perpetual term”); id., at 234 (statement of Quincy Jones) (”I’m particularly fascinated with Representative Hoke’s statement… . [W]hy not forever?”); id., at 277 (statement of Quincy Jones) (“If we can start with 70, add 20, it would be a good start”). And the statute ended up creating a term so long that (were the vesting of 19th-century real property at issue) it would typically violate the traditional rule against perpetuities. See 10 R. Powell, Real Property §§ 71.02[2]-[3], p. 71-11 (M. Wolf ed. 2002) (traditional rule that estate must vest, if at all, within lives in being plus 21 years); cf. id., § 71.03, at 71-15 (modern statutory perpetuity term of 90 years, 5 years shorter than 95-year copyright terms).

 

In any event, the incentive-related numbers are far too small for Congress to have concluded rationally, even with respect to new works, that the extension’s economic-incentive effect could justify the serious expression-related harms earlier described. See Part II-B, supra. And, of course, in respect to works already created – the source of many of the harms previously described – the statute creates no economic incentive at all.

 

Second, the Court relies heavily for justification upon international uniformity of terms. Ante, at 196, 205-206. Although it can be helpful to look to international norms and legal experience in understanding American law, in this case the justification based upon foreign rules is surprisingly weak. Those who claim that significant copyright-related benefits flow from greater international uniformity of terms point to the fact that the nations of the European Union have adopted a system of copyright terms uniform among themselves. And the extension before this Court implements a term of life plus 70 years that appears to conform with the European standard. But how does “uniformity” help to justify this statute?

 

Despite appearances, the statute does not create a uniform American-European term with respect to the lion’s share of the economically significant works that it affects – all works made “for hire” and all existing works created prior to 1978. … .

 

What benefit, then, might this partial future uniformity achieve? The majority refers to “greater incentive for American and other authors to create and disseminate their work in the United States,” and cites a law review article suggesting a need to “‘avoid competitive disadvantages.’” The Solicitor General elaborates on this theme, postulating that because uncorrected disuniformity would permit Europe, not the United States, to hold out the prospect of protection lasting for “life plus 70 years” (instead of “life plus 50 years”), a potential author might decide to publish initially in Europe, delaying American publication. And the statute, by creating a uniformly longer term, corrects for the disincentive that this disuniformity might otherwise produce.

 

That disincentive, however, could not possibly bring about serious harm of the sort that the Court, the Solicitor General, or the law review author fears. … . As we have seen, the present commercial value of any such difference amounts at most to comparative pennies. And a commercial decision that turned upon such a difference would have had to have rested previously upon a knife edge so fine as to be invisible. A rational legislature could not give major weight to an invisible, likely nonexistent incentive-related effect.

 

But if there is no incentive-related benefit, what is the benefit of the future uniformity that the statute only partially achieves? Unlike the Copyright Act of 1976, this statute does not constitute part of an American effort to conform to an important international treaty like the Berne Convention. … .

 

In sum, the partial, future uniformity that the 1998 Act promises cannot reasonably be said to justify extension of the copyright term for new works. And concerns with uniformity cannot possibly justify the extension of the new term to older works, for the statute there creates no uniformity at all.

 

Third, several publishers and filmmakers argue that the statute provides incentives to those who act as publishers to republish and to redistribute older copyrighted works. This claim cannot justify this statute, however, because the rationale is inconsistent with the basic purpose of the Copyright Clause – as understood by the Framers and by this Court. The Clause assumes an initial grant of monopoly, designed primarily to encourage creation, followed by termination of the monopoly grant in order to promote dissemination of already-created works. It assumes that it is the disappearance of the monopoly grant, not its perpetuation, that will, on balance, promote the dissemination of works already in existence. This view of the Clause does not deny the empirical possibility that grant of a copyright monopoly to the heirs or successors of a long-dead author could on occasion help publishers resurrect the work, say, of a long-lost Shakespeare. But it does deny Congress the Copyright Clause power to base its actions primarily upon that empirical possibility – lest copyright grants become perpetual, lest on balance they restrict dissemination, lest too often they seek to bestow benefits that are solely retroactive.

 

[Justice Breyer reviews the history and text of the clause to buttress this view of the Clause.]

 

[This view] finds empirical support in sources that underscore the wisdom of the Framers’ judgment. See CRS Report 3 (“[N]ew, cheaper editions can be expected when works come out of copyright”); see also Part II-B, supra. And it draws logical support from the endlessly self-perpetuating nature of the publishers’ claim and the difficulty of finding any kind of logical stopping place were this Court to accept such a uniquely publisher-related rationale. (Would it justify continuing to extend copyrights indefinitely, say, for those granted to F. Scott Fitzgerald or his lesser known contemporaries? Would it not, in principle, justify continued protection of the works of Shakespeare, Melville, Mozart, or perhaps Salieri, Mozart’s currently less popular contemporary? Could it justify yet further extension of the copyright on the song Happy Birthday to You (melody first published in 1893, song copyrighted after litigation in 1935), still in effect and currently owned by a subsidiary of AOL Time Warner? See Profitable “Happy Birthday,” Times of London, Aug. 5, 2000, p. 6.)

 

Given this support, it is difficult to accept the conflicting rationale that the publishers advance, namely, that extension, rather than limitation, of the grant will, by rewarding publishers with a form of monopoly, promote, rather than retard, the dissemination of works already in existence. Indeed, given these considerations, this rationale seems constitutionally perverse – unable, constitutionally speaking, to justify the blanket extension here at issue.

 

Fourth, the statute’s legislative history suggests another possible justification. That history refers frequently to the financial assistance the statute will bring the entertainment industry, particularly through the promotion of exports. … . I can find nothing in the Copyright Clause that would authorize Congress to enhance the copyright grant’s monopoly power, likely leading to higher prices both at home and abroad, solely in order to produce higher foreign earnings. That objective is not a copyright objective. Nor, standing alone, is it related to any other objective more closely tied to the Clause itself. Neither can higher corporate profits alone justify the grant’s enhancement. The Clause seeks public, not private, benefits.

 

Finally, the Court mentions as possible justifications “demographic, economic, and technological changes” – by which the Court apparently means the facts that today people communicate with the help of modern technology, live longer, and have children at a later age. The first fact seems to argue not for, but instead against, extension. See Part II-B, supra. The second fact seems already corrected for by the 1976 Act’s life-plus-50 term, which automatically grows with lifespans. And the third fact – that adults are having children later in life – is a makeweight at best, providing no explanation of why the 1976 Act’s term of 50 years after an author’s death – a longer term than was available to authors themselves for most of our Nation’s history – is an insufficient potential bequest. The weakness of these final rationales simply underscores the conclusion that emerges from consideration of earlier attempts at justification: There is no legitimate, serious copyright-related justification for this statute.

 

 

 

 

III

 

 

 

 

The Court is concerned that our holding in this case not inhibit the broad decisionmaking leeway that the Copyright Clause grants Congress. It is concerned about the implications of today’s decision for the Copyright Act of 1976 – an Act that changed copyright’s basic term from 56 years (assuming renewal) to life of the author plus 50 years. It is concerned about having to determine just how many years of copyright is too many – a determination that it fears would require it to find the “right” constitutional number, a task for which the Court is not well suited.

 

I share the Court’s initial concern, about intrusion upon the decisionmaking authority of Congress. But I do not believe it intrudes upon that authority to find the statute unconstitutional on the basis of (1) a legal analysis of the Copyright Clause’s objectives; (2) the total implausibility of any incentive effect; and (3) the statute’s apparent failure to provide significant international uniformity. Nor does it intrude upon congressional authority to consider rationality in light of the expressive values underlying the Copyright Clause, related as it is to the First Amendment, and given the constitutional importance of correctly drawing the relevant Clause/Amendment boundary. We cannot avoid the need to examine the statute carefully by saying that “Congress has not altered the traditional contours of copyright protection,” for the sentence points to the question, rather than the answer. Nor should we avoid that examination here. That degree of judicial vigilance – at the far outer boundaries of the Clause – is warranted if we are to avoid the monopolies and consequent restrictions of expression that the Clause, read consistently with the First Amendment, seeks to preclude. And that vigilance is all the more necessary in a new century that will see intellectual property rights and the forms of expression that underlie them play an ever more important role in the Nation’s economy and the lives of its citizens.

 

I do not share the Court’s concern that my view of the 1998 Act could automatically doom the 1976 Act. Unlike the present statute, the 1976 Act thoroughly revised copyright law and enabled the United States to join the Berne Convention – an international treaty that requires the 1976 Act’s basic life-plus-50 term as a condition for substantive protections from a copyright’s very inception, Berne Conv. Art. 7(1). Consequently, the balance of copyright-related harms and benefits there is far less one sided. The same is true of the 1909 and 1831 Acts, which, in any event, provided for maximum terms of 56 years or 42 years while requiring renewal after 28 years, with most copyrighted works falling into the public domain after that 28-year period, well before the putative maximum terms had elapsed. Regardless, the law provides means to protect those who have reasonably relied upon prior copyright statutes. And, in any event, we are not here considering, and we need not consider, the constitutionality of other copyright statutes.

 

Neither do I share the Court’s aversion to line-drawing in this case. Even if it is difficult to draw a single clear bright line, the Court could easily decide (as I would decide) that this particular statute simply goes too far. And such examples – of what goes too far – sometimes offer better constitutional guidance than more absolute-sounding rules. In any event, “this Court sits” in part to decide when a statute exceeds a constitutional boundary. In my view, “[t]ext, history, and precedent,” support both the need to draw lines in general and the need to draw the line here short of this statute.

 

Finally, the Court complains that I have not “restrained” my argument or “train[ed my] fire, as petitioners do, on Congress’ choice to place existing and future copyrights in parity.” The reason that I have not so limited my argument is my willingness to accept, for purposes of this opinion, the Court’s understanding that, for reasons of “[j]ustice, policy, and equity” – as well as established historical practice – it is not “categorically beyond Congress’ authority” to “exten[d] the duration of existing copyrights” to achieve such parity. I have accepted this view, however, only for argument’s sake – putting to the side, for the present, JUSTICE STEVENS’ persuasive arguments to the contrary. And I make this assumption only to emphasize the lack of rational justification for the present statute. A desire for “parity” between A (old copyrights) and B (new copyrights) cannot justify extending A when there is no rational justification for extending B. At the very least (if I put aside my rationality characterization), to ask B to support A here is like asking Tom Thumb to support Paul Bunyan’s ox. Where the case for extending new copyrights is itself so weak, what “justice,” what “policy,” what “equity” can warrant the tolls and barriers that extension of existing copyrights imposes?

 

 

 

 

IV

 

 

 

 

This statute will cause serious expression-related harm. It will likely restrict traditional dissemination of copyrighted works. It will likely inhibit new forms of dissemination through the use of new technology. It threatens to interfere with efforts to preserve our Nation’s historical and cultural heritage and efforts to use that heritage, say, to educate our Nation’s children. It is easy to understand how the statute might benefit the private financial interests of corporations or heirs who own existing copyrights. But I cannot find any constitutionally legitimate, copyright-related way in which the statute will benefit the public. Indeed, in respect to existing works, the serious public harm and the virtually nonexistent public benefit could not be more clear.

 

I have set forth the analysis upon which I rest these judgments. This analysis leads inexorably to the conclusion that the statute cannot be understood rationally to advance a constitutionally legitimate interest. The statute falls outside the scope of legislative power that the Copyright Clause, read in light of the First Amendment, grants to Congress. I would hold the statute unconstitutional.

 

I respectfully dissent.

 

 

 

APPENDIX TO OPINION OF BREYER, J.

 

 

 

 

A

 

 

 

 

The text’s estimates of the economic value of 1998 Act copyrights relative to the economic value of a perpetual copyright as well as the incremental value of a 20-year extension of a 75-year term rest upon the conservative future value and discount rate assumptions set forth in the brief of economist amici. Brief for George A. Akerlof et al. as Amici Curiae 5-7. Under these assumptions, if an author expects to live 30 years after writing a book, the copyright extension (by increasing the copyright term from “life of the author plus 50 years” to “life of the author plus 70 years”) increases the author’s expected income from that book – i. e., the economic incentive to write – by no more than about 0.33%.

 

The text assumes that the extension creates a term of 95 years (the term corresponding to works made for hire and for all existing pre-1978 copyrights). Under the economists’ conservative assumptions, the value of a 95-year copyright is slightly more than 99.8% of the value of a perpetual copyright. If a “life plus 70” term applies, and if an author lives 78 years after creation of a work (as with Irving Berlin and Alexander’s Ragtime Band), the same assumptions yield a figure of 99.996%.

 

The most unrealistically conservative aspect of these assumptions, i. e., the aspect most unrealistically favorable to the majority, is the assumption of a constant future income stream. In fact, as noted in the text, uncontested data indicate that no author could rationally expect that a stream of copyright royalties will be constant forever. Indeed, only about 2% of copyrights can be expected to retain commercial value at the end of 55 to 75 years. Thus, in the overwhelming majority of cases, the ultimate value of the extension to copyright holders will be zero, and the economic difference between the extended copyright and a perpetual copyright will be zero.

 

Nonetheless, there remains a small 2% or so chance that a given work will remain profitable. The CRS Report suggests a way to take account of both that likelihood and the related “decay” in a work’s commercial viability: Find the annual decay rate that corresponds to the percentage of works that become commercially unavailable in any given year, and then discount the revenue for each successive year accordingly. See CRS Report 7. Following this approach, if one estimates, conservatively, that a full 2% of all works survives at the end of 75 years, the corresponding annual decay rate is about 5%. I instead (and again conservatively) use the 3.8% decay rate the CRS has applied in the case of books whose copyrights were renewed between 1950 and 1970. Using this 3.8% decay rate and the economist amici’s proposed 7% discount rate, the value of a 95-year copyright is more realistically estimated not as 99.8%, but as 99.996% of the value of a perpetual copyright. The comparable “Irving Berlin” figure is 99.99999%. (With a 5% decay rate, the figures are 99.999% and 99.999998%, respectively.) Even these figures seem likely to be underestimates in the sense that they assume that, if a work is still commercially available, it earns as much as it did in a year shortly after its creation.

 

 

 

 

B

 

 

 

 

Conclusions regarding the economic significance of “works made for hire” are judgmental because statistical information about the ratio of “for hire” works to all works is scarce. But we know that, as of 1955, copyrights on “for hire” works accounted for 40% of newly registered copyrights. We also know that copyrights on works typically made for hire – feature-length movies – were renewed, and since the 1930’s apparently have remained commercially viable, at a higher than average rate. Further, we know that “harmonization” looks to benefit United States exports and that films and sound recordings account for the dominant share of export revenues earned by new copyrighted works of potential lasting commercial value (i. e., works other than computer software). It also appears generally accepted that, in these categories, “for hire” works predominate. Taken together, these circumstances support the conclusion in the text that the extension fails to create uniformity where it would appear to be most important – pre-1978 copyrighted works nearing the end of their pre-extension terms, and works made for hire.

 


  1. [The court listed the amici in this case. Among those for reversal were the American Association of Law Libraries, the College Art Association, the Eagle Forum Education & Legal Defense Fund, the Free Software Foundation, groups of intellectual property and constitutional law professors, and the Internet Archive. Among those writing in support of affirmance were the American Intellectual Property Law Association, the American Society of Composers, Authors and Publishers (ASCAP), Amsong, Inc., AOL Time Warner, the Association of American Publishers, the Bureau of National Affairs, the Directors Guild of America, Dr. Seuss Enterprises, the Motion Picture Association of America, the Recording Artists Coalition, the Recording Industry Association of America (RIAA), the Songwriters Guild of America, Senator Orrin G. Hatch, and Representative F. James Sensenbrenner, Jr.]

 

  1. JUSTICE BREYER’s dissent is not similarly restrained. He makes no effort meaningfully to distinguish existing copyrights from future grants. Under his reasoning, the CTEA’s 20-year extension is globally unconstitutional.

 

  1. Asserting that the last several decades have seen a proliferation of copyright legislation in departure from Congress’ traditional pace of legislative amendment in this area, petitioners cite nine statutes passed between 1962 and 1974, each of which incrementally extended existing copyrights for brief periods. As respondent (Attorney General Ashcroft) points out, however, these statutes were all temporary placeholders subsumed into the systemic changes effected by the 1976 Act.

 

  1. Petitioners argue that the 1790 Act must be distinguished from the later Acts on the ground that it covered existing works but did not extend existing copyrights. The parties disagree on the question whether the 1790 Act’s copyright term should be regarded in part as compensation for the loss of any then existing state- or common-law copyright protections. Without resolving that dispute, we underscore that the First Congress clearly did confer copyright protection on works that had already been created.

 

  1. We note again that JUSTICE BREYER makes no such concession. He does not train his fire, as petitioners do, on Congress’ choice to place existing and future copyrights in parity. Moving beyond the bounds of the parties’ presentations, and with abundant policy arguments but precious little support from precedent, he would condemn Congress’ entire product as irrational.

 

  1. Moreover, the precise duration of a federal copyright has never been fixed at the time of the initial grant. The 1790 Act provided a federal copyright term of 14 years from the work’s publication, renewable for an additional 14 years if the author survived and applied for an additional term. § 1. Congress retained that approach in subsequent statutes. Similarly, under the method for measuring copyright terms established by the 1976 Act and retained by the CTEA, the baseline copyright term is measured in part by the life of the author, rendering its duration indeterminate at the time of the grant.

 

  1. Responding to an inquiry whether copyrights could be extended “forever,” Register of Copyrights Marybeth Peters emphasized the dominant reason for the CTEA: “There certainly are proponents of perpetual copyright: We heard that in our proceeding on term extension. The Songwriters Guild suggested a perpetual term. However, our Constitution says limited times, but there really isn’t a very good indication on what limited times is. The reason why you’re going to life-plus-70 today is because Europe has gone that way … .”

 

  1. The author of the law review article cited in text, Shira Perlmutter, currently a vice president of AOL Time Warner, was at the time of the CTEA’s enactment Associate Register for Policy and International Affairs, United States Copyright Office.

 

  1. See also Austin, Does the Copyright Clause Mandate Isolationism?, 26 Colum. J. L. & Arts 17, 59 (2002) (cautioning against “an isolationist reading of the Copyright Clause that is in tension with … America’s international copyright relations over the last hundred or so years”)

 

  1. Members of Congress expressed the view that, as a result of increases in human longevity and in parents’ average age when their children are born, the pre-CTEA term did not adequately secure “the right to profit from licensing one’s work during one’s lifetime and to take pride and comfort in knowing that one’s children – and perhaps their children – might also benefit from one’s posthumous popularity.” Also cited was “the failure of the U.S. copyright term to keep pace with the substantially increased commercial life of copyrighted works resulting from the rapid growth in communications media.”

 

  1. JUSTICE BREYER urges that the economic incentives accompanying copyright term extension are too insignificant to “mov[e]” any author with a “rational economic perspective.” Calibrating rational economic incentives, however, like “fashion[ing] … new rules [in light of] new technology,” is a task primarily for Congress, not the courts. Congress heard testimony from a number of prominent artists; each expressed the belief that the copyright system’s assurance of fair compensation for themselves and their heirs was an incentive to create. See, e. g., House Hearings 233-239 (statement of Quincy Jones); Copyright Term Extension Act of 1995: Hearing before the Senate Committee on the Judiciary, 104th Cong., 1st Sess., 55-56 (1995) (statement of Bob Dylan); id., at 56-57 (statement of Don Henley); id., at 57 (statement of Carlos Santana). We would not take Congress to task for crediting this evidence which, as JUSTICE BREYER acknowledges, reflects general “propositions about the value of incentives” that are “undeniably true.”

     

    Congress also heard testimony from Register of Copyrights Marybeth Peters and others regarding the economic incentives created by the CTEA. According to the Register, extending the copyright for existing works “could … provide additional income that would finance the production and publication of new works.” House Hearings 158. “Authors would not be able to continue to create,” the Register explained, “unless they earned income on their finished works. The public benefits not only from an author’s original work but also from his or her further creations. Although this truism may be illustrated in many ways, one of the best examples is Noah Webster[,] who supported his entire family from the earnings on his speller and grammar during the twenty years he took to complete his dictionary.” Id., at 165.

 

  1. JUSTICE BREYER agrees that “Congress did not intend to act unconstitutionally” when it enacted the CTEA, yet in his very next breath, he seems to make just that accusation. What else is one to glean from his selection of scattered statements from individual Members of Congress? He does not identify any statement in the statutory text that installs a perpetual copyright, for there is none. But even if the statutory text were sufficiently ambiguous to warrant recourse to legislative history, JUSTICE BREYER’S selections are not the sort to which this Court accords high value: “In surveying legislative history we have repeatedly stated that the authoritative source for finding the Legislature’s intent lies in the Committee Reports on the bill, which ‘represen[t] the considered and collective understanding of those [Members of Congress] involved in drafting and studying proposed legislation.’” The House and Senate Reports accompanying the CTEA reflect no purpose to make copyright a forever thing. Notably, the Senate Report expressly acknowledged that the Constitution “clearly precludes Congress from granting unlimited protection for copyrighted works,” and disclaimed any intent to contravene that prohibition. Members of Congress instrumental in the CTEA’s passage spoke to similar effect.

     

    JUSTICE BREYER nevertheless insists that the “economic effect” of the CTEA is to make the copyright term “virtually perpetual.” Relying on formulas and assumptions provided in an amicus brief supporting petitioners, he stresses that the CTEA creates a copyright term worth 99.8% of the value of a perpetual copyright. If JUSTICE BREYER’S calculations were a basis for holding the CTEA unconstitutional, then the 1976 Act would surely fall as well, for – under the same assumptions he indulges – the term set by that Act secures 99.4% of the value of a perpetual term. Indeed, on that analysis even the “limited” character of the 1909 (97.7%) and 1831 (94.1%) Acts might be suspect. JUSTICE BREYER several times places the Founding Fathers on his side. It is doubtful, however, that those architects of our Nation, in framing the “limited Times” prescription, thought in terms of the calculator rather than the calendar.

 

  1. Respondent notes that the CTEA’s life-plus-70-years baseline term is expected to produce an average copyright duration of 95 years, and that this term “resembles some other long-accepted durational practices in the law, such as 99-year leases of real property and bequests within the rule against perpetuities.” Whether such referents mark the outer boundary of “limited Times” is not before us today. JUSTICE BREYER suggests that the CTEA’s baseline term extends beyond that typically permitted by the traditional rule against perpetuities. The traditional common-law rule looks to lives in being plus 21 years. Under that rule, the period before a bequest vests could easily equal or exceed the anticipated average copyright term under the CTEA. If, for example, the vesting period on a deed were defined with reference to the life of an infant, the sum of the measuring life plus 21 years could commonly add up to 95 years.

 

  1. As we have noted, petitioners seek to distinguish the 1790 Act from those that followed. They argue that by requiring authors seeking its protection to surrender whatever rights they had under state law, the 1790 Act enhanced uniformity and certainty and thus “promote[d] … Progress.” This account of the 1790 Act simply confirms, however, that the First Congress understood it could “promote … Progress” by extending copyright protection to existing works. Every subsequent adjustment of copyright’s duration, including the CTEA, reflects a similar understanding.

 

  1. The fact that patent and copyright involve different exchanges does not, of course, mean that we may not be guided in our “limited Times” analysis by Congress’ repeated extensions of existing patents. If patent’s quid pro quo is more exacting than copyright’s, then Congress’ repeated extension of existing patents without constitutional objection suggests even more strongly that similar legislation with respect to copyrights is constitutionally permissible.

 

 

 

3.4.4. Secondary Liability

 

Sony Corp. of America v. Universal City Studios,

464 U.S. 417 (1983)

 

 

 

Dean C. Dunlavey, Los Angeles, Cal., for petitioners.

 

Stephen A. Kroft, Beverly Hills, Cal., for respondents.

 

 

 

Justice Stevens delivered the opinion of the Court.

 

Petitioners manufacture and sell home video tape recorders. Respondents own the copyrights on some of the television programs that are broadcast on the public airwaves. Some members of the general public use video tape recorders sold by petitioners to record some of these broadcasts, as well as a large number of other broadcasts. The question presented is whether the sale of petitioners’ copying equipment to the general public violates any of the rights conferred upon respondents by the Copyright Act.

 

Respondents commenced this copyright infringement action against petitioners in the United States District Court for the Central District of California in 1976. Respondents alleged that some individuals had used Betamax video tape recorders (VTR’s) to record some of respondents’ copyrighted works which had been exhibited on commercially sponsored television and contended that these individuals had thereby infringed respondents’ copyrights. Respondents further maintained that petitioners were liable for the copyright infringement allegedly committed by Betamax consumers because of petitioners’ marketing of the Betamax VTR’s. Respondents sought no relief against any Betamax consumer. Instead, they sought money damages and an equitable accounting of profits from petitioners, as well as an injunction against the manufacture and marketing of Betamax VTR’s.

 

After a lengthy trial, the District Court denied respondents all the relief they sought and entered judgment for petitioners. 480 F.Supp. 429 (1979). The United States Court of Appeals for the Ninth Circuit reversed the District Court’s judgment on respondent’s copyright claim, holding petitioners liable for contributory infringement and ordering the District Court to fashion appropriate relief. 659 F.2d 963 (1981). We granted certiorari; since we had not completed our study of the case last Term, we ordered reargument. We now reverse.

 

An explanation of our rejection of respondents’ unprecedented attempt to impose copyright liability upon the distributors of copying equipment requires a quite detailed recitation of the findings of the District Court. In summary, those findings reveal that the average member of the public uses a VTR principally to record a program he cannot view as it is being televised and then to watch it once at a later time. This practice, known as “time-shifting,” enlarges the television viewing audience. For that reason, a significant amount of television programming may be used in this manner without objection from the owners of the copyrights on the programs. For the same reason, even the two respondents in this case, who do assert objections to time-shifting in this litigation, were unable to prove that the practice has impaired the commercial value of their copyrights or has created any likelihood of future harm. Given these findings, there is no basis in the Copyright Act upon which respondents can hold petitioners liable for distributing VTR’s to the general public. The Court of Appeals’ holding that respondents are entitled to enjoin the distribution of VTR’s, to collect royalties on the sale of such equipment, or to obtain other relief, if affirmed, would enlarge the scope of respondents’ statutory monopolies to encompass control over an article of commerce that is not the subject of copyright protection. Such an expansion of the copyright privilege is beyond the limits of the grants authorized by Congress.

 

 

 

 

I

 

 

 

 

The two respondents in this action, Universal Studios, Inc. and Walt Disney Productions, produce and hold the copyrights on a substantial number of motion pictures and other audiovisual works. In the current marketplace, they can exploit their rights in these works in a number of ways: by authorizing theatrical exhibitions, by licensing limited showings on cable and network television, by selling syndication rights for repeated airings on local television stations, and by marketing programs on prerecorded videotapes or videodiscs. Some works are suitable for exploitation through all of these avenues, while the market for other works is more limited.

 

Petitioner Sony manufactures millions of Betamax video tape recorders and markets these devices through numerous retail establishments, some of which are also petitioners in this action. Sony’s Betamax VTR is a mechanism consisting of three basic components: (1) a tuner, which receives electromagnetic signals transmitted over the television band of the public airwaves and separates them into audio and visual signals; (2) a recorder, which records such signals on a magnetic tape; and (3) an adapter, which converts the audio and visual signals on the tape into a composite signal that can be received by a television set.

 

Several capabilities of the machine are noteworthy. The separate tuner in the Betamax enables it to record a broadcast off one station while the television set is tuned to another channel, permitting the viewer, for example, to watch two simultaneous news broadcasts by watching one “live” and recording the other for later viewing. Tapes may be reused, and programs that have been recorded may be erased either before or after viewing. A timer in the Betamax can be used to activate and deactivate the equipment at predetermined times, enabling an intended viewer to record programs that are transmitted when he or she is not at home. Thus a person may watch a program at home in the evening even though it was broadcast while the viewer was at work during the afternoon. The Betamax is also equipped with a pause button and a fast-forward control. The pause button, when depressed, deactivates the recorder until it is released, thus enabling a viewer to omit a commercial advertisement from the recording, provided, of course, that the viewer is present when the program is recorded. The fast forward control enables the viewer of a previously recorded program to run the tape rapidly when a segment he or she does not desire to see is being played back on the television screen.

 

The respondents and Sony both conducted surveys of the way the Betamax machine was used by several hundred owners during a sample period in 1978. Although there were some differences in the surveys, they both showed that the primary use of the machine for most owners was “time-shifting,” – the practice of recording a program to view it once at a later time, and thereafter erasing it. Time-shifting enables viewers to see programs they otherwise would miss because they are not at home, are occupied with other tasks, or are viewing a program on another station at the time of a broadcast that they desire to watch. Both surveys also showed, however, that a substantial number of interviewees had accumulated libraries of tapes.1 Sony’s survey indicated that over 80% of the interviewees watched at least as much regular television as they had before owning a Betamax.2 Respondents offered no evidence of decreased television viewing by Betamax owners.3

 

Sony introduced considerable evidence describing television programs that could be copied without objection from any copyright holder, with special emphasis on sports, religious, and educational programming. For example, their survey indicated that 7.3% of all Betamax use is to record sports events, and representatives of professional baseball, football, basketball, and hockey testified that they had no objection to the recording of their televised events for home use.

 

Respondents offered opinion evidence concerning the future impact of the unrestricted sale of VTR’s on the commercial value of their copyrights. The District Court found, however, that they had failed to prove any likelihood of future harm from the use of VTR’s for time-shifting. Id., at 469.

 

… .

 

 

 

 

III

 

 

 

 

The Copyright Act does not expressly render anyone liable for infringement committed by another. In contrast, the Patent Act expressly brands anyone who “actively induces infringement of a patent” as an infringer, 35 U.S.C. § 271(b), and further imposes liability on certain individuals labeled “contributory” infringers, id., § 271(c). The absence of such express language in the copyright statute does not preclude the imposition of liability for copyright infringements on certain parties who have not themselves engaged in the infringing activity. For vicarious liability is imposed in virtually all areas of the law, and the concept of contributory infringement is merely a species of the broader problem of identifying the circumstances in which it is just to hold one individual accountable for the actions of another.

 

… .

 

When a charge of contributory infringement is predicated entirely on the sale of an article of commerce that is used by the purchaser to infringe a patent, the public interest in access to that article of commerce is necessarily implicated. A finding of contributory infringement does not, of course, remove the article from the market altogether; it does, however, give the patentee effective control over the sale of that item. Indeed, a finding of contributory infringement is normally the functional equivalent of holding that the disputed article is within the monopoly granted to the patentee.4

 

For that reason, in contributory infringement cases arising under the patent laws the Court has always recognized the critical importance of not allowing the patentee to extend his monopoly beyond the limits of his specific grant. These cases deny the patentee any right to control the distribution of unpatented articles unless they are “unsuited for any commercial noninfringing use.” Dawson Chemical Co. v. Rohm & Hass Co., 448 U.S. 176, 198 (1980). Unless a commodity “has no use except through practice of the patented method,” ibid, the patentee has no right to claim that its distribution constitutes contributory infringement. “To form the basis for contributory infringement the item must almost be uniquely suited as a component of the patented invention.” P. Rosenberg, Patent Law Fundamentals § 17.02[2] (1982). “[A] sale of an article which though adapted to an infringing use is also adapted to other and lawful uses, is not enough to make the seller a contributory infringer. Such a rule would block the wheels of commerce.” Henry v. A.B. Dick Co., 224 U.S. 1, 48 (1912).

 

We recognize there are substantial differences between the patent and copyright laws. But in both areas the contributory infringement doctrine is grounded on the recognition that adequate protection of a monopoly may require the courts to look beyond actual duplication of a device or publication to the products or activities that make such duplication possible. The staple article of commerce doctrine must strike a balance between a copyright holder’s legitimate demand for effective – not merely symbolic – protection of the statutory monopoly, and the rights of others freely to engage in substantially unrelated areas of commerce. Accordingly, the sale of copying equipment, like the sale of other articles of commerce, does not constitute contributory infringement if the product is widely used for legitimate, unobjectionable purposes. Indeed, it need merely be capable of substantial noninfringing uses.

 

 

 

 

IV

 

 

 

 

The question is thus whether the Betamax is capable of commercially significant noninfringing uses. In order to resolve that question, we need not explore all the different potential uses of the machine and determine whether or not they would constitute infringement. Rather, we need only consider whether on the basis of the facts as found by the district court a significant number of them would be non-infringing. Moreover, in order to resolve this case we need not give precise content to the question of how much use is commercially significant. For one potential use of the Betamax plainly satisfies this standard, however it is understood: private, noncommercial time-shifting in the home. It does so both (A) because respondents have no right to prevent other copyright holders from authorizing it for their programs, and (B) because the District Court’s factual findings reveal that even the unauthorized home time-shifting of respondents’ programs is legitimate fair use.

 

 

 

 

 

A. Authorized Time Shifting

 

 

 

 

 

Each of the respondents owns a large inventory of valuable copyrights, but in the total spectrum of television programming their combined market share is small. The exact percentage is not specified, but it is well below 10%.5 If they were to prevail, the outcome of this litigation would have a significant impact on both the producers and the viewers of the remaining 90% of the programming in the Nation. No doubt, many other producers share respondents’ concern about the possible consequences of unrestricted copying. Nevertheless the findings of the District Court make it clear that time-shifting may enlarge the total viewing audience and that many producers are willing to allow private time-shifting to continue, at least for an experimental time period.

 

The District Court found:

 

 

Even if it were deemed that home-use recording of copyrighted material constituted infringement, the Betamax could still legally be used to record noncopyrighted material or material whose owners consented to the copying. An injunction would deprive the public of the ability to use the Betamax for this noninfringing off-the-air recording.

* * * Defendants introduced considerable testimony at trial about the potential for such copying of sports, religious, educational and other programming. This included testimony from representatives of the Offices of the Commissioners of the National Football, Basketball, Baseball and Hockey Leagues and Associations, the Executive Director of National Religious Broadcasters and various educational communications agencies. Plaintiffs attack the weight of the testimony offered and also contend that an injunction is warranted because infringing uses outweigh noninfringing uses.

Whatever the future percentage of legal versus illegal home-use recording might be, an injunction which seeks to deprive the public of the very tool or article of commerce capable of some noninfringing use would be an extremely harsh remedy, as well as one unprecedented in copyright law.

 

 

480 F.Supp., at 468.

 

Although the District Court made these statements in the context of considering the propriety of injunctive relief, the statements constitute a finding that the evidence concerning “sports, religious, educational, and other programming” was sufficient to establish a significant quantity of broadcasting whose copying is now authorized, and a significant potential for future authorized copying. That finding is amply supported by the record. In addition to the religious and sports officials identified explicitly by the District Court, two items in the record deserve specific mention.

 

First is the testimony of John Kenaston, the station manager of Channel 58, an educational station in Los Angeles affiliated with the Public Broadcasting Service. He explained and authenticated the station’s published guide to its programs. For each program, the guide tells whether unlimited home taping is authorized, home taping is authorized subject to certain restrictions (such as erasure within seven days), or home taping is not authorized at all. The Spring 1978 edition of the guide described 107 programs. Sixty-two of those programs or 58% authorize some home taping. Twenty-one of them or almost 20% authorize unrestricted home taping.

 

Second is the testimony of Fred Rogers, president of the corporation that produces and owns the copyright on Mr. Rogers’ Neighborhood. The program is carried by more public television stations than any other program. Its audience numbers over 3,000,000 families a day. He testified that he had absolutely no objection to home taping for noncommercial use and expressed the opinion that it is a real service to families to be able to record children’s programs and to show them at appropriate times.6

 

If there are millions of owners of VTR’s who make copies of televised sports events, religious broadcasts, and educational programs such as Mister Rogers’ Neighborhood, and if the proprietors of those programs welcome the practice, the business of supplying the equipment that makes such copying feasible should not be stifled simply because the equipment is used by some individuals to make unauthorized reproductions of respondents’ works. The respondents do not represent a class composed of all copyright holders. Yet a finding of contributory infringement would inevitably frustrate the interests of broadcasters in reaching the portion of their audience that is available only through time-shifting.

 

Of course, the fact that other copyright holders may welcome the practice of time-shifting does not mean that respondents should be deemed to have granted a license to copy their programs. Third party conduct would be wholly irrelevant in an action for direct infringement of respondents’ copyrights. But in an action for contributory infringement against the seller of copying equipment, the copyright holder may not prevail unless the relief that he seeks affects only his programs, or unless he speaks for virtually all copyright holders with an interest in the outcome. In this case, the record makes it perfectly clear that there are many important producers of national and local television programs who find nothing objectionable about the enlargement in the size of the television audience that results from the practice of time-shifting for private home use. The seller of the equipment that expands those producers’ audiences cannot be a contributory infringer if, as is true in this case, it has had no direct involvement with any infringing activity.

 

 

 

 

 

B. Unauthorized Time Shifting

 

 

 

 

 

Even unauthorized uses of a copyrighted work are not necessarily infringing. An unlicensed use of the copyright is not an infringement unless it conflicts with one of the specific exclusive rights conferred by the copyright statute. Twentieth Century Music Corp. v. Aiken, 422 U.S. 151, 154-155, 95 S.Ct. 2040, 2043, 45 L.Ed.2d 84. Moreover, the definition of exclusive rights in § 106 of the present Act is prefaced by the words “subject to sections 107 through 118.” Those sections describe a variety of uses of copyrighted material that “are not infringements of copyright notwithstanding the provisions of § 106.” The most pertinent in this case is § 107, the legislative endorsement of the doctrine of “fair use.”

 

That section identifies various factors7 that enable a Court to apply an “equitable rule of reason” analysis to particular claims of infringement. Although not conclusive, the first factor requires that “the commercial or nonprofit character of an activity” be weighed in any fair use decision. If the Betamax were used to make copies for a commercial or profit-making purpose, such use would presumptively be unfair. The contrary presumption is appropriate here, however, because the District Court’s findings plainly establish that time-shifting for private home use must be characterized as a noncommercial, nonprofit activity. Moreover, when one considers the nature of a televised copyrighted audiovisual work, see 17 U.S.C. § 107(2), and that timeshifting merely enables a viewer to see such a work which he had been invited to witness in its entirety free of charge, the fact that the entire work is reproduced, see id., at § 107(3), does not have its ordinary effect of militating against a finding of fair use.

 

This is not, however, the end of the inquiry because Congress has also directed us to consider “the effect of the use upon the potential market for or value of the copyrighted work.” Id., at § 107(4). The purpose of copyright is to create incentives for creative effort. Even copying for noncommercial purposes may impair the copyright holder’s ability to obtain the rewards that Congress intended him to have. But a use that has no demonstrable effect upon the potential market for, or the value of, the copyrighted work need not be prohibited in order to protect the author’s incentive to create. The prohibition of such noncommercial uses would merely inhibit access to ideas without any countervailing benefit.

 

Thus, although every commercial use of copyrighted material is presumptively an unfair exploitation of the monopoly privilege that belongs to the owner of the copyright, noncommercial uses are a different matter. A challenge to a noncommercial use of a copyrighted work requires proof either that the particular use is harmful, or that if it should become widespread, it would adversely affect the potential market for the copyrighted work. Actual present harm need not be shown; such a requirement would leave the copyright holder with no defense against predictable damage. Nor is it necessary to show with certainty that future harm will result. What is necessary is a showing by a preponderance of the evidence that some meaningful likelihood of future harm exists. If the intended use is for commercial gain, that likelihood may be presumed. But if it is for a noncommercial purpose, the likelihood must be demonstrated.

 

In this case, respondents failed to carry their burden with regard to home time-shifting. The District Court described respondents’ evidence as follows:

 

 

Plaintiffs’ experts admitted at several points in the trial that the time-shifting without librarying would result in ‘not a great deal of harm.’ Plaintiffs’ greatest concern about time-shifting is with ‘a point of important philosophy that transcends even commercial judgment.’ They fear that with any Betamax usage, ‘invisible boundaries’ are passed: ‘the copyright owner has lost control over his program.’

 

 

480 F.Supp., at 467.

 

Later in its opinion, the District Court observed:

 

 

Most of plaintiffs’ predictions of harm hinge on speculation about audience viewing patterns and ratings, a measurement system which Sidney Sheinberg, MCA’s president, calls a ‘black art’ because of the significant level of imprecision involved in the calculations.

 

 

Id., at 469.

 

There was no need for the District Court to say much about past harm. “Plaintiffs have admitted that no actual harm to their copyrights has occurred to date.” Id., at 451.

 

On the question of potential future harm from time-shifting, the District Court offered a more detailed analysis of the evidence. It rejected respondents’ “fear that persons ‘watching’ the original telecast of a program will not be measured in the live audience and the ratings and revenues will decrease,” by observing that current measurement technology allows the Betamax audience to be reflected. Id., at 466. It rejected respondents’ prediction “that live television or movie audiences will decrease as more people watch Betamax tapes as an alternative,” with the observation that “[t]here is no factual basis for [the underlying] assumption.” Ibid. It rejected respondents’ “fear that time-shifting will reduce audiences for telecast reruns,” and concluded instead that “given current market practices, this should aid plaintiffs rather than harm them.” Ibid. And it declared that respondents’ suggestion “that theater or film rental exhibition of a program will suffer because of time-shift recording of that program” “lacks merit.” 480 F.Supp., at 467.

 

After completing that review, the District Court restated its overall conclusion several times, in several different ways. “Harm from time-shifting is speculative and, at best, minimal.” Ibid. “The audience benefits from the time-shifting capability have already been discussed. It is not implausible that benefits could also accrue to plaintiffs, broadcasters, and advertisers, as the Betamax makes it possible for more persons to view their broadcasts.” Ibid. “No likelihood of harm was shown at trial, and plaintiffs admitted that there had been no actual harm to date.” Id., at 468-469. “Testimony at trial suggested that Betamax may require adjustments in marketing strategy, but it did not establish even a likelihood of harm.” Id., at 469. “Television production by plaintiffs today is more profitable than it has ever been, and, in five weeks of trial, there was no concrete evidence to suggest that the Betamax will change the studios’ financial picture.” Ibid.

 

The District Court’s conclusions are buttressed by the fact that to the extent time-shifting expands public access to freely broadcast television programs, it yields societal benefits. Earlier this year, in Community Television of Southern California v. Gottfried, — U.S. —-, —- – —-, n. 12, 103 S.Ct. 885, 891-892, 74 L.Ed.2d 705 (1983), we acknowledged the public interest in making television broadcasting more available. Concededly, that interest is not unlimited. But it supports an interpretation of the concept of “fair use” that requires the copyright holder to demonstrate some likelihood of harm before he may condemn a private act of time-shifting as a violation of federal law.

 

When these factors are all weighed in the “equitable rule of reason” balance, we must conclude that this record amply supports the District Court’s conclusion that home time-shifting is fair use. In light of the findings of the District Court regarding the state of the empirical data, it is clear that the Court of Appeals erred in holding that the statute as presently written bars such conduct.

 

In summary, the record and findings of the District Court lead us to two conclusions. First, Sony demonstrated a significant likelihood that substantial numbers of copyright holders who license their works for broadcast on free television would not object to having their broadcasts time-shifted by private viewers. And second, respondents failed to demonstrate that time-shifting would cause any likelihood of nonminimal harm to the potential market for, or the value of, their copyrighted works. The Betamax is, therefore, capable of substantial noninfringing uses. Sony’s sale of such equipment to the general public does not constitute contributory infringement of respondent’s copyrights.

 

 

 

 

V

 

 

 

 

“The direction of Art. I is that Congress shall have the power to promote the progress of science and the useful arts. When, as here, the Constitution is permissive, the sign of how far Congress has chosen to go can come only from Congress.” Deepsouth Packing Co. v. Laitram Corp., 406 U.S. 518, 530, 92 S.Ct. 1700, 1707, 32 L.Ed.2d 273 (1972).

 

One may search the Copyright Act in vain for any sign that the elected representatives of the millions of people who watch television every day have made it unlawful to copy a program for later viewing at home, or have enacted a flat prohibition against the sale of machines that make such copying possible.

 

It may well be that Congress will take a fresh look at this new technology, just as it so often has examined other innovations in the past. But it is not our job to apply laws that have not yet been written. Applying the copyright statute, as it now reads, to the facts as they have been developed in this case, the judgment of the Court of Appeals must be reversed.

 

It is so ordered.

 

 

 

Justice Blackmun, with whom Justice Marshall, Justice Powell, and Justice Rehnquist join, dissenting.

 

 

 

… .

 

The fair use doctrine must strike a balance between the dual risks created by the copyright system: on the one hand, that depriving authors of their monopoly will reduce their incentive to create, and, on the other, that granting authors a complete monopoly will reduce the creative ability of others. The inquiry is necessarily a flexible one, and the endless variety of situations that may arise precludes the formulation of exact rules. But when a user reproduces an entire work and uses it for its original purpose, with no added benefit to the public, the doctrine of fair use usually does not apply. There is then no need whatsoever to provide the ordinary user with a fair use subsidy at the author’s expense.

 

The making of a videotape recording for home viewing is an ordinary rather than a productive use of the Studios’ copyrighted works. The District Court found that “Betamax owners use the copy for the same purpose as the original. They add nothing of their own.” 480 F.Supp., at 453. Although applying the fair use doctrine to home VTR recording, as Sony argues, may increase public access to material broadcast free over the public airwaves, I think Sony’s argument misconceives the nature of copyright. Copyright gives the author a right to limit or even to cut off access to his work. Fox Film Corp. v. Doyal, 286 U.S. 123, 127, 52 S.Ct. 546, 547, 76 L.Ed. 1010 (1932). A VTR recording creates no public benefit sufficient to justify limiting this right. Nor is this right extinguished by the copyright owner’s choice to make the work available over the airwaves. Section 106 of the 1976 Act grants the copyright owner the exclusive right to control the performance and the reproduction of his work, and the fact that he has licensed a single television performance is really irrelevant to the existence of his right to control its reproduction. Although a television broadcast may be free to the viewer, this fact is equally irrelevant; a book borrowed from the public library may not be copied any more freely than a book that is purchased.

 

It may be tempting, as, in my view, the Court today is tempted, to stretch the doctrine of fair use so as to permit unfettered use of this new technology in order to increase access to television programming. But such an extension risks eroding the very basis of copyright law, by depriving authors of control over their works and consequently of their incentive to create. Even in the context of highly productive educational uses, Congress has avoided this temptation; in passing the 1976 Act, Congress made it clear that off-the-air videotaping was to be permitted only in very limited situations. See 1976 House Report 71; 1975 Senate Report 64. And, the Senate report adds, “[t]he committee does not intend to suggest … that off-the-air recording for convenience would under any circumstances, be considered ‘fair use.’ ” Id., at 66. I cannot disregard these admonitions.

 

… .

 

I recognize, nevertheless, that there are situations where permitting even an unproductive use would have no effect on the author’s incentive to create, that is, where the use would not affect the value of, or the market for, the author’s work. Photocopying an old newspaper clipping to send to a friend may be an example; pinning a quotation on one’s bulletin board may be another. In each of these cases, the effect on the author is truly de minimis. Thus, even though these uses provide no benefit to the public at large, no purpose is served by preserving the author’s monopoly, and the use may be regarded as fair.

 

Courts should move with caution, however, in depriving authors of protection from unproductive “ordinary” uses. As has been noted above, even in the case of a productive use, § 107(4) requires consideration of “the effect of the use upon the potential market for or value of the copyrighted work” (emphasis added). “[A] particular use which may seem to have little or no economic impact on the author’s rights today can assume tremendous importance in times to come.” Register’s Supplementary Report 14. Although such a use may seem harmless when viewed in isolation, “[i]solated instances of minor infringements, when multiplied many times, become in the aggregate a major inroad on copyright that must be prevented.” 1975 Senate Report 65.

 

I therefore conclude that, at least when the proposed use is an unproductive one, a copyright owner need prove only a potential for harm to the market for or the value of the copyrighted work. See 3 M. Nimmer, Copyright § 13.05[E][4][c], p. 13-84 (1982). Proof of actual harm, or even probable harm, may be impossible in an area where the effect of a new technology is speculative, and requiring such proof would present the “real danger … of confining the scope of an author’s rights on the basis of the present technology so that, as the years go by, his copyright loses much of its value because of unforeseen technical advances.” Register’s Supplementary Report 14. Infringement thus would be found if the copyright owner demonstrates a reasonable possibility that harm will result from the proposed use. When the use is one that creates no benefit to the public at large, copyright protection should not be denied on the basis that a new technology that may result in harm has not yet done so.

 

The Studios have identified a number of ways in which VTR recording could damage their copyrights. VTR recording could reduce their ability to market their works in movie theaters and through the rental or sale of pre-recorded videotapes or videodiscs; it also could reduce their rerun audience, and consequently the license fees available to them for repeated showings. Moreover, advertisers may be willing to pay for only “live” viewing audiences, if they believe VTR viewers will delete commercials or if rating services are unable to measure VTR use; if this is the case, VTR recording could reduce the license fees the Studios are able to charge even for first-run showings. Library-building may raise the potential for each of the types of harm identified by the Studios, and time-shifting may raise the potential for substantial harm as well.8

 

… .

 


  1. As evidence of how a VTR may be used, respondents offered the testimony of William Griffiths. Griffiths, although named as an individual defendant, was a client of plaintiffs’ law firm. The District Court summarized his testimony as follows:

     

     

    He owns approximately 100 tapes. When Griffiths bought his Betamax, he intended not only to time-shift (record, play-back and then erase) but also to build a library of cassettes. Maintaining a library, however, proved too expensive, and he is now erasing some earlier tapes and reusing them.

    Griffiths copied about 20 minutes of a Universal motion picture called ‘Never Give An Inch,’ and two episodes from Universal television series entitled ‘Baa Baa Black Sheep’ and ‘Holmes and Yo Yo.’ He would have erased each of these but for the request of plaintiffs’ counsel that it be kept. Griffiths also testified that he had copied but already erased Universal films called ‘Alpha Caper’ (erased before anyone saw it) and ‘Amelia Earhart.’ At the time of his deposition Griffiths did not intend to keep any Universal film in his library.

    Griffiths has also recorded documentaries, news broadcasts, sporting events and political programs such as a rerun of the Nixon/Kennedy debate.

     

     

    480 F.Supp., at 436-437.

    Four other witnesses testified to having engaged in similar activity.

 

  1. The District Court summarized some of the findings in these surveys as follows:

     

     

    According to plaintiffs’ survey, 75.4% of the VTR owners use their machines to record for time-shifting purposes half or most of the time. Defendants’ survey showed that 96% of the Betamax owners had used the machine to record programs they otherwise would have missed.

    When plaintiffs asked interviewees how many cassettes were in their library, 55.8% said there were 10 or fewer. In defendants’ survey, of the total programs viewed by interviewees in the past month, 70.4% had been viewed only that one time and for 57.9%, there were no plans for further viewing.

     

     

    480 F.Supp., at 438.

 

  1. “81.9% of the defendants’ interviewees watched the same amount or more of regular television as they did before owning a Betamax. 83.2% reported their frequency of movie going was unaffected by Betamax.” 480 F.Supp., at 439.

 

  1. It seems extraordinary to suggest that the Copyright Act confers upon all copyright owners collectively, much less the two respondents in this case, the exclusive right to distribute VTR’s simply because they may be used to infringe copyrights. That, however, is the logical implication of their claim. The request for an injunction below indicates that respondents seek, in effect, to declare VTR’s contraband. Their suggestion in this Court that a continuing royalty pursuant to a judicially created compulsory license would be an acceptable remedy merely indicates that respondents, for their part, would be willing to license their claimed monopoly interest in VTR’s to petitioners in return for a royalty.

 

  1. The record suggests that Disney’s programs at the time of trial consisted of approximately one hour a week of network television and one syndicated series. Universal’s percentage in the Los Angeles market on commercial television stations was under 5%. See Tr. 532-533, 549-550.

 

  1. “Some public stations, as well as commercial stations, program the ‘Neighborhood’ at hours when some children cannot use it. I think that it’s a real service to families to be able to record such programs and show them at appropriate times. I have always felt that with the advent of all of this new technology that allows people to tape the ‘Neighborhood’ off-the-air, and I’m speaking for the ‘Neighborhood’ because that’s what I produce, that they then become much more active in the programming of their family’s television life. Very frankly, I am opposed to people being programmed by others. My whole approach in broadcasting has always been ‘You are an important person just the way you are. You can make healthy decisions.’ Maybe I’m going on too long, but I just feel that anything that allows a person to be more active in the control of his or her life, in a healthy way, is important.” T.R. 2920-2921. See also Def. Exh. PI, p. 85.

 

  1. Section 107 provides:

     

     

    Notwithstanding the provisions of section 106, the fair use of a copyrighted work, including such use by reproduction in copies or phonorecords or by any other means specified by that section, for purposes such as criticism, comment, news reporting, teaching (including multiple copies for classroom use), scholarship, or research, is not an infringement of copyright. In determining whether the use made of a work in any particular case is a fair use the factors to be considered shall include –

     

     

    (1) the purpose and character of the use, including whether such use is of a commercial nature or is for nonprofit educational purposes;

    (2) the nature of the copyrighted work;

    (3) the amount and substantiality of the portion used in relation to the copyrighted work as a whole; and

    (4) the effect of the use upon the potential market for or value of the copyrighted work.” 17 U.S.C. § 107.

     

     

 

  1. A VTR owner who has taped a favorite movie for repeated viewing will be less likely to rent or buy a tape containing the same movie, watch a televised rerun, or pay to see the movie at a theater. Although time-shifting may not replace theater or rerun viewing or the purchase of prerecorded tapes or discs, it may well replace rental usage; a VTR user who has recorded a first-run movie for later viewing will have no need to rent a copy when he wants to see it. Both library-builders and time-shifters may avoid commercials; the library builder may use the pause control to record without them, and all users may fast-forward through commercials on playback.

    The Studios introduced expert testimony that both time-shifting and librarying would tend to decrease their revenue from copyrighted works. See 480 F.Supp., at 440. The District Court’s findings also show substantial library-building and avoidance of commercials. Both sides submitted surveys showing that the average Betamax user owns between 25 and 32 tapes. The Studios’ survey showed that at least 40% of users had more than 10 tapes in a “library”; Sony’s survey showed that more than 40% of users planned to view their tapes more than once; and both sides’ surveys showed that commercials were avoided at least 25% of the time. Id., at 438-439.

 

 

 

Metro-Goldwyn-Mayer Studios Inc. v. Grokster, Ltd.,

545 U.S. 913 (2005)

 

 

 

Justice Souter delivered the opinion of the Court.

 

The question is under what circumstances the distributor of a product capable of both lawful and unlawful use is liable for acts of copyright infringement by third parties using the product. We hold that one who distributes a device with the object of promoting its use to infringe copyright, as shown by clear expression or other affirmative steps taken to foster infringement, is liable for the resulting acts of infringement by third parties.

 

 

 

I>

 

 

 

 

A

 

 

 

 

Respondents, Grokster, Ltd., and StreamCast Networks, Inc., defendants in the trial court, distribute free software products that allow computer users to share electronic files through peer-to-peer networks, so called because users’ computers communicate directly with each other, not through central servers. The advantage of peer-to-peer networks over information networks of other types shows up in their substantial and growing popularity. Because they need no central computer server to mediate the exchange of information or files among users, the high-bandwidth communications capacity for a server may be dispensed with, and the need for costly server storage space is eliminated. Since copies of a file (particularly a popular one) are available on many users’ computers, file requests and retrievals may be faster than on other types of networks, and since file exchanges do not travel through a server, communications can take place between any computers that remain connected to the network without risk that a glitch in the server will disable the network in its entirety. Given these benefits in security, cost, and efficiency, peer-to-peer networks are employed to store and distribute electronic files by universities, government agencies, corporations, and libraries, among others.1

 

Other users of peer-to-peer networks include individual recipients of Grokster’s and StreamCast’s software, and although the networks that they enjoy through using the software can be used to share any type of digital file, they have prominently employed those networks in sharing copyrighted music and video files without authorization. A group of copyright holders (MGM for short, but including motion picture studios, recording companies, songwriters, and music publishers) sued Grokster and StreamCast for their users’ copyright infringements, alleging that they knowingly and intentionally distributed their software to enable users to reproduce and distribute the copyrighted works in violation of the Copyright Act, 17 U.S.C. § 101 et seq. MGM sought damages and an injunction.

 

Discovery during the litigation revealed the way the software worked, the business aims of each defendant company, and the predilections of the users. Grokster’s eponymous software employs what is known as FastTrack technology, a protocol developed by others and licensed to Grokster. StreamCast distributes a very similar product except that its software, called Morpheus, relies on what is known as Gnutella technology. A user who downloads and installs either software possesses the protocol to send requests for files directly to the computers of others using software compatible with FastTrack or Gnutella. On the FastTrack network opened by the Grokster software, the user’s request goes to a computer given an indexing capacity by the software and designated a supernode, or to some other computer with comparable power and capacity to collect temporary indexes of the files available on the computers of users connected to it. The supernode (or indexing computer) searches its own index and may communicate the search request to other supernodes. If the file is found, the supernode discloses its location to the computer requesting it, and the requesting user can download the file directly from the computer located. The copied file is placed in a designated sharing folder on the requesting user’s computer, where it is available for other users to download in turn, along with any other file in that folder.

 

In the Gnutella network made available by Morpheus, the process is mostly the same, except that in some versions of the Gnutella protocol there are no supernodes… . .

 

Although Grokster and StreamCast do not therefore know when particular files are copied, a few searches using their software would show what is available on the networks the software reaches. MGM commissioned a statistician to conduct a systematic search, and his study showed that nearly 90% of the files available for download on the FastTrack system were copyrighted works.2 Grokster and StreamCast dispute this figure, raising methodological problems and arguing that free copying even of copyrighted works may be authorized by the rightholders. They also argue that potential noninfringing uses of their software are significant in kind, even if infrequent in practice. Some musical performers, for example, have gained new audiences by distributing their copyrighted works for free across peer-to-peer networks, and some distributors of unprotected content have used peer-to-peer networks to disseminate files, Shakespeare being an example. Indeed, StreamCast has given Morpheus users the opportunity to download the briefs in this very case, though their popularity has not been quantified.

 

As for quantification, the parties’ anecdotal and statistical evidence entered thus far to show the content available on the FastTrack and Gnutella networks does not say much about which files are actually downloaded by users, and no one can say how often the software is used to obtain copies of unprotected material. But MGM’s evidence gives reason to think that the vast majority of users’ downloads are acts of infringement, and because well over 100 million copies of the software in question are known to have been downloaded, and billions of files are shared across the FastTrack and Gnutella networks each month, the probable scope of copyright infringement is staggering.

 

Grokster and StreamCast concede the infringement in most downloads, and it is uncontested that they are aware that users employ their software primarily to download copyrighted files, even if the decentralized FastTrack and Gnutella networks fail to reveal which files are being copied, and when. From time to time, moreover, the companies have learned about their users’ infringement directly, as from users who have sent e-mail to each company with questions about playing copyrighted movies they had downloaded, to whom the companies have responded with guidance. And MGM notified the companies of 8 million copyrighted files that could be obtained using their software.

 

Grokster and StreamCast are not, however, merely passive recipients of information about infringing use. The record is replete with evidence that from the moment Grokster and StreamCast began to distribute their free software, each one clearly voiced the objective that recipients use it to download copyrighted works, and each took active steps to encourage infringement.

 

After the notorious file-sharing service, Napster, was sued by copyright holders for facilitation of copyright infringement, A&M Records, Inc. v. Napster, Inc., 114 F.Supp.2d 896 (N.D.Cal.2000), aff’d in part, rev’d in part, 239 F.3d 1004 (C.A.9 2001), StreamCast gave away a software program of a kind known as OpenNap, designed as compatible with the Napster program and open to Napster users for downloading files from other Napster and OpenNap users’ computers. Evidence indicates that “[i]t was always [StreamCast’s] intent to use [its OpenNap network] to be able to capture email addresses of [its] initial target market so that [it] could promote [its] StreamCast Morpheus interface to them,” App. 861; indeed, the OpenNap program was engineered “‘to leverage Napster’s 50 million user base.’”

 

StreamCast monitored both the number of users down-loading its OpenNap program and the number of music files they downloaded. It also used the resulting OpenNap network to distribute copies of the Morpheus software and to encourage users to adopt it. Internal company documents indicate that StreamCast hoped to attract large numbers of former Napster users if that company was shut down by court order or otherwise, and that StreamCast planned to be the next Napster. A kit developed by StreamCast to be delivered to advertisers, for example, contained press articles about StreamCast’s potential to capture former Napster users, and it introduced itself to some potential advertisers as a company “which is similar to what Napster was.” It broadcast banner advertisements to users of other Napster-compatible software, urging them to adopt its OpenNap. An internal e-mail from a company executive stated: “‘We have put this network in place so that when Napster pulls the plug on their free service … or if the Court orders them shut down prior to that … we will be positioned to capture the flood of their 32 million users that will be actively looking for an alternative.’”

 

Thus, StreamCast developed promotional materials to market its service as the best Napster alternative. One proposed advertisement read: “Napster Inc. has announced that it will soon begin charging you a fee. That’s if the courts don’t order it shut down first. What will you do to get around it?” Another proposed ad touted StreamCast’s software as the “# 1 alternative to Napster” and asked “[w]hen the lights went off at Napster … where did the users go?”3 StreamCast even planned to flaunt the illegal uses of its software; when it launched the OpenNap network, the chief technology officer of the company averred that “[t]he goal is to get in trouble with the law and get sued. It’s the best way to get in the new[s].”

 

The evidence that Grokster sought to capture the market of former Napster users is sparser but revealing, for Grokster launched its own OpenNap system called Swaptor and inserted digital codes into its Web site so that computer users using Web search engines to look for “Napster” or “[f]ree filesharing” would be directed to the Grokster Web site, where they could download the Grokster software. And Grokster’s name is an apparent derivative of Napster.

 

StreamCast’s executives monitored the number of songs by certain commercial artists available on their networks, and an internal communication indicates they aimed to have a larger number of copyrighted songs available on their networks than other file-sharing networks. The point, of course, would be to attract users of a mind to infringe, just as it would be with their promotional materials developed showing copyrighted songs as examples of the kinds of files available through Morpheus. Morpheus in fact allowed users to search specifically for “Top 40” songs, which were inevitably copyrighted. Similarly, Grokster sent users a newsletter promoting its ability to provide particular, popular copyrighted materials.

 

In addition to this evidence of express promotion, marketing, and intent to promote further, the business models employed by Grokster and StreamCast confirm that their principal object was use of their software to download copyrighted works. Grokster and StreamCast receive no revenue from users, who obtain the software itself for nothing. Instead, both companies generate income by selling advertising space, and they stream the advertising to Grokster and Morpheus users while they are employing the programs. As the number of users of each program increases, advertising opportunities become worth more. While there is doubtless some demand for free Shakespeare, the evidence shows that substantive volume is a function of free access to copyrighted work. Users seeking Top 40 songs, for example, or the latest release by Modest Mouse, are certain to be far more numerous than those seeking a free Decameron, and Grokster and StreamCast translated that demand into dollars.

 

Finally, there is no evidence that either company made an effort to filter copyrighted material from users’ downloads or otherwise impede the sharing of copyrighted files. Although Grokster appears to have sent e-mails warning users about infringing content when it received threatening notice from the copyright holders, it never blocked anyone from continuing to use its software to share copyrighted files. StreamCast not only rejected another company’s offer of help to monitor infringement, but blocked the Internet Protocol addresses of entities it believed were trying to engage in such monitoring on its networks.

 

… .

 

 

 

II

 

 

 

 

A

 

 

 

 

MGM and many of the amici fault the Court of Appeals’s holding for upsetting a sound balance between the respective values of supporting creative pursuits through copyright protection and promoting innovation in new communication technologies by limiting the incidence of liability for copyright infringement. The more artistic protection is favored, the more technological innovation may be discouraged; the administration of copyright law is an exercise in managing the trade-off. See Sony Corp. v. Universal City Studios, supra, at 442, 104 S.Ct. 774; see generally Ginsburg, Copyright and Control Over New Technologies of Dissemination, 101 Colum. L.Rev. 1613 (2001); Lichtman & Landes, Indirect Liability for Copyright Infringement: An Economic Perspective, 16 Harv. J.L. & Tech. 395 (2003).

 

The tension between the two values is the subject of this case, with its claim that digital distribution of copyrighted material threatens copyright holders as never before, because every copy is identical to the original, copying is easy, and many people (especially the young) use file-sharing software to download copyrighted works. This very breadth of the software’s use may well draw the public directly into the debate over copyright policy, Peters, Brace Memorial Lecture: Copyright Enters the Public Domain, 51 J. Copyright Soc. 701, 705-717 (2004) (address by Register of Copyrights), and the indications are that the ease of copying songs or movies using software like Grokster’s and Napster’s is fostering disdain for copyright protection, Wu, When Code Isn’t Law, 89 Va. L.Rev. 679, 724-726 (2003). As the case has been presented to us, these fears are said to be offset by the different concern that imposing liability, not only on infringers but on distributors of software based on its potential for unlawful use, could limit further development of beneficial technologies. See, e.g., Lemley & Reese, Reducing Digital Copyright Infringement Without Restricting Innovation, 56 Stan. L.Rev. 1345, 1386-1390 (2004).

 

The argument for imposing indirect liability in this case is, however, a powerful one, given the number of infringing downloads that occur every day using StreamCast’s and Grokster’s software. When a widely shared service or product is used to commit infringement, it may be impossible to enforce rights in the protected work effectively against all direct infringers, the only practical alternative being to go against the distributor of the copying device for secondary liability on a theory of contributory or vicarious infringement.

 

One infringes contributorily by intentionally inducing or encouraging direct infringement, see Gershwin Pub. Corp. v. Columbia Artists Management, Inc., 443 F.2d 1159, 1162 (C.A.2 1971), and infringes vicariously by profiting from direct infringement while declining to exercise a right to stop or limit it, Shapiro, Bernstein & Co. v. H.L. Green Co., 316 F.2d 304, 307 (C.A.2 1963).4 Although “[t]he Copyright Act does not expressly render anyone liable for infringement committed by another,” Sony Corp. v. Universal City Studios, 464 U.S., at 434, these doctrines of secondary liability emerged from common law principles and are well established in the law, id., at 486 (Blackmun, J., dissenting).

 

… .

 

The parties and many of the amici in this case think the key to resolving it is the Sony rule and, in particular, what it means for a product to be “capable of commercially significant noninfringing uses.” Sony Corp. v. Universal City Studios, supra, at 442. MGM advances the argument that granting summary judgment to Grokster and StreamCast as to their current activities gave too much weight to the value of innovative technology, and too little to the copyrights infringed by users of their software, given that 90% of works available on one of the networks was shown to be copyrighted. Assuming the remaining 10% to be its noninfringing use, MGM says this should not qualify as “substantial,” and the Court should quantify Sony to the extent of holding that a product used “principally” for infringement does not qualify. As mentioned before, Grokster and StreamCast reply by citing evidence that their software can be used to reproduce public domain works, and they point to copyright holders who actually encourage copying. Even if infringement is the principal practice with their software today, they argue, the noninfringing uses are significant and will grow.

 

We agree with MGM that the Court of Appeals misapplied Sony, which it read as limiting secondary liability quite beyond the circumstances to which the case applied. Sony barred secondary liability based on presuming or imputing intent to cause infringement solely from the design or distribution of a product capable of substantial lawful use, which the distributor knows is in fact used for infringement. The Ninth Circuit has read Sony’s limitation to mean that whenever a product is capable of substantial lawful use, the producer can never be held contributorily liable for third parties’ infringing use of it; it read the rule as being this broad, even when an actual purpose to cause infringing use is shown by evidence independent of design and distribution of the product, unless the distributors had “specific knowledge of infringement at a time at which they contributed to the infringement, and failed to act upon that information.” 380 F.3d, at 1162 (internal quotation marks and alterations omitted). Because the Circuit found the StreamCast and Grokster software capable of substantial lawful use, it concluded on the basis of its reading of Sony that neither company could be held liable, since there was no showing that their software, being without any central server, afforded them knowledge of specific unlawful uses.

 

This view of Sony, however, was error, converting the case from one about liability resting on imputed intent to one about liability on any theory. Because Sony did not displace other theories of secondary liability, and because we find below that it was error to grant summary judgment to the companies on MGM’s inducement claim, we do not revisit Sony further, as MGM requests, to add a more quantified description of the point of balance between protection and commerce when liability rests solely on distribution with knowledge that unlawful use will occur. It is enough to note that the Ninth Circuit’s judgment rested on an erroneous understanding of Sony and to leave further consideration of the Sony rule for a day when that may be required.

 

 

 

 

C

 

 

 

 

Sony’s rule limits imputing culpable intent as a matter of law from the characteristics or uses of a distributed product. But nothing in Sony requires courts to ignore evidence of intent if there is such evidence, and the case was never meant to foreclose rules of fault-based liability derived from the common law. Thus, where evidence goes beyond a product’s characteristics or the knowledge that it may be put to infringing uses, and shows statements or actions directed to promoting infringement, Sony’s staple-article rule will not preclude liability.

 

The classic case of direct evidence of unlawful purpose occurs when one induces commission of infringement by another, or “entic[es] or persuad[es] another” to infringe, Black’s Law Dictionary 790 (8th ed.2004), as by advertising. Thus at common law a copyright or patent defendant who “not only expected but invoked [infringing use] by advertisement” was liable for infringement “on principles recognized in every part of the law.” Kalem Co. v. Harper Brothers, 222 U.S., at 62-63 (copyright infringement).

 

The rule on inducement of infringement as developed in the early cases is no different today. Evidence of “active steps … taken to encourage direct infringement,” Oak Industries, Inc. v. Zenith Electronics Corp., 697 F.Supp. 988, 992 (N.D.Ill.1988), such as advertising an infringing use or instructing how to engage in an infringing use, show an affirmative intent that the product be used to infringe, and a showing that infringement was encouraged overcomes the law’s reluctance to find liability when a defendant merely sells a commercial product suitable for some lawful use.

 

For the same reasons that Sony took the staple-article doctrine of patent law as a model for its copyright safe-harbor rule, the inducement rule, too, is a sensible one for copyright. We adopt it here, holding that one who distributes a device with the object of promoting its use to infringe copyright, as shown by clear expression or other affirmative steps taken to foster infringement, is liable for the resulting acts of infringement by third parties. We are, of course, mindful of the need to keep from trenching on regular commerce or discouraging the development of technologies with lawful and unlawful potential. Accordingly, just as Sony did not find intentional inducement despite the knowledge of the VCR manufacturer that its device could be used to infringe, mere knowledge of infringing potential or of actual infringing uses would not be enough here to subject a distributor to liability. Nor would ordinary acts incident to product distribution, such as offering customers technical support or product updates, support liability in themselves. The inducement rule, instead, premises liability on purposeful, culpable expression and conduct, and thus does nothing to compromise legitimate commerce or discourage innovation having a lawful promise.

 

 

 

III

 

 

 

 

A

 

 

 

 

The only apparent question about treating MGM’s evidence as sufficient to withstand summary judgment under the theory of inducement goes to the need on MGM’s part to adduce evidence that StreamCast and Grokster communicated an inducing message to their software users. The classic instance of inducement is by advertisement or solicitation that broadcasts a message designed to stimulate others to commit violations. MGM claims that such a message is shown here. It is undisputed that StreamCast beamed onto the computer screens of users of Napster-compatible programs ads urging the adoption of its OpenNap program, which was designed, as its name implied, to invite the custom of patrons of Napster, then under attack in the courts for facilitating massive infringement. Those who accepted StreamCast’s OpenNap program were offered software to perform the same services, which a factfinder could conclude would readily have been understood in the Napster market as the ability to download copyrighted music files. Grokster distributed an electronic newsletter containing links to articles promoting its software’s ability to access popular copyrighted music. And anyone whose Napster or free file-sharing searches turned up a link to Grokster would have understood Grokster to be offering the same file-sharing ability as Napster, and to the same people who probably used Napster for infringing downloads; that would also have been the understanding of anyone offered Grokster’s suggestively named Swaptor software, its version of OpenNap. And both companies communicated a clear message by responding affirmatively to requests for help in locating and playing copyrighted materials.

 

In StreamCast’s case, of course, the evidence just described was supplemented by other unequivocal indications of unlawful purpose in the internal communications and advertising designs aimed at Napster users (“When the lights went off at Napster … where did the users go?” (ellipsis in original)). Whether the messages were communicated is not to the point on this record. The function of the message in the theory of inducement is to prove by a defendant’s own statements that his unlawful purpose disqualifies him from claiming protection (and incidentally to point to actual violators likely to be found among those who hear or read the message). Proving that a message was sent out, then, is the preeminent but not exclusive way of showing that active steps were taken with the purpose of bringing about infringing acts, and of showing that infringing acts took place by using the device distributed. Here, the summary judgment record is replete with other evidence that Grokster and StreamCast, unlike the manufacturer and distributor in Sony, acted with a purpose to cause copyright violations by use of software suitable for illegal use.

 

Three features of this evidence of intent are particularly notable. First, each company showed itself to be aiming to satisfy a known source of demand for copyright infringement, the market comprising former Napster users. StreamCast’s internal documents made constant reference to Napster, it initially distributed its Morpheus software through an OpenNap program compatible with Napster, it advertised its OpenNap program to Napster users, and its Morpheus software functions as Napster did except that it could be used to distribute more kinds of files, including copyrighted movies and software programs. Grokster’s name is apparently derived from Napster, it too initially offered an OpenNap program, its software’s function is likewise comparable to Napster’s, and it attempted to divert queries for Napster onto its own Web site. Grokster and StreamCast’s efforts to supply services to former Napster users, deprived of a mechanism to copy and distribute what were overwhelmingly infringing files, indicate a principal, if not exclusive, intent on the part of each to bring about infringement.

 

Second, this evidence of unlawful objective is given added significance by MGM’s showing that neither company attempted to develop filtering tools or other mechanisms to diminish the infringing activity using their software. While the Ninth Circuit treated the defendants’ failure to develop such tools as irrelevant because they lacked an independent duty to monitor their users’ activity, we think this evidence underscores Grokster’s and StreamCast’s intentional facilitation of their users’ infringement.5

 

Third, there is a further complement to the direct evidence of unlawful objective. It is useful to recall that StreamCast and Grokster make money by selling advertising space, by directing ads to the screens of computers employing their software. As the record shows, the more the software is used, the more ads are sent out and the greater the advertising revenue becomes. Since the extent of the software’s use determines the gain to the distributors, the commercial sense of their enterprise turns on high-volume use, which the record shows is infringing. This evidence alone would not justify an inference of unlawful intent, but viewed in the context of the entire record its import is clear.

 

The unlawful objective is unmistakable.

 

 

 

 

B

 

 

 

 

In addition to intent to bring about infringement and distribution of a device suitable for infringing use, the inducement theory of course requires evidence of actual infringement by recipients of the device, the software in this case. As the account of the facts indicates, there is evidence of infringement on a gigantic scale, and there is no serious issue of the adequacy of MGM’s showing on this point in order to survive the companies’ summary judgment requests. Although an exact calculation of infringing use, as a basis for a claim of damages, is subject to dispute, there is no question that the summary judgment evidence is at least adequate to entitle MGM to go forward with claims for damages and equitable relief.

 

 

 

 

 

* * *

 

 

 

 

 

In sum, this case is significantly different from Sony and reliance on that case to rule in favor of StreamCast and Grokster was error. Sony dealt with a claim of liability based solely on distributing a product with alternative lawful and unlawful uses, with knowledge that some users would follow the unlawful course. The case struck a balance between the interests of protection and innovation by holding that the product’s capability of substantial lawful employment should bar the imputation of fault and consequent secondary liability for the unlawful acts of others.

 

MGM’s evidence in this case most obviously addresses a different basis of liability for distributing a product open to alternative uses. Here, evidence of the distributors’ words and deeds going beyond distribution as such shows a purpose to cause and profit from third-party acts of copyright infringement. If liability for inducing infringement is ultimately found, it will not be on the basis of presuming or imputing fault, but from inferring a patently illegal objective from statements and actions showing what that objective was.

 

There is substantial evidence in MGM’s favor on all elements of inducement, and summary judgment in favor of Grokster and StreamCast was error. On remand, reconsideration of MGM’s motion for summary judgment will be in order.

 

The judgment of the Court of Appeals is vacated, and the case is remanded for further proceedings consistent with this opinion.

 

It is so ordered.

 

 

 

Justice Ginsburg, with whom The Chief Justice and Justice Kennedy join, concurring

 

I concur in the Court’s decision, which vacates in full the judgment of the Court of Appeals for the Ninth Circuit, and write separately to clarify why I conclude that the Court of Appeals misperceived, and hence misapplied, our holding in Sony Corp. of America v. Universal City Studios, Inc., 464 U.S. 417 (1984).

 

… .

 

This case differs markedly from Sony. Here, there has been no finding of any fair use and little beyond anecdotal evidence of noninfringing uses. In finding the Grokster and StreamCast software products capable of substantial noninfringing uses, the District Court and the Court of Appeals appear to have relied largely on declarations submitted by the defendants. These declarations include assertions (some of them hearsay) that a number of copyright owners authorize distribution of their works on the Internet and that some public domain material is available through peer-to-peer networks including those accessed through Grokster’s and StreamCast’s software.

 

The District Court declared it “undisputed that there are substantial noninfringing uses for Defendants’ software,” thus obviating the need for further proceedings. This conclusion appears to rest almost entirely on the collection of declarations submitted by Grokster and StreamCast. Review of these declarations reveals mostly anecdotal evidence, sometimes obtained second-hand, of authorized copyrighted works or public domain works available online and shared through peer-to-peer networks, and general statements about the benefits of peer-to-peer technology. These declarations do not support summary judgment in the face of evidence, proffered by MGM, of overwhelming use of Grokster’s and StreamCast’s software for infringement.6

 

Even if the absolute number of noninfringing files copied using the Grokster and StreamCast software is large, it does not follow that the products are therefore put to substantial noninfringing uses and are thus immune from liability. The number of noninfringing copies may be reflective of, and dwarfed by, the huge total volume of files shared. Further, the District Court and the Court of Appeals did not sharply distinguish between uses of Grokster’s and StreamCast’s software products (which this case is about) and uses of peer-to-peer technology generally (which this case is not about).

 

In sum, when the record in this case was developed, there was evidence that Grokster’s and StreamCast’s products were, and had been for some time, overwhelmingly used to infringe and that this infringement was the overwhelming source of revenue from the products. Fairly appraised, the evidence was insufficient to demonstrate, beyond genuine debate, a reasonable prospect that substantial or commercially significant noninfringing uses were likely to develop over time. On this record, the District Court should not have ruled dispositively on the contributory infringement charge by granting summary judgment to Grokster and StreamCast.

 

If, on remand, the case is not resolved on summary judgment in favor of MGM based on Grokster and StreamCast actively inducing infringement, the Court of Appeals, I would emphasize, should reconsider, on a fuller record, its interpretation of Sony’s product distribution holding.

 

 

 

Justice Breyer, with whom Justice Stevens and Justice O’Connor join, concurring.

 

I agree with the Court that the distributor of a dual-use technology may be liable for the infringing activities of third parties where he or she actively seeks to advance the infringement. I further agree that, in light of our holding today, we need not now “revisit” Sony Corp. of America v. Universal City Studios, Inc., 464 U.S. 417 (1984). Other Members of the Court, however, take up the Sony question: whether Grokster’s product is “capable of ‘substantial’ or ‘commercially significant’ noninfringing uses.” And they answer that question by stating that the Court of Appeals was wrong when it granted summary judgment on the issue in Grokster’s favor. I write to explain why I disagree with them on this matter.

 

I

 

 

 

The Court’s opinion in Sony and the record evidence (as described and analyzed in the many briefs before us) together convince me that the Court of Appeals’ conclusion has adequate legal support.

 

… .

 

When measured against Sony’s underlying evidence and analysis, the evidence now before us shows that Grokster passes Sony’s test-that is, whether the company’s product is capable of substantial or commercially significant noninfringing uses. For one thing, petitioners’ (hereinafter MGM) own expert declared that 75% of current files available on Grokster are infringing and 15% are “likely infringing.” That leaves some number of files near 10% that apparently are noninfringing, a figure very similar to the 9% or so of authorized time-shifting uses of the VCR that the Court faced in Sony.

 

As in Sony, witnesses here explained the nature of the noninfringing files on Grokster’s network without detailed quantification. Those files include:

 

  • Authorized copies of music by artists such as Wilco, Janis Ian, Pearl Jam, Dave Matthews, John Mayer, and others.

     

 

  • Free electronic books and other works from various online publishers, including Project Gutenberg.

     

 

  • Public domain and authorized software, such as WinZip 8.1.

     

 

  • Licensed music videos and television and movie segments distributed via digital video packaging with the permission of the copyright holder.

     

 

The nature of these and other lawfully swapped files is such that it is reasonable to infer quantities of current lawful use roughly approximate to those at issue in Sony. At least, MGM has offered no evidence sufficient to survive summary judgment that could plausibly demonstrate a significant quantitative difference. To be sure, in quantitative terms these uses account for only a small percentage of the total number of uses of Grokster’s product. But the same was true in Sony, which characterized the relatively limited authorized copying market as “substantial.” … .

 

Importantly, Sony also used the word “capable,” asking whether the product is “capable of” substantial noninfringing uses. Its language and analysis suggest that a figure like 10%, if fixed for all time, might well prove insufficient, but that such a figure serves as an adequate foundation where there is a reasonable prospect of expanded legitimate uses over time. And its language also indicates the appropriateness of looking to potential future uses of the product to determine its “capability.”

 

Here the record reveals a significant future market for noninfringing uses of Grokster-type peer-to-peer software. Such software permits the exchange of any sort of digital file-whether that file does, or does not, contain copyrighted material. As more and more uncopyrighted information is stored in swappable form, it seems a likely inference that lawful peer- to-peer sharing will become increasingly prevalent.

 

And that is just what is happening. Such legitimate noninfringing uses are coming to include the swapping of: research information (the initial purpose of many peer-to-peer networks); public domain films (e.g., those owned by the Prelinger Archive); historical recordings and digital educational materials ( e.g., those stored on the Internet Archive); digital photos (OurPictures, for example, is starting a P2P photo-swapping service); “shareware” and “freeware” (e.g., Linux and certain Windows software); secure licensed music and movie files (Intent MediaWorks, for example, protects licensed content sent across P2P networks); news broadcasts past and present (the BBC Creative Archive lets users “rip, mix and share the BBC”); user-created audio and video files (including “podcasts” that may be distributed through P2P software); and all manner of free “open content” works collected by Creative Commons (one can search for Creative Commons material on StreamCast). See Brief for Distributed Computing Industry Association as Amicus Curiae 15-26; Merges, A New Dynamism in the Public Domain, 71 U. Chi. L.Rev. 183 (2004). I can find nothing in the record that suggests that this course of events will not continue to flow naturally as a consequence of the character of the software taken together with the foreseeable development of the Internet and of information technology.

 

There may be other now-unforeseen noninfringing uses that develop for peer-to-peer software, just as the home-video rental industry (unmentioned in Sony) developed for the VCR. But the foreseeable development of such uses, when taken together with an estimated 10% noninfringing material, is sufficient to meet Sony’s standard. And while Sony considered the record following a trial, there are no facts asserted by MGM in its summary judgment filings that lead me to believe the outcome after a trial here could be any different. The lower courts reached the same conclusion.

 

Of course, Grokster itself may not want to develop these other noninfringing uses. But Sony’s standard seeks to protect not the Groksters of this world (which in any event may well be liable under today’s holding), but the development of technology more generally. And Grokster’s desires in this respect are beside the point.

 

 

 

II

 

 

 

The real question here, I believe, is not whether the record evidence satisfies Sony. As I have interpreted the standard set forth in that case, it does. And of the Courts of Appeals that have considered the matter, only one has proposed interpreting Sony more strictly than I would do – in a case where the product might have failed under any standard.

 

Instead, the real question is whether we should modify the Sony standard, as MGM requests, or interpret Sony more strictly, as I believe Justice GINSBURG’s approach would do in practice. Compare ante, at 2784-2787 (concurring) (insufficient opinion vidence in this case of both present lawful uses and of a reasonable prospect that substantial noninfringing uses would develop over time), with Sony, 464 U.S., at 442-447, 104 S.Ct. 774 (basing conclusion as to the likely existence of a substantial market for authorized copying upon general declarations, some survey data, and common sense).

 

As I have said, Sony itself sought to “strike a balance between a copyright holder’s legitimate demand for effective-not merely symbolic-protection of the statutory monopoly, and the rights of others freely to engage in substantially unrelated areas of commerce.” Thus, to determine whether modification, or a strict interpretation, of Sony is needed, I would ask whether MGM has shown that Sony incorrectly balanced copyright and new-technology interests. In particular: (1) Has Sony (as I interpret it) worked to protect new technology? (2) If so, would modification or strict interpretation significantly weaken that protection? (3) If so, would new or necessary copyright-related benefits outweigh any such weakening?

 

 

 

 

A

 

 

The first question is the easiest to answer. Sony’s rule, as I interpret it, has provided entrepreneurs with needed assurance that they will be shielded from copyright liability as they bring valuable new technologies to market.

 

Sony’s rule is clear. That clarity allows those who develop new products that are capable of substantial noninfringing uses to know, ex ante, that distribution of their product will not yield massive monetary liability. At the same time, it helps deter them from distributing products that have no other real function than-or that are specifically intended for-copyright infringement, deterrence that the Court’s holding today reinforces (by adding a weapon to the copyright holder’s legal arsenal).

 

Sony’s rule is strongly technology protecting. The rule deliberately makes it difficult for courts to find secondary liability where new technology is at issue. It establishes that the law will not impose copyright liability upon the distributors of dual-use technologies (who do not themselves engage in unauthorized copying) unless the product in question will be used almost exclusively to infringe copyrights (or unless they actively induce infringements as we today describe). Sony thereby recognizes that the copyright laws are not intended to discourage or to control the emergence of new technologies, including (perhaps especially) those that help disseminate information and ideas more broadly or more efficiently. Thus Sony’s rule shelters VCRs, typewriters, tape recorders, photocopiers, computers, cassette players, compact disc burners, digital video recorders, MP3 players, Internet search engines, and peer-to-peer software. But Sony’s rule does not shelter descramblers, even if one could theoretically use a descrambler in a noninfringing way.

 

Sony’s rule is forward looking. It does not confine its scope to a static snapshot of a product’s current uses (thereby threatening technologies that have undeveloped future markets). Rather, as the VCR example makes clear, a product’s market can evolve dramatically over time. And Sony-by referring to a capacity for substantial noninfringing uses-recognizes that fact. Sony’s word “capable” refers to a plausible, not simply a theoretical, likelihood that such uses will come to pass, and that fact anchors Sony in practical reality.

 

Sony’s rule is mindful of the limitations facing judges where matters of technology are concerned. Judges have no specialized technical ability to answer questions about present or future technological feasibility or commercial viability where technology professionals, engineers, and venture capitalists themselves may radically disagree and where answers may differ depending upon whether one focuses upon the time of product development or the time of distribution. Consider, for example, the question whether devices can be added to Grokster’s software that will filter out infringing files. MGM tells us this is easy enough to do, as do several amici that produce and sell the filtering technology. Grokster says it is not at all easy to do, and not an efficient solution in any event, and several apparently disinterested computer science professors agree. Which account should a judge credit? Sony says that the judge will not necessarily have to decide.

 

Given the nature of the Sony rule, it is not surprising that in the last 20 years, there have been relatively few contributory infringement suits – based on a product distribution theory – brought against technology providers (a small handful of federal appellate court cases and perhaps fewer than two dozen District Court cases in the last 20 years). I have found nothing in the briefs or the record that shows that Sony has failed to achieve its innovation-protecting objective.

 

 

B

 

 

The second, more difficult, question is whether a modified Sony rule (or a strict interpretation) would significantly weaken the law’s ability to protect new technology. Justice GINSBURG’s approach would require defendants to produce considerably more concrete evidence-more than was presented here-to earn Sony’s shelter. That heavier evidentiary demand, and especially the more dramatic (case-by-case balancing) modifications that MGM and the Government seek, would, I believe, undercut the protection that Sony now offers.

 

To require defendants to provide, for example, detailed evidence-say, business plans, profitability estimates, projected technological modifications, and so forth-would doubtless make life easier for copyrightholder plaintiffs. But it would simultaneously increase the legal uncertainty that surrounds the creation or development of a new technology capable of being put to infringing uses. Inventors and entrepreneurs (in the garage, the dorm room, the corporate lab, or the boardroom) would have to fear (and in many cases endure) costly and extensive trials when they create, produce, or distribute the sort of information technology that can be used for copyright infringement. They would often be left guessing as to how a court, upon later review of the product and its uses, would decide when necessarily rough estimates amounted to sufficient evidence. They would have no way to predict how courts would weigh the respective values of infringing and noninfringing uses; determine the efficiency and advisability of technological changes; or assess a product’s potential future markets. The price of a wrong guess-even if it involves a good-faith effort to assess technical and commercial viability-could be large statutory damages (not less than $750 and up to $30,000 per infringed work ). 17 U.S.C. § 504(c)(1). The additional risk and uncertainty would mean a consequent additional chill of technological development.

 

 

C

 

 

The third question-whether a positive copyright impact would outweigh any technology-related loss-I find the most difficult of the three. I do not doubt that a more intrusive Sony test would generally provide greater revenue security for copyright holders. But it is harder to conclude that the gains on the copyright swings would exceed the losses on the technology roundabouts.

 

For one thing, the law disfavors equating the two different kinds of gain and loss; rather, it leans in favor of protecting technology. As Sony itself makes clear, the producer of a technology which permits unlawful copying does not himself engage in unlawful copying-a fact that makes the attachment of copyright liability to the creation, production, or distribution of the technology an exceptional thing. Moreover, Sony has been the law for some time. And that fact imposes a serious burden upon copyright holders like MGM to show a need for change in the current rules of the game, including a more strict interpretation of the test.

 

In any event, the evidence now available does not, in my view, make out a sufficiently strong case for change. To say this is not to doubt the basic need to protect copyrighted material from infringement. The Constitution itself stresses the vital role that copyright plays in advancing the “useful Arts.” Art. I, § 8, cl. 8. No one disputes that “reward to the author or artist serves to induce release to the public of the products of his creative genius.” United States v. Paramount Pictures, Inc., 334 U.S. 131, 158 (1948). And deliberate unlawful copying is no less an unlawful taking of property than garden-variety theft. But these highly general principles cannot by themselves tell us how to balance the interests at issue in Sony or whether Sony’s standard needs modification. And at certain key points, information is lacking.

 

Will an unmodified Sony lead to a significant diminution in the amount or quality of creative work produced? Since copyright’s basic objective is creation and its revenue objectives but a means to that end, this is the underlying copyright question. And its answer is far from clear.

 

Unauthorized copying likely diminishes industry revenue, though it is not clear by how much. [citations to conflicting studies of effect of file sharing on music sales].

 

The extent to which related production has actually and resultingly declined remains uncertain, though there is good reason to believe that the decline, if any, is not substantial. See, e.g., M. Madden, Pew Internet & American Life Project, Artists, Musicians, and the Internet, p. 21, (Dec. 5, 2004), http://www.pewinternet.org/pdfs/PIP_Artists.Musicians_ Report.pdf (nearly 70% of musicians believe that file sharing is a minor threat or no threat at all to creative industries); Benkler, Sharing Nicely: On Shareable Goods and the Emergence of Sharing as a Modality of Economic Production, 114 Yale L. J. 273, 351-352 (2004) (“Much of the actual flow of revenue to artists-from performances and other sources-is stable even assuming a complete displacement of the CD market by peer-to-peer distribution ….[I]t would be silly to think that music, a cultural form without which no human society has existed, will cease to be in our world [because of illegal file swapping]”).

 

More importantly, copyright holders at least potentially have other tools available to reduce piracy and to abate whatever threat it poses to creative production. As today’s opinion makes clear, a copyright holder may proceed against a technology provider where a provable specific intent to infringe (of the kind the Court describes) is present. Services like Grokster may well be liable under an inducement theory.

 

In addition, a copyright holder has always had the legal authority to bring a traditional infringement suit against one who wrongfully copies. Indeed, since September 2003, the Recording Industry Association of America (RIAA) has filed “thousands of suits against people for sharing copyrighted material.” Walker, New Movement Hits Universities: Get Legal Music, Washington Post, Mar. 17, 2005, p. E1. These suits have provided copyright holders with damages; have served as a teaching tool, making clear that much file sharing, if done without permission, is unlawful; and apparently have had a real and significant deterrent effect. [citations omitted]

 

Further, copyright holders may develop new technological devices that will help curb unlawful infringement. Some new technology, called “digital ‘watermarking’” and “digital fingerprint[ing],” can encode within the file information about the author and the copyright scope and date, which “fingerprints” can help to expose infringers. [citations omitted]

 

At the same time, advances in technology have discouraged unlawful copying by making lawful copying (e.g., downloading music with the copyright holder’s permission) cheaper and easier to achieve. Several services now sell music for less than $1 per song. (Walmart.com, for example, charges $0.88 each.) Consequently, many consumers initially attracted to the convenience and flexibility of services like Grokster are now migrating to lawful paid services (services with copying permission) where they can enjoy at little cost even greater convenience and flexibility without engaging in unlawful swapping. See Wu, When Code Isn’t Law, 89 Va. L.Rev. 679, 731-735 (2003) (noting the prevalence of technological problems on unpaid swapping sites); K. Dean, P2P Tilts Toward Legitimacy, Wired News (Nov. 24, 2004), http:// www.wired.com/news/digiwood/0,1412,65836,00.html; Madden & Rainie, March 2005 Data Memo, supra, at 6-7 (percentage of current downloaders who have used paid services rose from 24% to 43% in a year; number using free services fell from 58% to 41%).

 

Thus, lawful music downloading services-those that charge the customer for downloading music and pay royalties to the copyright holder-have continued to grow and to produce substantial revenue. See Brief for Internet Law Faculty as Amicus Curiae 5-20; Bruno, Digital Entertainment: Piracy Fight Shows Encouraging Signs (Mar. 5, 2005), available at LEXIS, News Library, Billboard File (in 2004, consumers worldwide purchased more than 10 times the number of digital tracks purchased in 2003; global digital music market of $330 million in 2004 expected to double in 2005); Press Release, Informa Telecoms & media, Steady Download Growth Defies P2P (global digital revenues will likely exceed $3 billion in 2010); Ashton, [International Federation of the Phonographic Industry] Predicts Downloads Will Hit the Mainstream, Music Week, Jan. 29, 2005, p. 6 (legal music sites and portable MP3 players “are helping to transform the digital music market” into “an everyday consumer experience”). And more advanced types of non- music-oriented peer-to-peer networks have also started to develop, drawing in part on the lessons of Grokster.

 

Finally, as Sony recognized, the legislative option remains available. Courts are less well suited than Congress to the task of “accommodat[ing] fully the varied permutations of competing interests that are inevitably implicated by such new technology.” Sony, 464 U.S., at 431, 104 S.Ct. 774; see, e.g., Audio Home Recording Act of 1992, 106 Stat. 4237 (adding 17 U.S.C., ch. 10); Protecting Innovation and Art While Preventing Piracy: Hearing Before the Senate Committee on the Judiciary, 108th Cong., 2d Sess. (2004).

 

I do not know whether these developments and similar alternatives will prove sufficient, but I am reasonably certain that, given their existence, a strong demonstrated need for modifying Sony (or for interpreting Sony’s standard more strictly) has not yet been shown. That fact, along with the added risks that modification (or strict interpretation) would impose upon technological innovation, leads me to the conclusion that we should maintain Sony, reading its standard as I have read it. As so read, it requires affirmance of the Ninth Circuit’s determination of the relevant aspects of the Sony question.

 

 

 

* * *

 

 

 

For these reasons, I disagree with Justice GINSBURG, but I agree with the Court and join its opinion.

 


  1. Peer-to-peer networks have disadvantages as well. Searches on peer-to-peer networks may not reach and uncover all available files because search requests may not be transmitted to every computer on the network. There may be redundant copies of popular files. The creator of the software has no incentive to minimize storage or bandwidth consumption, the costs of which are borne by every user of the network. Most relevant here, it is more difficult to control the content of files available for retrieval and the behavior of users.>

 

  1. By comparison, evidence introduced by the plaintiffs in A&M Records, Inc. v. Napster, Inc., 239 F.3d 1004 (C.A.9 2001), showed that 87% of files available on the Napster file-sharing network were copyrighted, id., at 1013.

 

  1. The record makes clear that StreamCast developed these promotional materials but not whether it released them to the public. Even if these advertisements were not released to the public and do not show encouragement to infringe, they illuminate StreamCast’s purposes.

 

  1. We stated in Sony Corp. of America v. Universal City Studios, Inc., 464 U.S. 417 (1984), that “‘the lines between direct infringement, contributory infringement and vicarious liability are not clearly drawn’….[R]easoned analysis of [the Sony plaintiffs’ contributory infringement claim] necessarily entails consideration of arguments and case law which may also be forwarded under the other labels, and indeed the parties … rely upon such arguments and authority in support of their respective positions on the issue of contributory infringement,” id., at 435, n. 17. In the present case MGM has argued a vicarious liability theory, which allows imposition of liability when the defendant profits directly from the infringement and has a right and ability to supervise the direct infringer, even if the defendant initially lacks knowledge of the infringement. Because we resolve the case based on an inducement theory, there is no need to analyze separately MGM’s vicarious liability theory.

 

  1. Of course, in the absence of other evidence of intent, a court would be unable to find contributory infringement liability merely based on a failure to take affirmative steps to prevent infringement, if the device otherwise was capable of substantial noninfringing uses. Such a holding would tread too close to the Sony safe harbor.

 

  1. Justice BREYER finds support for summary judgment in this motley collection of declarations and in a survey conducted by an expert retained by MGM. That survey identified 75% of the files available through Grokster as copyrighted works owned or controlled by the plaintiffs, and 15% of the files as works likely copyrighted. As to the remaining 10% of the files, “there was not enough information to form reasonable conclusions either as to what those files even consisted of, and/or whether they were infringing or non-infringing.” Even assuming, as Justice BREYER does, that the Sony Court would have absolved Sony of contributory liability solely on the basis of the use of the Betamax for authorized time-shifting, summary judgment is not inevitably appropriate here. Sony stressed that the plaintiffs there owned “well below 10%” of copyrighted television programming, and found, based on trial testimony from representatives of the four major sports leagues and other individuals authorized to consent to home-recording of their copyrighted broadcasts, that a similar percentage of program copying was authorized. Here, the plaintiffs allegedly control copyrights for 70% or 75% of the material exchanged through the Grokster and StreamCast software, and the District Court does not appear to have relied on comparable testimony about authorized copying from copyright holders.

 

 

 

3.4.4.1. Problems

 

PROBLEMS

 

 

 

For each, state the possible theories of secondary liability for infringement that could be used, if any, based only on the facts given. (inducement, vicarious, or Sony-style)

 

1. A newspaper contains a classified ad section, a small subsection of which frequently features solicitations to trade copies of copyrighted compact discs. Newspaper is sued.

 

2. A developer sells software called StripIt that removes the restrictions on movies purchased from the iTunes Store. Developer is sued.

 

3. A developer sells software that records in an audio file all audio output by one’s computer. A web designer creates a popular, ad-supported website featuring how-to guides on using the software to record internet radio broadcasts, some of which are copyrighted and do not authorize duplication. Developer is sued.

 

4. Same as 3, but web designer is sued.

 

 

 

Answers

 

 

1. A newspaper contains a classified ad section, a small subsection of which frequently features solicitations to trade copies of copyrighted compact discs. Newspaper is sued.

 

 

Vicarious liability is the most promising avenue. Vicarious liability, in the copyright infringement context, is liability for the infringement of others when one has the right and practical ability to control that infringement. Here, the newspaper has the right to decide which ads to print. Further, the newspaper is certainly capable of screening ads for solicitations to trade infringing copies. This case would essentially be a low-tech version of Napster, but where the ability to screen for infringers is even easier.

 

Note that solicitations to trade the compact discs themselves, the actual discs bought at retail, would not form the basis of a secondary infringement claim. There would be no copy at all made in such circumstances. I’m free to sell or give away books I no longer want, CDs that I don’t want, or any other article subject to copyright.

 

You may be tempted to cite inducement as a possibility here. But the newspaper itself is not actively encouraging anyone to infringe. The evidence of inducement in Grokster went to evidence that the companies themselves were encouraging infringement – by marketing to infringers, advertising the utility of their produce to infringe, etc. There isn’t any of that here.

 

 

2. A developer sells software called StripIt that removes the restrictions on movies purchased from the iTunes Store. Developer is sued.

 

 

Contributory infringement, or what I’ve been calling Sony-style infringement, might be found here, depending on what other uses the software has. Cautionary note: there was no liability in Sony. So when I say Sony-style liability, I mean liability on the ground discussed in Sony. Sony itself was not found liable under what we’re calling a Sony theory. Contributory infringement is liability for merely placing a product on the market that is not capable substantial noninfringing uses. In other words, if your product is really only good for infringement, you can be liable for the illegal copying of those who use it.

 

Here, StripIt appears to be useful only to remove the protections that prevent unauthorized copying of digital movies purchased from iTunes. We’d want to know, though, whether it’s capable of other uses. Further, we’d want to know how often the product is used to enable users to make fair use copies of movies they have bought. (There is another statute, the controversial DMCA, that governs the intentional circumvention of anti-copying measures.)

 

 

3. A developer sells software that records in an audio file all audio output by one’s computer. A web designer creates a popular, ad-supported website featuring how-to guides on using the software to record internet radio broadcasts, some of which are copyrighted and do not authorize duplication. Developer is sued.

 

 

No liability. There is no evidence here that the developer has done anything to encourage anyone to use the product to make illegal copies. It’s doubtful therefore that there could be liability under an inducement theory. Nor is vicarious liability a realistic possibility. The courts aren’t going to require the developer to change the program to monitor people’s usage and route that information back to the developer. The developer simply doesn’t have the practical ability to control how the product is used once people download it. Finally, there almost certainly is no liability under a Sony theory. There are a great many uses for such a product that do not include copying copyrighted material. Indeed, web designer’s guides include information on recording noncopyrighted streams. So even if we restrict attention to copying streams, it would seem the product is capable of substantial noninfringing uses. This is not to mention the other kinds of recordings a user could make.

 

 

4. Same as 3, but web designer is sued.

 

 

The most promising ground here is inducement. Web designer hasn’t released a product that makes copies. Rather, the designer has encouraged others to make copies by telling the how to do it. That some of the guides aren’t for copyrighted streams doesn’t matter. Even if there is only one guide that instructs users on recording a copyrighted stream, designer might be held liable under an inducement theory – as the designer might be argued to have actively encouraged the guide’s readers to do what’s in the guide: copy a copyrighted stream.

 

 

Problem: Streamers

 

 

 

Lenny the law student is pretty good with computers. While task avoiding, Lenny developed an application for recording streamed audio from the internet. Called Streamers, the program works as follows. It assumes the user has certain software, not made by Lenny, already installed, namely a web browser, a calendar program, RealPlayer (for playing streamed audio), and a program called WireTap. WireTap, made by another software company, records any sounds output by the computer. So, after hitting record in WireTap and listening or watching a stream on RealPlayer, the audio file created by WireTap will contain the audio of that stream (along with any other sounds the user happens to trigger when WireTap is recording). Because the raw sound is recorded and then re-compressed, the resulting audio file typically is a little lower quality than the original stream.

 

Streamers works by coordinating these four applications. After typing into Streamers a URL and a recording schedule, the application puts the recordings into the user’s calendar with an alarm. When the time for a recording arrives, the alarm is triggered, and Streamers launches the web browser, triggering the desired stream to play, and then launches WireTap to begin recording. When the stream is finished, recording ends, and Streamers conveniently moves the resulting audio file into iTunes where it is ready for the user to listen to or put on an iPod. In this way, a user can develop a whole library of streams and schedules so that with no manual effort on the part of the user, he or she will always have a library of his or her favorite streams.

 

To Lenny’s surprise, shortly after posting his application, thousands of people downloaded it and began using it to keep up-to-date libraries of their favorite NPR and commercial radio shows, making use of the online streams many radio stations provide. Though they already could have listened to these while sitting at their computers, many were eager to get these programs onto their iPods or other portable players. A good many users had questions, and Lenny provided help over email for configuring Streamers to record various radio programs hosted on numerous websites.

 

Lenny comes to you, a well-known lawyer, with an email he has just received from VapidChannel Communications. The email demands Lenny remove Streamers from distribution and destroy all copies of the application and its source code. According to Vapid, Lenny has “promoted, encouraged, and enabled the illegal copying of Vapid’s copyright protected programs.” The email went on, “If you do not comply with Our demands within ten days, Vapid will be forced to file suit against You for copyright infringement and seek injunction and damages in an amount up to $300,000 per illegal copy.”

 

The letter further states that “Your application, Streamers, places you in violation of the anti-circumvention provision of the Digital Millenium Copyright Act, 17 U.S.C. § 1201, as it enables unauthorized copies of audio streams that have been effectively protected against being downloaded.”

 

Lenny is scared, makes nothing for Streamers, which is a free download, but does not want to be intimidated. What can you tell Lenny about his potential liability and what evidence, if any, might be relevant to the case?

 

 

 

Discussion

 

 

 

This problem obviously calls for an application and extension of the ideas in Grokster and Sony. L is not being threatened for making any copies of Vapid’s copyrighted programs. Rather, Vapid’s theory must be that he is liable for the illegal copying by others using his program, Streamers. Called “contributory infringement,” this basis for liability is not found in the Copyright Act itself but is a judicial doctrine meant to effectuate the Act’s purposes.

 

We have identified three possible circumstances under which one may be liable for the infringing acts of another. First, vicarious liability (think Napster) may be found where one has the ability and right to control the infringing acts of third parties but does not do so. Second, liability for inducement (think Grokster) attaches when one “distributes a device with the object of promoting its use to infringe copyright, as shown by clear expression or other affirmative steps taken to foster infringement.” Third, mere distribution, with no such expression or affirmative steps, may constitute contributory infringement if the device is not capable of significant noninfringing use.

 

A good answer here would run through these three possibilities. There’s clearly no control as there was in Napster, and so vicarious liability seems exceedingly unlikely. Inducement? Well, the facts did state that L provided some assistance to users seeking to record shows. It is not clear whether and to what extent L helped in the copying of copyrighted material. The problem asked what evidence might be relevant to the case, and the content of L’s customer support is likely important. If it can be shown that L provided no help toward what he knew to be infringing uses, then L can likely avoid an inducement claim, and he won’t have to defend the legality of the acts of any particular users he helped.

 

What about the third possibility (Sony)? Regardless of the support he provided or steps he took, could L be liable simply because of the infringement that Streamers makes possible? The question here would center on how much non-infringing use Streamers either permits or, if Ginsburg’s analysis rules the day, is likely actually to occur. L’s lawyer would clearly want to establish the large number of potential noninfringing uses that could be made of Streamers. Think about the different kinds of evidence we could consider. The abundance of streams that do no forbid copying of this sort? Just because there may be many such streams does not mean that their copying is a substantial part of the copying Streamers users actually engage in. Maybe the other side will gather evidence concerning the proportion of Streamers-made copies that are authorized. (Remember in discussing Grokster when we asked whether the percentage of noncopyrighted files or copies was the relevant benchmark for the Sony “significant” benchmark.)

 

But even if Vapid can show that the vast majority of Streamers-made copies are of non-authorized streams, meaning streams that the content owners would not allow to be copied, the analysis would not stop there. Just because a content owner does not authorize a copy does not mean that he or she has a valid legal claim against the copier. If the content is not copyrighted to begin with or if the copying is fair use, then there can be no infringement, and, thus, such copies cannot establish contributory infringement.

 

Remember that one need not register a work to gain a copyright. Your blog postings, etc., those can all be subject to copyright protection. Indeed, nearly all of the streams that Streamers users copy are likely subject to copyright. And so, unless the owners of those streams authorize copying, we have prima facie infringement. Exactly how many streams authorize such copying is something we’d have to investigate. But as to the unauthorized streams: is copying them with Streamers fair use if done (a) only to time shift or (b) to time shift and “device shift” (i.e., put the content on an iPod even when the content owner wants me to pay to do so)?

 

A very good answer here would discuss the fair use statute (looking to opinions interpreting it, like Suntrust Bank, for guidance) and examine its factors. Sony, though, tells us how these factors resolve with respect to television time-shifting, and it’s clearly relevant to, if not dispositive of, the copying here. Though the fair use factors appear both here and in Sony, on their surface, pretty heavily to favor the broadcasters, the fact in Sony that a user of a VCR is only shifting to a more convenient time the viewing of a program he or she was invited to view free of charge pretty much overwhelmed the analysis. The Court just didn’t see anything wrong with that kind of copying in light of the purposes of copyright, and the statutory factors that pointed the other way were deemed less important in this context.

 

At first glance, one might think that Sony definitively answers the question here with regard to time shifting. However, Vapid could argue that time shifting is not fair use in this context, at least with respect to streams which are available on demand on the internet. Unlike television broadcasting in the early 1980s, streams can be listened to anytime one is sitting at the computer with the necessary software. Time shifting is simply not as valuable or necessary here as it is to VCR users. What Streamers users are really doing, Vapid will argue, is “space shifting” or copying in order to convert the stream into a more versatile form that can be consumed in settings other than those for which the stream was designed (sitting in front of the computer scenarios). Vapid will assert that it can afford to put its content in free streams only because doing so drives traffic to its profit-generating businesses like commercial radio and paid downloads. Space shifting directly compromises those revenues.

 

A really great answer on this point would step through the fair use factors with respect to space shifting, compare it with the time shifting analysis as we understand it from Sony, and then situate that analysis within the broader purposes of the Copyright Act. The few decisions that have dealt with this issue have been inconsistent. This is the most important issue, setting the DMCA provision aside for a moment, in this case. If space shifting is infringement, then we’ll have a very hard time defending Streamers.

 

Ok, on to the DMCA. The provision cited in the letter is the so-called anti-circumvention provision. It prohibits the circumventing of “technological measures” intended to prevent copying of copyrighted works. Further, it prohibits distribution of devices or services that (1) are designed primarily to accomplish such circumvention, (2) have only limited other uses, or (3) are promoted as enabling circumvention. What’s troublesome is that this provision would seem to prohibit one from making a fair use copy if doing so requires getting around some protective measure. For example, suppose I wanted to make what is concededly a fair use copy of a DRM-protected iTunes Store purchase. Under the DMCA, I’m prohibited from making a copy by circumventing the DRM, even though under copyright law, I’m permitted to make a copy. And here, even if we conclude, per the above, that time and space shifting are fair uses of these streams, getting around a technological measure meant to prevent copying might still be a DMCA violation – even though it’s not a copyright violation.

 

This might go too far. After all, if the law gives the power to content producers to disable fair uses through legally protected (even if technically flawed) technological means, then fair use is a dead letter. And fair use was supposed to be a critical part of the great First Amendment – Copyright balance.

 

So what would an answer look like here? Well, I’d want you to start with the text of the statute. Does Streamers violate its terms? Does it circumvent by disabling the protections inherent in non-saveable streams or does it make use of something more like the analog hole? See here, e.g.: http://volokh.com/posts/1176397745.shtml Does your conclusion depend on your interpretation of the text? Which of those interpretations are sensible? Are the harshest interpretations unconstitutional?

 

The DMCA portion of this question was meant to expose you to this very tricky area of law and would not have been at all fair on a real test. But I do hope that you’ll give it some thought. Interesting trivia: as a law student I wrote an app called Streamers that works exactly as described in this problem.

 

3.5. Publicity Rights

 

Parks v. La Face Records,

76 F. Supp. 2d 775 (E.D. Mich. 1999).

 

 

 

Gregory J. Reed,Gregory J. ReedAssoc., Detroit, MI, for plaintiff.

 

Joseph M. Beck, Kilpatrick, Stockton, Atlanta, GA, Blanche B. Cook, Philip B. Phillips, Miller, Canfield, Detroit, MI, Conrad L. Mallett, Jr., Miller, Canfield, Detroit, MI, for defendants.

 

 

 

ORDER DENYING PLAINTIFF’S MOTION FOR SUMMARY JUDGMENT AND INJUNCTIVE RELIEF AND GRANTING DEFENDANTS’ MOTION FOR SUMMARY JUDGMENT

 

 

 

Hackett, District Judge.

 

Plaintiff Rosa Parks is a well-known public figure who has been recognized as an international symbol of freedom, humanity, dignity and strength for over 43 years. Plaintiff’s notoriety arose from her heroic stance against racial inequality in the South when on December 1, 1955, in Montgomery, Alabama, she refused to give up her seat to a white passenger and move to the back of the bus. This one defiant act precipitated a 381-day bus boycott that ended segregation on public transportation and ultimately sparked the Civil Rights Movement of the 1960’s. The above-captioned case presents a conflict between plaintiff’s right to protect her celebrated name and the First Amendment right of others to use her name in an expressive work. Specifically at issue is whether plaintiff can prevent the use of her name as the title of a rap song written, performed, marketed and distributed by defendants. Now before the court are the parties’ cross-motions for summary judgment. Oral argument was heard on November 4, 1999. For the following reasons, plaintiff’s motion shall be denied, and summary judgment shall be granted regrettably in defendants’ favor.

 

 

 

I. FACTS

 

 

 

The facts relevant to disposition of this matter are few and undisputed. Defendants are entertainers and producers of popular music. Specifically, Kenny Edmonds and Antonio Reid1 are recording artists and the members of the musical group Outkast, whose services are contractually rendered to LaFace Records. LaFace is a record company engaged in the business of creating, manufacturing and distributing musical sound recordings. LaFace’s recordings are manufactured and distributed by a variety of entities under the auspices of Arista Records and BMG Entertainment.

 

During the week of September 28, 1998, defendants released Outkast’s album called Aquemini, which included a total of 15 songs. The album’s first single release was track number three, a song entitled “Rosa Parks.” Plaintiff is not mentioned by name in the lyrics, and the song is not about her or the Civil Rights Movement. However, the chorus includes the words “Ah, ha, hush that fuss. Everybody move to the back of the bus,” which are repeated a total of ten times over the course of the work.

 

The band Outkast, the album Aquemini, and the song “Rosa Parks” have all enjoyed a great degree of critical acclaim. Aquemini has sold over two million copies (double platinum status), and “Rosa Parks” has enjoyed long-lasting success on the Billboard Charts. In fact, Outkast received their first-ever Grammy nomination for the song. Consequently, defendants have advertised and promoted the Aquemini album, the hit single “Rosa Parks” and its Grammy nomination in ways that are customary in the music business, such as in print advertisements, in a music video, and with stickers affixed to the front of each cassette and compact disc jewel case indicating that the album contains “The Hit Singles ‘Rosa Parks’ And ‘Skew It On The Bar-B.’”2

 

No legal relationship of any kind exists between plaintiff and defendants, and defendants did not attempt to secure plaintiff’s permission to use her name prior to the release of their Aquemini album. Plaintiff, however, opposes defendants’ use of her name without compensation. In particular, plaintiff is offended by the use of her name in association with music that contains, according to plaintiff, “profanity, racial slurs, and derogatory language directed at women.” To date, defendants have continued to use plaintiff’s name on and in connection with the song “Rosa Parks” and the album Aquemini. Accordingly, plaintiff filed this lawsuit seeking monetary and injunctive relief. The court now turns to the parties’ pending motions.

 

 

 

II. STANDARD OF REVIEW

 

 

 

Federal Rule of Civil Procedure 56(c) empowers the court to render summary judgment “forthwith if the pleadings, depositions, answers to interrogatories and admissions on file, together with the affidavits, if any, show that there is no genuine issue as to any material fact and that the moving party is entitled to judgment as a matter of law.” See F.D.I.C. v. Alexander, 78 F.3d 1103, 1106 (6th Cir.1996). The Supreme Court has affirmed the court’s use of summary judgment as an integral part of the fair and efficient administration of justice. The procedure is not a disfavored procedural shortcut. Celotex Corp. v. Catrett, 477 U.S. 317, 327, 106 S.Ct. 2548, 91 L.Ed.2d 265 (1986); see also Kutrom Corp. v. City of Center Line, 979 F.2d 1171, 1174 (6th Cir.1992).

 

The standard for determining whether summary judgment is appropriate is “‘whether the evidence presents a sufficient disagreement to require submission to a jury or whether it is so one-sided that one party must prevail as a matter of law.’” Winningham v. North Am. Resources Corp., 42 F.3d 981, 984 (6th Cir.1994) (citing Booker v. Brown & Williamson Tobacco Co., 879 F.2d 1304, 1310 (6th Cir.1989)). The evidence and all inferences therefrom must be construed in the light most favorable to the non-moving party. Matsushita Elec. Indus. Co., Ltd. v. Zenith Radio Corp., 475 U.S. 574, 587, 106 S.Ct. 1348, 89 L.Ed.2d 538 (1986); Enertech Elec., Inc. v. Mahoning County Comm’rs, 85 F.3d 257, 259 (6th Cir.1996); Wilson v. Stroh Companies, Inc., 952 F.2d 942, 945 (6th Cir.1992). “[T]he mere existence of some alleged factual dispute between the parties will not defeat an otherwise properly supported motion for summary judgment; the requirement is that there be no genuine issue of material fact.” Anderson v. Liberty Lobby, Inc., 477 U.S. 242, 247-48, 106 S.Ct. 2505, 91 L.Ed.2d 202 (1986); see also Hartleip v. McNeilab, Inc., 83 F.3d 767, 774 (6th Cir.1996).

 

If the movant establishes by use of the material specified in Rule 56(c) that there is no genuine issue of material fact and that it is entitled to judgment as a matter of law, the opposing party must come forward with “specific facts showing that there is a genuine issue for trial.” First Nat’l Bank v. Cities Serv. Co., 391 U.S. 253, 270, 88 S.Ct. 1575, 20 L.Ed.2d 569 (1968); see also Adams v. Philip Morris, Inc., 67 F.3d 580, 583 (6th Cir.1995). Mere allegations or denials in the non-movant’s pleadings will not meet this burden. Anderson, 477 U.S. at 248, 106 S.Ct. 2505. Further, the non-moving party cannot rest on its pleadings to avoid summary judgment. It must support its claim with some probative evidence. Kraft v. United States, 991 F.2d 292, 296 (6th Cir.), cert. denied, 510 U.S. 976, 114 S.Ct. 467, 126 L.Ed.2d 419 (1993).

 

 

 

III. DISCUSSION

 

 

 

 

A. Right of Publicity Claim (Count I)

 

 

 

 

In Count I of the complaint, plaintiff alleges that defendants’ use of her name violates her common law right of publicity. The right of publicity protects a celebrity’s commercial interest in her identity. “The theory of the right is that a celebrity’s identity can be valuable in the promotion of products, and the celebrity has an interest that may be protected from the unauthorized commercial exploitation of that identity.” Carson v. Here’s Johnny Portable Toilets, Inc., 698 F.2d 831, 835 (6th Cir.1983) (emphasis added). The right of publicity, however, does not authorize a celebrity to prevent the use of her name in an expressive work protected by the First Amendment. 4 J. Thomas McCarthy, McCarthy on Trademarks and Unfair Competition ss 28:40-28:41 (4th ed.1999); see also Hicks v. Casablanca Records, 464 F.Supp. 426, 430 (S.D.N.Y.1978) (“[M]ore so than posters, bubble gum cards, or some other such ‘merchandise,’ books and movies are vehicles through which ideas and opinions are disseminated and, as such, have enjoyed certain constitutional protections, not generally accorded ‘merchandise.’ “); Paulsen v. Personality Posters, Inc., 59 Misc.2d 444, 450, 299 N.Y.S.2d 501, 508 (1968) (“[E]ven where the ‘right of publicity’ is recognized, it does not invest a prominent person with the right to exploit financially every public use of name or picture. What is made actionable is the unauthorized use for advertising purposes in connection with the sale of a commodity.”).3 Clearly, “[m]usic, as a form of expression and communication, is protected under the First Amendment.” Ward v. Rock Against Racism, 491 U.S. 781, 790, 109 S.Ct. 2746, 105 L.Ed.2d 661 (1989); see also Hurley v. Irish-American Gay, Lesbian and Bisexual Group, 515 U.S. 557, 569, 115 S.Ct. 2338, 132 L.Ed.2d 487 (1995) (paintings, music and poetry are “unquestionably shielded” by the First Amendment).

 

Similarly, titles of artistic works that use the names of public figures and celebrities, like the works themselves, are entitled to First Amendment protection from right of publicity claims:

 

 

Titles, like the artistic works they identify, are of a hybrid nature, combining artistic expression and commercial promotion. The title of a movie may be both an integral element of the film-maker’s expression as well as a significant means of marketing the film to the public. The artistic and commercial elements of titles are inextricably intertwined. Film-makers and authors frequently rely on word-play, ambiguity, irony, and allusion in titling their works. Furthermore, their interest in freedom of artistic expression is shared by their audience. The subtleties of a title can enrich a reader’s or a viewer’s understanding of a work.

 

 

Rogers v. Grimaldi, 875 F.2d 994, 998 (2d Cir.1989). Furthermore, no publicity claim exists when a celebrity’s name is used in advertisements for a work protected by the First Amendment. See, e.g., Guglielmi v. Spelling-Goldberg Prods., 25 Cal.3d 860, 873, 160 Cal.Rptr. 352, 603 P.2d 454, 462 (1979) (Bird, C.J., concurring) (“Since the use of Valentino’s name and likeness in the film was not an actionable infringement of Valentino’s right of publicity, the use of his identity in advertisements for the film is similarly not actionable.”).

 

The right of publicity is therefore inapplicable under the First Amendment if the content of an expressive work bears any relationship to the use of a celebrity’s name. Rogers, 875 F.2d at 1004 (right of publicity unavailable unless use of celebrity’s name is “wholly unrelated” to the work). In addition, the right of publicity does not bar the use of a celebrity’s name in a title as long as the item is a literary work and not “‘simply a disguised commercial advertisement for the sale of goods or services.’” Id. (quoting Frosch v. Grosset & Dunlap, Inc., 75 A.D.2d 768, 769, 427 N.Y.S.2d 828, 829 (1980)).

 

In this case, plaintiff repeatedly contends that defendants’ “Rosa Parks” song is not expressly “about” her or her civil rights efforts, concluding that the song’s title is “unrelated” to its content and must thus have been selected “solely to attract attention.” The court finds plaintiff’s argument unpersuasive and unsupportable. There can be no reasonable dispute that Rosa Parks is universally known for and commonly associated with her refusal in late 1955 to obey the segregation laws in Montgomery, Alabama and “move to the back of the bus.” The song at issue makes unmistakable reference to that symbolic act a total of ten times. Admittedly, the song is not about plaintiff in a strictly biographical sense, but it need not be. Rather, defendants’ use of plaintiff’s name, along with the phrase “move to the back of the bus,” is metaphorical and symbolic. As a matter of law, this obvious relationship between the content of the song and its title bearing plaintiff’s name renders the right of publicity inapplicable.

 

Moreover, that plaintiff (and even the court) may find defendants’ music “profane and vulgar” and not regard it as artistically serious is of no consequence:

 

 

It is fundamental that courts may not muffle expression by passing judgment on its skill or clumsiness, its sensitivity or coarseness; nor on whether it pains or pleases. It is enough that the work is a form of expression “deserving of substantial freedom both as entertainment and as a form of social and literary criticism.”

 

 

University of Notre Dame Du Lac v. Twentieth Century-Fox Film Corp., 22 A.D.2d 452, 458, 256 N.Y.S.2d 301, 307,aff’d, 15 N.Y.2d 940, 259 N.Y.S.2d 832, 207 N.E.2d 508 (1965). Given that the court may not “pass on literary categories, or literary judgment,” defendants’ song enjoys First Amendment protection as long as it is a “[musical] work and not simply a disguised commercial advertisement for the sale of goods or services.” Frosch, 75 A.D.2d at 769, 427 N.Y.S.2d at 829 (holding that right of publicity did not apply to book entitled “Marilyn” and rejecting alleged distinction between a “biography” about Marilyn Monroe and “fiction”). Because the necessary linkage between title and content is easily satisfied here, plaintiff would be hard-pressed to demonstrate that defendants’ use of her name as the title to their song is “simply a disguised commercial advertisement” to sell a product, especially considering the song has received widespread acclaim and was nominated for a Grammy Award. This result is not altered by defendants’ promotion of their album and hit single because the fundamental right to free expression would be illusory if defendants were permitted to entitle their song “Rosa Parks,” but not advertise it to the public. The law imposes no such artificial limitation. See, e.g., Groden v. Random House, Inc., 61 F.3d 1045, 1050-51 (2d Cir.1995) (use of author’s photograph to promote critical work did not infringe right of publicity); Guglielmi, 25 Cal.3d at 873, 160 Cal.Rptr. 352, 603 P.2d at 462 (“It would be illogical to allow respondents to exhibit the film but effectively preclude any advance discussion or promotion of their lawful enterprise.”); Namath v. Sports Illustrated, 48 A.D.2d 487, 488, 371 N.Y.S.2d 10, 11 (1975), aff’d, 39 N.Y.2d 897, 386 N.Y.S.2d 397, 352 N.E.2d 584 (1976) (“The use of plaintiff’s photograph was merely incidental advertising of defendants’ magazine in which plaintiff had earlier been properly and fairly depicted.”).

 

Plaintiff also contends that because her name was not used in the news, to educate, for scholarly purposes or as a parody, defendants’ use is not protected by the First Amendment. However, the Supreme Court has expressly rejected such a notion and held that entertainment is entitled to the same constitutional protection as the exposition of ideas: “The line between the informing and the entertaining is too elusive for the protection of the basic right.” Winters v. New York, 333 U.S. 507, 510, 68 S.Ct. 665, 92 L.Ed. 840 (1948). Next, plaintiff incorrectly argues that First Amendment concerns are not implicated here because defendants’ use of plaintiff’s name was unnecessary and done only to sell their album.

 

 

If this analysis were used to determine whether an expression is entitled to constitutional protection, grave harm would result. Courts would be required not merely to determine whether there is some minimal relationship between the expression and the celebrity, but to compel the author to justify the use of the celebrity’s identity. Only upon satisfying a court of the necessity of weaving the celebrity’s identity into a particular publication would the shadow of liability and censorship fade. Such a course would inevitably chill the exercise of free speech limiting not only the manner and form of expression but the interchange of ideas as well.

 

 

Guglielmi, 25 Cal.3d at 869, 160 Cal.Rptr. 352, 603 P.2d at 460 (citation omitted).

 

In her final attempt to convince the court that the First Amendment does not shield defendants from liability on her publicity claim, plaintiff states: “If Defendants [sic] position is carried forward, then the Ku Klux Klan, skinheads or any race supremacist group can exploit the names of others.” Indeed, the possibly offensive use by such groups of a celebrity’s name is exactly the type of speech protected by the First Amendment, which would be a poor refuge for free expression if public figures could censor the use of their names whenever they found the speech to be distasteful. To the contrary, the First Amendment empowers the audience to regulate expressive speech. “[P]rominence invites creative comment. Surely, the range of free expression would be meaningfully reduced if prominent persons in the present and recent past were forbidden topics for the imaginations of authors.” Guglielmi, 25 Cal.3d at 869, 160 Cal.Rptr. 352, 603 P.2d at 460. Thus, plaintiff’s argument that this is “simply a property case” is offensive to the First Amendment. Accordingly, because the title “Rosa Parks” is not “wholly unrelated” to defendants’ song, and because the title is the name of an expressive work and not a disguised commercial for a product, the right of publicity does not apply to the undisputed facts of this case, which necessitates dismissal of Count I in plaintiff’s complaint.

 

 

 

 

B. Lanham Act and State Law Unfair Competition Claims (Counts II and V)

 

 

 

 

Plaintiff also raises a claim under the Lanham Act, 15 U.S.C. s 1125 (Count II), and a related state law unfair competition claim (Count V). Section 1125(a)(1)(A) provides:

 

 

Any person who, on or in connection with any goods or services, or any container for goods, uses in commerce any word, term, name, symbol, or device, or any combination thereof, or any false designation of origin, false or misleading description of fact, or false or misleading representation of fact, which is likely to cause confusion, or to cause mistake, or to deceive as to the affiliation, connection, or association of such person with another person, or as to the origin, sponsorship, or approval of his or her goods, services, or commercial activities by another person, … shall be liable in a civil action by any person who believes that he or she is or is likely to be damaged by such act.

 

 

Like the right of publicity, the Lanham Act ordinarily applies to commercial transactions involving ordinary goods and services, not expressive works in which First Amendment concerns are paramount. Thus, even if the legitimate goals of the Lanham Act and state unfair competition laws in preventing consumer confusion are implicated by the undisputed facts of this case, they are outweighed as a matter of law by defendants’ right to freedom of expression.

 

The court is well aware that “[p]oetic license is not without limits.” Rogers v. Grimaldi, 875 F.2d 994, 997 (2d Cir.1989). Nonetheless,

 

 

in general the [Lanham] Act should be construed to apply to artistic works only where the public interest in avoiding consumer confusion outweighs the public interest in free expression. In the context of allegedly misleading titles using a celebrity’s name, that balance will normally not support application of the Act unless the title has no artistic relevance to the underlying work whatsoever, or, if it has some artistic relevance, unless the title explicitly misleads as to the source or the content of the work.

 

 

Id. at 999 (emphasis added).

 

As discussed above, the direct artistic relevance between the title “Rosa Parks” and the content of defendants’ song is so obvious that the matter is not open to reasonable debate. Furthermore, it is abundantly clear that the title does not “explicitly” mislead as to source or content. Examples of blatantly misleading titles might include: “Nimmer on Copyright,” “Jane Fonda’s Workout Book,” a title “explicitly signifying endorsement, such as the phrase in a subtitle ‘an authorized biography,’” or “The True Life Story of Ginger and Fred.”Id. at 999-1000. Where such overt references are used in a title and are false as applied to the underlying work, the consumer’s interest in avoiding deception might warrant application of the Lanham Act.

 

 

Many titles, however, include a well-known name without any overt indication of authorship or endorsement-for example, the hit song “Bette Davis Eyes” and the recent film “Come Back to the Five and Dime, Jimmy Dean, Jimmy Dean.”To some people, these titles might implicitly suggest that the named celebrity had endorsed the work or had a role in producing it. Even if that suggestion is false, the title is artistically relevant to the work. In these circumstances, the slight risk that such use of a celebrity’s name might implicitly suggest endorsement or sponsorship to some people is outweighed by the danger of restricting artistic expression, and the Lanham Act is not applicable.

 

 

 

Similarly, … many titles with a celebrity’s name make no explicit statement that the work is about that person in any direct sense; the relevance of the title may be oblique and may become clear only after viewing or reading the work. As to such titles, the consumer interest in avoiding deception is too slight to warrant application of the Lanham Act.Though consumers frequently look to the title of a work to determine what it is about, they do not regard titles of artistic works in the same way as the names of ordinary commercial products…. [M]ost consumers are well aware that they cannot judge a book solely by its title any more than by its cover.

 

 

Id. (emphasis added) (citations omitted).

 

By entitling their song “Rosa Parks,” defendants made no explicit representation that their work was endorsed by or affiliated with plaintiff.4 Nevertheless, plaintiff contends that defendants’ use of her name on the cover of their album creates a grave likelihood of confusion because she recently licensed the use of her name in connection with an album of gospel recordings entitled “Verity Records Presents: A Tribute to Mrs. Rosa Parks.” To support her claim of likelihood of confusion, plaintiff submitted approximately 20 affidavits from consumers who were allegedly confused as to the source or content of defendants’ album. However, plaintiff’s anecdotal evidence, assuming its validity, indicates at most that some members of the public might draw the incorrect inference that plaintiff had some involvement with defendants’ album. “But that risk of misunderstanding, not engendered by any overt claim in the title, is so outweighed by the interests in artistic expression as to preclude application of the Lanham Act.”Id. at 1001; see also DeClemente v. Columbia Pictures Indus., Inc., 860 F.Supp. 30, 52 (E.D.N.Y.1994) (“[E]ven if the plaintiff’s allegations of secondary meaning and consumer confusion are true, the defendants’ first amendment interest in naming their films outweighs the plaintiff’s and even the public’s interest in preventing consumer confusion.”).5

 

Moreover, defendants’ album and its packaging unequivocally identify defendants as the source of the album. “The most common and effective means of apprising intending purchasers of the source of goods is a prominent disclosure on the container, package, wrapper, or label of the manufacturer’s or trader’s name … [and when] that is done, there is no basis for a charge of unfair competition.” Versa Prods. Co., Inc. v. Bifold Co. (Mfg.) Ltd., 50 F.3d 189, 203 (3d Cir.), cert. denied, 516 U.S. 808, 116 S.Ct. 54, 133 L.Ed.2d 19 (1995). In other words, “the plaintiff’s right can be protected sufficiently by requiring the defendant’s [products] to be clearly marked so as to indicate unmistakably that they are defendant’s and not the plaintiff’s goods.” Flagg Mfg. Co. v. Holway, 178 Mass. 83, 91, 59 N.E. 667, 667 (1901). Given the prominent appearance of defendant Outkast’s name on their album, there can be no likelihood of confusion under the Lanham Act between plaintiff’s and defendants’ albums as a matter of law.6

 

Likewise, plaintiff’s state law unfair competition claim also fails.7 As a threshold matter, plaintiff cannot proceed under M.C.L. s 429.428 because application of that statute contemplates a state registered trademark, which plaintiff does not have. Furthermore, plaintiff has not established that she has any common law trademark rights in her name, or that defendants made any trademark use of her name.9 In fact, plaintiff conceded during oral argument that she cannot maintain an action for trademark infringement. Finally, plaintiff cites the following from the Restatement (Third) of Unfair Competition ss 46-47 (1995):

 

 

One who appropriates the commercial value of a person’s identity by using without consent the person’s name, likeness, or other indicia of identity for purposes of trade is subject to liability…. The name, likeness, and other indicia of a person’s identity are used “for purposes of trade” … if they are used in advertising the user’s goods or services, or are placed on merchandise marketed by the user, or are used in connection with services rendered by the user.

 

 

Plaintiff’s purported reliance on the Restatement lacks credit, however, considering she neglected to mention the portion that unambiguously distinguishes it from this case: “However, uses ‘for purposes of trade’ does not ordinarily include the use of a person’s identity in news reporting, commentary, entertainment, works of fiction or nonfiction, or in advertising that is incidental to such uses.” Id. s 47. As a public figure, plaintiff simply is not entitled to control every conceivable presentation of her name and likeness. Consequently, defendants are entitled to summary judgment as to plaintiff’s unfair competition and Lanham Act claims.

 

 

 

 

C. Defamation Claim (Count III)

 

 

 

 

Next, plaintiff asserts a claim for defamation in Count III of her complaint. Specifically, plaintiff alleges that defendants “used derogatory and offensive language in their lyrics and associated said language with Plaintiff’s ‘persona,’ character and name without her consent,” thereby “cast[ing] Plaintiff in a false and unacceptable light.”

 

… .

 

… . [S]ummary judgment in defendants’ favor is appropriate as to Count III.

 

 

 

 

D. Intentional Infliction of Emotional Distress Claim (Count IV)

 

 

 

 

Count IV states a claim for intentional infliction of emotional distress. [The court dismissed this claim as a matter of law.]

 

 

 

 

E. Intentional Interference with Business Relations Claim (Count VI)

 

 

 

 

Plaintiff also asserts a claim for intentional interference with an advantageous business relationship, contending that defendants’ “unauthorized use of her name in the present context will substantially interfere with sale and distribution of the authorized [tribute gospel] recording.”

 

… .

 

… . [D]efendants clearly have not engaged in any per se wrongful conduct by permissibly titling their song “Rosa Parks.” Nor have defendants committed any lawful act with malice that is unjustified in law. Accordingly, defendants are entitled to summary judgment as to Count VI.

 

 

 

 

F. Remaining State Law Claims (Counts VII-IX)

 

 

 

 

Plaintiff’s remaining claims include: unjust enrichment (Count VII); negligence (Count VIII); and, conspiracy (Count IX). Because all three claims incorrectly presuppose a legal duty not to use plaintiff’s name as defendants did, they too must be dismissed.

 

… .

 

 

 

IV. CONCLUSION

 

 

 

The undisputed facts in this case require that summary judgment be granted in defendants’ favor. Therefore,

 

IT IS ORDERED that plaintiff’s motion for summary judgment and injunctive relief hereby is DENIED.

 

IT IS FURTHER ORDERED that defendants’ motion for summary judgment hereby is GRANTED.

 


  1. Although named in the caption of plaintiff’s complaint and other pleadings, Edmonds and Reid were never served with process, and, therefore, are not parties to this litigation in their individual capacities.

 

  1. The sticker also states in smaller print: “Parental Advisory. Explicit Content.”

 

  1. Cf. Abdul-Jabbar v. General Motors Corp., 85 F.3d 407 (9th Cir.1996) (involved use of basketball player’s name in an automobile advertisement); Winterland Concessions Co. v. Sileo, 528 F.Supp. 1201 (N.D.Ill.1981), aff’d, 830 F.2d 195 (7th Cir.1987) (involved the use of names of entertainers and musical groups on t-shirts); Uhlaender v. Henricksen, 316 F.Supp. 1277 (D.Minn.1970) (involved use of major league baseball players’ names in connection with a board game); Palmer v. Schonhorn Enter., Inc., 96 N.J.Super. 72, 232 A.2d 458 (1967) (involved use of a golf player’s name and likeness in connection with a golf game).

 

  1. By contrast, in Cher v. Forum International, Ltd., 692 F.2d 634 (9th Cir.1982), cert. denied, 462 U.S. 1120, 103 S.Ct. 3089, 77 L.Ed.2d 1350 (1983), relied upon by plaintiff, the defendant explicitly misrepresented that Cher had granted an “exclusive” interview and endorsed the magazine. Similarly, in National Bank of Commerce v. Shaklee Corp., 503 F.Supp. 533 (W.D.Tex.1980), the plaintiff’s name was used without permission in an explicit “endorsement” of products, and in Fairfield v. American Photocopy Equipment Co., 138 Cal.App.2d 82, 291 P.2d 194 (1955), the defendant explicitly misrepresented that the plaintiff was a “satisfied user” of defendant’s products.

 

  1. Furthermore, in conventional “product” trademark cases, “merely occasional” instances of alleged actual confusion do not support the existence of likelihood of confusion. See, e.g., S.C. Johnson & Son, Inc. v. Johnson, 266 F.2d 129, 141 (6th Cir.), cert. denied, 361 U.S. 820, 80 S.Ct. 65, 4 L.Ed.2d 65 (1959).

 

  1. Compare the Lanham Act cases cited by plaintiff all of which involved the use of famous persons’ names in connection with recordings by them, thereby directly and intentionally misrepresenting that the plaintiffs had approved the unauthorized distributions of their work. See PPX Enter., Inc. v. Audiofidelity Enter., Inc., 818 F.2d 266, 268 (2d Cir.1987) (defendants marketed eight albums allegedly containing feature performances by the electric guitarist Jimi Hendrix, when in fact the albums “either did not contain Hendrix performances at all or contained performances in which Hendrix was merely a background performer or undifferentiated session player”); Apple Corps Ltd. v. Adirondack Group, 124 Misc.2d 351, 355, 476 N.Y.S.2d 716, 719 (1983) (defendants distributed “recordings of inferior quality which were made as a lark”); Benson v. Paul Winley Record Sales Corp., 452 F.Supp. 516, 518 (S.D.N.Y.1978) (defendants “made [the plaintiff] to appear as the central and controlling artist when in fact he was not,” and also misrepresented to the public that the songs on the album were recent releases by the plaintiff). On the other hand, defendants here have not used plaintiff’s name on a work by plaintiff, nor have defendants misappropriated any written or recorded material of plaintiff’s.

 

  1. The Sixth Circuit noted in Carson v. Here’s Johnny Portable Toilets, Inc., that Lanham Act and Michigan state law unfair competition claims were both governed by the same standards. 698 F.2d at 833; see also Sports Auth., Inc. v. Abercrombie & Fitch, Inc., 965 F.Supp. 925, 934 n. 5 (E.D.Mich.1997) (“the Court’s analysis under the Lanham Act will also apply to [plaintiff’s Michigan] common law claim”).

 

  1. M.C.L. s 429.42 states in relevant part: [A]ny person who shall:

     

     

    (a) Use, without the consent of the registrant, any reproduction, counterfeit, copy or colorable imitation of a mark registered under this act in connection with the sale, offering for sale, or advertising of any goods or services on or in connection with which such use is likely to cause confusion or mistake or to deceive as to the source of origin of such goods or services … is liable to a civil action by the owner of the registered mark.

     

     

    (emphasis added).

 

  1. “[W]hether alleging infringement of a registered trademark … or infringement of an unregistered trademark …, it is clear that a plaintiff must show that it has actually used the designation at issue as a trademark, and that the defendant has also used the same or a similar designation as a trademark.” Rock and Roll Hall of Fame and Museum, Inc. v. Gentile Prods., 134 F.3d 749, 753 (6th Cir.1998). Compare defendants’ use of plaintiff’s name here for the purpose of sending an artistic message, with a hypothetical claim by defendants that they were selling “Rosa Parks”-brand discs and cassettes.

 

 

 

Midler v. Ford Motor Company,

849 F.2d 460 (9th. Cir. 1986).

 

 

 

Peter Laird, Los Angeles, Cal., for plaintiff-appellant.

 

Robert M. Callagy, New York City, for defendants-appellees.

 

Appeal from the United States District Court for the Central District of California.

 

Before Hug, Tang and Noonan, Circuit Judges.

 

 

 

Noonan, Circuit Judge:

 

This case centers on the protectibility of the voice of a celebrated chanteuse from commercial exploitation without her consent. Ford Motor Company and its advertising agency, Young & Rubicam, Inc., in 1985 advertised the Ford Lincoln Mercury with a series of nineteen 30 or 60 second television commercials in what the agency called “The Yuppie Campaign.” The aim was to make an emotional connection with Yuppies, bringing back memories of when they were in college. Different popular songs of the seventies were sung on each commercial. The agency tried to get “the original people,” that is, the singers who had popularized the songs, to sing them. Failing in that endeavor in ten cases the agency had the songs sung by “sound alikes.” Bette Midler, the plaintiff and appellant here, was done by a sound alike.

 

Midler is a nationally known actress and singer. She won a Grammy as early as 1973 as the Best New Artist of that year. Records made by her since then have gone Platinum and Gold. She was nominated in 1979 for an Academy award for Best Female Actress in The Rose, in which she portrayed a pop singer. Newsweek in its June 30, 1986 issue described her as an “outrageously original singer/comedian.” Time hailed her in its March 2, 1987 issue as “a legend” and “the most dynamic and poignant singer-actress of her time.”

 

When Young & Rubicam was preparing the Yuppie Campaign it presented the commercial to its client by playing an edited version of Midler singing “Do You Want To Dance,” taken from the 1973 Midler album, “The Divine Miss M.” After the client accepted the idea and form of the commercial, the agency contacted Midler’s manager, Jerry Edelstein. The conversation went as follows: “Hello, I am Craig Hazen from Young and Rubicam. I am calling you to find out if Bette Midler would be interested in doing …? Edelstein: “Is it a commercial?” “Yes.” “We are not interested.”

 

Undeterred, Young & Rubicam sought out Ula Hedwig whom it knew to have been one of “the Harlettes” a backup singer for Midler for ten years. Hedwig was told by Young & Rubicam that “they wanted someone who could sound like Bette Midler’s recording of [Do You Want To Dance].” She was asked to make a “demo” tape of the song if she was interested. She made an a capella demo and got the job.

 

At the direction of Young & Rubicam, Hedwig then made a record for the commercial. The Midler record of “Do You Want To Dance” was first played to her. She was told to “sound as much as possible like the Bette Midler record,” leaving out only a few “aahs” unsuitable for the commercial. Hedwig imitated Midler to the best of her ability.

 

After the commercial was aired Midler was told by “a number of people” that it “sounded exactly” like her record of “Do You Want To Dance.” Hedwig was told by “many personal friends” that they thought it was Midler singing the commercial. Ken Fritz, a personal manager in the entertainment business not associated with Midler, declares by affidavit that he heard the commercial on more than one occasion and thought Midler was doing the singing.

 

Neither the name nor the picture of Midler was used in the commercial; Young & Rubicam had a license from the copyright holder to use the song. At issue in this case is only the protection of Midler’s voice. The district court described the defendants’ conduct as that “of the average thief.” They decided, “If we can’t buy it, we’ll take it.” The court nonetheless believed there was no legal principle preventing imitation of Midler’s voice and so gave summary judgment for the defendants. Midler appeals.

 

The First Amendment protects much of what the media do in the reproduction of likenesses or sounds. A primary value is freedom of speech and press. Time, Inc. v. Hill, 385 U.S. 374, 388, 87 S.Ct. 534, 542, 17 L.Ed.2d 456 (1967). The purpose of the media’s use of a person’s identity is central. If the purpose is “informative or cultural” the use is immune; “if it serves no such function but merely exploits the individual portrayed, immunity will not be granted.” Felcher and Rubin, “Privacy, Publicity and the Portrayal of Real People by the Media,”88 Yale L.J. 1577, 1596 (1979). Moreover, federal copyright law preempts much of the area. “Mere imitation of a recorded performance would not constitute a copyright infringement even where one performer deliberately sets out to simulate another’s performance as exactly as possible.” Notes of Committee on the Judiciary, 17 U.S.C.A. s 114(b). It is in the context of these First Amendment and federal copyright distinctions that we address the present appeal.

 

Nancy Sinatra once sued Goodyear Tire and Rubber Company on the basis of an advertising campaign by Young & Rubicam featuring “These Boots Are Made For Walkin’,” a song closely identified with her; the female singers of the commercial were alleged to have imitated her voice and style and to have dressed and looked like her. The basis of Nancy Sinatra’s complaint was unfair competition; she claimed that the song and the arrangement had acquired “a secondary meaning” which, under California law, was protectible. This court noted that the defendants “had paid a very substantial sum to the copyright proprietor to obtain the license for the use of the song and all of its arrangements.” To give Sinatra damages for their use of the song would clash with federal copyright law. Summary judgment for the defendants was affirmed. Sinatra v. Goodyear Tire & Rubber Co., 435 F.2d 711, 717-718 (9th Cir.1970), cert. denied, 402 U.S. 906, 91 S.Ct. 1376, 28 L.Ed.2d 646 (1971). If Midler were claiming a secondary meaning to “Do You Want To Dance” or seeking to prevent the defendants from using that song, she would fail like Sinatra. But that is not this case. Midler does not seek damages for Ford’s use of “Do You Want To Dance,” and thus her claim is not preempted by federal copyright law. Copyright protects “original works of authorship fixed in any tangible medium of expression.” 17 U.S.C. s 102(a). A voice is not copyrightable. The sounds are not “fixed.” What is put forward as protectible here is more personal than any work of authorship.

 

Bert Lahr once sued Adell Chemical Co. for selling Lestoil by means of a commercial in which an imitation of Lahr’s voice accompanied a cartoon of a duck. Lahr alleged that his style of vocal delivery was distinctive in pitch, accent, inflection, and sounds. The First Circuit held that Lahr had stated a cause of action for unfair competition, that it could be found “that defendant’s conduct saturated plaintiff’s audience, curtailing his market.” Lahr v. Adell Chemical Co., 300 F.2d 256, 259 (1st Cir.1962). That case is more like this one. But we do not find unfair competition here. One-minute commercials of the sort the defendants put on would not have saturated Midler’s audience and curtailed her market. Midler did not do television commercials. The defendants were not in competition with her. See Halicki v. United Artists Communications, Inc., 812 F.2d 1213 (9th Cir.1987).

 

California Civil Code section 3344 is also of no aid to Midler. The statute affords damages to a person injured by another who uses the person’s “name, voice, signature, photograph or likeness, in any manner.” The defendants did not use Midler’s name or anything else whose use is prohibited by the statute. The voice they used was Hedwig’s, not hers. The term “likeness” refers to a visual image not a vocal imitation. The statute, however, does not preclude Midler from pursuing any cause of action she may have at common law; the statute itself implies that such common law causes of action do exist because it says its remedies are merely “cumulative.” Id. s 3344(g).

 

The companion statute protecting the use of a deceased person’s name, voice, signature, photograph or likeness states that the rights it recognizes are “property rights.” Id. s 990(b). By analogy the common law rights are also property rights. Appropriation of such common law rights is a tort in California. Motschenbacher v. R.J. Reynolds Tobacco Co., 498 F.2d 821 (9th Cir.1974). In that case what the defendants used in their television commercial for Winston cigarettes was a photograph of a famous professional racing driver’s racing car. The number of the car was changed and a wing-like device known as a “spoiler” was attached to the car; the car’s features of white pinpointing, an oval medallion, and solid red coloring were retained. The driver, Lothar Motschenbacher, was in the car but his features were not visible. Some persons, viewing the commercial, correctly inferred that the car was his and that he was in the car and was therefore endorsing the product. The defendants were held to have invaded a “proprietary interest” of Motschenbacher in his own identity. Id.at 825.

 

Midler’s case is different from Motschenbacher’s. He and his car were physically used by the tobacco company’s ad; he made part of his living out of giving commercial endorsements. But, as Judge Koelsch expressed it in Motschenbacher, California will recognize an injury from “an appropriation of the attributes of one’s identity.” Id. at 824. It was irrelevant that Motschenbacher could not be identified in the ad. The ad suggested that it was he. The ad did so by emphasizing signs or symbols associated with him. In the same way the defendants here used an imitation to convey the impression that Midler was singing for them.

 

Why did the defendants ask Midler to sing if her voice was not of value to them? Why did they studiously acquire the services of a sound-alike and instruct her to imitate Midler if Midler’s voice was not of value to them? What they sought was an attribute of Midler’s identity. Its value was what the market would have paid for Midler to have sung the commercial in person.

 

A voice is more distinctive and more personal than the automobile accouterments protected in Motschenbacher. A voice is as distinctive and personal as a face. The human voice is one of the most palpable ways identity is manifested. We are all aware that a friend is at once known by a few words on the phone. At a philosophical level it has been observed that with the sound of a voice, “the other stands before me.” D. Ihde, Listening and Voice 77 (1976). A fortiori, these observations hold true of singing, especially singing by a singer of renown. The singer manifests herself in the song. To impersonate her voice is to pirate her identity. See W. Keeton, D. Dobbs, R. Keeton, D. Owen, Prosser & Keeton on Torts 852 (5th ed. 1984).

 

We need not and do not go so far as to hold that every imitation of a voice to advertise merchandise is actionable. We hold only that when a distinctive voice of a professional singer is widely known and is deliberately imitated in order to sell a product, the sellers have appropriated what is not theirs and have committed a tort in California. Midler has made a showing, sufficient to defeat summary judgment, that the defendants here for their own profit in selling their product did appropriate part of her identity.

 

REVERSED AND REMANDED FOR TRIAL.

 

 

White v. Samsung Electronics America, Inc.,

971 F.2d 1395 (9th Cir. 1992).

 

Blaine Greenberg, John Genga, Hill Wynne Troop & Meisinger, Los Angeles, Cal., for plaintiff-appellant.

 

Anthony Liebig, Kenneth Kulzick, Liebig & Kulzick, Los Angeles, Cal., for defendants-appellees.

 

Appeal from the United States District Court for the Central District of California.

 

Before: Goodwin, Pregerson, and Alarcon, Circuit Judges.

 

 

 

Goodwin, Senior Circuit Judge:

 

This case involves a promotional “fame and fortune” dispute. In running a particular advertisement without Vanna White’s permission, defendants Samsung Electronics America, Inc. (Samsung) and David Deutsch Associates, Inc. (Deutsch) attempted to capitalize on White’s fame to enhance their fortune. White sued, alleging infringement of various intellectual property rights, but the district court granted summary judgment in favor of the defendants. We affirm in part, reverse in part, and remand.

 

Plaintiff Vanna White is the hostess of “Wheel of Fortune,” one of the most popular game shows in television history. An estimated forty million people watch the program daily. Capitalizing on the fame which her participation in the show has bestowed on her, White markets her identity to various advertisers.

 

The dispute in this case arose out of a series of advertisements prepared for Samsung by Deutsch. The series ran in at least half a dozen publications with widespread, and in some cases national, circulation. Each of the advertisements in the series followed the same theme. Each depicted a current item from popular culture and a Samsung electronic product. Each was set in the twenty-first century and conveyed the message that the Samsung product would still be in use by that time. By hypothesizing outrageous future outcomes for the cultural items, the ads created humorous effects. For example, one lampooned current popular notions of an unhealthy diet by depicting a raw steak with the caption: “Revealed to be health food. 2010 A.D.” Another depicted irreverent “news”-show host Morton Downey Jr. in front of an American flag with the caption: “Presidential candidate. 2008 A.D.”

 

The advertisement which prompted the current dispute was for Samsung video-cassette recorders (VCRs). The ad depicted a robot, dressed in a wig, gown, and jewelry which Deutsch consciously selected to resemble White’s hair and dress. The robot was posed next to a game board which is instantly recognizable as the Wheel of Fortune game show set, in a stance for which White is famous. The caption of the ad read: “Longest-running game show. 2012 A.D.” Defendants referred to the ad as the “Vanna White” ad. Unlike the other celebrities used in the campaign, White neither consented to the ads nor was she paid.

 

Following the circulation of the robot ad, White sued Samsung and Deutsch in federal district court under: (1) California Civil Code s 3344; (2) the California common law right of publicity; and (3) s 43(a) of the Lanham Act, 15 U.S.C. s 1125(a). The district court granted summary judgment against White on each of her claims. White now appeals.

 

 

 

I. Section 3344

 

 

 

White first argues that the district court erred in rejecting her claim under section 3344. Section 3344(a) provides, in pertinent part, that “[a]ny person who knowingly uses another’s name, voice, signature, photograph, or likeness, in any manner, … for purposes of advertising or selling, … without such person’s prior consent … shall be liable for any damages sustained by the person or persons injured as a result thereof.”

 

White argues that the Samsung advertisement used her “likeness” in contravention of section 3344. In Midler v. Ford Motor Co., 849 F.2d 460 (9th Cir.1988), this court rejected Bette Midler’s section 3344 claim concerning a Ford television commercial in which a Midler “sound-alike” sang a song which Midler had made famous. In rejecting Midler’s claim, this court noted that “[t]he defendants did not use Midler’s name or anything else whose use is prohibited by the statute. The voice they used was [another person’s], not hers. The term ‘likeness’ refers to a visual image not a vocal imitation.” Id. at 463.

 

In this case, Samsung and Deutsch used a robot with mechanical features, and not, for example, a manikin molded to White’s precise features. Without deciding for all purposes when a caricature or impressionistic resemblance might become a “likeness,” we agree with the district court that the robot at issue here was not White’s “likeness” within the meaning of section 3344. Accordingly, we affirm the court’s dismissal of White’s section 3344 claim.

 

 

 

II. Right of Publicity

 

 

 

White next argues that the district court erred in granting summary judgment to defendants on White’s common law right of publicity claim. In Eastwood v. Superior Court, 149 Cal.App.3d 409 (1983), the California court of appeal stated that the common law right of publicity cause of action “may be pleaded by alleging (1) the defendant’s use of the plaintiff’s identity; (2) the appropriation of plaintiff’s name or likeness to defendant’s advantage, commercially or otherwise; (3) lack of consent; and (4) resulting injury.” Id. at 417. The district court dismissed White’s claim for failure to satisfy Eastwood’s second prong, reasoning that defendants had not appropriated White’s “name or likeness” with their robot ad. We agree that the robot ad did not make use of White’s name or likeness. However, the common law right of publicity is not so confined.

 

The Eastwood court did not hold that the right of publicity cause of action could be pleaded only by alleging an appropriation of name or likeness. Eastwood involved an unauthorized use of photographs of Clint Eastwood and of his name. Accordingly, the Eastwood court had no occasion to consider the extent beyond the use of name or likeness to which the right of publicity reaches. That court held only that the right of publicity cause of action “may be” pleaded by alleging, inter alia, appropriation of name or likeness, not that the action may be pleaded only in those terms.

 

The “name or likeness” formulation referred to in Eastwood originated not as an element of the right of publicity cause of action, but as a description of the types of cases in which the cause of action had been recognized. The source of this formulation is Prosser, Privacy, 48 Cal.L.Rev. 383, 401-07 (1960), one of the earliest and most enduring articulations of the common law right of publicity cause of action. In looking at the case law to that point, Prosser recognized that right of publicity cases involved one of two basic factual scenarios: name appropriation, and picture or other likeness appropriation. Id. at 401-02, nn. 156-57.

 

Even though Prosser focused on appropriations of name or likeness in discussing the right of publicity, he noted that “[i]t is not impossible that there might be appropriation of the plaintiff’s identity, as by impersonation, without the use of either his name or his likeness, and that this would be an invasion of his right of privacy.” Id. at 401, n. 155.1 At the time Prosser wrote, he noted however, that “[n]o such case appears to have arisen.” Id.

 

Since Prosser’s early formulation, the case law has borne out his insight that the right of publicity is not limited to the appropriation of name or likeness. In Motschenbacher v. R.J. Reynolds Tobacco Co., 498 F.2d 821 (9th Cir.1974), the defendant had used a photograph of the plaintiff’s race car in a television commercial. Although the plaintiff appeared driving the car in the photograph, his features were not visible. Even though the defendant had not appropriated the plaintiff’s name or likeness, this court held that plaintiff’s California right of publicity claim should reach the jury.

 

In Midler, this court held that, even though the defendants had not used Midler’s name or likeness, Midler had stated a claim for violation of her California common law right of publicity because “the defendants … for their own profit in selling their product did appropriate part of her identity” by using a Midler sound-alike. Id. at 463-64.

 

In Carson v. Here’s Johnny Portable Toilets, Inc., 698 F.2d 831 (6th Cir.1983), the defendant had marketed portable toilets under the brand name “Here’s Johnny”-Johnny Carson’s signature “Tonight Show” introduction-without Carson’s permission. The district court had dismissed Carson’s Michigan common law right of publicity claim because the defendants had not used Carson’s “name or likeness.” Id. at 835. In reversing the district court, the sixth circuit found “the district court’s conception of the right of publicity … too narrow” and held that the right was implicated because the defendant had appropriated Carson’s identity by using, inter alia, the phrase “Here’s Johnny.” Id. at 835-37.

 

These cases teach not only that the common law right of publicity reaches means of appropriation other than name or likeness, but that the specific means of appropriation are relevant only for determining whether the defendant has in fact appropriated the plaintiff’s identity. The right of publicity does not require that appropriations of identity be accomplished through particular means to be actionable. It is noteworthy that the Midler and Carson defendants not only avoided using the plaintiff’s name or likeness, but they also avoided appropriating the celebrity’s voice, signature, and photograph. The photograph in Motschenbacher did include the plaintiff, but because the plaintiff was not visible the driver could have been an actor or dummy and the analysis in the case would have been the same.

 

Although the defendants in these cases avoided the most obvious means of appropriating the plaintiffs’ identities, each of their actions directly implicated the commercial interests which the right of publicity is designed to protect. As the Carson court explained:

 

 

the right of publicity has developed to protect the commercial interest of celebrities in their identities. The theory of the right is that a celebrity’s identity can be valuable in the promotion of products, and the celebrity has an interest that may be protected from the unauthorized commercial exploitation of that identity…. If the celebrity’s identity is commercially exploited, there has been an invasion of his right whether or not his “name or likeness” is used.

 

 

Carson, 698 F.2d at 835. It is not important how the defendant has appropriated the plaintiff’s identity, but whether the defendant has done so. Motschenbacher, Midler, and Carson teach the impossibility of treating the right of publicity as guarding only against a laundry list of specific means of appropriating identity. A rule which says that the right of publicity can be infringed only through the use of nine different methods of appropriating identity merely challenges the clever advertising strategist to come up with the tenth.

 

Indeed, if we treated the means of appropriation as dispositive in our analysis of the right of publicity, we would not only weaken the right but effectively eviscerate it. The right would fail to protect those plaintiffs most in need of its protection. Advertisers use celebrities to promote their products. The more popular the celebrity, the greater the number of people who recognize her, and the greater the visibility for the product. The identities of the most popular celebrities are not only the most attractive for advertisers, but also the easiest to evoke without resorting to obvious means such as name, likeness, or voice.

 

Consider a hypothetical advertisement which depicts a mechanical robot with male features, an African-American complexion, and a bald head. The robot is wearing black hightop Air Jordan basketball sneakers, and a red basketball uniform with black trim, baggy shorts, and the number 23 (though not revealing “Bulls” or “Jordan” lettering). The ad depicts the robot dunking a basketball one-handed, stiff-armed, legs extended like open scissors, and tongue hanging out. Now envision that this ad is run on television during professional basketball games. Considered individually, the robot’s physical attributes, its dress, and its stance tell us little. Taken together, they lead to the only conclusion that any sports viewer who has registered a discernible pulse in the past five years would reach: the ad is about Michael Jordan.

 

Viewed separately, the individual aspects of the advertisement in the present case say little. Viewed together, they leave little doubt about the celebrity the ad is meant to depict. The female-shaped robot is wearing a long gown, blond wig, and large jewelry. Vanna White dresses exactly like this at times, but so do many other women. The robot is in the process of turning a block letter on a game-board. Vanna White dresses like this while turning letters on a game-board but perhaps similarly attired Scrabble-playing women do this as well. The robot is standing on what looks to be the Wheel of Fortune game show set. Vanna White dresses like this, turns letters, and does this on the Wheel of Fortune game show. She is the only one. Indeed, defendants themselves referred to their ad as the “Vanna White” ad. We are not surprised.

 

Television and other media create marketable celebrity identity value. Considerable energy and ingenuity are expended by those who have achieved celebrity value to exploit it for profit. The law protects the celebrity’s sole right to exploit this value whether the celebrity has achieved her fame out of rare ability, dumb luck, or a combination thereof. We decline Samsung and Deutch’s invitation to permit the evisceration of the common law right of publicity through means as facile as those in this case. Because White has alleged facts showing that Samsung and Deutsch had appropriated her identity, the district court erred by rejecting, on summary judgment, White’s common law right of publicity claim.

 

 

 

III. The Lanham Act

 

 

 

White’s final argument is that the district court erred in denying her claim under s 43(a) of the Lanham Act, 15 U.S.C. s 1125(a). The version of section 43(a) applicable to this case2 provides, in pertinent part, that “[a]ny person who shall … use, in connection with any goods or services … any false description or representation … shall be liable to a civil action … by any person who believes that he is or is likely to be damaged by the use of any such false description or designation.” 15 U.S.C. s 1125(a).

 

To prevail on her Lanham Act claim, White is required to show that in running the robot ad, Samsung and Deutsch created a likelihood of confusion.

 

This circuit recognizes several different multi-factor tests for determining whether a likelihood of confusion exists. None of these tests is correct to the exclusion of the others. Normally, in reviewing the district court’s decision, this court will look to the particular test that the district court used. However, because the district court in this case apparently did not use any of the multi-factor tests in making its likelihood of confusion determination, and because this case involves an appeal from summary judgment and we review de novo the district court’s determination, we will look for guidance to the 8-factor test enunciated in AMF, Inc. v. Sleekcraft Boats, 599 F.2d 341 (9th Cir.1979). According to AMF, factors relevant to a likelihood of confusion include:

 

 

(1) strength of the plaintiff’s mark;

 

 

 

(2) relatedness of the goods;

 

 

 

(3) similarity of the marks;

 

 

 

(4) evidence of actual confusion;

 

 

 

(5) marketing channels used;

 

 

 

(6) likely degree of purchaser care;

 

 

 

(7) defendant’s intent in selecting the mark;

 

 

 

(8) likelihood of expansion of the product lines.

 

 

599 F.2d at 348-49. We turn now to consider White’s claim in light of each factor.

 

In cases involving confusion over endorsement by a celebrity plaintiff, “mark” means the celebrity’s persona. See Allen, 610 F.Supp. at 627. The “strength” of the mark refers to the level of recognition the celebrity enjoys among members of society. See Academy, 944 F.2d at 1455. If Vanna White is unknown to the segment of the public at whom Samsung’s robot ad was directed, then that segment could not be confused as to whether she was endorsing Samsung VCRs. Conversely, if White is well-known, this would allow the possibility of a likelihood of confusion. For the purposes of the Sleekcraft test, White’s “mark,” or celebrity identity, is strong.

 

In cases concerning confusion over celebrity endorsement, the plaintiff’s “goods” concern the reasons for or source of the plaintiff’s fame. Because White’s fame is based on her televised performances, her “goods” are closely related to Samsung’s VCRs. Indeed, the ad itself reinforced the relationship by informing its readers that they would be taping the “longest-running game show” on Samsung’s VCRs well into the future.

 

The third factor, “similarity of the marks,” both supports and contradicts a finding of likelihood of confusion. On the one hand, all of the aspects of the robot ad identify White; on the other, the figure is quite clearly a robot, not a human. This ambiguity means that we must look to the other factors for resolution.

 

The fourth factor does not favor White’s claim because she has presented no evidence of actual confusion.

 

Fifth, however, White has appeared in the same stance as the robot from the ad in numerous magazines, including the covers of some. Magazines were used as the marketing channels for the robot ad. This factor cuts toward a likelihood of confusion.

 

Sixth, consumers are not likely to be particularly careful in determining who endorses VCRs, making confusion as to their endorsement more likely.

 

Concerning the seventh factor, “defendant’s intent,” the district court found that, in running the robot ad, the defendants had intended a spoof of the “Wheel of Fortune.” The relevant question is whether the defendants “intended to profit by confusing consumers” concerning the endorsement of Samsung VCRs. Toho, 645 F.2d 788. We do not disagree that defendants intended to spoof Vanna White and “Wheel of Fortune.” That does not preclude, however, the possibility that defendants also intended to confuse consumers regarding endorsement. The robot ad was one of a series of ads run by defendants which followed the same theme. Another ad in the series depicted Morton Downey Jr. as a presidential candidate in the year 2008. Doubtless, defendants intended to spoof presidential elections and Mr. Downey through this ad. Consumers, however, would likely believe, and would be correct in so believing, that Mr. Downey was paid for his permission and was endorsing Samsung products. Looking at the series of advertisements as a whole, a jury could reasonably conclude that beneath the surface humor of the series lay an intent to persuade consumers that celebrity Vanna White, like celebrity Downey, was endorsing Samsung products.

 

Finally, the eighth factor, “likelihood of expansion of the product lines,” does not appear apposite to a celebrity endorsement case such as this.

 

Application of the Sleekcraft factors to this case indicates that the district court erred in rejecting White’s Lanham Act claim at the summary judgment stage. In so concluding, we emphasize two facts, however. First, construing the motion papers in White’s favor, as we must, we hold only that White has raised a genuine issue of material fact concerning a likelihood of confusion as to her endorsement. Cohen v. Paramount Pictures Corp., 845 F.2d 851, 852-53 (9th Cir.1988). Whether White’s Lanham Act claim should succeed is a matter for the jury. Second, we stress that we reach this conclusion in light of the peculiar facts of this case. In particular, we note that the robot ad identifies White and was part of a series of ads in which other celebrities participated and were paid for their endorsement of Samsung’s products.

 

 

 

IV. The Parody Defense

 

 

 

In defense, defendants cite a number of cases for the proposition that their robot ad constituted protected speech. The only cases they cite which are even remotely relevant to this case are Hustler Magazine v. Falwell, 485 U.S. 46, 108 S.Ct. 876, 99 L.Ed.2d 41 (1988) and L.L. Bean, Inc. v. Drake Publishers, Inc., 811 F.2d 26 (1st Cir.1987). Those cases involved parodies of advertisements run for the purpose of poking fun at Jerry Falwell and L.L. Bean, respectively. This case involves a true advertisement run for the purpose of selling Samsung VCRs. The ad’s spoof of Vanna White and Wheel of Fortune is subservient and only tangentially related to the ad’s primary message: “buy Samsung VCRs.” Defendants’ parody arguments are better addressed to non-commercial parodies.3 The difference between a “parody” and a “knock-off” is the difference between fun and profit.

 

 

 

V. Conclusion

 

 

 

In remanding this case, we hold only that White has pleaded claims which can go to the jury for its decision.

 

AFFIRMED IN PART, REVERSED IN PART, and REMANDED.

 

 

 

Alarcon, Circuit Judge, concurring in part, dissenting in part

 

Vanna White seeks recovery from Samsung based on three theories: the right to privacy, the right to publicity, and the Lanham Act. I concur in the majority’s conclusions on the right to privacy. I respectfully dissent from its holdings on the right to publicity and the Lanham Act claims.

 

 

 

I. RIGHT TO PRIVACY (CAL. CIV. CODE 3344(a))

 

 

 

I agree with the majority’s conclusion that no reasonable jury could find that the robot was a “likeness” of Vanna White within the meaning of California Civil Code section 3344(a).

 

 

 

II. RIGHT TO PUBLICITY

 

 

 

I must dissent from the majority’s holding on Vanna White’s right to publicity claim. The district court found that, since the commercial advertisement did not show a “likeness” of Vanna White, Samsung did not improperly use the plaintiff’s identity. The majority asserts that the use of a likeness is not required under California common law. According to the majority, recovery is authorized if there is an appropriation of one’s “identity.” I cannot find any holding of a California court that supports this conclusion. Furthermore, the record does not support the majority’s finding that Vanna White’s “identity” was appropriated.

 

The district court relied on Eastwood v. Superior Court, 149 Cal.App.3d 409, 198 Cal.Rptr. 342, (1983), in holding that there was no cause of action for infringement on the right to publicity because there had been no use of a likeness. In Eastwood, the California Court of Appeal described the elements of the tort of “commercial appropriation of the right of publicity” as “(1) the defendant’s use of the plaintiff’s identity; (2) the appropriation of plaintiff’s name or likeness to defendant’s advantage, …; (3) lack of consent; and (4) resulting injury.” Id. at 417, 198 Cal.Rptr. 342. (Emphasis added).

 

All of the California cases that my research has disclosed hold that a cause of action for appropriation of the right to publicity requires proof of the appropriation of a name or likeness. See, e.g., Lugosi v. Universal Pictures, 25 Cal.3d 813, 603 P.2d 425, 160 Cal.Rptr. 323 (1979) (“The so-called right of publicity means in essence that the reaction of the public to name and likeness … endows the name and likeness of the person involved with commercially exploitable opportunities.”); Guglielmi v. Spelling-Goldberg Prods., 25 Cal.3d 860, 603 P.2d 454, 457, 160 Cal.Rptr. 352, 355 (1979) (use of name of Rudolph Valentino in fictional biography allowed); Eastwood v. Superior Court, supra (use of photo and name of actor on cover of tabloid newspaper); In re Weingand, 231 Cal.App.2d 289, 41 Cal.Rptr. 778 (1964) (aspiring actor denied court approval to change name to “Peter Lorie” when famous actor Peter Lorre objected); Fairfield v. American Photocopy Equip. Co., 138 Cal.App.2d 82, 291 P.2d 194 (1955), later app. 158 Cal.App.2d 53, 322 P.2d 93 (1958) (use of attorney’s name in advertisement); Gill v. Curtis Publishing Co., 38 C.2d 273, 239 P.2d 630 (1952) (use of photograph of a couple in a magazine).

 

Notwithstanding the fact that California case law clearly limits the test of the right to publicity to name and likeness, the majority concludes that “the common law right of publicity is not so confined.” Majority opinion at p. 1397. The majority relies on two factors to support its innovative extension of the California law. The first is that the Eastwood court’s statement of the elements was permissive rather than exclusive. The second is that Dean Prosser, in describing the common law right to publicity, stated that it might be possible that the right extended beyond name or likeness. These are slender reeds to support a federal court’s attempt to create new law for the state of California.

 

In reaching its surprising conclusion, the majority has ignored the fact that the California Court of Appeal in Eastwood specifically addressed the differences between the common law right to publicity and the statutory cause of action codified in California Civil Code section 3344. The court explained that “[t]he differences between the common law and the statutory actions are: (1) Section 3344, subdivision (a) requires knowing use whereas under case law, mistake and inadvertence are not a defense against commercial appropriation and (2) section 3344, subdivision (g) expressly provides that its remedies are cumulative and in addition to any provided by law.” Eastwood, 149 Cal.App.3d at n. 6, 198 Cal.Rptr. 342 (emphasis in original). The court did not include appropriations of identity by means other than name or likeness among its list of differences between the statute and the common law.

 

The majority also relies on Dean Prosser’s statement that “[i]t is not impossible that there might be an appropriation of the plaintiff’s identity, as by impersonation, without the use of either his name or his likeness, and that this would be an invasion of his right of privacy.” Prosser, Privacy,48 Cal.L.Rev. 383, 401 n. 155 (1960). As Dean Prosser noted, however, “[n]o such case appears to have arisen.” Id.

 

The majority states that the case law has borne out Dean Prosser’s insight that the right to publicity is not limited to name or likeness. As noted above, however, the courts of California have never found an infringement on the right to publicity without the use of the plaintiff’s name or likeness.

 

The interest of the California Legislature as expressed in California Civil Code section 3344 appears to preclude the result reached by the majority. The original section 3344 protected only name or likeness. In 1984, ten years after our decision in Motschenbacher v. R.J. Reynolds Tobacco Company, 498 F.2d 821 (9th Cir.1974) and 24 years after Prosser speculated about the future development of the law of the right of publicity, the California legislature amended the statute. California law now makes the use of someone’s voice or signature, as well as name or likeness, actionable. Cal.Civ.Code sec. 2233(a) (Deering 1991 Supp.). Thus, California, after our decision in Motschenbacher specifically contemplated protection for interests other than name or likeness, but did not include a cause of action for appropriation of another person’s identity. The ancient maxim, inclusio unius est exclusio alterius, would appear to bar the majority’s innovative extension of the right of publicity. The clear implication from the fact that the California Legislature chose to add only voice and signature to the previously protected interests is that it wished to limit the cause of action to enumerated attributes.

 

The majority has focused on federal decisions in its novel extension of California Common Law. Those decisions do not provide support for the majority’s decision.

 

In each of the federal cases relied upon by the majority, the advertisement affirmatively represented that the person depicted therein was the plaintiff. In this case, it is clear that a metal robot and not the plaintiff, Vanna White, is depicted in the commercial advertisement. The record does not show an appropriation of Vanna White’s identity.

 

In Motschenbacher, a picture of a well-known race driver’s car, including its unique markings, was used in an advertisement. Id. at 822. Although the driver could be seen in the car, his features were not visible. Id. The distinctive markings on the car were the only information shown in the ad regarding the identity of the driver. These distinctive markings compelled the inference that Motschenbacher was the person sitting in the racing car. We concluded that “California appellate courts would … afford legal protection to an individual’s proprietary interest in his own identity.” Id. at 825. (Emphasis added). Because the distinctive markings on the racing car were sufficient to identify Motschenbacher as the driver of the car, we held that an issue of fact had been raised as to whether his identity had been appropriated. Id. at 827.

 

In Midler v. Ford Motor Co., 849 F.2d 460 (9th Cir.1988), a singer who had been instructed to sound as much like Bette Midler as possible, sang a song in a radio commercial made famous by Bette Midler. Id. at 461. A number of persons told Bette Midler that they thought that she had made the commercial. Id. at 462. Aside from the voice, there was no information in the commercial from which the singer could be identified. We noted that “[t]he human voice is one of the most palpable ways identity is manifested.” Id. at 463. We held that, “[t]o impersonate her voice is to pirate her identity,”id., and concluded that Midler had raised a question of fact as to the misappropriation of her identity.

 

In Carson v. Here’s Johnny Portable Toilets, Inc., 698 F.2d 831 (6th Cir.1983), the Sixth Circuit was called upon to interpret Michigan’s common-law right to publicity. The case involved a manufacturer who used the words, “Here’s Johnny,” on portable toilets. Id. at 832-33. These same words were used to introduce the star of a popular late-night television program. There was nothing to indicate that this use of the phrase on the portable toilets was not associated with Johnny Carson’s television program. The court found that “[h]ere there was an appropriation of Carson’s identity,” which violated the right to publicity. Id. at 837.

 

The common theme in these federal cases is that identifying characteristics unique to the plaintiffs were used in a context in which they were the only information as to the identity of the individual. The commercial advertisements in each case showed attributes of the plaintiff’s identities which made it appear that the plaintiff was the person identified in the commercial. No effort was made to dispel the impression that the plaintiffs were the source of the personal attributes at issue. The commercials affirmatively represented that the plaintiffs were involved. See, e.g., Midler at 462 (“The [Motschenbacher] ad suggested that it was he…. In the same way the defendants here used an imitation to convey the impression that Midler was singing for them.”). The proper interpretation of Motschenbacher, Midler, and Carson is that where identifying characteristics unique to a plaintiff are the only information as to the identity of the person appearing in an ad, a triable issue of fact has been raised as to whether his or her identity as been appropriated.

 

The case before this court is distinguishable from the factual showing made in Motschenbacher, Midler, and Carson. It is patently clear to anyone viewing the commercial advertisement that Vanna White was not being depicted. No reasonable juror could confuse a metal robot with Vanna White.

 

The majority contends that “the individual aspects of the advertisement … [v]iewed together leave little doubt about the celebrity the ad is meant to depict.” Majority Opinion at p. 1399. It derives this conclusion from the fact that Vanna White is “the only one” who “dresses like this, turns letters, and does this on the Wheel of Fortune game show.” Id. In reaching this conclusion, the majority confuses Vanna White, the person, with the role she has assumed as the current hostess on the “Wheel of Fortune” television game show. A recognition of the distinction between a performer and the part he or she plays is essential for a proper analysis of the facts of this case. As is discussed below, those things which Vanna White claims identify her are not unique to her. They are, instead, attributes of the role she plays. The representation of those attributes, therefore, does not constitute a representation of Vanna White. See Nurmi v. Peterson, 10 U.S.P.Q.2d 1775 (C.D.Cal.1989) (distinguishing between performer and role).

 

Vanna White is a one-role celebrity. She is famous solely for appearing as the hostess on the “Wheel of Fortune” television show. There is nothing unique about Vanna White or the attributes which she claims identify her. Although she appears to be an attractive woman, her face and figure are no more distinctive than that of other equally comely women. She performs her role as hostess on “Wheel of Fortune” in a simple and straight-forward manner. Her work does not require her to display whatever artistic talent she may possess.

 

The majority appears to argue that because Samsung created a robot with the physical proportions of an attractive woman, posed it gracefully, dressed it in a blond wig, an evening gown, and jewelry, and placed it on a set that resembles the Wheel of Fortune layout, it thereby appropriated Vanna White’s identity. But an attractive appearance, a graceful pose, blond hair, an evening gown, and jewelry are attributes shared by many women, especially in Southern California. These common attributes are particularly evident among game-show hostesses, models, actresses, singers, and other women in the entertainment field. They are not unique attributes of Vanna White’s identity. Accordingly, I cannot join in the majority’s conclusion that, even if viewed together, these attributes identify Vanna White and, therefore, raise a triable issue as to the appropriation of her identity.

 

The only characteristic in the commercial advertisement that is not common to many female performers or celebrities is the imitation of the “Wheel of Fortune” set. This set is the only thing which might possibly lead a viewer to think of Vanna White. The Wheel of Fortune set, however, is not an attribute of Vanna White’s identity. It is an identifying characteristic of a television game show, a prop with which Vanna White interacts in her role as the current hostess. To say that Vanna White may bring an action when another blond female performer or robot appears on such a set as a hostess will, I am sure, be a surprise to the owners of the show. Cf. Baltimore Orioles, Inc. v. Major League Baseball Players Ass’n, 805 F.2d 663 (7th Cir.1986) (right to publicity in videotaped performances preempted by copyright of owner of telecast).

 

The record shows that Samsung recognized the market value of Vanna White’s identity. No doubt the advertisement would have been more effective if Vanna White had appeared in it. But the fact that Samsung recognized Vanna White’s value as a celebrity does not necessarily mean that it appropriated her identity. The record shows that Samsung dressed a robot in a costume usually worn by television game-show hostesses, including Vanna White. A blond wig, and glamorous clothing are not characteristics unique to the current hostess of Wheel of Fortune. This evidence does not support the majority’s determination that the advertisement was meant to depict Vanna White. The advertisement was intended to depict a robot, playing the role Vanna White currently plays on the Wheel of Fortune. I quite agree that anyone seeing the commercial advertisement would be reminded of Vanna White. Any performance by another female celebrity as a game-show hostess, however, will also remind the viewer of Vanna White because Vanna White’s celebrity is so closely associated with the role. But the fact that an actor or actress became famous for playing a particular role has, until now, never been sufficient to give the performer a proprietary interest in it. I cannot agree with the majority that the California courts, which have consistently taken a narrow view of the right to publicity, would extend law to these unique facts.

 

 

 

III. THE LANHAM ACT

 

 

 

Vanna White’s Lanham Act claim is easily resolved by applying the proper legal standard. Vanna White seeks damages for violation of section 43(a) of the Lanham Act. To succeed, Vanna White must prove actual deception of the consuming public. Harper House, Inc. v. Thomas Nelson, Inc., 889 F.2d 197, 208 (9th Cir.1989) (claim for damages under section 43(a) requires showing the defendant “actually deceived a significant portion of the consuming public.”); see also PPX Enterprises, Inc. v. Audiofidelity Enterprises, Inc., 818 F.2d 266, 271 (2d Cir.1987) (“to establish entitlement to damages for violation of section 43(a): [Plaintiffs] must establish actual confusion or deception resulting from the violation.”); J. Gilson, Trademark Protection and Practice section 7.02 at 7-137 to 7-138 (1991) (plaintiffs must show actual deception to obtain damages under section 43(a)). Vanna White offered no evidence that any portion of the consuming public was deceived. The district court was correct in granting summary judgment on Vanna White’s Lanham Act claim.

 

The majority finds that because a majority of factors set forth in AMF, Inc. v. Sleekcraft Boats, 599 F.2d 341 (9th Cir.1979), favor Vanna White, the district court erred in granting summary judgment.

 

The AMF test is designed to aid in determining whether two marks are so sufficiently similar that it is likely that a consumer would confuse them. Where the marks are so obviously different that no confusion could possibly occur, the test is unnecessary. That is the situation in this matter. The attempt to use the Lanham Act to prevent “misappropriations” of which a court does not approve results in the distortion of the law which makes it more difficult to apply the law in appropriate cases. See Hanson & Walls, Protecting Trademark Goodwill: Towards a Federal Standard of Misappropriation, 81 Trademark Rep. 480, 511-513 (1991). This case is an example of such distortion.

 

The majority assumes the conclusion that the AMF test is designed to disclose. In repeatedly stating that the robot “identifies” Vanna White, the majority has usurped the fact finding function of the district court.

 

The majority holds that the first factor of the AMF test, strength of the mark, weighs in Vanna White’s favor. It equates this factor with the strength of Vanna White’s fame, citing Allen v. National Video, Inc., 610 F.Supp. 612 (D.C.N.Y.1985). Allen involved a celebrity look-alike who bore a remarkable resemblance to Woody Allen. Id. at 617. The instant matter involves a robot that bears no resemblance to Vanna White.

 

It is unclear whether the “mark” for which Vanna White seeks protection is her screen image or the imitation Wheel of Fortune. Although Vanna White is certainly famous for being famous, there is no evidence in the record that consumers identify the specific characteristics at issue, i.e., blond hair and fancy dress, solely with Vanna White. The majority ignores this important distinction.

 

The majority has glossed over the third AMF factor-similarity of the marks-the most important factor in this case. The majority finds this factor “ambiguous” because the common characteristics “identify” Vanna White. Majority Opinion at p. 1400. We are required, however, to compare marks in their entirety. California Cooler, Inc. v. Loretto Winery, 774 F.2d 1451 (9th Cir.1985). In this matter, the consumer is confronted with two entities. One is Vanna White. The other is a robot. No one could reasonably confuse the two.

 

Certain aspects of a mark may have a greater impact than other aspects. When a mark has certain salient characteristics, they are given greater weight. Country Floors, Inc. v. Gepner, 930 F.2d 1056 (3d Cir.1991); Henri’s Food Products Co. v. Kraft, Inc., 717 F.2d 352 (7th Cir.1983); Giant Food, Inc. v. Nations’ Foodservice, Inc., 710 F.2d 1565 (Fed.Cir.1983). The face of Vanna White and the features of the robot are obviously more important characteristics than their hair, dress, physical proportions, jewelry, or the decoration of the set. Thus, the features of the robot and Vanna White should be given great weight in the analysis. It should be clear to anyone viewing the commercial advertisement that the crude features of the robot are very dissimilar to Vanna White’s attractive and human face.

 

The majority’s analysis of the intent or seventh factor in AMF is similarly suspect. The question presented here is whether there is any evidence in the record that Samsung intended to confuse consumers. It did not.

 

Where the circumstances are sufficient to eliminate any likelihood of confusion, this court has repeatedly held that there is no claim for a violation of the Lanham Act. See Toho Co., Ltd. v. Sears, Roebuck & Co., 645 F.2d 788 (9th Cir.1981) (“Bagzilla” garbage bags did not infringe “Godzilla” mark); Walt Disney Prods. v. Air Pi rates, 581 F.2d 751 (9th Cir.1978), cert. denied sub nom O’Neill v. Walt Disney Prods, 439 U.S. 1132, 99 S.Ct. 1054, 59 L.Ed.2d 94 (1979) (“Silly Sympathies” in adult comic books did not infringe on Disney’s “Silly Symphonies”). The use of a robot in the commercial advertisement makes it clear that Vanna White did not endorse Samsung’s product.

 

Although likelihood of confusion may usually be a factual question, “courts retain an important authority to monitor the outer limits of substantial similarity within which a jury is permitted to make the factual determination whether there is a likelihood of confusion.” Warner Bros., Inc. v. American Broadcasting Cos., Inc., 720 F.2d 231, 246 (2d Cir.1983). “[S]ummary judgment is appropriate if the court is satisfied that the products or marks are so dissimilar that no question of fact is presented.” Universal City Studios, Inc. v. Nintendo Co., Ltd., 746 F.2d 112 (2d Cir.1984).

 

“There is no issue for trial unless there is sufficient evidence favoring the nonmoving party for a jury to return a verdict for that party.” Anderson v. Liberty Lobby, Inc., 477 U.S. 242, 249, 106 S.Ct. 2505, 2510, 91 L.Ed.2d 202 (1986). Vanna White has presented no evidence of actual deception. Thus, she has failed to raise a genuine issue of material fact that would support her Lanham Act claim.

 

 

 

IV. SAMSUNG’S FIRST AMENDMENT DEFENSE

 

 

 

The majority gives Samsung’s First Amendment defense short shrift because “[t]his case involves a true advertisement run for the purpose of selling Samsung VCRs.” Majority opinion at p. 1401. I respectfully disagree with the majority’s analysis of this issue as well.

 

The majority’s attempt to distinguish this case from Hustler Magazine v. Falwell, 485 U.S. 46, 108 S.Ct. 876, 99 L.Ed.2d 41 (1988), and L.L. Bean, Inc. v. Drake Publishers, Inc., 811 F.2d 26 (1st Cir.1987), is unpersuasive. The majority notes that the parodies in those cases were made for the purpose of poking fun at the Reverend Jerry Falwell and L.L. Bean. But the majority fails to consider that the defendants in those cases were making fun of the Reverend Jerry Falwell and L.L. Bean for the purely commercial purpose of selling soft-core pornographic magazines.

 

Generally, a parody does not constitute an infringement on the original work if it takes no more than is necessary to “conjure up” the original. Walt Disney Prods. v. Air Pirates, 581 F.2d 751, 756 (9th Cir.1978). The majority has failed to consider these factors properly in deciding that Vanna White may bring an action for damages solely because the popularity of the fame show, Wheel of Fortune.

 

The effect of the majority’s holding on expressive conduct is difficult to estimate. The majority’s position seems to allow any famous person or entity to bring suit based on any commercial advertisement that depicts a character or role performed by the plaintiff. Under the majority’s view of the law, Gene Autry could have brought an action for damages against all other singing cowboys. Clint Eastwood would be able to sue anyone who plays a tall, soft-spoken cowboy, unless, of course, Jimmy Stewart had not previously enjoined Clint Eastwood. Johnny Weismuller would have been able to sue each actor who played the role of Tarzan. Sylvester Stallone could sue actors who play blue-collar boxers. Chuck Norris could sue all karate experts who display their skills in motion pictures. Arnold Schwarzenegger could sue body builders who are compensated for appearing in public.

 

The majority’s reading of the Lanham Act would provide a basis for “commercial” enterprises to maintain an action for section 43(a) violations even in the absence of confusion or deception. May Black and Decker, maker of the “Dustbuster” portable vacuum, now sue “Bust-dusters,” the Los Angeles topless cleaning service. Can the Los Angeles Kings hockey team state a cause of action against the City of Las Vegas for its billboards reading “L.A. has the Kings, but we have the Aces.”

 

Direct competitive advertising could also be affected. Will BMW, which advertises its automobiles as “the ultimate driving machine,” be able to maintain an action against Toyota for advertising one of its cars as “the ultimate saving machine”? Can Coca Cola sue Pepsi because it depicted a bottle of Coca Cola in its televised “taste test”? Indeed, any advertisement which shows a competitor’s product, or any recognizable brand name, would appear to be liable for damages under the majority’s view of the applicable law. Under the majority’s analysis, even the depiction of an obvious facsimile of a competitor’s product may provide sufficient basis for the maintenance of an action for damages.

 

 

 

V. CONCLUSION

 

 

 

The protection of intellectual property presents the courts with the necessity of balancing competing interests. On the one hand, we wish to protect and reward the work and investment of those who create intellectual property. In so doing, however, we must prevent the creation of a monopoly that would inhibit the creative expressions of others. We have traditionally balanced those interests by allowing the copying of an idea, but protecting a unique expression of it. Samsung clearly used the idea of a glamorous female game show hostess. Just as clearly, it avoided appropriating Vanna White’s expression of that role. Samsung did not use a likeness of her. The performer depicted in the commercial advertisement is unmistakably a lifeless robot. Vanna White has presented no evidence that any consumer confused the robot with her identity. Indeed, no reasonable consumer could confuse the robot with Vanna White or believe that, because the robot appeared in the advertisement, Vanna White endorsed Samsung’s product.

 

I would affirm the district court’s judgment in all respects.

 


  1. Under Professor Prosser’s scheme, the right of publicity is the last of the four categories of the right to privacy. Prosser, 48 Cal. L. Rev. at 389.

 

  1. The statute was amended after White filed her complaint. The amendments would not have altered the analysis in this case however.

 

  1. In warning of a first amendment chill to expressive conduct, the dissent reads this decision too broadly. See Dissent at 1407. This case concerns only the market which exists in our society for the exploitation of celebrity to sell products, and an attempt to take a free ride on a celebrity’s celebrity value. Commercial advertising which relies on celebrity fame is different from other forms of expressive activity in two crucial ways.

     

    First, for celebrity exploitation advertising to be effective, the advertisement must evoke the celebrity’s identity. The more effective the evocation, the better the advertisement. If, as Samsung claims, its ad was based on a “generic” game-show hostess and not on Vanna White, the ad would not have violated anyone’s right of publicity, but it would also not have been as humorous or as effective.

     

    Second, even if some forms of expressive activity, such as parody, do rely on identity evocation, the first amendment hurdle will bar most right of publicity actions against those activities. Cf. Falwell, 485 U.S. at 46, 108 S.Ct. at 876. In the case of commercial advertising, however, the first amendment hurdle is not so high. Central Hudson Gas & Electric Corp. v. Public Service Comm’n of New York, 447 U.S. 557, 566, 100 S.Ct. 2343, 2351, 65 L.Ed.2d 341 (1980). Realizing this, Samsung attempts to elevate its ad above the status of garden-variety commercial speech by pointing to the ad’s parody of Vanna White. Samsung’s argument is unavailing. See Board of Trustees, State Univ. of N.Y. v. Fox, 492 U.S. 469, 474-75, 109 S.Ct. 3028, 3031, 106 L.Ed.2d 388 (1988); Bolger v. Youngs Drug Products Corp., 463 U.S. 60, 67-68, 103 S.Ct. 2875, 2880-81, 77 L.Ed.2d 469 (1983). Unless the first amendment bars all right of publicity actions-and it does not, see Zachini v. Scripps-Howard Broadcasting Co., 433 U.S. 562, 97 S.Ct. 2849, 53 L.Ed.2d 965 (1977)-then it does not bar this case.

 

 

 

Gordon Sumner, p/k/a Sting v Michael Urvan,

Case No. D2000-0596 (WIPO Arbitration, Jul. 25, 2000)

 

 

 

 

1. The Parties

 

 

 

1.1 The Complainant is Gordon Sumner, professionally known as “Sting”, a citizen of the United Kingdom who maintains a residence in the United States. The Respondent is Michael Urvan, of Marietta, Georgia, United States of America.

 

 

 

 

 

… .

 

 

 

 

 

4. Factual Background

 

 

 

Complainant’s Activities and Trademarks

 

 

 

 

4.1 In his Complaint, the Complainant asserted the following in relation to his activities and trademarks. The Complainant is a world famous musician, recording and performing artist who has, for over twenty years, rendered high-quality musical services under his name, trademark and service mark STING. Since at least as early as 1978, the Complainant has exclusively and continuously used the STING mark in connection with approximately twenty record albums, almost all of which have gone multi-platinum in the United States and enjoyed great commercial success worldwide. The Complainant has also used the STING mark in connection with innumerable world-wide concert tours involving venues with significant capacities, the majority of which sell out. The STING mark is internationally known and famous as a result of the Complainant’s extensive, high-profile, and overwhelmingly commercially successful activities in the music industry. The Complainant is the owner of the STING mark as a trademark and service mark. The name STING has become synonymous in the minds of the public with the Complainant and his activities in the music industry, and serves as a symbol of the goodwill and excellent reputation associated with Sting. The STING mark is famous and entitled to the widest scope of protection afforded by law, including protection against dilution.

 

 

 

 

 

4.2 In his Response, the Respondent asserted that there are 20 trademark registrations of the word STING in the US, but none of them are registered by the Complainant. The word STING is a common word in the English language, and so registration of it as a domain name is not a violation of the Uniform Policy. The Respondent is not a competitor of the Complainant and the Respondent does not attempt to cause any confusion with him.

 

 

 

 

Respondent’s Activities

 

 

 

 

4.3 The Complainant asserted the following in relation to the Respondent’s activities and use of the domain name. Until the Respondent was contacted by a representative of the Complainant, the Respondent made no use of the domain name. After being contacted by a representative of the Complainant, Respondent linked the domain name to another site called “GunBroker.Com”, which is a site that facilitates “person to person” selling of guns. During or about February of 2000, and again during or about May of 2000, the Respondent offered to sell the domain name to the Complainant for $25,000.00. Since offering to sell the domain name to the Complainant for $25,000.00, the Respondent has frequently changed the web site identified by the domain name, usually with an “under construction” message, and in some cases providing a link to a third-party operated unauthorized web site relating to the Complainant.

 

 

 

 

 

4.4 In his Response, the Respondent asserted that he has been using the nickname “Sting” and more recently “=Sting=” publicly on the Internet for at least 8 years. The Respondent registered the domain name in July 1995, approximately 5 years before this dispute was commenced. The Respondent did not register the domain name to sell it, nor did he register the domain to hold it hostage for any reason. The Respondent engaged in work on web site to which he intended the domain name “sting.com” to resolve, prior to any notification of this dispute. The Respondent did not point the domain name “sting.com” to the “GunBroker.com” website – this occurred for a short time as a result of an error on the part of the Respondent’s web service provider. The Complainant’s assertion that the Respondent initiated contact with the Complainant is false – the first contact was initiated by the Complainant on May 16, 2000.

 

 

 

 

 

 

… .

 

 

 

 

 

6. Discussion and Findings

 

 

 

Domain Name Identical or Confusingly Similar to Complainant’s Mark

 

 

 

 

6.1 The relevant part of the domain name “sting.com” is “sting”. The Complainant asserts, the Respondent admits, and this Administrative Panel finds, that the domain name is identical to the word STING.

 

 

 

 

 

6.2 The Complainant is not the owner of a trademark or service mark registration for the word STING. It is, however, clear that the Uniform Policy is not limited to a “registered” mark; an unregistered, or common law, mark is sufficient for the purposes of paragraph 4(a)(i). The Complainant did not provide any documentary evidence in support of his assertion that he is the owner of the unregistered trademark and/or service mark STING. However, the Uniform Policy is not limited to trademarks or service marks “owned” by the Complainant; it is sufficient for the purposes of paragraph 4(a)(i) that there be a trademark or service mark “in which the Complainant has rights”. The Complainant asserted, and this Administrative Panel through the equivalent of taking judicial notice finds, that the Complainant is a world famous entertainer who is known by the name STING.

 

 

 

 

 

6.3 The question that arises is whether being known under a particular name is the same as having rights in that name as a “trademark or service mark”. The answer to this question is not straightforward. On the one hand, there are a number of cases under the Uniform Policy in which the Panel has treated the name of a famous or at least widely known person as constituting an unregistered trademark or service mark sufficient for the purposes of paragraph 4(a)(i) (eg. Julia Fiona Roberts v Russell Boyd WIPO Case No. D2000-0210; Jeannette Winterson v Mark Hogarth WIPO Case No. D2000-0235; Steven Rattner v BuyThisDomainName (John Pepin) WIPO Case No. D2000-0402).

 

 

 

 

 

6.4 On the other hand, the Report of the WIPO Internet Domain Name Process of April 30, 1999, on which ICANN based the Uniform Policy, at paragraphs 165-168, states as follows (footnote citations deleted, emphasis added):

 

 

The preponderance of views, however, was in favor of restricting the scope of the procedure, at least initially, in order to deal first with the most offensive forms of predatory practices and to establish the procedure on a sound footing. Two limitations on the scope of the procedure were, as indicated above, favored by these commentators. The first limitation would confine the availability of the procedure to cases of deliberate, bad faith abusive registrations. The definition of such abusive registrations is discussed in the next section. The second limitation would define abusive registration by reference only to trademarks and service marks. Thus, registrations that violate trade names, geographical indications or personality rights would not be considered to fall within the definition of abusive registration for the purposes of the administrative procedure. Those in favor of this form of limitation pointed out that the violation of trademarks (and service marks) was the most common form of abuse and that the law with respect to trade names, geographical indications and personality rights is less evenly harmonized throughout the world, although international norms do exist requiring the protection of trade names and geographical indications. We are persuaded by the wisdom of proceeding firmly but cautiously and of tackling, at the first stage, problems which all agree require a solution. … [W]e consider that it is premature to extend the notion of abusive registration beyond the violation of trademarks and service marks at this stage. After experience has been gained with the operation of the administrative procedure and time has allowed for an assessment of its efficacy and of the problems, if any, which remain outstanding, the question of extending the notion of abusive registration to other intellectual property rights can always be re-visited.

 

 

It is clear from this statement that personality rights were not intended to be made subject to the proposed dispute resolution procedure. In adopting the procedure proposed in the WIPO Report, ICANN did not vary this limitation on its application. It must be concluded, therefore, that ICANN did not intend the procedure to apply to personality rights.

 

 

 

 

 

6.5 In the opinion of this Administrative Panel, it is doubtful whether the Uniform Policy is applicable to this dispute. Although it is accepted that the Complainant is world famous under the name STING, it does not follow that he has rights in STING as a trademark or service mark. Unlike the personal names in issue in the cases Julia Fiona Roberts v Russell Boyd, Jeannette Winterson v Mark Hogarth, and Steven Rattner v BuyThisDomainName (John Pepin), the personal name in this case is also a common word in the English language, with a number of different meanings. The following are the entries for “sting” from Merriam- Webster’s Collegiate Dictionary:

 

 

sting vb stung ; sting.ing [ME, fr. OE stingan; akin to ON stinga to sting and prob. to Gk stachys spike of grain, stochos target, aim] vt (bef. 12c) 1: to prick painfully: as a: to pierce or wound with a poisonous or irritating process b: to affect with sharp quick pain or smart “hail stung their faces” 2: to cause to suffer acutely “stung with remorse” 3: overcharge, cheat ~ vi 1: to wound one with or as if with a sting 2: to feel a keen burning pain or smart; also: to cause such pain – sting.ing.ly adv

 

 

 

 

 

 

 

sting n (bef. 12c) 1 a: the act of stinging; specif: the thrust of a stinger into the flesh b: a wound or pain caused by or as if by stinging 2: stinger 2 3: a sharp or stinging element, force, or quality 4: an elaborate confidence game; specif: such a game worked by undercover police in order to trap criminals

 

 

 

 

 

 

6.6 In light of the fact that the word “sting” is in common usage in the English language, with a number of meanings, this case can be distinguished from the other cases cited above in which the Complainants’ personal name was found also to be an unregistered trademark or service mark to which the Uniform Policy applies. This Administrative Panel is inclined to the view, therefore, that the Complainant’s name STING is not a trademark or service mark within the scope of paragraph 4(a)(i) of the Uniform Policy. However, it is not necessary to reach a formal decision on this issue, because this Administrative Panel finds against the Complainant on other grounds, namely that the requirement of paragraph 4(a)(iii) is not met, as discussed below.

 

 

 

 

Respondent’s Rights or Legitimate Interests in the Domain Name

 

 

 

 

6.7 The Respondent provided evidence of circumstances of the type specified in paragraph 4(c) of the Uniform Policy as giving rise to a right to or legitimate interest in the domain name. In particular, the Respondent provided in Exhibit C of the Response copies of various email communications to him prior to the commencement of this dispute, showing that the “UserName”, the “nickname”, the “Screen Name”, or the “Account PIC” under which the Respondent had registered for global internet gaming services consisted of or included the word “sting”. In Exhibit D to the Response, the Respondent provided copies of web page printouts from The Champions League of Quake, a service which monitors Quake servers and keeps track of the scores of registered players of this game. Those printouts show that the Respondent played this game using the player names “sting” or “=sting=”. In addition, the Respondent provided evidence in Exhibit E to the Response of preparations by him to establish a web site at the URL http://www.sting.com.

 

 

 

 

 

6.8 Although this evidence is not irrelevant to the issue of whether or not the Respondent has a right to or a legitimate interest in the domain name, it is certainly at the weaker end of the spectrum of such evidence. The Respondent’s use of the name “sting” or “=sting=” for gaming does not establish that he has been “commonly known” by the domain name as contemplated by paragraph 4(c)(ii). The word is undistinctive, and most likely is used by numerous people in cyberspace. In practice, this word provides the Respondent with anonymity rather than with a name by which he is commonly known. The Respondent’s evidence of his preparations to establish a web site at the URL http://www.sting.com does not establish the circumstances contemplated by paragraph 4(c)(i), because there is no evidence that this proposed use of the domain name is in connection with a bona fide offering of goods or services.

 

 

 

 

 

6.9 In short, a more substantive use of the word “sting” than that proven by the Respondent is required to show a right or legitimate interest in the domain name “sting.com” (although this proven use is relevant to the issue of bad faith). On balance, therefore, this Administrative Panel finds that the Respondent does not have a right to or a legitimate interest in the domain name, in the sense in which that concept is used in paragraph 4(a)(ii) of the Uniform Policy.

 

 

 

 

Domain Name Registered and Used in Bad Faith

 

 

 

 

6.10 The Complainant has not satisfied this Administrative Panel that the Respondent registered and is using the domain name in bad faith. The Complainant asserted that the Respondent offered to sell the domain name to the Complainant for $25,000, but the Complainant provided no evidence in support of this assertion. In particular, the Complainant provided no evidence of the Respondent’s alleged communications with the Complainant on this issue. The Respondent admitted that he offered to sell the domain name to the Complainant, but only after the Complainant solicited that offer. (The Respondent did not specify the price at which he offered to sell the domain name, but he did not dispute the Complainant’s assertion of $25,000, so this Administrative Panel assumes the offered price was for that amount, or at least for an amount in excess of the Respondent’s out-of-pocket expenses.) Although this evidence is consistent with the Complainant’s contention that the Respondent acquired the domain name primarily for the purpose of selling it to the Complainant, as required by paragraph 4(b)(i), this evidence does not prove that. This evidence is equally consistent with the Respondent’s contention that he acquired the domain name five years ago in good faith. In the absence of any evidence whatsoever from the Complainant going to the assertion of the Respondent’s offer to sell the domain name, this Administrative Panel finds that the Complainant has not met the burden of proof on this issue.

 

 

 

 

 

6.11 This Administrative Panel does not accept the Complainant’s contention that the linking of the domain name to the “GunBroker.com” web site constituted intentionally attempting to attract, for commercial gain, Internet users to an on-line location by creating a likelihood of confusion with the STING mark as to source, sponsorship, affiliation, or endorsement, and so constitutes an activity which corresponds to that listed in paragraph 4(b)(iv) of the Uniform Policy as evidence of bad faith registration and use of the domain name. Again, the Complainant provided no evidence in support of this contention. In particular, the Complainant provided no evidence as to the contents of the “GunBroker.com” site, and thus no evidence establishing that a likelihood of confusion with the STING mark was created as to source, sponsorship, affiliation or endorsement of the site. The Respondent admitted that the domain name did point to the “GunBroker.com” site for a period of time, but provided evidence to the effect that this was due to an error on the part of the Respondent’s web service provider. The evidence is therefore consistent with the Respondent’s contention that there was no intentional attempt to attract internet users for commercial gain. Once again, the Complainant has failed to satisfy its burden of proof on this point.

 

 

 

 

 

6.12 Finally, this Administrative Panel does not accept the Complainant’s contention that “it is not possible to conceive of any plausible actual or contemplated active use of the [D]omain [N]ame by the Respondent that would not be illegitimate, such as by being a passing off, an infringement of consumer protection legislation, or an infringement of the Complainant’s rights under trademark law”. The words in quotation marks come from Telstra Corporation Limited v Nuclear Marshmallows WIPO Case No. D2000-0003. In the Telstra case, the trademark in question was an invented word. In this case the mark in question is a common word in the English language, with a number of meanings. Unlike the situation in the Telstra case, therefore, it is far from inconceivable that there is a plausible legitimate use to which the Respondent could put the domain name. The Respondent has asserted a legitimate use to which he has put, and intends to put, the domain name. Whilst the evidence provided in support of this assertion is not particularly strong, it is at least consistent with that assertion, and with his overall contention that he did not register and has not been using the domain name in bad faith. The Complainant has thus failed to satisfy the burden of proof on this point.

 

 

 

 

7. Decision

 

 

 

 

7.1 This Administrative Panel decides that the Complainant has not proven each of the three elements in paragraph 4(a) of the Uniform Policy in relation to the domain name the subject of the Complaint.

 

 

 

 

 

7.2 Pursuant to paragraph 4(i) of the Uniform Policy and paragraph 15 of the Uniform Rules, this Administrative Panel denies the request that the Registrar, Network Solutions, Inc, be required either to transfer to the Complainant, Gordon Sumner, p/k/a Sting, or to cancel, the domain name “sting.com”.

 

 

 

License

Icon for the Creative Commons Attribution-ShareAlike 4.0 International License

Property Volume One by Christian Turner is licensed under a Creative Commons Attribution-ShareAlike 4.0 International License, except where otherwise noted.